06.01.2015 Views

The Real And Complex Number Systems

The Real And Complex Number Systems

The Real And Complex Number Systems

SHOW MORE
SHOW LESS

You also want an ePaper? Increase the reach of your titles

YUMPU automatically turns print PDFs into web optimized ePapers that Google loves.

<strong>The</strong> <strong>Real</strong> <strong>And</strong> <strong>Complex</strong> <strong>Number</strong> <strong>Systems</strong><br />

Integers<br />

1.1 Prove that there is no largest prime.<br />

Proof: Suppose p is the largest prime. <strong>The</strong>n p! + 1 is NOT a prime. So,<br />

there exists a prime q such that<br />

q |p! + 1 ⇒ q |1<br />

which is impossible. So, there is no largest prime.<br />

Remark: <strong>The</strong>re are many and many proofs about it. <strong>The</strong> proof that we<br />

give comes from Archimedes 287-212 B. C. In addition, Euler Leonhard<br />

(1707-1783) find another method to show it. <strong>The</strong> method is important since<br />

it develops to study the theory of numbers by analytic method. <strong>The</strong> reader<br />

can see the book, An Introduction To <strong>The</strong> <strong>The</strong>ory Of <strong>Number</strong>s by<br />

Loo-Keng Hua, pp 91-93. (Chinese Version)<br />

1.2 If n is a positive integer, prove the algebraic identity<br />

∑n−1<br />

a n − b n = (a − b) a k b n−1−k<br />

k=0<br />

Proof: It suffices to show that<br />

∑n−1<br />

x n − 1 = (x − 1) x k .<br />

k=0<br />

1


Consider the right hand side, we have<br />

∑n−1<br />

∑n−1<br />

∑n−1<br />

(x − 1) x k = x k+1 −<br />

k=0<br />

=<br />

k=0<br />

n∑<br />

x k −<br />

k=1<br />

= x n − 1.<br />

k=0<br />

∑n−1<br />

x k<br />

k=0<br />

x k<br />

1.3 If 2 n − 1 is a prime, prove that n is prime. A prime of the form<br />

2 p − 1, where p is prime, is called a Mersenne prime.<br />

Proof: If n is not a prime, then say n = ab, where a > 1 and b > 1. So,<br />

we have<br />

∑b−1<br />

2 ab − 1 = (2 a − 1) (2 a ) k<br />

which is not a prime by Exercise 1.2. So, n must be a prime.<br />

Remark: <strong>The</strong> study of Mersenne prime is important; it is related<br />

with so called Perfect number. In addition, there are some OPEN problem<br />

about it. For example, is there infinitely many Mersenne nembers<br />

<strong>The</strong> reader can see the book, An Introduction To <strong>The</strong> <strong>The</strong>ory<br />

Of <strong>Number</strong>s by Loo-Keng Hua, pp 13-15. (Chinese Version)<br />

1.4 If 2 n + 1 is a prime, prove that n is a power of 2. A prime of the<br />

form 2 2m + 1 is called a Fermat prime. Hint. Use exercise 1.2.<br />

Proof: If n is a not a power of 2, say n = ab, where b is an odd integer.<br />

So,<br />

2 a + 1 ∣ ∣ 2 ab + 1<br />

and 2 a + 1 < 2 ab + 1. It implies that 2 n + 1 is not a prime. So, n must be a<br />

power of 2.<br />

k=0<br />

Remark: (1) In the proof, we use the identity<br />

2n−2<br />

∑<br />

x 2n−1 + 1 = (x + 1) (−1) k x k .<br />

k=0<br />

2


Proof: Consider<br />

2n−2<br />

∑<br />

(x + 1) (−1) k x k =<br />

k=0<br />

=<br />

2n−2<br />

∑<br />

k=0<br />

2n−1<br />

(−1) k x k+1 +<br />

2n−2<br />

∑<br />

k=0<br />

2n−2<br />

(−1) k x k<br />

∑<br />

∑<br />

(−1) k+1 x k + (−1) k x k<br />

k=1<br />

= x 2n+1 + 1.<br />

k=0<br />

(2) <strong>The</strong> study of Fermat number is important; for the details the reader<br />

can see the book, An Introduction To <strong>The</strong> <strong>The</strong>ory Of <strong>Number</strong>s by<br />

Loo-Keng Hua, pp 15. (Chinese Version)<br />

1.5 <strong>The</strong> Fibonacci numbers 1, 1, 2, 3, 5, 8, 13, ... are defined by the recursion<br />

formula x n+1 = x n + x n−1 , with x 1 = x 2 = 1. Prove that (x n , x n+1 ) = 1<br />

and that x n = (a n − b n ) / (a − b) , where a and b are the roots of the quadratic<br />

equation x 2 − x − 1 = 0.<br />

So,<br />

Proof: Let d = g.c.d. (x n , x n+1 ) , then<br />

Continue the process, we finally have<br />

So, d = 1 since d is positive.<br />

Observe that<br />

and thus we consider<br />

i.e., consider<br />

If we let<br />

d |x n and d |x n+1 = x n + x n−1 .<br />

d |x n−1 .<br />

d |1 .<br />

x n+1 = x n + x n−1 ,<br />

x n+1 = x n + x n−1 ,<br />

x 2 = x + 1 with two roots, a and b.<br />

F n = (a n − b n ) / (a − b) ,<br />

3


then it is clear that<br />

So, F n = x n for all n.<br />

F 1 = 1, F 2 = 1, and F n+1 = F n + F n−1 for n > 1.<br />

Remark: <strong>The</strong> study of the Fibonacci numbers is important; the reader<br />

can see the book, Fibonacci and Lucas <strong>Number</strong>s with Applications<br />

by Koshy and Thomas.<br />

1.6 Prove that every nonempty set of positive integers contains a smallest<br />

member. This is called the well–ordering Principle.<br />

Proof: Given (φ ≠) S (⊆ N) , we prove that if S contains an integer<br />

k, then S contains the smallest member. We prove it by Mathematical<br />

Induction of second form as follows.<br />

As k = 1, it trivially holds. Assume that as k = 1, 2, ..., m holds, consider<br />

as k = m + 1 as follows. In order to show it, we consider two cases.<br />

(1) If there is a member s ∈ S such that s < m + 1, then by Induction<br />

hypothesis, we have proved it.<br />

(2) If every s ∈ S, s ≥ m + 1, then m + 1 is the smallest member.<br />

Hence, by Mathematical Induction, we complete it.<br />

Remark: We give a fundamental result to help the reader get more. We<br />

will prove the followings are equivalent:<br />

(A. Well–ordering Principle) every nonempty set of positive integers<br />

contains a smallest member.<br />

(B. Mathematical Induction of first form) Suppose that S (⊆ N) ,<br />

if S satisfies that<br />

<strong>The</strong>n S = N.<br />

(1). 1 in S<br />

(2). As k ∈ S, then k + 1 ∈ S.<br />

(C. Mathematical Induction of second form) Suppose that S (⊆ N) ,<br />

if S satisfies that<br />

(1). 1 in S<br />

(2). As 1, ..., k ∈ S, then k + 1 ∈ S.<br />

4


<strong>The</strong>n S = N.<br />

Proof: (A ⇒ B): If S ≠ N, then N − S ≠ φ. So, by (A), there exists<br />

the smallest integer w such that w ∈ N − S. Note that w > 1 by (1), so we<br />

consider w − 1 as follows.<br />

Since w − 1 /∈ N − S, we know that w − 1 ∈ S. By (2), we know that<br />

w ∈ S which contadicts to w ∈ N − S. Hence, S = N.<br />

(B ⇒ C): It is obvious.<br />

(C ⇒ A): We have proved it by this exercise.<br />

Rational and irrational numbers<br />

1.7 Find the rational number whose decimal expansion is 0.3344444444....<br />

Proof: Let x = 0.3344444444..., then<br />

x = 3 10 + 3<br />

10 + 4<br />

2 10 + ... + 4 + .., where n ≥ 3<br />

3 10n = 33<br />

10 + 4 (<br />

1 + 1 2 10 3 10 + ... + 1 )<br />

10 + .. n<br />

= 33<br />

10 + 4 ( 1<br />

2 10 3 1 − 1<br />

10<br />

= 33<br />

10 + 4<br />

2 900<br />

= 301<br />

900 .<br />

)<br />

1.8 Prove that the decimal expansion of x will end in zeros (or in nines)<br />

if, and only if, x is a rational number whose denominator is of the form 2 n 5 m ,<br />

where m and n are nonnegative integers.<br />

Proof: (⇐)Suppose that x =<br />

k , if n ≥ m, we have<br />

2 n 5 m<br />

k5 n−m<br />

2 n 5 n = 5n−m k<br />

10 n .<br />

So, the decimal expansion of x will end in zeros. Similarly for m ≥ n.<br />

(⇒)Suppose that the decimal expansion of x will end in zeros (or in<br />

nines).<br />

5


For case x = a 0 .a 1 a 2 · · · a n . <strong>The</strong>n<br />

∑ n<br />

k=0<br />

x =<br />

10n−k a k<br />

=<br />

10 n<br />

∑ n<br />

k=0 10n−k a k<br />

2 n 5 n .<br />

For case x = a 0 .a 1 a 2 · · · a n 999999 · · · . <strong>The</strong>n<br />

∑ n<br />

k=0<br />

x =<br />

10n−k a k<br />

+ 9<br />

9<br />

+ ... + + ...<br />

2 n 5 n 10n+1 10n+m ∑ n<br />

k=0<br />

=<br />

10n−k a k<br />

+ 9 ∑ ∞<br />

10 −j<br />

2 n 5 n 10 n+1 j=0<br />

∑ n<br />

k=0<br />

=<br />

10n−k a k<br />

+ 1<br />

2 n 5 n 10 n<br />

= 1 + ∑ n<br />

k=0 10n−k a k<br />

.<br />

2 n 5 n<br />

So, in both case, we prove that x is a rational number whose denominator is<br />

of the form 2 n 5 m , where m and n are nonnegative integers.<br />

1.9 Prove that √ 2 + √ 3 is irrational.<br />

Proof: If √ 2 + √ 3 is rational, then consider<br />

(√<br />

3 +<br />

√<br />

2<br />

) (√<br />

3 −<br />

√<br />

2<br />

)<br />

= 1<br />

which implies that √ 3 − √ 2 is rational. Hence, √ 3 would be rational. It is<br />

impossible. So, √ 2 + √ 3 is irrational.<br />

Remark: (1) √ p is an irrational if p is a prime.<br />

Proof: If √ p ∈ Q, write √ p = a , where g.c.d. (a, b) = 1. <strong>The</strong>n<br />

b<br />

Write a = pq. So,<br />

By (*) and (*’), we get<br />

b 2 p = a 2 ⇒ p ∣ ∣ a 2 ⇒ p |a (*)<br />

b 2 p = p 2 q 2 ⇒ b 2 = pq 2 ⇒ p ∣ ∣ b 2 ⇒ p |b . (*’)<br />

p |g.c.d. (a, b) = 1<br />

which implies that p = 1, a contradiction. So, √ p is an irrational if p is a<br />

prime.<br />

6


Note: <strong>The</strong>re are many and many methods to prove it. For example, the<br />

reader can see the book, An Introduction To <strong>The</strong> <strong>The</strong>ory Of <strong>Number</strong>s<br />

by Loo-Keng Hua, pp 19-21. (Chinese Version)<br />

(2) Suppose a, b ∈ N. Prove that √ a+ √ b is rational if and only if, a = k 2<br />

and b = h 2 for some h, k ∈ N.<br />

Proof: (⇐) It is clear.<br />

(⇒) Consider<br />

( √a +<br />

√<br />

b<br />

) ( √a −<br />

√<br />

b<br />

)<br />

= a 2 − b 2 ,<br />

then √ a ∈ Q and √ b ∈ Q. <strong>The</strong>n it is clear that a = h 2 and b = h 2 for some<br />

h, k ∈ N.<br />

1.10 If a, b, c, d are rational and if x is irrational, prove that (ax + b) / (cx + d)<br />

is usually irrational. When do exceptions occur<br />

Proof: We claim that (ax + b) / (cx + d) is rational if and only if ad = bc.<br />

(⇒)If (ax + b) / (cx + d) is rational, say (ax + b) / (cx + d) = q/p. We<br />

consider two cases as follows.<br />

(i) If q = 0, then ax + b = 0. If a ≠ 0, then x would be rational. So, a = 0<br />

and b = 0. Hence, we have<br />

ad = 0 = bc.<br />

(ii) If q ≠ 0, then (pa − qc) x+(pb − qd) = 0. If pa−qc ≠ 0, then x would<br />

be rational. So, pa − qc = 0 and pb − qd = 0. It implies that<br />

qcb = qad ⇒ ad = bc.<br />

(⇐)Suppose ad = bc. If a = 0, then b = 0 or c = 0. So,<br />

If a ≠ 0, then d = bc/a. So,<br />

{<br />

ax + b 0 if a = 0 and b = 0<br />

cx + d = b<br />

.<br />

if a = 0 and c = 0<br />

ax + b<br />

cx + d =<br />

d<br />

ax + b<br />

cx + bc/a<br />

=<br />

a (ax + b)<br />

c (ax + b) = a c .<br />

Hence, we proved that if ad = bc, then (ax + b) / (cx + d) is rational.<br />

7


1.11 Given any real x > 0, prove that there is an irrational number<br />

between 0 and x.<br />

Proof: If x ∈ Q c , we choose y = x/2 ∈ Q c . <strong>The</strong>n 0 < y < x. If x ∈ Q,<br />

we choose y = x/ √ 2 ∈ Q, then 0 < y < x.<br />

Remark: (1) <strong>The</strong>re are many and many proofs about it. We may prove<br />

it by the concept of Perfect set. <strong>The</strong> reader can see the book, Principles<br />

of Mathematical Analysis written by Walter Rudin, <strong>The</strong>orem 2.43,<br />

pp 41. Also see the textbook, Exercise 3.25.<br />

(2) Given a and b ∈ R with a < b, there exists r ∈ Q c , and q ∈ Q such<br />

that a < r < b and a < q < b.<br />

Proof: We show it by considering four cases. (i) a ∈ Q, b ∈ Q. (ii)<br />

a ∈ Q, b ∈ Q c . (iii) a ∈ Q c , b ∈ Q. (iv) a ∈ Q c , b ∈ Q c .(<br />

)<br />

1 − √ 1<br />

2<br />

b.<br />

(i) (a ∈ Q, b ∈ Q) Choose q = a+b<br />

2<br />

and r = √ 1<br />

2<br />

a +<br />

(ii) (a ∈ Q, b ∈ Q c ) Choose r = a+b<br />

2<br />

and let c = 1<br />

< b−a, then a+c := q.<br />

2 n<br />

(iii) (a ∈ Q c , b ∈ Q) Similarly for (iii).<br />

(iv) (a ∈ Q c , b ∈ Q c ) It suffices to show that there exists a rational<br />

number q ∈ (a, b) by (ii). Write<br />

Choose n large enough so that<br />

b = b 0 .b 1 b 2 · · · b n · ··<br />

a < q = b 0 .b 1 b 2 · · · b n < b.<br />

(It works since b − q = 0.000..000b n+1 ... ≤ 1<br />

10 n )<br />

1.12 If a/b < c/d with b > 0, d > 0, prove that (a + c) / (b + d) lies<br />

bwtween the two fractions a/b and c/d<br />

Proof: It only needs to conisder the substraction. So, we omit it.<br />

Remark: <strong>The</strong> result of this exercise is often used, so we suggest the<br />

reader keep it in mind.<br />

1.13 Let a and b be positive integers. Prove that √ 2 always lies between<br />

the two fractions a/b and (a + 2b) / (a + b) . Which fraction is closer to √ 2<br />

Proof: Suppose a/b ≤ √ 2, then a ≤ √ 2b. So,<br />

a + 2b<br />

a + b − √ (√ ) (√ )<br />

2 − 1 2b − a<br />

2 =<br />

≥ 0.<br />

a + b<br />

8


In addition,<br />

(√ a<br />

)<br />

2 − −<br />

b<br />

( a + 2b<br />

a + b − √ )<br />

2 = 2 √ 2 −<br />

( a<br />

b + a + 2b )<br />

a + b<br />

= 2 √ 2 − a2 + 2ab + 2b 2<br />

ab + b 2<br />

1<br />

[(<br />

= 2 √ )<br />

2 − 2 ab +<br />

ab + b 2<br />

≥<br />

= 0.<br />

1<br />

ab + b 2 [ (<br />

2 √ 2 − 2<br />

)<br />

a√ a + 2<br />

(<br />

2 √ ) ]<br />

2 − 2 b 2 − a 2<br />

(<br />

2 √ ) ( ) ] 2<br />

a<br />

2 − 2 √2 − a 2<br />

So, a+2b is closer to √ 2.<br />

a+b<br />

Similarly, we also have if a/b > √ 2, then a+2b<br />

a+b<br />

to √ 2 in this case.<br />

Remark: Note that<br />

a<br />

b < √ 2 < a + 2b<br />

a + b < 2b<br />

a<br />

< √ 2. Also, a+2b<br />

a+b<br />

by Exercise 12 and 13.<br />

is closer<br />

<strong>And</strong> we know that a+2b is closer to √ 2. We can use it to approximate √ 2.<br />

a+b<br />

Similarly for the case<br />

2b<br />

a < a + 2b<br />

a + b < √ 2 < a b .<br />

1.14 Prove that √ n − 1 + √ n + 1 is irrational for every integer n ≥ 1.<br />

Proof: Suppose that √ n − 1 + √ n + 1 is rational, and thus consider<br />

( √n<br />

+ 1 +<br />

√<br />

n − 1<br />

) ( √n<br />

+ 1 −<br />

√<br />

n − 1<br />

)<br />

= 2<br />

which implies that √ n + 1 − √ n − 1 is rational. Hence, √ n + 1 and √ n − 1<br />

are rational. So, n − 1 = k 2 and n + 1 = h 2 , where k and h are positive<br />

integer. It implies that<br />

h = 3 2 and k = 1 2<br />

which is absurb. So, √ n − 1 + √ n + 1 is irrational for every integer n ≥ 1.<br />

9


1.15 Given a real x and an integer N > 1, prove that there exist integers<br />

h and k with 0 < k ≤ N such that |kx − h| < 1/N. Hint. Consider the N +1<br />

numbers tx − [tx] for t = 0, 1, 2, ..., N and show that some pair differs by at<br />

most 1/N.<br />

Proof: Given N > 1, and thus consider tx − [tx] for t = 0, 1, 2, ..., N as<br />

follows. Since<br />

0 ≤ tx − [tx] := a t < 1,<br />

so there exists two numbers a i and a j where i ≠ j such that<br />

|a i − a j | < 1 N ⇒ |(i − j) x − p| < 1 , where p = [jx] − [ix] .<br />

N<br />

Hence, there exist integers h and k with 0 < k ≤ N such that |kx − h| < 1/N.<br />

1.16 If x is irrational prove that there are infinitely many rational numbers<br />

h/k with k > 0 such that |x − h/k| < 1/k 2 . Hint. Assume there are<br />

only a finite number h 1 /k 1 , ..., h r /k r and obtain a contradiction by applying<br />

Exercise 1.15 with N > 1/δ, where δ is the smallest of the numbers<br />

|x − h i /k i | .<br />

Proof: Assume there are only a finite number h 1 /k 1 , ..., h r /k r and let<br />

δ = min r i=1 |x − h i /k i | > 0 since x is irrational. Choose N > 1/δ, then by<br />

Exercise 1.15, we have<br />

∣<br />

1 ∣∣∣<br />

N < δ ≤ x − h k ∣ < 1<br />

kN<br />

which implies that<br />

1<br />

N < 1<br />

kN<br />

which is impossible. So, there are infinitely many rational numbers h/k with<br />

k > 0 such that |x − h/k| < 1/k 2 .<br />

Remark: (1) <strong>The</strong>re is another proof by continued fractions. <strong>The</strong><br />

reader can see the book, An Introduction To <strong>The</strong> <strong>The</strong>ory Of <strong>Number</strong>s<br />

by Loo-Keng Hua, pp 270. (Chinese Version)<br />

(2) <strong>The</strong> exercise is useful to help us show the following lemma. {ar + b : a ∈ Z, b ∈ Z} ,<br />

where r ∈ Q c is dense in R. It is equivalent to {ar : a ∈ Z} , where r ∈ Q c is<br />

dense in [0, 1] modulus 1.<br />

10


Proof: Say {ar + b : a ∈ Z, b ∈ Z} = S, and since r ∈ Q c , then by Exercise<br />

1.16, there are infinitely many rational numbers h/k with k > 0 such<br />

that |kr − h| < 1 . Consider (x − δ, x + δ) := I, where δ > 0, and thus choosing<br />

k 0 large enough so that 1/k 0 < δ. Define L = |k 0 r − h 0 | , then we have<br />

k<br />

sL ∈ I for some s ∈ Z. So, sL = (±) [(sk 0 ) r − (sh 0 )] ∈ S. That is, we have<br />

proved that S is dense in R.<br />

1.17 Let x be a positive rational number of the form<br />

x =<br />

n∑<br />

k=1<br />

where each a k is nonnegative integer with a k ≤ k − 1 for k ≥ 2 and a n > 0.<br />

Let [x] denote the largest integer in x. Prove that a 1 = [x] , that a k =<br />

[k!x] − k [(k − 1)!x] for k = 2, ..., n, and that n is the smallest integer such<br />

that n!x is an integer. Conversely, show that every positive rational number<br />

x can be expressed in this form in one and only one way.<br />

and<br />

Proof: (⇒)First,<br />

[ ]<br />

n∑ a k<br />

[x] = a 1 +<br />

k!<br />

k=2<br />

[ n∑<br />

]<br />

a k<br />

= a 1 + since a 1 ∈ N<br />

k!<br />

k=2<br />

n∑ a k<br />

n∑<br />

= a 1 since<br />

k! ≤ k − 1<br />

k!<br />

k=2<br />

k=2<br />

Second, fixed k and consider<br />

k!x = k!<br />

n∑<br />

j=1<br />

(k − 1)!x = (k − 1)!<br />

=<br />

k−1<br />

a j<br />

j! = k! ∑<br />

n∑<br />

j=1<br />

j=1<br />

a k<br />

k! ,<br />

n∑<br />

k=2<br />

1<br />

(k − 1)! − 1 k! = 1 − 1 n! < 1.<br />

a j<br />

j! + a k + k!<br />

k−1<br />

a j<br />

j! = (k − 1)! ∑<br />

j=1<br />

n∑<br />

j=k+1<br />

a j<br />

j!<br />

a j<br />

n∑<br />

j! + (k − 1)!<br />

j=k<br />

a j<br />

j! .<br />

11


So,<br />

and<br />

[<br />

∑k−1<br />

[k!x] = k!<br />

j=1<br />

∑k−1<br />

= k!<br />

j=1<br />

a j<br />

j! + a k + k!<br />

a j<br />

j! + a k since k!<br />

[<br />

∑k−1<br />

k [(k − 1)!x] = k (k − 1)!<br />

j=1<br />

∑k−1<br />

= k (k − 1)!<br />

∑k−1<br />

= k!<br />

j=1<br />

a j<br />

j!<br />

j=1<br />

n∑<br />

j=k+1<br />

n∑<br />

a j<br />

j!<br />

j=k+1<br />

]<br />

a j<br />

j! < 1<br />

a j<br />

n∑<br />

j! + (k − 1)!<br />

j=k<br />

]<br />

a j<br />

j! .<br />

a j<br />

n∑<br />

j! since (k − 1)!<br />

j=k<br />

a j<br />

j! < 1<br />

which implies that<br />

a k = [k!x] − k [(k − 1)!x] for k = 2, ..., n.<br />

Last, in order to show that n is the smallest integer such that n!x is an<br />

integer. It is clear that<br />

n∑ a k<br />

n!x = n!<br />

k! ∈ Z.<br />

In addition,<br />

So, we have proved it.<br />

k=1<br />

(n − 1)!x = (n − 1)!<br />

n∑<br />

k=1<br />

n−1<br />

∑<br />

= (n − 1)!<br />

k=1<br />

/∈ Z since a n<br />

n<br />

12<br />

a k<br />

k!<br />

a k<br />

k! + a n<br />

n<br />

/∈ Z.


(⇐)It is clear since every a n is uniquely deermined.<br />

Upper bounds<br />

1.18 Show that the sup and the inf of a set are uniquely determined whenever<br />

they exists.<br />

Proof: Given a nonempty set S (⊆ R) , and assume sup S = a and<br />

sup S = b, we show a = b as follows. Suppose that a > b, and thus choose<br />

ε = a−b , then there exists a x ∈ S such that<br />

2<br />

which implies that<br />

b < a + b<br />

2<br />

= a − ε < x < a<br />

b < x<br />

which contradicts to b = sup S. Similarly for a < b. Hence, a = b.<br />

1.19 Find the sup and inf of each of the following sets of real numbers:<br />

(a) All numbers of the form 2 −p + 3 −q + 5 −r , where p, q, and r take on all<br />

positive integer values.<br />

Proof: Define S = {2 −p + 3 −q + 5 −r : p, q, r ∈ N}. <strong>The</strong>n it is clear that<br />

sup S = 1 + 1 + 1 , and inf S = 0.<br />

2 3 5<br />

(b) S = {x : 3x 2 − 10x + 3 < 0}<br />

Proof: Since 3x 2 − 10x + 3 = (x − 3) (3x − 1) , we know that S = ( 1<br />

3 , 3) .<br />

Hence, sup S = 3 and inf S = 1 3 .<br />

(c) S = {x : (x − a) (x − b) (x − c) (x − d) < 0} , where a < b < c < d.<br />

Proof: It is clear that S = (a, b)∪(c, d) . Hence, sup S = d and inf S = a.<br />

1.20 Prove the comparison property for suprema (<strong>The</strong>orem 1.16)<br />

Proof: Since s ≤ t for every s ∈ S and t ∈ T, fixed t 0 ∈ T, then s ≤ t 0<br />

for all s ∈ S. Hence, by Axiom 10, we know that sup S exists. In addition,<br />

it is clear sup S ≤ sup T.<br />

Remark: <strong>The</strong>re is a useful result, we write it as a reference. Let S and T<br />

be two nonempty subsets of R. If S ⊆ T and sup T exists, then sup S exists<br />

and sup S ≤ sup T.<br />

13


Proof: Since sup T exists and S ⊆ T, we know that for every s ∈ S, we<br />

have<br />

s ≤ sup T.<br />

Hence, by Axiom 10, we have proved the existence of sup S. In addition,<br />

sup S ≤ sup T is trivial.<br />

1.21 Let A and B be two sets of positive numbers bounded above, and<br />

let a = sup A, b = sup B. Let C be the set of all products of the form xy,<br />

where x ∈ A and y ∈ B. Prove that ab = sup C.<br />

Proof: Given ε > 0, we want to find an element c ∈ C such that ab−ε <<br />

c. If we can show this, we have proved that sup C exists and equals ab.<br />

Since sup A = a > 0 and sup B = b > 0, we can choose n large enough<br />

such that a − ε/n > 0, b − ε/n > 0, and n > a + b. So, for this ε ′ = ε/n,<br />

there exists a ′ ∈ A and b ′ ∈ B such that<br />

which implies that<br />

a − ε ′ < a ′ and b − ε ′ < b ′<br />

ab − ε ′ (a + b − ε ′ ) < a ′ b ′ since a − ε ′ > 0 and b − ε ′ > 0<br />

which implies that<br />

which implies that<br />

ab − ε n (a + b) < a′ b ′ := c<br />

ab − ε < c.<br />

1.22 Given x > 0, and an integer k ≥ 2. Let a 0 denote the largest integer<br />

≤ x and, assumeing that a 0 , a 1 , ..., a n−1 have been defined, let a n denote the<br />

largest integer such that<br />

a 0 + a 1<br />

k + a 2<br />

k 2 + ... + a n<br />

k n ≤ x.<br />

Note: When k = 10 the integers a 0 , a 1 , ... are the digits in a decimal<br />

representation of x. For general k they provide a representation in<br />

the scale of k.<br />

(a) Prove that 0 ≤ a i ≤ k − 1 for each i = 1, 2, ...<br />

14


Proof: Choose a 0 = [x], and thus consider<br />

[kx − ka 0 ] := a 1<br />

then<br />

0 ≤ k (x − a 0 ) < k ⇒ 0 ≤ a 1 ≤ k − 1<br />

and<br />

a 0 + a 1<br />

k ≤ x ≤ a 0 + a 1<br />

k + 1 k .<br />

Continue the process, we then have<br />

0 ≤ a i ≤ k − 1 for each i = 1, 2, ...<br />

and<br />

a 0 + a 1<br />

k + a 2<br />

k 2 + ... + a n<br />

k n ≤ x < a 0 + a 1<br />

k + a 2<br />

k 2 + ... + a n<br />

k n + 1<br />

k n . (*)<br />

(b) Let r n = a 0 + a 1 k −1 + a 2 k −2 + ... + a n k −n and show that x is the sup<br />

of the set of rational numbers r 1 , r 2 , ...<br />

Proof: It is clear by (a)-(*).<br />

Inequality<br />

1.23 Prove Lagrange’s identity for real numbers:<br />

( n∑<br />

) 2 ( n∑<br />

) ( n∑<br />

)<br />

a k b k = a 2 k b 2 k −<br />

∑<br />

(a k b j − a j b k ) 2 .<br />

k=1<br />

k=1 k=1 1≤k


and<br />

( n∑<br />

) ( n∑<br />

)<br />

a k b k a k b k = ∑<br />

a k b k a j b j =<br />

k=1<br />

k=1<br />

1≤k,j≤n<br />

So,<br />

( n∑<br />

k=1<br />

a k b k<br />

) 2<br />

=<br />

=<br />

=<br />

( n∑<br />

k=1<br />

( n∑<br />

k=1<br />

( n∑<br />

k=1<br />

a 2 k<br />

a 2 k<br />

a 2 k<br />

) ( n∑<br />

k=1<br />

) ( n∑<br />

k=1<br />

) ( n∑<br />

k=1<br />

b 2 k<br />

b 2 k<br />

b 2 k<br />

)<br />

n∑<br />

k=1<br />

a 2 kb 2 k + ∑ k≠j<br />

+ ∑ a k b k a j b j − ∑ a 2 kb 2 j<br />

k≠j<br />

k≠j<br />

)<br />

∑<br />

+ 2 a k b k a j b j −<br />

∑<br />

)<br />

−<br />

1≤k| ≤ ‖x‖ ‖y‖ by Remark (1).<br />

1.24 Prove that for arbitrary real a k , b k , c k we have<br />

( n∑<br />

) 4 ( n∑<br />

) ( n∑<br />

) 2 ( n∑<br />

)<br />

a k b k c k ≤ a 4 k b 2 k c 4 k .<br />

k=1<br />

k=1 k=1 k=1<br />

16


Proof: Use Cauchy-Schwarz inequality twice, we then have<br />

( n∑<br />

) ⎡ 4 ( n∑<br />

) ⎤ 2<br />

2<br />

a k b k c k = ⎣ a k b k c k<br />

⎦<br />

k=1<br />

k=1<br />

( n∑<br />

) 2 ( n∑<br />

) 2<br />

≤ a 2 kc 2 k b 2 k<br />

k=1<br />

k=1<br />

( n∑<br />

) 2 ( n∑<br />

) ( n∑<br />

≤ a 4 k c 4 k<br />

k=1<br />

k=1 k=1<br />

( n∑<br />

) ( n∑<br />

) 2 ( n∑<br />

= a 4 k b 2 k<br />

k=1 k=1 k=1<br />

b 2 k<br />

c 4 k<br />

) 2<br />

)<br />

.<br />

1.25 Prove that Minkowski’s inequality:<br />

( n∑<br />

k=1<br />

(a k + b k ) 2 ) 1/2<br />

≤<br />

a 2 k) 1/2<br />

+<br />

b 2 k) 1/2<br />

.<br />

( n∑<br />

k=1<br />

( n∑<br />

k=1<br />

This is the triangle inequality ‖a + b‖ ≤ ‖a‖+‖b‖ for n−dimensional vectors,<br />

where a = (a 1 , ..., a n ) , b = (b 1 , ..., b n ) and<br />

‖a‖ =<br />

( n∑<br />

k=1<br />

a 2 k) 1/2<br />

.<br />

Proof: Consider<br />

n∑<br />

n∑ n∑<br />

n∑<br />

(a k + b k ) 2 = a 2 k + b 2 k + 2 a k b k<br />

k=1<br />

k=1<br />

k=1<br />

k=1<br />

(<br />

n∑ n∑<br />

n∑<br />

) 1/2 ( n∑ 1/2<br />

≤ a 2 k + b 2 k + 2 a 2 k bk) 2 by Cauchy-Schwarz inequality<br />

k=1 k=1<br />

k=1<br />

k=1<br />

⎡<br />

1/2 ) ⎤ 1/2<br />

2<br />

= ⎣ ak) 2 +<br />

⎦ .<br />

( n∑<br />

( n∑<br />

k=1<br />

k=1<br />

b 2 k<br />

17


So,<br />

( n∑<br />

k=1<br />

(a k + b k ) 2 ) 1/2<br />

≤<br />

a 2 k) 1/2<br />

+<br />

b 2 k) 1/2<br />

.<br />

( n∑<br />

k=1<br />

( n∑<br />

k=1<br />

1.26 If a 1 ≥ ... ≥ a n and b 1 ≥ ... ≥ b n , prove that<br />

( n∑<br />

) ( n∑<br />

) ( n∑<br />

)<br />

a k b k ≤ n a k b k .<br />

k=1 k=1<br />

k=1<br />

Hint. ∑ 1≤j≤k≤n (a k − a j ) (b k − b j ) ≥ 0.<br />

Proof: Consider<br />

0 ≤ ∑<br />

(a k − a j ) (b k − b j ) = ∑<br />

1≤j≤k≤n<br />

which implies that<br />

Since<br />

∑<br />

1≤j≤k≤n<br />

∑<br />

1≤j≤k≤n<br />

a k b j + a j b k =<br />

1≤j≤k≤n<br />

a k b j + a j b k ≤<br />

=<br />

=<br />

∑<br />

1≤j


In addition,<br />

∑<br />

a k b k + a j b j<br />

1≤j≤k≤n<br />

=<br />

= n<br />

n∑<br />

a k b k + na 1 b 1 +<br />

k=1<br />

n∑<br />

a k b k + (n − 1) a 2 b 2 + ... +<br />

k=2<br />

n∑<br />

a k b k + a 1 b 1 + a 2 b 2 + ... + a n b n<br />

k=1<br />

= (n + 1)<br />

n∑<br />

a k b k<br />

k=1<br />

which implies that, by (**),<br />

( n∑<br />

) ( n∑<br />

) ( n∑<br />

)<br />

a k b k ≤ n a k b k .<br />

k=1 k=1<br />

k=1<br />

n∑<br />

k=n−1<br />

a k b k + 2a n−1 b n−1 + ∑ k=n<br />

a k b k<br />

<strong>Complex</strong> numbers<br />

1.27 Express the following complex numbers in the form a + bi.<br />

(a) (1 + i) 3<br />

Solution: (1 + i) 3 = 1 + 3i + 3i 2 + i 3 = 1 + 3i − 3 − i = −2 + 2i.<br />

(b) (2 + 3i) / (3 − 4i)<br />

Solution: 2+3i<br />

3−4i = (2+3i)(3+4i)<br />

(3−4i)(3+4i) = −6+17i<br />

25<br />

= −6<br />

25 + 17<br />

25 i.<br />

(c) i 5 + i 16<br />

Solution: i 5 + i 16 = i + 1.<br />

(d) 1 2 (1 + i) (1 + i−8 )<br />

Solution: 1 2 (1 + i) (1 + i−8 ) = 1 + i.<br />

1.28 In each case, determine all real x and y which satisfy the given<br />

relation.<br />

19


(a) x + iy = |x − iy|<br />

Proof: Since |x − iy| ≥ 0, we have<br />

(b) x + iy = (x − iy) 2<br />

x ≥ 0 and y = 0.<br />

Proof: Since (x − iy) 2 = x 2 − (2xy) i − y 2 , we have<br />

x = x 2 − y 2 and y = −2xy.<br />

We consider tow cases: (i) y = 0 and (ii) y ≠ 0.<br />

(i) As y = 0 : x = 0 or 1.<br />

(ii) As y ≠ 0 : x = −1/2, and y = ± √ 3<br />

2 .<br />

(c) ∑ 100<br />

k=0 ik = x + iy<br />

Proof: Since ∑ 100<br />

k=0 ik = 1−i101<br />

1−i<br />

= 1−i<br />

1−i<br />

= 1, we have x = 1 and y = 0.<br />

1.29 If z = x+iy, x and y real, the complex conjugate of z is the complex<br />

number ¯z = x − iy. Prove that:<br />

(a) Conjugate of (z 1 + z 2 ) = ¯z 1 + ¯z 2<br />

Proof: Write z 1 = x 1 + iy 1 and z 2 = x 2 + iy 2 , then<br />

z 1 + z 2 = (x 1 + x 2 ) + i (y 1 + y 2 )<br />

= (x 1 + x 2 ) − i (y 1 + y 2 )<br />

= (x 1 − iy 1 ) + (x 2 − iy 2 )<br />

= ¯z 1 + ¯z 2 .<br />

(b) z 1 z 2 = ¯z 1¯z 2<br />

Proof: Write z 1 = x 1 + iy 1 and z 2 = x 2 + iy 2 , then<br />

z 1 z 2 = (x 1 x 2 − y 1 y 2 ) + i (x 1 y 2 + x 2 y 1 )<br />

= (x 1 x 2 − y 1 y 2 ) − i (x 1 y 2 + x 2 y 1 )<br />

and<br />

¯z 1¯z 2 = (x 1 − iy 1 ) (x 2 − iy 2 )<br />

= (x 1 x 2 − y 1 y 2 ) − i (x 1 y 2 + x 2 y 1 ) .<br />

20


So, z 1 z 2 = ¯z 1¯z 2<br />

(c) z¯z = |z| 2<br />

Proof: Write z = x + iy and thus<br />

z¯z = x 2 + y 2 = |z| 2 .<br />

(d) z + ¯z =twice the real part of z<br />

Proof: Write z = x + iy, then<br />

z + ¯z = 2x,<br />

twice the real part of z.<br />

(e) (z − ¯z) /i =twice the imaginary part of z<br />

Proof: Write z = x + iy, then<br />

z − ¯z<br />

i<br />

= 2y,<br />

twice the imaginary part of z.<br />

1.30 Describe geometrically the set of complex numbers z which satisfies<br />

each of the following conditions:<br />

(a) |z| = 1<br />

Solution: <strong>The</strong> unit circle centered at zero.<br />

(b) |z| < 1<br />

Solution: <strong>The</strong> open unit disk centered at zero.<br />

(c) |z| ≤ 1<br />

Solution: <strong>The</strong> closed unit disk centered at zero.<br />

(d) z + ¯z = 1<br />

Solution: Write z = x + iy, then z + ¯z = 1 means that x = 1/2. So, the<br />

set is the line x = 1/2.<br />

(e) z − ¯z = i<br />

21


Proof: Write z = x + iy, then z − ¯z = i means that y = 1/2. So, the set<br />

is the line y = 1/2.<br />

(f) z + ¯z = |z| 2<br />

Proof: Write z = x + iy, then 2x = x 2 + y 2 ⇔ (x − 1) 2 + y 2 = 1. So, the<br />

set is the unit circle centered at (1, 0) .<br />

1.31 Given three complex numbers z 1 , z 2 , z 3 such that |z 1 | = |z 2 | = |z 3 | =<br />

1 and z 1 + z 2 + z 3 = 0. Show that these numbers are vertices of an equilateral<br />

triangle inscribed in the unit circle with center at the origin.<br />

Proof: It is clear that three numbers are vertices of triangle inscribed in<br />

the unit circle with center at the origin. It remains to show that |z 1 − z 2 | =<br />

|z 2 − z 3 | = |z 3 − z 1 | . In addition, it suffices to show that<br />

Note that<br />

which is equivalent to<br />

which is equivalent to<br />

|z 1 − z 2 | = |z 2 − z 3 | .<br />

|2z 1 + z 3 | = |2z 3 + z 1 | by z 1 + z 2 + z 3 = 0<br />

|2z 1 + z 3 | 2 = |2z 3 + z 1 | 2<br />

(2z 1 + z 3 ) (2¯z 1 + ¯z 3 ) = (2z 3 + z 1 ) (2¯z 3 + ¯z 1 )<br />

which is equivalent to<br />

|z 1 | = |z 3 | .<br />

1.32 If a and b are complex numbers, prove that:<br />

(a) |a − b| 2 ≤ ( 1 + |a| 2) ( 1 + |b| 2)<br />

Proof: Consider<br />

(<br />

1 + |a|<br />

2 ) ( 1 + |b| 2) − |a − b| 2 = (1 + āa) ( 1 + ¯bb ) − (a − b) ( ā − ¯b )<br />

= (1 + āb) ( 1 + a¯b )<br />

= |1 + āb| 2 ≥ 0,<br />

22


so, |a − b| 2 ≤ ( 1 + |a| 2) ( 1 + |b| 2)<br />

(b) If a ≠ 0, then |a + b| = |a| + |b| if, and only if, b/a is real and<br />

nonnegative.<br />

Proof: (⇒)Since |a + b| = |a| + |b| , we have<br />

which implies that<br />

which implies that<br />

which implies that<br />

<strong>The</strong>n<br />

(⇐) Suppose that<br />

|a + b| 2 = (|a| + |b|) 2<br />

Re (āb) = |a| |b| = |ā| |b|<br />

b<br />

a = āb<br />

āa<br />

b<br />

a<br />

āb = |ā| |b|<br />

=<br />

|ā| |b|<br />

|a| 2 ≥ 0.<br />

= k, where k ≥ 0.<br />

|a + b| = |a + ka| = (1 + k) |a| = |a| + k |a| = |a| + |b| .<br />

1.33 If a and b are complex numbers, prove that<br />

|a − b| = |1 − āb|<br />

if, and only if, |a| = 1 or |b| = 1. For which a and b is the inequality<br />

|a − b| < |1 − āb| valid<br />

Proof: (⇔) Since<br />

|a − b| = |1 − āb|<br />

⇔ ( ā − ¯b ) (a − b) = (1 − āb) ( 1 − a¯b )<br />

⇔ |a| 2 + |b| 2 = 1 + |a| 2 |b| 2<br />

⇔ ( |a| 2 − 1 ) ( |b| 2 − 1 ) = 0<br />

⇔ |a| 2 = 1 or |b| 2 = 1.<br />

23


By the preceding, it is easy to know that<br />

|a − b| < |1 − āb| ⇔ 0 < ( |a| 2 − 1 ) ( |b| 2 − 1 ) .<br />

So, |a − b| < |1 − āb| if, and only if, |a| > 1 and |b| > 1. (Or |a| < 1 and<br />

|b| < 1).<br />

1.34 If a and c are real constant, b complex, show that the equation<br />

az¯z + b¯z + ¯bz + c = 0 (a ≠ 0, z = x + iy)<br />

represents a circle in the x − y plane.<br />

Proof: Consider<br />

[ (<br />

z¯z −<br />

b ¯b ¯z −<br />

−a −a z + b ) ] b<br />

=<br />

−a −a<br />

−ac + |b|2<br />

a 2 ,<br />

so, we have<br />

( )∣ b ∣∣∣<br />

2<br />

∣ z − =<br />

−a<br />

−ac + |b|2<br />

a 2 .<br />

Hence, as |b| 2 − ac > 0, it is a circle.<br />

−ac+|b| 2<br />

a 2<br />

< 0, it is not a circle.<br />

Remark: <strong>The</strong> idea is easy from the fact<br />

We square both sides and thus<br />

|z − q| = r.<br />

z¯z − q¯z − ¯qz + ¯qq = r 2 .<br />

As −ac+|b|2<br />

a 2 = 0, it is a point. As<br />

1.35 Recall the definition of the inverse tangent: given a real number t,<br />

tan −1 (t) is the unique real number θ which satisfies the two conditions<br />

− π 2 < θ < +π , tan θ = t.<br />

2<br />

If z = x + iy, show that<br />

(a) arg (z) = tan ( ) −1 y<br />

x , if x > 0<br />

24


Proof: Note that in this text book, we say arg (z) is the principal argument<br />

of z, denoted by θ = arg z, where −π < θ ≤ π.<br />

So, as x > 0, arg z = tan −1 ( y<br />

x)<br />

.<br />

(b) arg (z) = tan −1 ( y<br />

x)<br />

+ π, if x < 0, y ≥ 0<br />

Proof: As x < 0, and y ≥ 0. <strong>The</strong> point (x, y) is lying on S = {(x, y) : x < 0, y ≥ 0} .<br />

Note that −π < arg z ≤ π, so we have arg (z) = tan −1 ( y<br />

x)<br />

+ π.<br />

(c) arg (z) = tan −1 ( y<br />

x)<br />

− π, if x < 0, y < 0<br />

Proof: Similarly for (b). So, we omit it.<br />

(d) arg (z) = π 2 if x = 0, y > 0; arg (z) = − π 2<br />

if x = 0, y < 0.<br />

Proof: It is obvious.<br />

1.36 Define the folowing ”pseudo-ordering” of the complex numbers:<br />

we say z 1 < z 2 if we have either<br />

(i) |z 1 | < |z 2 | or (ii) |z 1 | = |z 2 | and arg (z 1 ) < arg (z 2 ) .<br />

Which of Axioms 6,7,8,9 are satisfied by this relation<br />

Proof: (1) For axiom 6, we prove that it holds as follows. Given z 1 =<br />

r 1 e i arg(z 1) , and r 2 e i arg(z 2) , then if z 1 = z 2 , there is nothing to prove it. If<br />

z 1 ≠ z 2 , there are two possibilities: (a) r 1 ≠ r 2 , or (b) r 1 = r 2 and arg (z 1 ) ≠<br />

arg (z 2 ) . So, it is clear that axiom 6 holds.<br />

(2) For axiom 7, we prove that it does not hold as follows. Given z 1 = 1<br />

and z 2 = −1, then it is clear that z 1 < z 2 since |z 1 | = |z 2 | = 1 and arg (z 1 ) =<br />

0 < arg (z 2 ) = π. However, let z 3 = −i, we have<br />

z 1 + z 3 = 1 − i > z 2 + z 3 = −1 − i<br />

since<br />

and<br />

|z 1 + z 3 | = |z 2 + z 3 | = √ 2<br />

arg (z 1 + z 3 ) = − π 4 > −3π 4 = arg (z 2 + z 3 ) .<br />

(3) For axiom 8, we prove that it holds as follows. If z 1 > 0 and z 2 > 0,<br />

then |z 1 | > 0 and |z 2 | > 0. Hence, z 1 z 2 > 0 by |z 1 z 2 | = |z 1 | |z 2 | > 0.<br />

(4) For axiom 9, we prove that it holds as follows. If z 1 > z 2 and z 2 > z 3 ,<br />

we consider the following cases. Since z 1 > z 2 , we may have (a) |z 1 | > |z 2 | or<br />

(b) |z 1 | = |z 2 | and arg (z 1 ) < arg (z 2 ) .<br />

As |z 1 | > |z 2 | , it is clear that |z 1 | > |z 3 | . So, z 1 > z 3 .<br />

25


As |z 1 | = |z 2 | and arg (z 1 ) < arg (z 2 ) , we have arg (z 1 ) > arg (z 3 ) . So,<br />

z 1 > z 3 .<br />

1.37 Which of Axioms 6,7,8,9 are satisfied if the pseudo-ordering is<br />

defined as follows We say (x 1 , y 1 ) < (x 2 , y 2 ) if we have either (i) x 1 < x 2 or<br />

(ii) x 1 = x 2 and y 1 < y 2 .<br />

Proof: (1) For axiom 6, we prove that it holds as follows. Given x =<br />

(x 1 , y 1 ) and y = (x 2 , y 2 ) . If x = y, there is nothing to prove it. We consider<br />

x ≠ y : As x ≠ y, we have x 1 ≠ x 2 or y 1 ≠ y 2 . Both cases imply x < y or<br />

y < x.<br />

(2) For axiom 7, we prove that it holds as follows. Given x = (x 1 , y 1 ) ,<br />

y = (x 2 , y 2 ) and z = (z 1 , z 3 ) . If x < y, then there are two possibilities: (a)<br />

x 1 < x 2 or (b) x 1 = x 2 and y 1 < y 2 .<br />

For case (a), it is clear that x 1 + z 1 < y 1 + z 1 . So, x + z < y + z.<br />

For case (b), it is clear that x 1 + z 1 = y 1 + z 1 and x 2 + z 2 < y 2 + z 2 . So,<br />

x + z < y + z.<br />

(3) For axiom 8, we prove that it does not hold as follows. Consider<br />

x = (1, 0) and y = (0, 1) , then it is clear that x > 0 and y > 0. However,<br />

xy = (0, 0) = 0.<br />

(4) For axiom 9, we prove that it holds as follows. Given x = (x 1 , y 1 ) ,<br />

y = (x 2 , y 2 ) and z = (z 1 , z 3 ) . If x > y and y > z, then we consider the<br />

following cases. (a) x 1 > y 1 , or (b) x 1 = y 1 .<br />

For case (a), it is clear that x 1 > z 1 . So, x > z.<br />

For case (b), it is clear that x 2 > y 2 . So, x > z.<br />

1.38 State and prove a theorem analogous to <strong>The</strong>orem 1.48, expressing<br />

arg (z 1 /z 2 ) in terms of arg (z 1 ) and arg (z 2 ) .<br />

Proof: Write z 1 = r 1 e i arg(z 1) and z 2 = r 2 e i arg(z 2) , then<br />

z 1<br />

z 2<br />

= r 1<br />

r 2<br />

e i[arg(z 1)−arg(z 2 )] .<br />

Hence,<br />

where<br />

( )<br />

z1<br />

arg = arg (z 1 ) − arg (z 2 ) + 2πn (z 1 , z 2 ) ,<br />

z 2<br />

⎧<br />

⎨ 0 if − π < arg (z 1 ) − arg (z 2 ) ≤ π<br />

n (z 1 , z 2 ) = 1 if − 2π < arg (z 1 ) − arg (z 2 ) ≤ −π<br />

⎩<br />

−1 if π < arg (z 1 ) − arg (z 2 ) < 2π<br />

.<br />

26


1.39 State and prove a theorem analogous to <strong>The</strong>orem 1.54, expressing<br />

Log (z 1 /z 2 ) in terms of Log (z 1 ) and Log (z 2 ) .<br />

Proof: Write z 1 = r 1 e i arg(z 1) and z 2 = r 2 e i arg(z 2) , then<br />

Hence,<br />

∣ Log (z 1 /z 2 ) = log<br />

z 1 ∣∣∣<br />

∣ + i arg<br />

z 2<br />

z 1<br />

z 2<br />

= r 1<br />

r 2<br />

e i[arg(z 1)−arg(z 2 )] .<br />

(<br />

z1<br />

z 2<br />

)<br />

= log |z 1 | − log |z 2 | + i [arg (z 1 ) − arg (z 2 ) + 2πn (z 1 , z 2 )] by xercise 1.38<br />

= Log (z 1 ) − Log (z 2 ) + i2πn (z 1 , z 2 ) .<br />

1.40 Prove that the nth roots of 1 (also called the nth roots of unity)<br />

are given by α, α 2 , ..., α n , where α = e 2πi/n , and show that the roots ≠ 1<br />

satisfy the equation<br />

1 + x + x 2 + ... + x n−1 = 0.<br />

Proof: By <strong>The</strong>orem 1.51, we know that the roots of 1 are given by<br />

α, α 2 , ..., α n , where α = e 2πi/n . In addition, since<br />

which implies that<br />

x n = 1 ⇒ (x − 1) ( 1 + x + x 2 + ... + x n−1) = 0<br />

1 + x + x 2 + ... + x n−1 = 0 if x ≠ 1.<br />

So, all roots except 1 satisfy the equation<br />

1 + x + x 2 + ... + x n−1 = 0.<br />

1.41 (a) Prove that |z i | < e π for all complex z ≠ 0.<br />

Proof: Since<br />

z i = e iLog(z) = e − arg(z)+i log|z| ,<br />

27


we have<br />

∣ ∣z i ∣ ∣ = e<br />

− arg(z) < e π<br />

by −π < arg (z) ≤ π.<br />

(b) Prove that there is no constant M > 0 such that |cos z| < M for all<br />

complex z.<br />

Proof: Write z = x + iy and thus,<br />

cos z = cos x cosh y − i sin x sinh y<br />

which implies that<br />

|cos x cosh y| ≤ |cos z| .<br />

Let x = 0 and y be real, then<br />

e y<br />

2 ≤ 1 ∣ e y + e −y∣ ∣ ≤ |cos z| .<br />

2<br />

So, there is no constant M > 0 such that |cos z| < M for all complex z.<br />

Remark: <strong>The</strong>re is an important theorem related with this exercise. We<br />

state it as a reference. (Liouville’s <strong>The</strong>orem) A bounded entire function<br />

is constant. <strong>The</strong> reader can see the book, <strong>Complex</strong> Analysis by Joseph<br />

Bak, and Donald J. Newman, pp 62-63. Liouville’s <strong>The</strong>orem can<br />

be used to prove the much important theorem, Fundamental <strong>The</strong>orem of<br />

Algebra.<br />

1.42 If w = u + iv (u, v real), show that<br />

z w = e u log|z|−v arg(z) e i[v log|z|+u arg(z)] .<br />

Proof: Write z w = e wLog(z) , and thus<br />

wLog (z) = (u + iv) (log |z| + i arg (z))<br />

= [u log |z| − v arg (z)] + i [v log |z| + u arg (z)] .<br />

So,<br />

z w = e u log|z|−v arg(z) e i[v log|z|+u arg(z)] .<br />

28


1.43 (a) Prove that Log (z w ) = wLog z +2πin.<br />

Proof: Write w = u + iv, where u and v are real. <strong>The</strong>n<br />

Log (z w ) = log |z w | + i arg (z w )<br />

= log [ e u log|z|−v arg(z)] + i [v log |z| + u arg (z)] + 2πin by Exercise1.42<br />

= u log |z| − v arg (z) + i [v log |z| + u arg (z)] + 2πin.<br />

On the other hand,<br />

wLogz + 2πin = (u + iv) (log |z| + i arg (z)) + 2πin<br />

= u log |z| − v arg (z) + i [v log |z| + u arg (z)] + 2πin.<br />

Hence, Log (z w ) = wLog z +2πin.<br />

Remark: <strong>The</strong>re is another proof by considering<br />

e Log(zw) = z w = e wLog(z)<br />

which implies that<br />

Log (z w ) = wLogz + 2πin<br />

for some n ∈ Z.<br />

(b) Prove that (z w ) α = z wα e 2πinα , where n is an integer.<br />

Proof: By (a), we have<br />

(z w ) α = e αLog(zw) = e α(wLogz+2πin) = e αwLogz e 2πinα = z αw e 2πinα ,<br />

where n is an integer.<br />

1.44 (i) If θ and a are real numbers, −π < θ ≤ π, prove that<br />

(cos θ + i sin θ) a = cos (aθ) + i sin (aθ) .<br />

Proof: Write cos θ + i sin θ = z, we then have<br />

(cos θ + i sin θ) a = z a = e aLogz = e a[log|eiθ |+i arg(e iθ )] = e<br />

iaθ<br />

= cos (aθ) + i sin (aθ) .<br />

29


Remark: Compare with the Exercise 1.43-(b).<br />

(ii) Show that, in general, the restriction −π < θ ≤ π is necessary in (i)<br />

by taking θ = −π, a = 1 2 .<br />

Proof: As θ = −π, and a = 1 , we have<br />

2<br />

(<br />

(−1) 1 1<br />

2 = e 2 Log(−1) = e π −π<br />

2 i = i ≠ −i = cos<br />

2<br />

) ( −π<br />

+ i sin<br />

2<br />

)<br />

.<br />

(iii) If a is an integer, show that the formula in (i) holds without any<br />

restriction on θ. In this case it is known as DeMorvre’s theorem.<br />

Proof: By Exercise 1.43, as a is an integer we have<br />

(z w ) a = z wa ,<br />

where z w = e iθ . <strong>The</strong>n<br />

(<br />

e<br />

iθ ) a<br />

= e iθa = cos (aθ) + i sin (aθ) .<br />

1.45 Use DeMorvre’s theorem (Exercise 1.44) to derive the triginometric<br />

identities<br />

sin 3θ = 3 cos 2 θ sin θ − sin 3 θ<br />

cos 3θ = cos 3 θ − 3 cos θ sin 2 θ,<br />

valid for real θ. Are these valid when θ is complex<br />

Proof: By Exercise 1.44-(iii), we have for any real θ,<br />

By Binomial <strong>The</strong>orem, we have<br />

(cos θ + i sin θ) 3 = cos (3θ) + i sin (3θ) .<br />

sin 3θ = 3 cos 2 θ sin θ − sin 3 θ<br />

and<br />

cos 3θ = cos 3 θ − 3 cos θ sin 2 θ.<br />

30


For complex θ, we show that it holds as follows. Note that sin z = eiz −e −iz<br />

2i<br />

and cos z = eiz +e −iz<br />

, we have<br />

2<br />

( ) e<br />

3 cos 2 z sin z − sin 3 iz + e −iz 2 ( ) ( ) e iz − e −iz e iz − e −iz 3<br />

z = 3<br />

−<br />

2<br />

2i<br />

2i<br />

( ) ( ) e 2zi + e −2zi + 2 e iz − e −iz<br />

= 3<br />

+ e3zi − 3e iz + 3e −iz − e −3zi<br />

4<br />

2i<br />

8i<br />

= 1 [ (<br />

3 e 2zi + e −2zi + 2 ) ( e zi − e −zi) + ( e 3zi − 3e iz + 3e −iz − e −3zi)]<br />

8i<br />

= 1 [(<br />

3e 3zi + 3e iz − 3e −iz − 3e −3zi) + ( e 3zi − 3e iz + 3e −iz − e −3zi)]<br />

8i<br />

= 4 (<br />

e 3zi − e −3zi)<br />

8i<br />

= 1 (<br />

e 3zi − e −3zi)<br />

2i<br />

Similarly, we also have<br />

= sin 3z.<br />

cos 3 z − 3 cos z sin 2 z = cos 3z.<br />

1.46 Define tan z = sin z/ cos z and show that for z = x + iy, we have<br />

tan z =<br />

sin 2x + i sinh 2y<br />

cos 2x + cosh 2y .<br />

31


Proof: Since<br />

tan z = sin z sin (x + iy) sin x cosh y + i cos x sinh y<br />

= =<br />

cos z cos (x + iy) cos x cosh y − i sin x sinh y<br />

(sin x cosh y + i cos x sinh y) (cos x cosh y + i sin x sinh y)<br />

=<br />

(cos x cosh y − i sin x sinh y) (cos x cosh y + i sin x sinh y)<br />

(<br />

sin x cos x cosh 2 y − sin x cos x sinh 2 y ) + i ( sin 2 x cosh y sinh y + cos 2 x cosh y sinh y )<br />

=<br />

(cos x cosh y) 2 − (i sin x sinh y) 2<br />

= sin x cos x ( cosh 2 y − sinh 2 y ) + i (cosh y sinh y)<br />

cos 2 x cosh 2 y + sin 2 x sinh 2 since sin 2 x + cos 2 x = 1<br />

y<br />

(sin x cos x) + i (cosh y sinh y)<br />

=<br />

cos 2 x + sinh 2 since cosh 2 y = 1 + sinh 2 y<br />

y<br />

1<br />

2<br />

=<br />

sin 2x + i sinh 2y<br />

2<br />

cos 2 x + sinh 2 since 2 cosh y sinh y = sinh 2y and 2 sin x cos x = sin 2x<br />

y<br />

sin 2x + i sinh 2y<br />

=<br />

2 cos 2 x + 2 sinh 2 y<br />

sin 2x + i sinh 2y<br />

=<br />

2 cos 2 x − 1 + 2 sinh 2 y + 1<br />

sin 2x + i sinh 2y<br />

=<br />

cos 2x + cosh 2y since cos 2x = 2 cos2 x − 1 and 2 sinh 2 y + 1 = cosh 2y.<br />

1.47 Let w be a given complex number. If w ≠ ±1, show that there exists<br />

two values of z = x + iy satisfying the conditions cos z = w and −π < x ≤ π.<br />

Find these values when w = i and when w = 2.<br />

Proof: Since cos z = eiz +e −iz<br />

, if we let e iz = u, then cos z = w implies<br />

2<br />

that<br />

w = u2 + 1<br />

⇒ u 2 − 2wu + 1 = 0<br />

2u<br />

which implies that<br />

So, by <strong>The</strong>orem 1.51,<br />

(u − w) 2 = w 2 − 1 ≠ 0 since w ≠ ±1.<br />

e iz = u = w + ∣ w 2 − 1 ∣ 1/2 e iφ k<br />

, where φ k = arg (w2 − 1)<br />

2<br />

(<br />

)<br />

= w ± ∣ w 2 − 1 ∣ 1/2 e i arg(w 2 −1)<br />

2<br />

+ 2πk , k = 0, 1.<br />

2<br />

32


So,<br />

ix−y = i (x + iy) = iz = log<br />

∣ w ± ∣ w 2 − 1 ∣ ⎛<br />

(<br />

)⎞<br />

1/2 e i arg (w 2 −1)<br />

2<br />

∣ +i arg ⎝w ± ∣ w 2 − 1 ∣ 1/2 e i arg ( w 2 −1 )<br />

2<br />

⎠<br />

Hence, there exists two values of z = x+iy satisfying the conditions cos z = w<br />

and<br />

⎛<br />

(<br />

)⎞<br />

−π < x = arg ⎝w ± ∣ w 2 − 1 ∣ 1/2 e i arg(w 2 −1)<br />

2<br />

⎠ ≤ π.<br />

For w = i, we have<br />

(<br />

iz = log ∣ 1 ± √ )<br />

2 i∣ + i arg<br />

which implies that<br />

z = arg<br />

((<br />

1 ± √ ) )<br />

2 i<br />

For w = 2, we have<br />

∣<br />

iz = log ∣2 ± √ 3∣ + i arg<br />

((<br />

1 ± √ ) )<br />

2 i<br />

(<br />

− i log ∣ 1 ± √ )<br />

2 i∣ .<br />

(<br />

2 ± √ )<br />

3<br />

which implies that<br />

(<br />

z = arg 2 ± √ )<br />

3<br />

∣<br />

− i log ∣2 ± √ 3∣ .<br />

1.48 Prove Lagrange’s identity for complex numbers:<br />

∣ n∑ ∣∣∣∣<br />

2 n∑ n∑<br />

a k b k = |a k | 2 |b k | 2 − ∑ ( ) 2<br />

ak¯bj − ā j b k .<br />

∣<br />

k=1<br />

k=1<br />

k=1<br />

1≤k


Proof: By Exercise 1.44 (i), we have<br />

sin nθ =<br />

[ n+1<br />

2 ]<br />

∑<br />

k=1<br />

( n2k−1<br />

)<br />

sin 2k−1 θ cos n−(2k−1) θ<br />

⎧<br />

⎫<br />

⎪⎨ [ n+1<br />

2<br />

∑<br />

]<br />

(<br />

= sin n θ<br />

n2k−1<br />

)<br />

⎪⎬<br />

cot n−(2k−1) θ<br />

⎪⎩<br />

⎪⎭<br />

k=1<br />

= sin n θ { ( n 1) cot n−1 θ − ( n 3) cot n−3 θ + ( n 5) cot n−5 θ − +... } .<br />

(b) If 0 < θ < π/2, prove that<br />

sin (2m + 1) θ = sin 2m+1 θP m<br />

(<br />

cot 2 θ )<br />

where P m is the polynomial of degree m given by<br />

P m (x) = ( )<br />

2m+1<br />

1 x m − ( )<br />

2m+1<br />

3 x m−1 + ( )<br />

2m+1<br />

5 x m−2 − +...<br />

Use this to show that P m has zeros at the m distinct points x k = cot 2 {πk/ (2m + 1)}<br />

for k = 1, 2, ..., m.<br />

Proof: By (a),<br />

sin (2m + 1) θ<br />

= sin 2m+1 θ<br />

{ (2m+1<br />

1<br />

) (<br />

cot 2 θ ) m<br />

−<br />

( 2m+1<br />

3<br />

= sin 2m+1 θP m<br />

(<br />

cot 2 θ ) , where P m (x) =<br />

) (<br />

cot 2 θ ) m−1 (<br />

+<br />

2m+1<br />

) (<br />

5 cot 2 θ ) }<br />

m−2<br />

− +...<br />

m+1<br />

∑<br />

k=1<br />

( 2m+1<br />

)<br />

2k−1 x m+1−k . (*)<br />

In addition, by (*), sin (2m + 1) θ = 0 if, and only if, P m (cot 2 θ) = 0. Hence,<br />

P m has zeros at the m distinct points x k = cot 2 {πk/ (2m + 1)} for k =<br />

1, 2, ..., m.<br />

(c) Show that the sum of the zeros of P m is given by<br />

m∑<br />

cot 2 πk<br />

2m + 1<br />

k=1<br />

34<br />

m (2m − 1)<br />

= ,<br />

3


and the sum of their squares is given by<br />

m∑<br />

cot 4 πk<br />

2m + 1 = m (2m − 1) (4m2 + 10m − 9)<br />

.<br />

45<br />

k=1<br />

Note. <strong>The</strong>re identities can be used to prove that ∑ ∞<br />

∑ n=1 n−2 = π 2 /6 and<br />

∞<br />

n=1 n−4 = π 4 /90. (See Exercises 8.46 and 8.47.)<br />

Proof: By (b), we know that sum of the zeros of P m is given by<br />

( ( m∑ m∑<br />

x k = cot 2 πk − 2m+1<br />

))<br />

2m + 1 = − ( 3 m (2m − 1)<br />

2m+1<br />

) = .<br />

3<br />

k=1<br />

k=1<br />

<strong>And</strong> the sum of their squares is given by<br />

m∑ m∑<br />

x 2 k = cot 4 πk<br />

2m + 1<br />

k=1 k=1<br />

( m<br />

) 2 (<br />

∑<br />

∑<br />

= x k − 2<br />

k=1<br />

1<br />

1≤i


So, let z = 1, we obtain that<br />

n−1<br />

∏ (<br />

n =<br />

) n−1<br />

∏<br />

[(<br />

1 − e<br />

2πik/n<br />

= 1 − cos 2πk<br />

n<br />

k=1<br />

n−1<br />

∏<br />

=<br />

k=1<br />

n−1<br />

∏<br />

=<br />

k=1<br />

n−1<br />

∏<br />

= 2 n−1<br />

(<br />

2 sin 2 πk<br />

n<br />

(<br />

2 sin πk<br />

n<br />

k=1<br />

n−1<br />

∏<br />

= 2 n−1<br />

=<br />

[<br />

k=1<br />

∏<br />

2 n−1 n−1<br />

k=1<br />

n−1<br />

∏<br />

= 2 n−1<br />

k=1<br />

)<br />

− i<br />

(<br />

sin πk<br />

n<br />

(<br />

sin πk<br />

n<br />

k=1<br />

) (<br />

sin πk<br />

n<br />

(<br />

2 sin πk πk<br />

cos<br />

n n<br />

)<br />

) (<br />

cos<br />

)<br />

e i( 3π 2 + πk<br />

n )<br />

(<br />

sin πk ) ] e ∑ n−1 3π<br />

k=1 2 + πk<br />

n<br />

n<br />

(<br />

sin πk<br />

n<br />

)<br />

.<br />

)<br />

)<br />

πk<br />

− i cos<br />

n<br />

( 3π<br />

2 + πk )<br />

+ i sin<br />

n<br />

(<br />

− i sin 2πk )]<br />

n<br />

( 3π<br />

2 + πk ))<br />

n<br />

36


Some Basic Notations Of Set <strong>The</strong>ory<br />

References<br />

<strong>The</strong>re are some good books about set theory; we write them down. We<br />

wish the reader can get more.<br />

1. Set <strong>The</strong>ory and Related Topics by Seymour Lipschutz.<br />

2. Set <strong>The</strong>ory by Charles C. Pinter.<br />

3. <strong>The</strong>ory of sets by Kamke.<br />

4. Naive set by Halmos.<br />

2.1 Prove <strong>The</strong>orem 2.2. Hint. (a, b) = (c, d) means {{a} , {a, b}} =<br />

{{c} , {c, d}} . Now appeal to the definition of set equality.<br />

Proof: (⇐) It is trivial.<br />

(⇒) Suppose that (a, b) = (c, d) , it means that {{a} , {a, b}} = {{c} , {c, d}} .<br />

It implies that<br />

{a} ∈ {{c} , {c, d}} and {a, b} ∈ {{c} , {c, d}} .<br />

So, if a ≠ c, then {a} = {c, d} . It implies that c ∈ {a} which is impossible.<br />

Hence, a = c. Similarly, we have b = d.<br />

2.2 Let S be a relation and let D (S) be its domain. <strong>The</strong> relation S is<br />

said to be<br />

(i) reflexive if a ∈ D (S) implies (a, a) ∈ S,<br />

(ii) symmetric if (a, b) ∈ S implies (b, a) ∈ S,<br />

(iii) transitive if (a, b) ∈ S and (b, c) ∈ S implies (a, c) ∈ S.<br />

A relation which is symmetric, reflexive, and transitive is called an equivalence<br />

relation. Determine which of these properties is possessed by S, if S<br />

is the set of all pairs of real numbers (x, y) such that<br />

(a) x ≤ y<br />

Proof: Write S = {(x, y) : x ≤ y} , then we check that (i) reflexive, (ii)<br />

symmetric, and (iii) transitive as follows. It is clear that D (S) = R.<br />

1


(i) Since x ≤ x, (x, x) ∈ S. That is, S is reflexive.<br />

(ii) If (x, y) ∈ S, i.e., x ≤ y, then y ≤ x. So, (y, x) ∈ S. That is, S is<br />

symmetric.<br />

(iii) If (x, y) ∈ S and (y, z) ∈ S, i.e., x ≤ y and y ≤ z, then x ≤ z. So,<br />

(x, z) ∈ S. That is, S is transitive.<br />

(b) x < y<br />

Proof: Write S = {(x, y) : x < y} , then we check that (i) reflexive, (ii)<br />

symmetric, and (iii) transitive as follows. It is clear that D (S) = R.<br />

(i) It is clear that for any real x, we cannot have x < x. So, S is not<br />

reflexive.<br />

(ii) It is clear that for any real x, and y, we cannot have x < y and y < x<br />

at the same time. So, S is not symmetric.<br />

(iii) If (x, y) ∈ S and (y, z) ∈ S, then x < y and y < z. So, x < z wich<br />

implies (x, z) ∈ S. That is, S is transitive.<br />

(c) x < |y|<br />

Proof: Write S = {(x, y) : x < |y|} , then we check that (i) reflexive, (ii)<br />

symmetric, and (iii) transitive as follows. It is clear that D (S) = R.<br />

(i) Since it is impossible for 0 < |0| , S is not reflexive.<br />

(ii) Since (−1, 2) ∈ S but (2, −1) /∈ S, S is not symmetric.<br />

(iii) Since (0, −1) ∈ S and (−1, 0) ∈ S, but (0, 0) /∈ S, S is not transitive.<br />

(d) x 2 + y 2 = 1<br />

Proof: Write S = {(x, y) : x 2 + y 2 = 1} , then we check that (i) reflexive,<br />

(ii) symmetric, and (iii) transitive as follows. It is clear that D (S) = [−1, 1] ,<br />

an closed interval with endpoints, −1 and 1.<br />

(i) Since 1 ∈ D (S) , and it is impossible for (1, 1) ∈ S by 1 2 + 1 2 ≠ 1, S<br />

is not reflexive.<br />

(ii) If (x, y) ∈ S, then x 2 + y 2 = 1. So, (y, x) ∈ S. That is, S is symmetric.<br />

(iii) Since (1, 0) ∈ S and (0, 1) ∈ S, but (1, 1) /∈ S, S is not transitive.<br />

(e) x 2 + y 2 < 0<br />

Proof: Write S = {(x, y) : x 2 + y 2 < 1} = φ, then S automatically satisfies<br />

(i) reflexive, (ii) symmetric, and (iii) transitive.<br />

(f) x 2 + x = y 2 + y<br />

Proof: Write S = {(x, y) : x 2 + x = y 2 + y} = {(x, y) : (x − y) (x + y − 1) = 0} ,<br />

then we check that (i) reflexive, (ii) symmetric, and (iii) transitive as follows.<br />

It is clear that D (S) = R.<br />

2


(i) If x ∈ R, it is clear that (x, x) ∈ S. So, S is reflexive.<br />

(ii) If (x, y) ∈ S, it is clear that (y, x) ∈ S. So, S is symmetric.<br />

(iii) If (x, y) ∈ S and (y, z) ∈ S, it is clear that (x, z) ∈ S. So, S is<br />

transitive.<br />

2.3 <strong>The</strong> following functions F and G are defined for all real x by the<br />

equations given. In each case where the composite function G ◦ F can be<br />

formed, give the domain of G◦F and a formula (or formulas) for (G ◦ F ) (x) .<br />

(a) F (x) = 1 − x, G (x) = x 2 + 2x<br />

Proof: Write<br />

G ◦ F (x) = G [F (x)] = G [1 − x] = (1 − x) 2 + 2 (1 − x) = x 2 − 4x + 3.<br />

It is clear that the domain of G ◦ F (x) is R.<br />

(b) F (x) = x + 5, G (x) = |x| /x if x ≠ 0, G (0) = 0.<br />

Proof: Write<br />

G ◦ F (x) = G [F (x)] =<br />

{<br />

G (x + 5) =<br />

|x+5|<br />

if x ≠ −5.<br />

x+5<br />

0 if x = −5.<br />

It is clear that the domain of G ◦ F (x) is R.<br />

{ { 2x, if 0 ≤ x ≤ 1<br />

x<br />

(c) F (x) =<br />

G (x) =<br />

2 , if 0 ≤ x ≤ 1<br />

1, otherwise,<br />

0, otherwise.<br />

Proof: Write<br />

⎧<br />

⎨ 4x 2 if x ∈ [0, 1/2]<br />

G ◦ F (x) = G [F (x)] = 0 if x ∈ (1/2, 1]<br />

⎩<br />

1 if x ∈ R − [0, 1]<br />

It is clear that the domain of G ◦ F (x) is R.<br />

Find F (x) if G (x) and G [F (x)] are given as follows:<br />

(d) G (x) = x 3 , G [F (x)] = x 3 − 3x 2 + 3x − 1.<br />

Proof: With help of (x − 1) 3 = x 3 − 3x 2 + 3x − 1, it is easy to know<br />

that F (x) = 1 − x. In addition, there is not other function H (x) such that<br />

G [H (x)] = x 3 − 3x 2 + 3x − 1 since G (x) = x 3 is 1-1.<br />

(e) G (x) = 3 + x + x 2 , G [F (x)] = x 2 − 3x + 5.<br />

3<br />

.


Proof: Write G (x) = ( x + 1 2) 2<br />

+<br />

11<br />

4 , then<br />

G [F (x)] =<br />

(<br />

F (x) + 1 ) 2<br />

+ 11<br />

2 4 = x2 − 3x + 5<br />

which implies that<br />

which implies that<br />

(2F (x) + 1) 2 = (2x − 3) 2<br />

F (x) = x − 2 or − x + 1.<br />

2.4 Given three functions F, G, H, what restrictions must be placed on<br />

their domains so that the following four composite functions can be defined<br />

G ◦ F, H ◦ G, H ◦ (G ◦ F ) , (H ◦ G) ◦ F.<br />

Proof: It is clear for answers,<br />

R (F ) ⊆ D (G) and R (G) ⊆ D (H) .<br />

Assuming that H ◦ (G ◦ F ) and (H ◦ G) ◦ F can be defined, prove that<br />

associative law:<br />

H ◦ (G ◦ F ) = (H ◦ G) ◦ F.<br />

Proof: Given any x ∈ D (F ) , then<br />

((H ◦ G) ◦ F ) (x) = (H ◦ G) (F (x))<br />

= H (G (F (x)))<br />

= H ((G ◦ F ) (x))<br />

= (H ◦ (G ◦ F )) (x) .<br />

So, H ◦ (G ◦ F ) = (H ◦ G) ◦ F.<br />

2.5 Prove the following set-theoretic identities for union and intersection:<br />

4


(a) A ∪ (B ∪ C) = (A ∪ B) ∪ C, A ∩ (B ∩ C) = (A ∩ B) ∩ C.<br />

Proof: For the part, A∪(B ∪ C) = (A ∪ B)∪C : Given x ∈ A∪(B ∪ C) ,<br />

we have x ∈ A or x ∈ B ∪ C. That is, x ∈ A or x ∈ B or x ∈ C. Hence,<br />

x ∈ A∪B or x ∈ C. It implies x ∈ (A ∪ B)∪C. Similarly, if x ∈ (A ∪ B)∪C,<br />

then x ∈ A ∪ (B ∪ C) . <strong>The</strong>refore, A ∪ (B ∪ C) = (A ∪ B) ∪ C.<br />

For the part, A∩(B ∩ C) = (A ∩ B)∩C : Given x ∈ A∩(B ∩ C) , we have<br />

x ∈ A and x ∈ B ∩C. That is, x ∈ A and x ∈ B and x ∈ C. Hence, x ∈ A∩B<br />

and x ∈ C. It implies x ∈ (A ∩ B) ∩ C. Similarly, if x ∈ (A ∩ B) ∩ C, then<br />

x ∈ A ∩ (B ∩ C) . <strong>The</strong>refore, A ∩ (B ∩ C) = (A ∩ B) ∩ C.<br />

(b) A ∩ (B ∪ C) = (A ∩ B) ∪ (A ∩ C) .<br />

Proof: Given x ∈ A ∩ (B ∪ C) , then x ∈ A and x ∈ B ∪ C. We consider<br />

two cases as follows.<br />

If x ∈ B, then x ∈ A ∩ B. So, x ∈ (A ∩ B) ∪ (A ∩ C) .<br />

If x ∈ C, then x ∈ A ∩ C. So, x ∈ (A ∩ B) ∪ (A ∩ C) .<br />

So, we have shown that<br />

A ∩ (B ∪ C) ⊆ (A ∩ B) ∪ (A ∩ C) . (*)<br />

Conversely, given x ∈ (A ∩ B) ∪ (A ∩ C) , then x ∈ A ∩ B or x ∈ A ∩ C.<br />

We consider two cases as follows.<br />

If x ∈ A ∩ B, then x ∈ A ∩ (B ∪ C) .<br />

If x ∈ A ∩ C, then x ∈ A ∩ (B ∪ C) .<br />

So, we have shown that<br />

(A ∩ B) ∪ (A ∩ C) ⊆ A ∩ (B ∪ C) . (**)<br />

By (*) and (**), we have proved it.<br />

(c) (A ∪ B) ∩ (A ∪ C) = A ∪ (B ∩ C)<br />

Proof: Given x ∈ (A ∪ B) ∩ (A ∪ C) , then x ∈ A ∪ B and x ∈ A ∪ C.<br />

We consider two cases as follows.<br />

If x ∈ A, then x ∈ A ∪ (B ∩ C) .<br />

If x /∈ A, then x ∈ B and x ∈ C. So, x ∈ B ∩ C. It implies that<br />

x ∈ A ∪ (B ∩ C) .<br />

<strong>The</strong>refore, we have shown that<br />

(A ∪ B) ∩ (A ∪ C) ⊆ A ∪ (B ∩ C) . (*)<br />

5


Conversely, if x ∈ A ∪ (B ∩ C) , then x ∈ A or x ∈ B ∩ C. We consider<br />

two cases as follows.<br />

If x ∈ A, then x ∈ (A ∪ B) ∩ (A ∪ C) .<br />

If x ∈ B ∩ C, then x ∈ A ∪ B and x ∈ A ∪ C. So, x ∈ (A ∪ B) ∩ (A ∪ C) .<br />

<strong>The</strong>refore, we have shown that<br />

A ∪ (B ∩ C) ⊆ (A ∪ B) ∩ (A ∪ C) . (*)<br />

By (*) and (**), we have proved it.<br />

(d) (A ∪ B) ∩ (B ∪ C) ∩ (C ∪ A) = (A ∩ B) ∪ (A ∩ C) ∪ (B ∩ C)<br />

Proof: Given x ∈ (A ∪ B) ∩ (B ∪ C) ∩ (C ∪ A) , then<br />

x ∈ A ∪ B and x ∈ B ∪ C and x ∈ C ∪ A. (*)<br />

We consider the cases to show x ∈ (A ∩ B) ∪ (A ∩ C) ∪ (B ∩ C) as follows.<br />

For the case (x ∈ A):<br />

If x ∈ B, then x ∈ A ∩ B.<br />

If x /∈ B, then by (*), x ∈ C. So, x ∈ A ∩ C.<br />

Hence, in this case, we have proved that x ∈ (A ∩ B)∪(A ∩ C)∪(B ∩ C) .<br />

For the case (x /∈ A):<br />

If x ∈ B, then by (*), x ∈ C. So, x ∈ B ∩ C.<br />

If x /∈ B, then by (*), it is impossible.<br />

Hence, in this case, we have proved that x ∈ (A ∩ B)∪(A ∩ C)∪(B ∩ C) .<br />

From above,<br />

(A ∪ B) ∩ (B ∪ C) ∩ (C ∪ A) ⊆ (A ∩ B) ∪ (A ∩ C) ∪ (B ∩ C)<br />

Similarly, we also have<br />

(A ∩ B) ∪ (A ∩ C) ∪ (B ∩ C) ⊆ (A ∪ B) ∩ (B ∪ C) ∩ (C ∪ A) .<br />

So, we have proved it.<br />

Remark: <strong>The</strong>re is another proof, we write it as a reference.<br />

Proof: Consider<br />

(A ∪ B) ∩ (B ∪ C) ∩ (C ∪ A)<br />

= [(A ∪ B) ∩ (B ∪ C)] ∩ (C ∪ A)<br />

= [B ∪ (A ∩ C)] ∩ (C ∪ A)<br />

= [B ∩ (C ∪ A)] ∪ [(A ∩ C) ∩ (C ∪ A)]<br />

= [(B ∩ C) ∪ (B ∩ A)] ∪ (A ∩ C)<br />

= (A ∩ B) ∪ (A ∩ C) ∪ (B ∩ C) .<br />

6


(e) A ∩ (B − C) = (A ∩ B) − (A ∩ C)<br />

Proof: Given x ∈ A ∩ (B − C) , then x ∈ A and x ∈ B − C. So, x ∈ A<br />

and x ∈ B and x /∈ C. So, x ∈ A ∩ B and x /∈ C. Hence,<br />

x ∈ (A ∩ B) − C ⊆ (A ∩ B) − (A ∩ C) . (*)<br />

Conversely, given x ∈ (A ∩ B) − (A ∩ C) , then x ∈ A ∩ B and x /∈ A ∩ C.<br />

So, x ∈ A and x ∈ B and x /∈ C. So, x ∈ A and x ∈ B − C. Hence,<br />

By (*) and (**), we have proved it.<br />

(f) (A − C) ∩ (B − C) = (A ∩ B) − C<br />

x ∈ A ∩ (B − C) (**)<br />

Proof: Given x ∈ (A − C) ∩ (B − C) , then x ∈ A − C and x ∈ B − C.<br />

So, x ∈ A and x ∈ B and x /∈ C. So, x ∈ (A ∩ B) − C. Hence,<br />

(A − C) ∩ (B − C) ⊆ (A ∩ B) − C. (*)<br />

Conversely, given x ∈ (A ∩ B)−C, then x ∈ A and x ∈ B and x /∈ C. Hence,<br />

x ∈ A − C and x ∈ B − C. Hence,<br />

By (*) and (**), we have proved it.<br />

(A ∩ B) − C ⊆ (A − C) ∩ (B − C) . (**)<br />

(g) (A − B) ∪ B = A if, and only if, B ⊆ A<br />

Proof: (⇒) Suppose that (A − B) ∪ B = A, then it is clear that B ⊆ A.<br />

(⇐) Suppose that B ⊆ A, then given x ∈ A, we consider two cases.<br />

If x ∈ B, then x ∈ (A − B) ∪ B.<br />

If x /∈ B, then x ∈ A − B. Hence, x ∈ (A − B) ∪ B.<br />

From above, we have<br />

A ⊆ (A − B) ∪ B.<br />

In addition, it is obviously (A − B) ∪ B ⊆ A since A − B ⊆ A and B ⊆ A.<br />

2.6 Let f : S → T be a function. If A and B are arbitrary subsets of S,<br />

prove that<br />

f (A ∪ B) = f (A) ∪ f (B) and f (A ∩ B) ⊆ f (A) ∩ f (B) .<br />

7


Generalize to arbitrary unions and intersections.<br />

Proof: First, we prove f (A ∪ B) = f (A) ∪ f (B) as follows. Let y ∈<br />

f (A ∪ B) , then y = f (a) or y = f (b) , where a ∈ A and b ∈ B. Hence,<br />

y ∈ f (A) ∪ f (B) . That is,<br />

f (A ∪ B) ⊆ f (A) ∪ f (B) .<br />

Conversely, if y ∈ f (A) ∪ f (B) , then y = f (a) or y = f (b) , where a ∈ A<br />

and b ∈ B. Hence, y ∈ f (A ∪ B) . That is,<br />

f (A) ∪ f (B) ⊆ f (A ∪ B) .<br />

So, we have proved that f (A ∪ B) = f (A) ∪ f (B) .<br />

For the part f (A ∩ B) ⊆ f (A) ∩ f (B) : Let y ∈ f (A ∩ B) , then y =<br />

f (x) , where x ∈ A∩B. Hence, y ∈ f (A) and y ∈ f (B) . That is, f (A ∩ B) ⊆<br />

f (A) ∩ f (B) .<br />

For arbitrary unions and intersections, we have the following facts, and<br />

the proof is easy from above. So, we omit the detail.<br />

<strong>And</strong><br />

f (∪ i∈I A i ) = ∪ i∈I f (A i ) , where I is an index set.<br />

f (∩ i∈I A i ) ⊆ ∩ i∈I f (A i ) , where I is an index set.<br />

Remark: We should note why the equality does NOT hold for the case<br />

of intersection. for example, consider A = {1, 2} and B = {1, 3} , where<br />

f (1) = 1 and f (2) = 2 and f (3) = 2.<br />

f (A ∩ B) = f ({1}) = {1} ⊆ {1, 2} ⊆ f ({1, 2}) ∩ f ({1, 3}) = f (A) ∩ f (B) .<br />

2.7 Let f : S → T be a function. If Y ⊆ T, we denote by f −1 (Y ) the<br />

largest subset of S which f maps into Y. That is,<br />

f −1 (Y ) = {x : x ∈ S and f (x) ∈ Y } .<br />

<strong>The</strong> set f −1 (Y ) is called the inverse image of Y under f. Prove that the<br />

following for arbitrary subsets X of S and Y of T.<br />

(a) X ⊆ f −1 [f (X)]<br />

8


Proof: Given x ∈ X, then f (x) ∈ f (X) . Hence, x ∈ f −1 [f (X)] by<br />

definition of the inverse image of f (X) under f. So, X ⊆ f −1 [f (X)] .<br />

Remark: <strong>The</strong> equality may not hold, for example, let f (x) = x 2 on R,<br />

and let X = [0, ∞), we have<br />

f −1 [f (X)] = f −1 [[0, ∞)] = R.<br />

(b) f (f −1 (Y )) ⊆ Y<br />

Proof: Given y ∈ f (f −1 (Y )) , then there exists a point x ∈ f −1 (Y )<br />

such that f (x) = y. Since x ∈ f −1 (Y ) , we know that f (x) ∈ Y. Hence,<br />

y ∈ Y. So, f (f −1 (Y )) ⊆ Y<br />

Remark: <strong>The</strong> equality may not hold, for example, let f (x) = x 2 on R,<br />

and let Y = R, we have<br />

f ( f −1 (Y ) ) = f (R) = [0, ∞) ⊆ R.<br />

(c) f −1 [Y 1 ∪ Y 2 ] = f −1 (Y 1 ) ∪ f −1 (Y 2 )<br />

Proof: Given x ∈ f −1 [Y 1 ∪ Y 2 ] , then f (x) ∈ Y 1 ∪ Y 2 . We consider two<br />

cases as follows.<br />

If f (x) ∈ Y 1 , then x ∈ f −1 (Y 1 ) . So, x ∈ f −1 (Y 1 ) ∪ f −1 (Y 2 ) .<br />

If f (x) /∈ Y 1 , i.e., f (x) ∈ Y 2 , then x ∈ f −1 (Y 2 ) . So, x ∈ f −1 (Y 1 ) ∪<br />

f −1 (Y 2 ) .<br />

From above, we have proved that<br />

f −1 [Y 1 ∪ Y 2 ] ⊆ f −1 (Y 1 ) ∪ f −1 (Y 2 ) . (*)<br />

Conversely, since f −1 (Y 1 ) ⊆ f −1 [Y 1 ∪ Y 2 ] and f −1 (Y 2 ) ⊆ f −1 [Y 1 ∪ Y 2 ] ,<br />

we have<br />

f −1 (Y 1 ) ∪ f −1 (Y 2 ) ⊆ f −1 [Y 1 ∪ Y 2 ] . (**)<br />

From (*) and (**), we have proved it.<br />

(d) f −1 [Y 1 ∩ Y 2 ] = f −1 (Y 1 ) ∩ f −1 (Y 2 )<br />

Proof: Given x ∈ f −1 (Y 1 ) ∩ f −1 (Y 2 ) , then f (x) ∈ Y 1 and f (x) ∈ Y 2 .<br />

So, f (x) ∈ Y 1 ∩ Y 2 . Hence, x ∈ f −1 [Y 1 ∩ Y 2 ] . That is, we have proved that<br />

f −1 (Y 1 ) ∩ f −1 (Y 2 ) ⊆ f −1 [Y 1 ∩ Y 2 ] . (*)<br />

9


Conversely, since f −1 [Y 1 ∩ Y 2 ] ⊆ f −1 (Y 1 ) and f −1 [Y 1 ∩ Y 2 ] ⊆ f −1 (Y 2 ) ,<br />

we have<br />

f −1 [Y 1 ∩ Y 2 ] ⊆ f −1 (Y 1 ) ∩ f −1 (Y 2 ) . (**)<br />

From (*) and (**), we have proved it.<br />

(e) f −1 (T − Y ) = S − f −1 (Y )<br />

Proof: Given x ∈ f −1 (T − Y ) , then f (x) ∈ T − Y. So, f (x) /∈ Y. We<br />

want to show that x ∈ S − f −1 (Y ) . Suppose NOT, then x ∈ f −1 (Y ) which<br />

implies that f (x) ∈ Y. That is impossible. Hence, x ∈ S − f −1 (Y ) . So, we<br />

have<br />

f −1 (T − Y ) ⊆ S − f −1 (Y ) . (*)<br />

Conversely, given x ∈ S −f −1 (Y ) , then x /∈ f −1 (Y ) . So, f (x) /∈ Y. That<br />

is, f (x) ∈ T − Y. Hence, x ∈ f −1 (T − Y ) . So, we have<br />

From (*) and (**), we have proved it.<br />

S − f −1 (Y ) ⊆ f −1 (T − Y ) . (**)<br />

(f) Generalize (c) and (d) to arbitrary unions and intersections.<br />

Proof: We give the statement without proof since it is the same as (c)<br />

and (d). In general, we have<br />

and<br />

f −1 (∪ i∈I A i ) = ∪ i∈I f −1 (A i ) .<br />

f −1 (∩ i∈I A i ) = ∩ i∈I f −1 (A i ) .<br />

Remark: From above sayings and Exercise 2.6, we found that the<br />

inverse image f −1 and the operations of sets, such as intersection and union,<br />

can be exchanged. However, for a function, we only have the exchange of<br />

f and the operation of union. <strong>The</strong> reader also see the Exercise 2.9 to get<br />

more.<br />

2.8 Refer to Exercise 2.7. Prove that f [f −1 (Y )] = Y for every subset Y<br />

of T if, and only if, T = f (S) .<br />

Proof: (⇒) It is clear that f (S) ⊆ T. In order to show the equality, it<br />

suffices to show that T ⊆ f (S) . Consider f −1 (T ) ⊆ S, then we have<br />

f ( f −1 (T ) ) ⊆ f (S) .<br />

10


By hyppothesis, we get T ⊆ f (S) .<br />

(⇐) Suppose NOT, i.e., f [f −1 (Y )] is a proper subset of Y for some<br />

Y ⊆ T by Exercise 2.7 (b). Hence, there is a y ∈ Y such that y /∈<br />

f [f −1 (Y )] . Since Y ⊆ f (S) = T, f (x) = y for some x ∈ S. It implies that<br />

x ∈ f −1 (Y ) . So, f (x) ∈ f [f −1 (Y )] which is impossible by the choice of y.<br />

Hence, f [f −1 (Y )] = Y for every subset Y of T.<br />

2.9 Let f : S → T be a function. Prove that the following statements<br />

are equivalent.<br />

(a) f is one-to-one on S.<br />

(b) f (A ∩ B) = f (A) ∩ f (B) for all subsets A, B of S.<br />

(c) f −1 [f (A)] = A for every subset A of S.<br />

(d) For all disjoint subsets A and B of S, the image f (A) and f (B) are<br />

disjoint.<br />

(e) For all subsets A and B of S with B ⊆ A, we have<br />

f (A − B) = f (A) − f (B) .<br />

Proof: (a) ⇒ (b) : Suppose that f is 1-1 on S. By Exercise 2.6, we<br />

have proved that f (A ∩ B) ⊆ f (A) ∩ f (B) for all A, B of S. In order to<br />

show the equality, it suffices to show that f (A) ∩ f (B) ⊆ f (A ∩ B) .<br />

Given y ∈ f (A) ∩ f (B) , then y = f (a) and y = f (b) where a ∈ A<br />

and b ∈ B. Since f is 1-1, we have a = b. That is, y ∈ f (A ∩ B) . So,<br />

f (A) ∩ f (B) ⊆ f (A ∩ B) .<br />

(b) ⇒ (c) : Suppose that f (A ∩ B) = f (A) ∩ f (B) for all subsets A, B<br />

of S. If A ≠ f −1 [f (A)] for some A of S, then by Exercise 2.7 (a), there is<br />

an element a /∈ A and a ∈ f −1 [f (A)] . Consider<br />

φ = f (A ∩ {a}) = f (A) ∩ f ({a}) by (b) (*)<br />

Since a ∈ f −1 [f (A)] , we have f (a) ∈ f (A) which contradicts to (*). Hence,<br />

no such a exists. That is, f −1 [f (A)] = A for every subset A of S.<br />

(c) ⇒ (d) : Suppose that f −1 [f (A)] = A for every subset A of S. If<br />

A ∩ B = φ, then Consider<br />

φ = A ∩ B<br />

= f −1 [f (A)] ∩ f −1 [f (B)]<br />

= f −1 (f (A) ∩ f (B)) by Exercise 2.7 (d)<br />

11


which implies that f (A) ∩ f (B) = φ.<br />

(d) ⇒ (e) : Suppose that for all disjoint subsets A and B of S, the image<br />

f (A) and f (B) are disjoint. If B ⊆ A, then since (A − B) ∩ B = φ, we have<br />

which implies that<br />

f (A − B) ∩ f (B) = φ<br />

f (A − B) ⊆ f (A) − f (B) . (**)<br />

Conversely, we consider if y ∈ f (A) − f (B) , then y = f (x) , where x ∈ A<br />

and x /∈ B. It implies that x ∈ A − B. So, y = f (x) ∈ f (A − B) . That is,<br />

By (**) and (***), we have proved it.<br />

f (A) − f (B) ⊆ f (A − B) . (***)<br />

(d) ⇒ (a) : Suppose that f (A − B) = f (A) − f (B) for all subsets A and<br />

B of S with B ⊆ A. If f (a) = f (b) , consider A = {a, b} and B = {b} , we<br />

have<br />

f (A − B) = φ<br />

which implies that A = B. That is, a = b. So, f is 1-1.<br />

2.10 Prove that if A˜B and B˜C, then A˜C.<br />

Proof: Since A˜B and B˜C, then there exists bijection f and g such<br />

that<br />

f : A → B and g : B → C.<br />

So, if we consider g ◦ f : A → C, then A˜C since g ◦ f is bijection.<br />

2.11 If {1, 2, ..., n} ˜ {1, 2, ..., m} , prove that m = n.<br />

Proof: Since {1, 2, ..., n} ˜ {1, 2, ..., m} , there exists a bijection function<br />

f : {1, 2, ..., n} → {1, 2, ..., m} .<br />

Since f is 1-1, then n ≤ m. Conversely, consider f −1 is 1-1 to get m ≤ n. So,<br />

m = n.<br />

2.12 If S is an infinite set, prove that S contains a countably infinite<br />

subset. Hint. Choose an element a 1 in S and consider S − {a 1 } .<br />

12


Proof: Since S is an infinite set, then choose a 1 in S and thus S − {a 1 }<br />

is still infinite. From this, we have S − {a 1 , .., a n } is infinite. So, we finally<br />

have<br />

{a 1 , ..., a n , ...} (⊆ S)<br />

which is countably infinite subset.<br />

2.13 Prove that every infinite set S contains a proper subset similar to S.<br />

Proof: By Exercise 2.12, we write S = ˜S ∪ {x 1 , ..., x n , ...} , where ˜S ∩<br />

{x 1 , ..., x n , ...} = φ and try to show<br />

as follows. Define<br />

by<br />

˜S ∪ {x 2 , ..., x n , ...} ˜S<br />

f : ˜S ∪ {x 2 , ..., x n , ...} → S = ˜S ∪ {x 1 , ..., x n , ...}<br />

f (x) =<br />

{<br />

x if x ∈ ˜S<br />

x i if x = x i+1<br />

.<br />

<strong>The</strong>n it is clear that f is 1-1 and onto. So, we have proved that every infinite<br />

set S contains a proper subset similar to S.<br />

Remark: In the proof, we may choose the map<br />

f : ˜S ∪ {x N+1 , ..., x n , ...} → S = ˜S ∪ {x 1 , ..., x n , ...}<br />

by<br />

f (x) =<br />

{<br />

x if x ∈ ˜S<br />

x i if x = x i+N<br />

.<br />

2.14 If A is a countable set and B an uncountable set, prove that B − A<br />

is similar to B.<br />

Proof: In order to show it, we consider some cases as follows. (i) B∩A =<br />

φ (ii) B ∩ A is a finite set, and (iii) B ∩ A is an infinite set.<br />

For case (i), B − A = B. So, B − A is similar to B.<br />

For case (ii), it follows from the Remark in Exercise 2.13.<br />

For case (iii), note that B ∩ A is countable, and let C = B − A, we have<br />

B = C ∪ (B ∩ A) . We want to show that<br />

(B − A) ˜B ⇔ C˜C ∪ (B ∩ A) .<br />

13


By Exercise 2.12, we write C = ˜C ∪ D, where D is countably infinite and<br />

˜C ∩ D = φ. Hence,<br />

( )<br />

˜C ∪ D<br />

C˜C ∪ (B ∩ A) ⇔<br />

⇔<br />

(<br />

˜C ∪ D<br />

)<br />

˜<br />

[ ]<br />

˜ ˜C ∪ (D ∪ (B ∩ A))<br />

(<br />

˜C ∪ D<br />

′)<br />

( ) ( )<br />

where D ′ = D∪(B ∩ A) which is countably infinite. Since ˜C ∪ D ˜ ˜C ∪ D<br />

′<br />

is clear, we have proved it.<br />

2.15 A real number is called algebraic if it is a root of an algebraic<br />

equation f (x) = 0, where f (x) = a 0 + a 1 x + ... + a n x n is a polynomial<br />

with integer coefficients. Prove that the set of all polynomials with integer<br />

coefficients is countable and deduce that the set of algebraic numbers is also<br />

countable.<br />

Proof: Given a positive integer N (≥ 2) , there are only finitely many<br />

eqautions with<br />

n∑<br />

n + |a k | = N, where a k ∈ Z. (*)<br />

k=1<br />

Let S N = {f : f (x) = a 0 + a 1 x + ... + a n x n satisfies (*)} , then S N is a finite<br />

set. Hence, ∪ ∞ n=2S n is countable. Clearly, the set of all polynomials with<br />

integer coefficients is a subset of ∪ ∞ n=2S n . So, the set of all polynomials with<br />

integer coefficients is countable. In addition, a polynomial of degree k has at<br />

most k roots. Hence, the set of algebraic numbers is also countable.<br />

2.16 Let S be a finite set consisting of n elements and let T be the<br />

collection of all subsets of S. Show that T is a finite set and find the number<br />

of elements in T.<br />

Proof: Write S = {x 1 , ..., x n } , then T =the collection of all subsets of<br />

S. Clearly, T is a finite set with 2 n elements.<br />

2.17 Let R denote the set of real numbers and let S denote the set<br />

of all real-valued functions whose domain in R. Show that S and R are not<br />

equinumrous. Hint. Assume S˜R and let f be a one-to-one function such<br />

that f (R) = S. If a ∈ R, let g a = f (a) be the real-valued function in S which<br />

correspouds to real number a. Now define h by the equation h (x) = 1+g x (x)<br />

if x ∈ R, and show that h /∈ S.<br />

14


Proof: Assume S˜R and let f be a one-to-one function such that f (R) =<br />

S. If a ∈ R, let g a = f (a) be the real-valued function in S which correspouds<br />

to real number a. Define h by the equation h (x) = 1 + g x (x) if x ∈ R, then<br />

which implies that<br />

h = f (b) = g b<br />

h (b) := 1 + g b (b) = g b (b)<br />

which is impossible. So, S and R are not equinumrous.<br />

Remark: <strong>The</strong>re is a similar exercise, we write it as a reference. <strong>The</strong><br />

cardinal number of C [a, b] is 2 ℵ 0<br />

, where ℵ 0 = # (N) .<br />

Proof: First, # (R) = 2 ℵ 0<br />

≤ # (C [a, b]) by considering constant function.<br />

Second, we consider the map<br />

by<br />

f : C [a, b] → P (Q × Q) , the power set of Q × Q<br />

f (ϕ) = {(x, y) ∈ Q × Q : x ∈ [a, b] and y ≤ ϕ (x)} .<br />

Clearly, f is 1-1. It implies that # (C [a, b]) ≤ # (P (Q × Q)) = 2 ℵ 0<br />

.<br />

So, we have proved that # (C [a, b]) = 2 ℵ 0<br />

.<br />

Note: For notations, the reader can see the textbook, in Chapter 4, pp<br />

102. Also, see the book, Set <strong>The</strong>ory and Related Topics by Seymour<br />

Lipschutz, Chapter 9, pp 157-174. (Chinese Version)<br />

2.18 Let S be the collection of all sequences whose terms are the integers<br />

0 and 1. Show that S is uncountable.<br />

Proof: Let E be a countable subet of S, and let E consists of the sequences<br />

s 1 , .., s n , .... We construct a sequence s as follows. If the nth digit<br />

in s n is 1, we let the nth digit of s be 0, and vice versa. <strong>The</strong>n the sequence<br />

s differes from every member of E in at least one place; hence s /∈ E. But<br />

clearly s ∈ S, so that E is a proper subset of S.<br />

We have shown that every countable subset of S is a proper subset of S.<br />

It follows that S is uncountable (for otherwise S would be a proper subset<br />

of S, which is absurb).<br />

Remark: In this exercise, we have proved that R, the set of real numbers,<br />

is uncountable. It can be regarded as the Exercise 1.22 for k = 2. (Binary<br />

System).<br />

15


2.19 Show that the following sets are countable:<br />

(a) the set of circles in the complex plane having the ratiional radii and<br />

centers with rational coordinates.<br />

Proof: Write the set of circles in the complex plane having the ratiional<br />

radii and centers with rational coordinates, {C (x n ; q n ) : x n ∈ Q × Q and q n ∈ Q} :=<br />

S. Clearly, S is countable.<br />

(b) any collection of disjoint intervals of positive length.<br />

Proof: Write the collection of disjoint intervals of positive length, {I : I is an interval of positive<br />

S. Use the reason in Exercise 2.21, we have proved that S is countable.<br />

2.20 Let f be a real-valued function defined for every x in the interval<br />

0 ≤ x ≤ 1. Suppose there is a positive number M having the following<br />

property: for every choice of a finite number of points x 1 , x 2 , ..., x n in the<br />

interval 0 ≤ x ≤ 1, the sum<br />

|f (x 1 ) + ... + f (x n )| ≤ M.<br />

Let S be the set of those x in 0 ≤ x ≤ 1 for which f (x) ≠ 0. Prove that S<br />

is countable.<br />

Proof: Let S n = {x ∈ [0, 1] : |f (x)| ≥ 1/n} , then S n is a finite set by<br />

hypothesis. In addition, S = ∪ ∞ n=1S n . So, S is countable.<br />

2.21 Find the fallacy in the following ”proof” that the set of all intervals<br />

of positive length is countable.<br />

Let {x 1 , x 2 , ...} denote the countable set of rational numbers and let I<br />

be any interval of positive length. <strong>The</strong>n I contains infinitely many rational<br />

points x n , but among these there will be one with smallest index n. Define<br />

a function F by means of the eqaution F (I) = n if x n is the rational number<br />

with smallest index in the interval I. This function establishes a one-to-one<br />

correspondence between the set of all intervals and a subset of the positive<br />

integers. Hence, the set of all intervals is countable.<br />

Proof: Note that F is not a one-to-one correspondence between the set<br />

of all intervals and a subset of the positive integers. So, this is not a proof.<br />

In fact, the set of all intervals of positive length is uncountable.<br />

Remark: Compare with Exercise 2.19, and the set of all intervals of<br />

positive length is uncountable is clear by considering {(0, x) : 0 < x < 1} .<br />

16


2.22 Let S denote the collection of all subsets of a given set T. Let f : S →<br />

R be a real-valued function defined on S. <strong>The</strong> function f is called additive<br />

if f (A ∪ B) = f (A) + f (B) whenever A and B are disjoint subsets of T. If<br />

f is additive, prove that for any two subsets A and B we have<br />

f (A ∪ B) = f (A) + f (B − A)<br />

and<br />

f (A ∪ B) = f (A) + f (B) − f (A ∩ B) .<br />

Proof: Since A ∩ (B − A) = φ and A ∪ B = A ∪ (B − A) , we have<br />

f (A ∪ B) = f (A ∪ (B − A)) = f (A) + f (B − A) . (*)<br />

In addition, since(B − A) ∩ (A ∩ B) = φ and B = (B − A) ∪ (A ∩ B) , we<br />

have<br />

f (B) = f ((B − A) ∪ (A ∩ B)) = f (B − A) + f (A ∩ B)<br />

which implies that<br />

By (*) and (**), we have proved that<br />

f (B − A) = f (B) − f (A ∩ B) (**)<br />

f (A ∪ B) = f (A) + f (B) − f (A ∩ B) .<br />

2.23 Refer to Exercise 2.22. Assume f is additive and assume also that<br />

the following relations hold for two particular subsets A and B of T :<br />

and<br />

and<br />

f (A ∪ B) = f (A ′ ) + f (B ′ ) − f (A ′ ) f (B ′ )<br />

f (A ∩ B) = f (A) f (B)<br />

f (A) + f (B) ≠ f (T ) ,<br />

where A ′ = T − A, B ′ = T − B. Prove that these relations determine f (T ) ,<br />

and compute the value of f (T ) .<br />

17


Proof: Write<br />

f (T ) = f (A) + f (A ′ ) = f (B) + f (B ′ ) ,<br />

then<br />

[f (T )] 2 = [f (A) + f (A ′ )] [f (B) + f (B ′ )]<br />

= f (A) f (B) + f (A) f (B ′ ) + f (A ′ ) f (B) + f (A ′ ) f (B ′ )<br />

= f (A) f (B) + f (A) [f (T ) − f (B)] + [f (T ) − f (A)] f (B) + f (A ′ ) f (B ′ )<br />

= [f (A) + f (B)] f (T ) − f (A) f (B) + f (A ′ ) f (B ′ )<br />

= [f (A) + f (B)] f (T ) − f (A) f (B) + f (A ′ ) + f (B ′ ) − f (A ∪ B)<br />

= [f (A) + f (B)] f (T ) − f (A) f (B) + [f (T ) − f (A)] + [f (T ) − f (B)]<br />

− [f (A) + f (B) − f (A ∩ B)]<br />

= [f (A) + f (B) + 2] f (T ) − f (A) f (B) − 2 [f (A) + f (B)] + f (A ∩ B)<br />

= [f (A) + f (B) + 2] f (T ) − 2 [f (A) + f (B)]<br />

which implies that<br />

[f (T )] 2 − [f (A) + f (B) + 2] f (T ) + 2 [f (A) + f (B)] = 0<br />

which implies that<br />

x 2 − (a + 2) x + 2a = 0 ⇒ (x − a) (x − 2) = 0<br />

where a = f (A) + f (B) . So, x = 2 since x ≠ a by f (A) + f (B) ≠ f (T ) .<br />

18


Charpter 3 Elements of Point set Topology<br />

Open and closed sets in R 1 and R 2<br />

3.1 Prove that an open interval in R 1 is an open set and that a closed interval is a<br />

closed set.<br />

proof: 1. Let a, b be an open interval in R 1 , and let x a, b. Consider<br />

minx a, b x : L. <strong>The</strong>n we have Bx, L x L, x L a, b. Thatis,x is an<br />

interior point of a, b. Sincex is arbitrary, we have every point of a, b is interior. So,<br />

a, b is open in R 1 .<br />

2. Let a, b be a closed interval in R 1 , and let x be an adherent point of a, b. Wewant<br />

to show x a, b. Ifx a, b, then we have x a or x b. Consider x a, then<br />

Bx, a 2<br />

x a, b 3x 2<br />

a , x 2<br />

a a, b <br />

which contradicts the definition of an adherent point. Similarly for x b.<br />

<strong>The</strong>refore, we have x a, b if x is an adherent point of a, b. Thatis,a, b contains<br />

its all adherent points. It implies that a, b is closed in R 1 .<br />

3.2 Determine all the accumulation points of the following sets in R 1 and decide<br />

whether the sets are open or closed (or neither).<br />

(a) All integers.<br />

Solution: Denote the set of all integers by Z. Letx Z, and consider<br />

Bx, x1 x S . So,Z has no accumulation points.<br />

2<br />

However, Bx, x1 S x . SoZ contains its all adherent points. It means that<br />

2<br />

Z is closed. Trivially, Z is not open since Bx, r is not contained in Z for all r 0.<br />

Remark: 1. Definition of an adherent point: Let S be a subset of R n ,andx a point in<br />

R n , x is not necessarily in S. <strong>The</strong>n x is said to be adherent to S if every n ball Bx<br />

contains at least one point of S. To be roughly, Bx S .<br />

2. Definition of an accumulation point: Let S be a subset of R n ,andx a point in R n ,<br />

then x is called an accumulation point of S if every n ball Bx contains at least one point<br />

of S distinct from x. To be roughly, Bx x S . Thatis,x is an accumulation<br />

point if, and only if, x adheres to S x. Note that in this sense,<br />

Bx x S Bx S x.<br />

3. Definition of an isolated point: If x S, but x is not an accumulation point of S, then<br />

x is called an isolated point.<br />

4. Another solution for Z is closed: Since R Z nZ n, n 1, we know that R Z<br />

is open. So, Z is closed.<br />

5. In logics, if there does not exist any accumulation point of a set S, then S is<br />

automatically a closed set.<br />

(b) <strong>The</strong> interval a, b.<br />

solution: In order to find all accumulation points of a, b, we consider 2 cases as<br />

follows.<br />

1. a, b :Letx a, b, then Bx, r x a, b for any r 0. So, every<br />

point of a, b is an accumulation point.<br />

2. R 1 a, b , a b, : For points in b, and , a, it is easy to know<br />

that these points cannot be accumulation points since x b, or x , a, there


exists an n ball Bx, r x such that Bx, r x x a, b . For the point a, it is easy<br />

to know that Ba, r a a, b . That is, in this case, there is only one<br />

accumulation point a of a, b.<br />

So, from 1 and 2, we know that the set of the accumulation points of a, b is a, b.<br />

Since a a, b, we know that a, b cannot contain its all accumulation points. So,<br />

a, b is not closed.<br />

Since an n ball Bb, r is not contained in a, b for any r 0, we know that the point<br />

b is not interior to a, b. So,a, b is not open.<br />

(c) All numbers of the form 1/n, (n 1,2,3,....<br />

Solution: Write the set 1/n : n 1,2,... 1, 1/2, 1/3, . . . , 1/n,... : S.<br />

Obviously, 0 is the only one accumulation point of S. So,S is not closed since S does not<br />

contain the accumulation point 0. Since 1 S, andB1, r is not contained in S for any<br />

r 0, S is not open.<br />

Remark: Every point of 1/n : n 1,2,3,... is isolated.<br />

(d) All rational numbers.<br />

Solutions: Denote all rational numbers by Q. It is trivially seen that the set of<br />

accumulation points is R 1 .<br />

So, Q is not closed. Consider x Q, anyn ball Bx is not contained in Q. Thatis,x<br />

is not an interior point of Q. In fact, every point of Q is not an interior point of Q. So,<br />

Q is not open.<br />

(e) All numbers of the form 2 n 5 m , (m, n 1,2,....<br />

Solution: Write the set<br />

2 n 5 m : m, n 1, 2, . . m m1 1 2 5m , 1 4 5m ,..., 1<br />

2<br />

n 5m ,... : S<br />

1 2 1 5 , 1 2 1 5 ,..., 1 2 2 5 1<br />

m ,... <br />

1 4 1 5 , 1 4 1 5 ,..., 1 2 4 5 1<br />

m ,... <br />

........................................................<br />

<br />

2 1 n 1 5 , 1<br />

2<br />

n 1 5 ,..., 1 2 2<br />

n 5 1<br />

m ... <br />

.....................................................<br />

So,wefindthatS 1 : n 1, 2, . . . 1 : m 1, 2, . . .<br />

2 n 5<br />

0. So,S is not<br />

m<br />

closed since it does not contain 0. Since 1 S, andB 1 , r is not contained in S for any<br />

2<br />

2<br />

r 0, S is not open.<br />

Remark: By (1)-(3), we can regard them as three sequences<br />

1<br />

2 m 5m , 1 5 m m<br />

and 1<br />

m1 4 m1 2<br />

n m 5m , respectively.<br />

m1<br />

<strong>And</strong> it means that for (1), the sequence 5 m m m1<br />

moves 1 . Similarly for others. So, it is<br />

2<br />

easy to see why 1 is an accumulation point of 1 2 2 5m m<br />

m1 . <strong>And</strong> thus get the set of all<br />

accumulation points of 2 n 5 m : m, n 1, 2, . . .<br />

(f) All numbers of the form 1 n 1/m, (m, n 1,2,....<br />

Solution: Write the set of all numbers 1 n 1/m, (m, n 1, 2, . . . as<br />

1 m 1 m m<br />

1 1<br />

m1 m : S.<br />

m1<br />

1<br />

2<br />

3


<strong>And</strong> thus by the remark in (e), it is easy to know that S 1, 1. So,S is not closed<br />

since S S. Since2 S, andB2, r is not contained in S for any r 0, S is not open.<br />

(g) All numbers of the form 1/n 1/m, (m, n 1,2,....<br />

Solution: Write the set of all numbers 1/n 1/m, (m, n 1, 2, . . . as<br />

1 1/m m m1<br />

1/2 1/m m m1<br />

...1/n 1/m m m1<br />

...: S.<br />

We find that S 1/n : n N 1/m : m N 0 1/n : n N 0. So,S is<br />

not closed since S S. Since1 S, andB1, r is not contained in S for any r 0, S is<br />

not open.<br />

(h) All numbers of the form 1 n /1 1/n, (n 1,2,....<br />

Soluton: Write the set of all numbers 1 n /1 1/n, (n 1, 2, . . . as<br />

k<br />

1<br />

1 1 2k k1<br />

<br />

1<br />

1 1<br />

2k1<br />

k<br />

k1<br />

: S.<br />

We find that S 1, 1. So,S is not closed since S S. Since 1 S, andB 1<br />

2 2<br />

not contained in S for any r 0, S is not open.<br />

, r is<br />

3.3 <strong>The</strong> same as Exercise 3.2 for the following sets in R 2 .<br />

(a) All complex z such that |z| 1.<br />

Solution: Denote z C :|z| 1 by S. It is easy to know that S z C :|z| 1.<br />

So, S is not closed since S S. Letz S, then |z| 1. Consider Bz, |z|1 S, soevery<br />

2<br />

point of S is interior. That is, S is open.<br />

(b) All complex z such that |z| 1.<br />

Solution: Denote z C :|z| 1 by S. It is easy to know that S z C :|z| 1.<br />

So, S is closed since S S. Since1 S, andB1, r is not contained in S for any r 0, S<br />

is not open.<br />

(c) All complex numbers of the form 1/n i/m, (m, n 1, 2, . . . .<br />

Solution: Write the set of all complex numbers of the form 1/n i/m,<br />

(m, n 1, 2, . . . as<br />

1 m i m<br />

1 i m<br />

m1 2 m ... 1<br />

m1<br />

n m i m<br />

...: S.<br />

m1<br />

We know that S 1/n : n 1,2,... i/m : m 1, 2, . . . 0. So,S is not closed<br />

since S S. Since1 i S, andB1 i, r is not contained in S for any r 0, S is not<br />

open.<br />

(d) All points x, y such that x 2 y 2 1.<br />

Solution: Denote x, y : x 2 y 2 1 by S. We know that<br />

S x, y : x 2 y 2 1. So,S is not closed since S S. Letp x, y S, then<br />

x 2 y 2 1. Itiseasytofindthatr 0 such that Bp, r S. So,S is open.<br />

(e) All points x, y such that x 0.<br />

Solution: Write all points x, y such that x 0asx, y : x 0 : S. It is easy to<br />

know that S x, y : x 0. So,S is not closed since S S. Letx S, then it is easy<br />

to find r x 0 such that Bx, r x S. So,S is open.<br />

(f) All points x, y such that x 0.<br />

Solution: Write all points x, y such that x 0asx, y : x 0 : S. It is easy to


know that S x, y : x 0. So,S is closed since S S. Since0, 0 S, and<br />

B0, 0, r is not contained in S for any r 0, S is not open.<br />

3.4 Prove that every nonempty open set S in R 1 contains both rational and irratonal<br />

numbers.<br />

proof: Given a nonempty open set S in R 1 .Letx S, then there exists r 0 such that<br />

Bx, r S since S is open. <strong>And</strong> in R 1 , the open ball Bx, r x r, x r. Since any<br />

interval contains both rational and irrational numbers, we have S contains both rational and<br />

irrational numbers.<br />

3.5 Prove that the only set in R 1 which are both open and closed are the empty set<br />

and R 1 itself. Is a similar statement true for R 2 <br />

Proof: Let S be the set in R 1 , and thus consider its complement T R 1 S. <strong>The</strong>n we<br />

have both S and T are open and closed. Suppose that S R 1 and S , we will show that<br />

it is impossible as follows.<br />

Since S R 1 ,andS , then T and T R 1 . Choose s 0 S and t 0 T, then we<br />

consider the new point s 0t 0<br />

which is in S or T since R S T. If s 0t 0<br />

S, wesay<br />

2 2<br />

s 0 t 0<br />

s<br />

2 1, otherwise, we say s 0t 0<br />

t<br />

2 1.<br />

Continue these steps, we finally have two sequences named s n S and t m T.<br />

In addition, the two sequences are convergent to the same point, say p by our construction.<br />

So, we get p S and p T since both S and T are closed.<br />

However, it leads us to get a contradiction since p S T . Hence S R 1 or<br />

S .<br />

Remark: 1. In the proof, the statement is true for R n .<br />

2. <strong>The</strong> construction is not strange for us since the process is called Bolzano Process.<br />

3. 6 Prove that every closed set in R 1 is the intersection of a countable collection of<br />

open sets.<br />

proof: Given a closed set S, and consider its complement R 1 S which is open. If<br />

R 1 S , there is nothing to prove. So, we can assume that R 1 S .<br />

Let x R 1 S, then x is an interior point of R 1 S. So, there exists an open interval<br />

a, b such that x a, b R 1 S. In order to show our statement, we choose a smaller<br />

interval a x , b x so that x a x , b x and a x , b x a, b R 1 S. Hence, we have<br />

R 1 S xR 1 S a x , b x <br />

which implies that<br />

S R 1 xR 1 S a x , b x <br />

xR 1 S R 1 a x , b x <br />

n n1 R 1 a n , b n (by Lindelof Convering <strong>The</strong>orem).<br />

Remark: 1. <strong>The</strong>re exists another proof by Representation <strong>The</strong>orem for Open Sets on<br />

<strong>The</strong> <strong>Real</strong> Line.<br />

2. Note that it is true for that every closed set in R 1 is the intersection of a countable<br />

collection of closed sets.<br />

3. <strong>The</strong> proof is suitable for R n if the statement is that every closed set in R n is the<br />

intersection of a countable collection of open sets. All we need is to change intervals into<br />

disks.


3.7 Prove that a nonempty, bounded closed set S in R 1 is either a closed interval, or<br />

that S can be obtained from a closed interval by removing a countable disjoint collection of<br />

open intervals whose endpoints belong to S.<br />

proof: If S is an interval, then it is clear that S is a closed interval. Suppose that S is not<br />

an interval. Since S is bounded and closed, both sup S and inf S are in S. So,R 1 S<br />

inf S,supS S. Denote inf S,supS by I. Consider R 1 S is open, then by<br />

Representation <strong>The</strong>orem for Open Sets on <strong>The</strong> <strong>Real</strong> Line, we have<br />

R 1 S m m1 I m<br />

I S<br />

which implies that<br />

S I m m1 I m .<br />

That is, S can be obtained from a closed interval by removing a countable disjoint<br />

collection of open intervals whose endpoints belong to S.<br />

Open and closed sets in R n<br />

3.8 Prove that open n balls and n dimensional open intervals are open sets in R n .<br />

proof: Given an open n ball Bx, r. Choose y Bx, r and thus consider<br />

By, d Bx, r, whered min|x y|, r |x y|. <strong>The</strong>n y is an interior point of Bx, r.<br />

Since y is arbitrary, we have all points of Bx, r are interior. So, the open n ball Bx, r is<br />

open.<br />

Given an n dimensional open interval a 1 , b 1 a 2 , b 2 ...a n , b n : I. Choose<br />

x x 1, x 2 ,...,x n I and thus consider r minin i1 r i ,wherer i minx i a i , b i x i .<br />

<strong>The</strong>n Bx, r I. Thatis,x is an interior point of I. Sincex is arbitrary, we have all points<br />

of I are interior. So, the n dimensional open interval I is open.<br />

3.9 Prove that the interior of a set in R n isopeninR n .<br />

Proof: Let x intS, then there exists r 0 such that Bx, r S. Choose any point of<br />

Bx, r, sayy. <strong>The</strong>n y is an interior point of Bx, r since Bx, r is open. So, there exists<br />

d 0 such that By, d Bx, r S. Soy is also an interior point of S. Sincey is<br />

arbitrary, we find that every point of Bx, r is interior to S. Thatis,Bx, r intS. Sincex<br />

is arbitrary, we have all points of intS are interior. So, intS is open.<br />

Remark: 1 It should be noted that S is open if, and only if S intS.<br />

2. intintS intS.<br />

3. If S T, then intS intT.<br />

3.10 If S R n , prove that intS is the union of all open subsets of R n which are<br />

contained in S. This is described by saying that intS is the largest open subset of S.<br />

proof: It suffices to show that intS AS A,whereA is open. To show the statement,<br />

we consider two steps as follows.<br />

1. Let x intS, then there exists r 0 such that Bx, r S. So,<br />

x Bx, r AS A.Thatis,intS AS A.<br />

2. Let x AS A, then x A for some open set A S. SinceA is open, x is an<br />

interior point of A. <strong>The</strong>re exists r 0 such that Bx, r A S. Sox is an interior point<br />

of S, i.e., x intS. Thatis, AS A intS.<br />

From 1 and 2, we know that intS AS A,whereA is open.<br />

Let T be an open subset of S such that intS T. SinceintS AS A,whereA is open,


we have intS T AS A which implies intS T by intS AS A. Hence, intS is the<br />

largest open subset of S.<br />

3.11 If S and T are subsets of R n , prove that<br />

intS intT intS T and intS intT intS T.<br />

Proof: For the part intS intT intS T, we consider two steps as follows.<br />

1. Since intS S and intT T, wehaveintS intT S T which implies<br />

that Note that intS intT is open.<br />

intS intT intintS intT intS T.<br />

2. Since S T S and S T T, wehaveintS T intS and<br />

intS T intT. So,<br />

intS T intS intT.<br />

From 1 and 2, we know that intS intT intS T.<br />

For the part intS intT intS T, we consider intS S and intT T. So,<br />

intS intT S T<br />

which implies that Note that intS intT is open.<br />

intintS intT intS intT intS T.<br />

Remark: It is not necessary that intS intT intS T. For example, let S Q,<br />

and T Q c , then intS , andintT . However, intS T intR 1 R.<br />

3.12 Let S denote the derived set and S theclosureofasetSinR n . Prove that<br />

(a) S is closed in R n ; that is S S .<br />

proof: Let x be an adherent point of S . In order to show S is closed, it suffices to<br />

show that x is an accumulation point of S. Assume x is not an accumulation point of S, i.e.,<br />

there exists d 0 such that<br />

Bx, d x S . *<br />

Since x adheres to S , then Bx, d S . So, there exists y Bx, d such that y is an<br />

accumulation point of S. Note that x y, by assumption. Choose a smaller radius d so that<br />

By, d Bx, d x and By, d S .<br />

It implies<br />

By, d S Bx, d x S by (*)<br />

which is absurb. So, x is an accumulation point of S. Thatis,S contains all its adherent<br />

points. Hence S is closed.<br />

(b) If S T, then S T .<br />

Proof: Let x S , then Bx, r x S for any r 0. It implies that<br />

Bx, r x T for any r 0sinceS T. Hence, x is an accumulation point of T.<br />

That is, x T .So,S T .<br />

(c) S T S T <br />

Proof: For the part S T S T , we show it by two steps.<br />

1. Since S S T and T S T, wehaveS S T and T S T by (b).<br />

So,<br />

S T S T <br />

2. Let x S T , then Bx, r x S T . Thatis,


Bx, r x S Bx, r x T .<br />

So, at least one of Bx, r x S and Bx, r x T is not empty. If<br />

Bx, r x S , then x S .<strong>And</strong>ifBx, r x T , then x T .So,<br />

S T S T .<br />

From 1 and 2, we have S T S T .<br />

Remark: Note that since S T S T ,wehaveclS T clS clT,<br />

where clS is the closure of S.<br />

(d) S S .<br />

Proof: Since S S S , then S S S S S S since S S by<br />

(a).<br />

(e) S is closed in R n .<br />

Proof: Since S S S by (d), then S cantains all its accumulation points. Hence, S<br />

is closed.<br />

Remark: <strong>The</strong>re is another proof which is like (a). But it is too tedious to write.<br />

(f) S is the intersection of all closed subsets of R n containing S. That is, S is the<br />

smallest closed set containing S.<br />

Proof: It suffices to show that S AS A,whereA is closed. To show the statement,<br />

we consider two steps as follows.<br />

1. Since S is closed and S S, then AS A S.<br />

2. Let x S, then Bx, r S for any r 0. So, if A S, then<br />

Bx, r A for any r 0. It implies that x is an adherent point of A. Hence if A S,<br />

and A is closed, we have x A. Thatis,x AS A.So,S AS A.<br />

From 1 and 2, we have S AS A.<br />

Let S T S, whereT is closed. <strong>The</strong>n S AS A T. It leads us to get T S.<br />

That is, S is the smallest closed set containing S.<br />

Remark: In the exercise, there has something to remeber. We list them below.<br />

Remark 1. If S T, then S T .<br />

2. If S T, then S T.<br />

3. S S S .<br />

4. S is closed if, and only if S S.<br />

5. S is closed.<br />

6. S is the smallest closed set containing S.<br />

3.13 Let S and T be subsets of R n . Prove that clS T clS clT and that<br />

S clT clS T if S is open, where clS is the closure of S.<br />

Proof: Since S T S and S T T, then clS T clS and,<br />

clS T clT. So,clS T clS clT.<br />

Given an open set S , and let x S clT, then we have


1. x S and S is open.<br />

Bx, d S for some d 0.<br />

<br />

Bx, r S Bx, r if r d.<br />

Bx, r S Bx, d if r d.<br />

and<br />

2. x clT<br />

Bx, r T for any r 0.<br />

From 1 and 2, we know<br />

Bx, r S T Bx, r S T Bx, r T if r d.<br />

Bx, r S T Bx, r S T Bx, d T if r d.<br />

So, it means that x is an adherent point of S T. Thatis,x clS T. Hence,<br />

S clT clS T.<br />

Remark: It is not necessary that clS T clS clT. For example, S Q and<br />

T Q c , then clS T and clS clT R 1 .<br />

Note. <strong>The</strong> statements in Exercises 3.9 through 3.13 are true in any metric space.<br />

3.14 AsetSinR n is called convex if, for every pair of points x and y in S and every<br />

real satisfying 0 1, we have x 1 y S. Interpret this statement<br />

geometrically (in R 2 and R 3 and prove that<br />

(a) Every n ball in R n is convex.<br />

Proof: Given an n ball Bp, r, and let x, y Bp, r. Consider x 1 y, where<br />

0 1.<br />

<strong>The</strong>n<br />

x 1 y p x p 1 y p<br />

x p 1 y p<br />

r 1 r<br />

r.<br />

So, we have x 1 y Bp, r for 0 1. Hence, by the definition of convex,<br />

we know that every n ball in R n is convex.<br />

(b) Every n dimensional open interval is convex.<br />

Proof: Given an n dimensional open interval I a 1 , b 1 ...a n , b n .Letx, y I,<br />

and thus write x x 1 , x 2 ,...,x n and y y 1 , y 2 ,...y n . Consider<br />

x 1 y x 1 1 y 1 , x 2 1 y 2 ,...,x n 1 y n where 0 1.<br />

<strong>The</strong>n<br />

a i x i 1 y i b i ,wherei 1, 2, . . , n.<br />

So, we have x 1 y I for 0 1. Hence, by the definition of convex, we<br />

know that every n dimensional open interval is convex.<br />

(c) <strong>The</strong> interior of a convex is convex.<br />

Proof: Given a convex set S, and let x, y intS. <strong>The</strong>n there exists r 0 such that<br />

Bx, r S, andBy, r S. Consider x 1 y : p S, where 0 1, since S<br />

is convex.


Claim that Bp, r S as follows.<br />

Let q Bp, r, We want to find two special points x Bx, r, andy By, r such<br />

that q x 1 y.<br />

Since the three n balls Bx, r, By, r, andBp, r have the same radius. By<br />

parallelogram principle, we let x q x p, andy q y p, then<br />

x x q p r, andy y q p r.<br />

It implies that x Bx, r, andy By, r. In addition,<br />

x 1 y<br />

q x p 1 q y p<br />

q.<br />

Since x, y S, andS is convex, then q x 1 y S. It implies that Bp, r S<br />

since q is arbitrary. So, we have proved the claim. That is, for 0 1,<br />

x 1 y p intS if x, y intS, andS is convex. Hence, by the definition of<br />

convex, we know that the interior of a convex is convex.<br />

(d) <strong>The</strong> closure of a convex is convex.<br />

Proof: Given a convex set S, and let x, y S. Consider x 1 y : p, where<br />

0 1, and claim that p S, i.e., we want to show that Bp, r S .<br />

Suppose NOT, there exists r 0 such that<br />

Bp, r S . *<br />

Since x, y S, then Bx, r S and By, r S . <strong>And</strong> let x Bx, r S and<br />

2 2 2<br />

y By, r S. Consider<br />

2<br />

x 1 y p x 1 y x 1 y<br />

x x 1 y 1 y<br />

<br />

x x x x <br />

1 y 1 y 1 y 1 y<br />

x x 1 y y | |x y<br />

<br />

2 r | |x y<br />

r<br />

if we choose a suitable number , where 0 1.<br />

Hence, we have the point x 1 y Bp, r. Note that x, y S and S is convex,<br />

we have x 1 y S. It leads us to get a contradiction by (*). Hence, we have proved<br />

the claim. That is, for 0 1, x 1 y p S if x, y S. Hence, by the<br />

definition of convex, we know that the closure of a convex is convex.<br />

3.15 Let F be a collection of sets in R n , and let S AF A and T AF A. For each<br />

of the following statements, either give a proof or exhibit a counterexample.<br />

F.<br />

(a) If x is an accumulation point of T, then x is an accumulation point of each set A in<br />

Proof: Let x be an accumulation point of T, then Bx, r x T for any<br />

r 0. Note that for any A F, wehaveT A. Hence Bx, r x A for any<br />

r 0. That is, x is an accumulation point of A for any A F.<br />

<strong>The</strong> conclusion is that If x is an accumulation point of T AF A, then x is an<br />

accumulation point of each set A in F.


(b) If x is an accumulation point of S, then x is an accumulation point of at least one set<br />

AinF.<br />

Proof: No! For example, Let S R n ,andF be the collection of sets consisting of a<br />

single point x R n . <strong>The</strong>n it is trivially seen that S AF A.<strong>And</strong>ifx is an accumulation<br />

point of S, then x is not an accumulation point of each set A in F.<br />

3.16 Prove that the set S of rational numbers in the inerval 0, 1 cannot be<br />

expressed as the intersection of a countable collection of open sets. Hint: Write<br />

S x 1 , x 2 ,..., assume that S k k1 S k , where each S k is open, and construct a<br />

sequence Q n of closed intervals such that Q n1 Q n S n and such that x n Q n .<br />

<strong>The</strong>n use the Cantor intersection theorem to obtain a contradiction.<br />

Proof: We prove the statement by method of contradiction. Write S x 1 , x 2 ,..., and<br />

assume that S k k1 S k , where each S k is open.<br />

Since x 1 S 1 , there exists a bounded and open interval I 1 S 1 such that x 1 I 1 .<br />

Choose a closed interval Q 1 I 1 such that x 1 Q 1 .SinceQ 1 is an interval, it contains<br />

infinite rationals, call one of these, x 2 .Sincex 2 S 2 , there exists an open interval I 2 S 2<br />

and I 2 Q 1 . Choose a closed interval Q 2 I 2 such that x 2 Q 2 . Suppose Q n has been<br />

constructed so that<br />

1. Q n is a closed interval.<br />

2. Q n Q n1 S n1 .<br />

3. x n Q n .<br />

Since Q n is an interval, it contains infinite rationals, call one of these, x n1 .Since<br />

x n1 S n1 , there exists an open interval I n1 S n1 and I n1 Q n . Choose a closed<br />

interval Q n1 I n1 such that x n1 Q n1 .So,Q n1 satisfies our induction hypothesis, and<br />

the construction can process.<br />

Note that<br />

1. For all n, Q n is not empty.<br />

2. For all n, Q n is bounded since I 1 is bounded.<br />

3. Q n1 Q n .<br />

4. x n Q n .<br />

<strong>The</strong>n n n1 Q n by Cantor Intersection <strong>The</strong>orem.<br />

Since Q n S n , n n1 Q n n n1 S n S. So, we have<br />

S n n1 Q n n n1 Q n <br />

which is absurb since S n n1 Q n by the fact x n Q n . Hence, we have proved that<br />

our assumption does not hold. That is, S the set of rational numbers in the inerval 0, 1<br />

cannot be expressed as the intersection of a countable collection of open sets.<br />

Remark: 1. Often, the property is described by saying Q is not an G set.<br />

2. It should be noted that Q c is an G set.<br />

3. For the famous <strong>The</strong>orem called Cantor Intersection <strong>The</strong>orem, the reader should<br />

see another classical text book, Principles of Mathematical Analysis written by Walter<br />

Rudin, <strong>The</strong>orem 3.10 in page 53.<br />

4. For the method of proof, the reader should see another classical text book, Principles<br />

of Mathematical Analysis written by Walter Rudin, <strong>The</strong>orem 2.43, in page 41.


Covering theorems in R n<br />

3.17 If S R n , prove that the collection of isolated points of S is countable.<br />

Proof: Denote the collection of isolated points of S by F. Letx F, there exists an<br />

n ball Bx, r x x S . Write Q n x 1 , x 2 ,..., then there are many numbers in<br />

Q n lying on Bx, r x x. We choose the smallest index, say m mx, and denote x by<br />

x m .<br />

So, F x m : m P, whereP N, a subset of positive integers. Hence, F is<br />

countable.<br />

3.18 Prove that the set of open disks in the xy plane with center x, x and radius<br />

x 0, x rational, is a countable covering of the set x, y : x 0, y 0.<br />

Proof: Denote the set of open disks in the xy plane with center x, x and radius x 0<br />

by S. Choose any point a, b, wherea 0, and b 0. We want to find an 2 ball<br />

Bx, x, x S which contains a, b. It suffices to find x Q such that<br />

x, x a, b x. Since<br />

x, x a, b x x, x a, b 2 x 2 x 2 2a bx a 2 b 2 0.<br />

Since x 2 2a bx a 2 b 2 x a b 2 2ab, we can choose a suitable rational<br />

number x such that x 2 2a bx a 2 b 2 0sincea 0, and b 0. Hence, for any<br />

point a, b, wherea 0, and b 0, we can find an 2 ball Bx, x, x S which<br />

contains a, b.<br />

That is, S is a countable covering of the set x, y : x 0, y 0.<br />

Remark: <strong>The</strong> reader should give a geometric appearance or draw a graph.<br />

3.19 <strong>The</strong> collection Fof open intervals of the form 1/n,2/n, where n 2, 3, . . . , is an<br />

open covering of the open interval 0, 1. Prove (without using <strong>The</strong>orem 3.31) that no<br />

finite subcollection of F covers 0, 1.<br />

Proof: Write F as 1 ,1, 1 , 2 ,..., 1 2 3 3 n , 2 n ,.... Obviously, F is an open covering<br />

of 0, 1. Assume that there exists a finite subcollection of F covers 0, 1, and thus write<br />

them as F n 1 1<br />

1<br />

, m1 ,...., n 1 1<br />

k<br />

, mk . Choose p 0, 1 so that p min 1ik n 1 i<br />

. <strong>The</strong>n<br />

p n 1 1<br />

i<br />

, mi , where 1 i k. It contracdicts the fact F covers 0, 1.<br />

Remark: <strong>The</strong> reader should be noted that if we use <strong>The</strong>orem 3.31, then we cannot get<br />

the correct proof. In other words, the author T. M. Apostol mistakes the statement.<br />

3.20 Give an example of a set S which is closed but not bounded and exhibit a<br />

coubtable open covering F such that no finite subset of F covers S.<br />

Solution: Let S R 1 , then R 1 is closed but not bounded. <strong>And</strong> let<br />

F n, n 2 : n Z, then F is a countable open covering of S. In additon, it is<br />

trivially seen that no finite subset of F covers S.<br />

3.21 Given a set S in R n with the property that for every x in S there is an n ball Bx<br />

such that Bx S is coubtable. Prove that S is countable.<br />

Proof: Note that F Bx : x S forms an open covering of S. SinceS R n , then<br />

there exists a countable subcover F F of S by Lindelof Covering <strong>The</strong>orem. Write<br />

F Bx n : n N. Since<br />

S S nN Bx n nN S Bx n ,<br />

and<br />

S Bx n is countable by hypothesis.


<strong>The</strong>n S is countable.<br />

Remark: <strong>The</strong> reader should be noted that exercise 3.21 is equivalent to exercise 3.23.<br />

3.22 Prove that a collection of disjoint open sets in R n is necessarily countable. Give an<br />

example of a collection of disjoint closed sets which is not countable.<br />

Proof: Let F be a collection of disjoint open sets in R n , and write Q n x 1 , x 2 ,....<br />

Choose an open set S in F, then there exists an n ball By, r S. In this ball, there<br />

are infinite numbers in Q n . We choose the smallest index, say m my. <strong>The</strong>n we have<br />

F S m : m P N which is countable.<br />

For the example that a collection of disjoint closed sets which is not countable, we give<br />

it as follows. Let G x : x R n , then we complete it.<br />

3.23 Assume that S R n . A point x in R n is said to be condensation point of S if every<br />

n ball Bx has the property that Bx S is not countable. Prove that if S is not<br />

countable, then there exists a point x in S such that x is a condensation point of S.<br />

Proof: It is equivalent to exercise 3.21.<br />

Remark: Compare with two definitions on a condensation point and an accumulation<br />

point, it is easy to know that a condensation point is an accumulation point. However, am<br />

accumulation point is not a condensation point, for example, S 1/n : n N. Wehave<br />

0 is an accumulation point of S, but not a condensation point of S.<br />

3.24 Assume that S R n and assume that S is not countable. Let T denote the set of<br />

condensation points of S. Prove that<br />

(a) S T is countable.<br />

Proof: If S T is uncountable, then there exists a point x in S T such that xisa<br />

condensation point of S T by exercise 3.23. Obviously, x S is also a condensation<br />

point of S. It implies x T. So, we have x S T which is absurb since x S T.<br />

Remark: <strong>The</strong> reader should regard T as a special part of S, and the advantage of T<br />

helps us realize the uncountable set S R n . Compare with Cantor-Bendixon <strong>The</strong>orem<br />

in exercise 3.25.<br />

(b) S T is not countable.<br />

Proof: Suppose S T is countable, then S S T S T is countable by (a)<br />

which is absurb. So, S T is not countable.<br />

(c) T is a closed set.<br />

Proof: Let x be an adherent point of T, then Bx, r T for any r 0. We want to<br />

show x T. That is to show x is a condensation point of S. Claim that Bx, r S is<br />

uncountable for any r 0.<br />

Suppose NOT, then there exists an n ball Bx, d S which is countable. Since x is an<br />

adherent point of T, then Bx, d T . Choose y Bx, d T so that By, Bx, d<br />

and By, S is uncountable. However, we get a contradiction since<br />

By, S is uncountable Bx, d S is countable .<br />

Hence, Bx, r S is uncountable for any r 0. That is, x T. SinceT contains its all<br />

adherent points, T is closed.<br />

(d) T contains no isolated points.<br />

Proof: Let x T, andifx is an isolated point of T, then there exists an n ball Bx, d<br />

such that Bx, d T x. On the other hand, x T means that Bx, d x S is


uncountable. Hence, by exercise 3.23, we know that there exists y Bx, d x S<br />

such that y is a condensation point of Bx, d x S. So,y is a condensation point of<br />

S. It implies y T. It is impossible since<br />

1. y x T.<br />

2. y Bx, d.<br />

3. Bx, d T x.<br />

Hence, x is not an isolated point of T, ifx T. Thatis,T contains no isolatd points.<br />

Remark: Use exercise 3.25, by (c) and (d) we know that T is perfect.<br />

Note that Exercise 3.23 is a special case of (b).<br />

3.25 AsetinR n is called perfect if S S, that is, if S is a closed set which contains<br />

no isolated points. Prove that every uncountable closed set F in R n canbeexpressedinthe<br />

form F A B, where A is perfect and B is countable (Cantor-Bendixon theorem).<br />

Hint. Use Exercise 3.24.<br />

Proof: Let F be a uncountable closed set in R n . <strong>The</strong>n by exercise 3.24,<br />

F F T F T, whereT is the set of condensation points of F. Note that since F is<br />

closed, T F by the fact, a condensation point is an accumulation point. Define<br />

F T A and F T B, then B is countable and A T is perfect.<br />

Remark: 1. <strong>The</strong> reader should see another classical text book, Principles of<br />

Mathematical Analysis written by Walter Rudin, <strong>The</strong>orem 2.43, in page 41. Since the<br />

theorem is famous, we list it below.<br />

<strong>The</strong>orem 2.43 Let P be a nonempty perfect set in R k . <strong>The</strong>n P is uncountable.<br />

<strong>The</strong>orem Modefied 2.43 Let P be a nonempty perfect set in a complete separable<br />

metric space. <strong>The</strong>n P is uncountable.<br />

2. Let S has measure zero in R 1 . Prove that there is a nonempty perfect set P in R 1 such<br />

that P S .<br />

Proof: SinceS has measure zero, there exists a collection of open intervals I k such<br />

that<br />

S I k and |I k | 1.<br />

Consider its complement I k c which is closed with positive measure. Since the<br />

complement has a positive measure, we know that it is uncountable. Hence, by<br />

Cantor-Bendixon <strong>The</strong>orem, we know that<br />

I k c A B, whereA is perfect and B is countable.<br />

So, let A P, wehaveproveit.<br />

Note: From the similar method, we can show that given any set S in R 1 with measure<br />

0 d , there is a non-empty perfect set P such that P S . In particular, S Q,<br />

S the set of algebraic numbers, and so on. In addition, even for cases in R k , it still holds.<br />

Metric Spaces<br />

3.26 In any metric space M, d prove that the empty set and the whole set M are both<br />

open and closed.<br />

proof: In order to show the statement, it suffices to show that M is open and closed<br />

since M M . Letx M, then for any r 0, B M x, r M. Thatis,x is an interior


point of M. Sincx is arbitrary, we know that every point of M is interior. So, M is open.<br />

Let x be an adherent point of M, it is clearly x M since we consider all points lie in<br />

M. Hence, M contains its all adherent points. It implies that M is closed.<br />

Remark: <strong>The</strong> reader should regard the statement as a common sense.<br />

3.27 Consider the following two metrics in R n :<br />

in<br />

d 1 x, y max 1in |x i y i |, d 2 x, y |xi i1<br />

y i |.<br />

In each of the following metric spaces prove that the ball Ba; r has the geometric<br />

appearance indicated:<br />

(a) In R 2 , d 1 , a square with sides parallel to the coordinate axes.<br />

Solution: It suffices to consider the case B0, 0,1. Letx x 1 , x 2 B0, 0,1,<br />

then we have<br />

|x 1 | 1, and |x 2 | 1.<br />

So, it means that the ball B0, 0,1 is a square with sides lying on the coordinate axes.<br />

Hence, we know that Ba; r is a square with sides parallel to the coordinate axes.<br />

(b) In R 2 , d 2 , a square with diagonals parallel to the axes.<br />

Solution: It suffices to consider the case B0, 0,1. Letx x 1 , x 2 B0, 0,1,<br />

then we have<br />

|x 1 x 2 | 1.<br />

So, it means that the ball B0, 0,1 is a square with diagonals lying on the coordinate<br />

axes. Hence, we know that Ba; r is a square with diagonals parallel to the coordinate<br />

axes.<br />

(c) A cube in R 3 , d 1 .<br />

Solution:It suffices to consider the case B0, 0, 0,1. Let<br />

x x 1 , x 2 , x 3 B0, 0, 0,1, then we have<br />

|x 1 | 1, |x 2 | 1, and |x 3 | 1.<br />

So, it means that the ball B0, 0, 0,1 is a cube with length 2. Hence, we know that<br />

Ba; r is a cube with length 2a.<br />

(d) An octahedron in R 3 , d 2 .<br />

Solution: It suffices to consider the case B0, 0, 0,1. Let<br />

x x 1 , x 2 , x 3 B0, 0, 0,1, then we have<br />

|x 1 x 2 x 3 | 1.<br />

It means that the ball B0, 0, 0,1 is an octahedron. Hence, Ba; r is an octahedron.<br />

Remark: <strong>The</strong> exercise tells us one thing that Ba; r may not be an n ball if we<br />

consider some different matrices.<br />

3.28 Let d 1 and d 2 be the metrics of Exercise 3.27 and let x y denote the usual<br />

Euclidean metric. Prove that the following inequalities for all x and y in R n :<br />

d 1 x, y x y d 2 x, y and d 2 x, y n x y nd 1 x, y.<br />

Proof: List the definitions of the three metrics, and compare with them as follows.


<strong>The</strong>n we have<br />

(a)<br />

(b)<br />

(c)<br />

(d)<br />

x y <br />

1. d 1 x, y max 1in |x i y i |.<br />

2. x y in i1<br />

x i y i 2 1/2<br />

.<br />

in<br />

3. d 2 x, y |xi i1<br />

y i |.<br />

d 1 x, y max<br />

1in |x i y i | <br />

<br />

<br />

in<br />

1/2<br />

x i y i 2<br />

i1<br />

in<br />

1/2<br />

x i y i 2<br />

i1<br />

in<br />

|x i y i |<br />

i1<br />

2 1/2<br />

in<br />

1/2<br />

n x y n x i y i 2<br />

i1<br />

d 2 x, y 2 <br />

n n max |x i y i |<br />

1in<br />

d 1 x, y.<br />

in<br />

|x i y i |<br />

i1<br />

2<br />

max |x i y i | 2 1/2<br />

1in<br />

x y.<br />

in<br />

|x i y i | d 2 x, y.<br />

i1<br />

<br />

2 1/2<br />

in<br />

1/2<br />

n x i y i 2<br />

i1<br />

n max<br />

1in |x i y i |<br />

in<br />

x i y i 2 2|x i y i ||x j y j |<br />

i1<br />

1ijn<br />

in<br />

in<br />

x i y i 2 n 1 x i y i 2 by A. P. G. P.<br />

i1<br />

i1<br />

in<br />

n x i y i 2<br />

i1<br />

nx y 2 .<br />

So,<br />

d 2 x, y n x y.<br />

From (a)-(d), we have proved these inequalities.<br />

Remark: 1. Let M be a given set and suppose that M, d and M, d are metric spaces.<br />

We define the metrics d and d are equivalent if, and only if, there exist positive constants<br />

, such that<br />

dx, y dx, y dx, y.<br />

<strong>The</strong> concept is much important for us to consider the same set with different metrics. For


example, in this exercise, Since three metrics are equivalent, it is easy to know that<br />

R k , d 1 , R k , d 2 ,andR k , . are complete. (For definition of complete metric space,<br />

the reader can see this text book, page 74.)<br />

2. It should be noted that on a finite dimensional vector space X, any two norms are<br />

equivalent.<br />

3.29 If M, d is a metric space, define d x, y dx,y . Prove that 1dx,y d isalsoametric<br />

for M. Note that 0 d x, y 1 for all x, yinM.<br />

Proof: In order to show that d isametricforM, we consider the following four steps.<br />

(1) For x M, d x, x 0sincedx, x 0.<br />

(2) For x y, d x, y dx,y 0sincedx, y 0.<br />

1dx,y<br />

(3) For x, y M, d x, y dx,y dy,x d y, x<br />

1dx,y 1dy,x<br />

(4) For x, y, z M,<br />

d dx, y<br />

x, y <br />

1 dx, y 1 1<br />

1 dx, y<br />

1 1 since dx, y dx, z dz, y<br />

1 dx, z dz, y<br />

dx, z dz, y<br />

<br />

1 dx, z dz, y<br />

dx, z<br />

<br />

1 dx, z dz, y dz, y<br />

1 dx, z dz, y<br />

dx, z<br />

<br />

1 dx, z dz, y<br />

1 dz, y<br />

d x, z d z, y<br />

Hence, from (1)-(4), we know that d is also a metric for M. Obviously,<br />

0 d x, y 1 for all x, y in M.<br />

Remark: 1. <strong>The</strong> exercise tells us how to form a new metric from an old metric. Also,<br />

the reader should compare with exercise 3.37. This is another construction.<br />

2. Recall Discrete metric d, we find that given any set nonempty S, S, d is a metric<br />

space, and thus use the exercise, we get another metric space S, d , and so on. Hence,<br />

here is a common sense that given any nonempty set, we can use discrete metric to form<br />

many and many metric spaces.<br />

3.30 Prove that every finite subset of a metric space is closed.<br />

Proof: Let x be an adherent point of a finite subet S x i : i 1, 2, . . . , n of a metric<br />

space M, d. <strong>The</strong>n for any r 0, Bx, r S . Ifx S, then B M x, S where<br />

min 1ijn dx i , x j . It is impossible. Hence, x S. Thatis,S contains its all adherent<br />

points. So, S is closed.<br />

3.31 In a metric space M, d the closed ball of radius r 0 about a point a in M is the<br />

set B a; r x : dx, a r.<br />

(a) Prove that B a; r is a closed set.<br />

Proof: Let x M B a; r, then dx, a r. Consider Bx, , where dx,ar ,<br />

2<br />

then if y Bx, , wehavedy, a dx, a dx, y dx, a dx,ar r. Hence,<br />

2<br />

Bx, M B a; r. That is, every point of M B a; r is interior. So, M B a; r is<br />

open, or equivalently, B a; r is a closed set.


(b) Give an example of a metric space in which B a; r is not the closure of the open<br />

ball Ba; r.<br />

Solution: Consider discrete metric space M, then we have let x M<br />

<strong>The</strong> closure of Ba;1 a<br />

and<br />

B a;1 M.<br />

Hence, if we let a is a proper subset of M, then B a;1 is not the closure of the open ball<br />

Ba;1.<br />

3.32 In a metric space M, if subsets satisfy A S A , where A is the closure of A,<br />

thenAissaidtobedense in S. For example, the set Q of rational numbers is dense in R. If<br />

A is dense in S and if S is dense in T, prove that A is dense in T.<br />

Proof: Since A is dense in S and S is dense in T, wehaveA S and S T. <strong>The</strong>n<br />

A T. Thatis,A is dense in T.<br />

3.33 Refer to exercise 3.32. A metric space M is said to be separable if there is a<br />

countable subset A which is dense in M. For example, R 1 is separable becasue the set Q of<br />

rational numbrs is a countable dense subset. Prove that every Euclidean space R k is<br />

separable.<br />

Proof: Since Q k is a countable subset of R k ,andQ k<br />

R k , then we know that R k is<br />

separable.<br />

3.34 Refer to exercise 3.33. Prove that the Lindelof covering theorem (<strong>The</strong>orem 3.28)<br />

is valid in any separable metric space.<br />

Proof: Let M, d be a separable metric space. <strong>The</strong>n there exists a countable subset<br />

S x n : n N M which is dense in M. Given a set A M, and an open covering F<br />

of A. Write P Bx n , r m : x n S, r m Q.<br />

Claim that if x M, andG is an open set in M which contains x. <strong>The</strong>n<br />

x Bx n , r m G for some Bx n , r m P.<br />

Since x G, there exists Bx, r x G for some r x 0. Note that x clS since S is<br />

dense in M. <strong>The</strong>n, Bx, r x /2 S . So, if we choose x n Bx, r x /2 S and r m Q<br />

with r x /2 r m r x /3, then we have<br />

x Bx n , r m <br />

and<br />

Bx n , r m Bx, r x <br />

since if y Bx n , r m , then<br />

dy, x dy, x n dx n , x<br />

r m r x<br />

2<br />

r x<br />

3 r x<br />

2<br />

r x<br />

So, we have prvoed the claim x Bx n , r m Bx, r x G or some Bx n , r m P.<br />

Use the claim to show the statement as follows. Write A GF G, and let x A, then<br />

there is an open set G in F such that x G. By the claim, there is Bx n , r m : B nm in P<br />

such that x B nm G. <strong>The</strong>re are, of course, infinitely many such B nm corresponding to<br />

each G, but we choose only one of these, for example, the one of smallest index, say<br />

q qx. <strong>The</strong>n we have x B qx G.


<strong>The</strong> set of all B qx obtained as x varies over all elements of A is a countable collection<br />

of open sets which covers A. To get a countable subcollection of F which covers A, we<br />

simply correlate to each set B qx one of the sets G of F which contained B qx . This<br />

complete the proof.<br />

3.35 Refer to exercise 3.32. If A is dense in S and B is open in S, prove that<br />

B clA B, where clA B means the closure of A B.<br />

Hint. Exercise 3.13.<br />

Proof: Since A is dense in S and B is open in S, A S and S B B. <strong>The</strong>n<br />

B S B<br />

A B, B is open in S<br />

clA B<br />

by exercise 3.13.<br />

3.36 Refer to exercise 3.32. If each of A and B is dense in S and if B is open in S, prove<br />

that A BisdenseinS.<br />

Proof: Since<br />

clA B, B is open<br />

clA B by exercise 3.13<br />

S B since A is dense in S<br />

B since B is open in S<br />

then<br />

clA B B<br />

which implies<br />

clA B S<br />

since B is dense in S.<br />

3.37 Given two metric spaces S 1 , d 1 and S 2 , d 2 , ametric for the Cartesian<br />

product S 1 S 2 can be constructed from d 1 d 2 in may ways. For example, if x x 1 , x 2 <br />

and y y 1 , y 2 are in S 1 S 2 , let x, y d 1 x 1 , y 1 d 2 x 2 , y 2 . Prove that is a<br />

metric for S 1 S 2 and construct further examples.<br />

Proof: In order to show that isametricforS 1 S 2 , we consider the following four<br />

steps.<br />

(1) For x x 1 , x 2 S 1 S 2 , x, x d 1 x 1 , x 1 d 2 x 2 , x 2 0 0 0.<br />

(2) For x y, x, y d 1 x 1 , y 1 d 2 x 2 , y 2 0 since if x, y 0, then x 1 y 1<br />

and x 2 y 2 .<br />

(3) For x, y S 1 S 2 ,<br />

x, y d 1 x 1 , y 1 d 2 x 2 , y 2 <br />

d 1 y 1 , x 1 d 2 y 2 , x 2 <br />

y, x.<br />

(4) For x, y, z S 1 S 2 ,<br />

x, y d 1 x 1 , y 1 d 2 x 2 , y 2 <br />

d 1 x 1 , z 1 d 1 z 1 , y 1 d 2 x 2 , z 2 d 2 z 2 , y 2 <br />

d 1 x 1 , z 1 d 2 x 2 , z 2 d 1 z 1 , y 1 d 2 z 2 , y 2 <br />

x, z z, y.


Hence from (1)-(4), we know that isametricforS 1 S 2 .<br />

For other metrics, we define<br />

1 x, y : d 1 x 1 , y 1 d 2 x 2 , y 2 for , 0.<br />

d<br />

2 x, y : d 1 x 1 , y 1 2 x 2 , y 2 <br />

1 d 2 x 2 , y 2 <br />

and so on. (<strong>The</strong> proof is similar with us by above exercises.)<br />

Compact subsets of a metric space<br />

3.38 Assume S T M. <strong>The</strong>n S is compact in M, d if, and only if, S is compact in<br />

the metric subspace T, d.<br />

Proof: Suppose that S is compact in M, d. LetF O : O is open in T be an<br />

open covering of S. SinceO is open in T, there exists the corresponding G which is open<br />

in M such that G T O . It is clear that G forms an open covering of S. Sothereis<br />

a finite subcovering G 1 ,...,G n of S since S is compact in M, d. Thatis,S kn k1 G k .<br />

It implies that<br />

S T S<br />

T kn k1 G k . <br />

kn k1 T G k <br />

kn k1 O k F.<br />

So, we find a fnite subcovering O 1 ,...,O n of S. Thatis,S is compact in T, d.<br />

Suppose that S is compact in T, d. LetG G : G is open in M be an open<br />

covering of S. SinceG T : O is open in T, the collection O forms an open<br />

covering of S. So, there is a finite subcovering O 1 ,...,O n of S since S is compact in<br />

T, d. Thatis,S kn k1 O k . It implis that<br />

S kn k1 O k kn k1 G k .<br />

So, we find a finite subcovering G 1 ,...,G n of S. Thatis,S is compact in M, d.<br />

Remark: <strong>The</strong> exercise tells us one thing that the property of compact is not changed,<br />

but we should note the property of being open may be changed. For example, in the<br />

2 dimensional Euclidean space, an open interval a, b is not open since a, b cannot<br />

contain any 2 ball.<br />

3.39 If S is a closed and T is compact, then S T is compact.<br />

Proof: Since T is compact, T is closed. We have S T is closed. Since S T T, by<br />

<strong>The</strong>orem 3.39, we know that S T is compact.<br />

3.40 <strong>The</strong> intersection of an arbitrary collection of compact subsets of M is compact.<br />

Proof: Let F T : T is compacet in M , and thus consider TF<br />

T, whereF F.<br />

We have TF<br />

T is closed. Choose S F . then we have TF<br />

T S. Hence, by<br />

<strong>The</strong>orem 3.39 TF<br />

T is compact.<br />

3.41 <strong>The</strong> union of a finite number of compact subsets of M is cmpact.<br />

Proof: Denote T k is a compact subset of M : k 1, 2, . . n by S. LetF be an open<br />

covering of kn k1 T k . If there does NOT exist a finite subcovering of kn k1 T k , then there<br />

does not exist a finite subcovering of T m for some T m S. SinceF is also an open<br />

covering of T m , it leads us to get T m is not compact which is absurb. Hence, if F is an open<br />

covering of kn k1 T k , then there exists a finite subcovering of kn k1 T k .So,kn k1 T k is


compact.<br />

3.42 Consider the metric space Q of rational numbers with the Euclidean metric of<br />

R 1 . Let S consists of all rational numbers in the open interval a, b, whereaandbare<br />

irrational. <strong>The</strong>n S is a closed and bounded subset of Q which is not compact.<br />

Proof: Obviously, S is bounded. Let x Q S, then x a, orx b. Ifx a, then<br />

B Q x, d x d, x d Q Q S, whered a x. Similarly, x b. Hence, x is an<br />

interior point of Q S. Thatis,Q S is open, or equivalently, S is closed.<br />

Remark: 1. <strong>The</strong> exercise tells us an counterexample about that in a metric space, a<br />

closed and bounded subset is not necessary to be compact.<br />

2. Here is another counterexample. Let M be an infinite set, and thus consider the<br />

metric space M, d with discrete metric d. <strong>The</strong>nbythefactBx,1/2 x for any x M,<br />

we know that F Bx,1/2 : x M forms an open covering of M. It is clear that there<br />

does not exist a finite subcovering of M. Hence, M is not compact.<br />

3.In any metric space M, d, we have three equivalent conditions on compact which<br />

list them below. Let S M.<br />

(a) Given any open covering of S, there exists a finite subcovering of S.<br />

(b) Every infinite subset of S has an accumulation point in S.<br />

(c) S is totally bounded and complete.<br />

4. It should be note that if we consider the Euclidean spaceR n , d, wehavefour<br />

equivalent conditions on compact which list them below. Let S R n .<br />

Remark (a) Given any open covering of S, there exists a finite subcovering of S.<br />

(b) Every infinite subset of S has an accumulation point in S.<br />

(c) S is totally bounded and complete.<br />

(d) S is bounded and closed.<br />

5. <strong>The</strong> concept of compact is familar with us since it can be regarded as a extension of<br />

Bolzano Weierstrass <strong>The</strong>orem.<br />

Miscellaneous properties of the interior and the boundary<br />

If A and B denote arbitrary subsets of a metric space M, prove that:<br />

3.43 intA M clM A.<br />

Proof: In order to show the statement, it suffices to show that M intA clM A.<br />

1. Let x M intA, we want to show that x clM A, i.e.,<br />

Bx, r M A for all r 0. Suppose Bx, d M A for some d 0. <strong>The</strong>n<br />

Bx, d A which implies that x intA. It leads us to get a conradiction since<br />

x M intA. Hence, if x M intA, then x clM A. Thatis,<br />

M intA clM A.<br />

2. Let x clM A, we want to show that x M intA, i.e., x is not an interior<br />

point of A. Suppose x is an interior point of A, then Bx, d A for some d 0. However,<br />

since x clM A, then Bx, d M A . It leads us to get a conradiction since<br />

Bx, d A. Hence, if x clM A, then x M intA. Thatis,clM A M intA.<br />

From 1 and 2, we know that M intA clM A, or equvilantly,<br />

intA M clM A.


3.44 intM A M A .<br />

Proof: Let B M A, and by exercise 3.33, we know that<br />

M intB clM B<br />

which implies that<br />

intB M clM B<br />

which implies that<br />

intM A M clA.<br />

3.45 intintA intA.<br />

Proof: Since S is open if, and only if, S intS. Hence, Let S intA, wehavethe<br />

equality intintA intA.<br />

3.46<br />

(a) intn<br />

i1 A i n i1 intA i , where each A i M.<br />

Proof: We prove the equality by considering two steps.<br />

(1) Since n<br />

i1<br />

i 1, 2, . . . , n. Hence, intn<br />

i1<br />

A i A i for all i 1,2,...,n, then intn<br />

i1<br />

A i n i1 intA i .<br />

(2) Since intA i A i , then n<br />

i1 intA i n<br />

i1<br />

have<br />

n<br />

i1<br />

From (1) and (2), we know that intn<br />

i1<br />

intA i intn i1 A i .<br />

A i i1<br />

A i .Sincen<br />

i1<br />

n<br />

intA i .<br />

A i intA i for all<br />

intA i is open, we<br />

Remark: Note (2), we use the theorem, a finite intersection of an open sets is open.<br />

Hence, we ask whether an infinite intersection has the same conclusion or not.<br />

Unfortunately, the answer is NO! Just see (b) and (c) in this exercise.<br />

(b) int AF A AF intA, if F is an infinite collection of subsets of M.<br />

Proof: Since AF A A for all A F. <strong>The</strong>n int AF A intA for all A F.<br />

Hence, int AF A AF intA.<br />

(c) Give an example where eqaulity does not hold in (b).<br />

Proof: Let F 1<br />

n , 1 n : n N, then int AF A , and AF intA 0. So,<br />

we can see that in this case, int AF A is a proper subset of AF intA. Hence, the<br />

equality does not hold in (b).<br />

Remark: <strong>The</strong> key to find the counterexample, it is similar to find an example that an<br />

infinite intersection of opens set is not open.<br />

3.47<br />

(a) AF intA int AF A.<br />

Proof: Since intA A, AF intA AF A.Wehave AF intA int AF A<br />

since AF intA is open.<br />

(b) Give an example of a finite collection F in which equality does not hold in (a).<br />

Solution: Consider F Q, Q c , then we have intQ intQ c and<br />

intQ Q c intR 1 R 1 . Hence, intQ intQ c is a proper subset of<br />

intQ Q c R 1 . That is, the equality does not hold in (a).<br />

3.48


(a) intA if A is open or if A is closed in M.<br />

Proof: (1) Suppose that A is open. We prove it by the method of contradiction. Assume<br />

that intA , and thus choose<br />

x intA<br />

intclA clM A<br />

intclA M A<br />

intclA intM A since intS T intS intT.<br />

Since<br />

x intclA Bx, r 1 clA A A <br />

and<br />

x intM A Bx, r 2 M A A c *<br />

we choose r minr 1 , r 2 , then Bx, r A A A c A A c . However,<br />

x A and x A Bx, r A for this r. **<br />

Hence, we get a contradiction since<br />

Bx, r A by (*)<br />

and<br />

Bx, r A by (**).<br />

That is, intA if A is open.<br />

(2) Suppose that A is closed, then we have M A is open. By (1), we have<br />

intM A .<br />

Note that<br />

M A clM A clM M A<br />

clM A clA<br />

A<br />

. Hence, intA if A is closed.<br />

(b) Give an example in which intA M.<br />

Solution: Let M R 1 ,andA Q, then<br />

A clA clM A clQ clQ c R 1 . Hence, we have R 1 intA M.<br />

3.49 If intA intB and if A is closed in M, then intA B .<br />

Proof: Assume that intA B , then choose x intA B, then there exists<br />

Bx, r A B for some r 0. In addition, since intA , we find that Bx, r A.<br />

Hence, Bx, r B A . It implies Bx, r M A . Choose<br />

y Bx, r M A, then we have<br />

y Bx, r By, 1 Bx, r, where 0 1 r<br />

and<br />

y M A By, 2 M A, forsome 2 0.<br />

Choose min 1 , 2 , then we have<br />

By, Bx, r M A<br />

A B A c<br />

B.


That is, intB which is absurb. Hence, we have intA B .<br />

3.50 Give an example in which intA intB but intA B M.<br />

Solution: Consider the Euclidean sapce R 1 , |. |. LetA Q, andB Q c , then<br />

intA intB but intA B R 1 .<br />

3.51 A clA clM A and A M A.<br />

Proof: By the definition of the boundary of a set, it is clear that<br />

A clA clM A. In addition, A clA clM A, and<br />

M A clM A clM M A clM A clA. Hence, we have<br />

A M A.<br />

Remark: It had better regard the exercise as a formula.<br />

3.52 If clA clB , then A B A B.<br />

Proof: We prove it by two steps.<br />

(1) Let x A B, then for all r 0,<br />

Bx, r A B Bx, r A Bx, r B <br />

and<br />

Bx, r A B c Bx, r A c B c *<br />

Note that at least one of Bx, r A and Bx, r B is not empty. Without loss of<br />

generality, we say Bx, r A . <strong>The</strong>n by (*), we have for all r 0,<br />

Bx, r A , andBx, r A c .<br />

That is, x A. Hence, we have proved A B A B.<br />

(2) Let x A B. Without loss of generality, we let x A. <strong>The</strong>n<br />

Bx, r A , andBx, r A c .<br />

Since Bx, r A , wehave<br />

Bx, r A B Bx, r A Bx, r B . **<br />

Claim that Bx, r A B c Bx, r A c B c . Suppsoe NOT, it means that<br />

Bx, r A c B c . <strong>The</strong>n we have<br />

Bx, r A Bx, r clA<br />

and<br />

Bx, r B Bx, r clB.<br />

It implies that by hypothesis, Bx, r clA clB which is absurb. Hence, we have<br />

proved the claim. We have proved that<br />

Bx, r A B by(**).<br />

and<br />

Bx, r A B c .<br />

That is, x A B. Hence, we have proved A B A B.<br />

From (1) and (2), we have proved that A B A B.<br />

Supplement on a separable metric space<br />

Definition (Base) A collection V of open subsets of X is said to be a base for X if the<br />

following is true: For every x X and every open set G X such that x G, we<br />

have<br />

x V G for some .


In other words, every open set in X is the union of a subcollection of V .<br />

<strong>The</strong>orem Every separable metric space has a countable base.<br />

Proof: Let M, d be a separable metric space with S x 1 ,...,x n ,...<br />

satisfying clS M. Consider a collection Bx i , 1 : i, k N, then given any<br />

k<br />

x M and x G, whereG is open in X, wehaveBx, G for some 0.<br />

Since S is dense in M, we know that there is a set Bx i , 1 for some i, k, such that<br />

k<br />

x Bx i , 1 Bx, G. So, we know that M has a countable base.<br />

k<br />

Corollary R k ,wherek N, has a countable base.<br />

Proof: SinceR k is separable, by <strong>The</strong>orem 1, we know that R k has a countable<br />

base.<br />

<strong>The</strong>orem Every compact metric space is separable.<br />

Proof: LetK, d be a compact metric space, and given a radius 1/n, wehave<br />

p<br />

K i1 B x n i ,1/n .<br />

Let S x n i : i, n N , then it is clear S is countable. In order to show that S is<br />

dense in K, givenx K, we want to show that x is an adherent point of S.<br />

Consider Bx, for any 0, there is a point x n i in S such that<br />

B x n i ,1/n Bx, since 1/n 0. Hence, we have shown that Bx, S .<br />

That is, x clS which implies that K clS. So, we finally have K is separable.<br />

Corollary Every compact metric space has a countable base.<br />

Proof: It is immediately from <strong>The</strong>orem 1.<br />

Remark This corallary can be used to show that Arzela-Ascoli <strong>The</strong>orem.


Limits <strong>And</strong> Continuity<br />

Limits of sequence<br />

4.1 Prove each of the following statements about sequences in C.<br />

(a) z n 0if|z| 1; z n diverges if |z| 1.<br />

Proof: For the part: z n 0if|z| 1. Given 0, we want to find that there exists a<br />

positive integer N such that as n N, wehave<br />

|z n 0| .<br />

Note that log|z| 0since|z| 1, hence if we choose a positive integer N log |z|<br />

1,<br />

then as n N, wehave<br />

|z n 0| .<br />

For the part: z n diverges if |z| 1. Assume that z n converges to L, thengiven<br />

1, there exists a positive integer N 1 such that as n N 1 ,wehave<br />

|z n L| 1 <br />

|z| n 1 |L|. *<br />

However, note that log|z| 0since|z| 1, if we choose a positive integer<br />

N max log |z|<br />

1 |L| 1, N 1 , then we have<br />

|z| N 1 |L|<br />

which contradicts (*). Hence, z n diverges if |z| 1.<br />

Remark: 1. Given any complex number z C 0, lim n|z| 1/n 1.<br />

2. Keep lim n n! 1/n in mind.<br />

3. In fact, z n is unbounded if |z| 1. ( z n diverges if |z| 1. ) Since given<br />

M 1, and choose a positive integer N log |z|<br />

M 1, then |z| N M.<br />

(b) If z n 0andifc n is bounded, then c n z n 0.<br />

Proof: Since c n is bounded, say its bound M, i.e., |c n| M for all n N. In<br />

addition, since z n 0, given 0, there exists a positive integer N such that as n N,<br />

we have<br />

|z n 0| /M<br />

which implies that as n N, wehave<br />

|c n z n| M|z n| .<br />

That is, lim n c n z n 0.<br />

(c) z n /n! 0 for every complex z.<br />

Proof: Given a complex z, and thus find a positive integer N such that |z| N/2.<br />

Consider (let n N).<br />

z n<br />

z N<br />

z nN<br />

n! N! N 1N 2 n<br />

Hence, z n /n! 0 for every complex z.<br />

<br />

zN<br />

N!<br />

1<br />

2<br />

nN<br />

0asn .<br />

Remark: <strong>The</strong>re is another proof by using the fact <br />

n1<br />

a n converges which implies<br />

z<br />

a n 0. Since n<br />

n1<br />

converges by ratio test for every complex z, then we have<br />

n!


z n /n! 0 for every complex z.<br />

(d) If a n n 2 2 n, then a n 0.<br />

Proof: Since<br />

0 a n n 2 2 n 2<br />

n 2 2 n 1 n for all n N,<br />

and lim n 1/n 0, we have a n 0asn by Sandwich <strong>The</strong>orem.<br />

4.2 If a n2 a n1 a n /2 for all n 1, show that a n a 1 2a 2 /3. Hint:<br />

a n2 a n1 1 2 a n a n1 .<br />

Proof: Since a n2 a n1 a n /2 for all n 1, we have b n1 b n /2, where<br />

b n a n1 a n . So, we have<br />

n<br />

b n1 1 b1 0asn . *<br />

2<br />

Consider<br />

which implies that<br />

So we have<br />

n1<br />

a n2 a 2 b k 1<br />

2 n<br />

k2<br />

k1<br />

b n <br />

3a n1<br />

2<br />

b k <br />

a 1 2a 2<br />

2<br />

a n a 1 2a 2 /3 by (*).<br />

1<br />

2<br />

a n1 a 1 <br />

.<br />

4.3 If 0 x 1 1andifx n1 1 1 x n for all n 1, prove that x n is a<br />

decreasing sequence with limit 0. Prove also that x n1 /x n 1 2 .<br />

Proof: Claim that 0 x n 1 for all n N. We prove the claim by Mathematical<br />

Induction. As n 1, there is nothing to prove. Suppose that n k holds, i.e., 0 x k 1,<br />

then as n k 1, we have<br />

0 x k1 1 1 x k 1 by induction hypothesis.<br />

So, by Mathematical Induction, we have proved the claim. Use the claim, and then we<br />

have<br />

x<br />

x n1 x n 1 x n 1 x n n x n 1<br />

1 x n 2 1 x n 0since0 x n 1.<br />

So, we know that the sequence x n is a decreasing sequence. Since 0 x n 1 for all<br />

n N, by Completeness of R, (Thatis,a monotonic sequence in R which is bounded is<br />

a convergent sequence.) Hence, we have proved that x n is a convergent sequence,<br />

denoted its limit by x. Note that since<br />

x n1 1 1 x n for all n N,<br />

we have x lim n x n1 lim n 1 1 x n 1 1 x which implies xx 1 0.<br />

Since x n is a decreasing sequence with 0 x n 1 for all n N, we finally have x 0.<br />

For proof of x n1 /x n 1 .Since 2<br />

1 1 x<br />

lim<br />

x0 x 1 2<br />

then we have<br />

*


x n1<br />

x n<br />

1 1 x n<br />

x n<br />

1 2 .<br />

Remark: In (*), it is the derivative of 1 1 x at the point x 0. Of course, we can<br />

prove (*) by L-Hospital Rule.<br />

4.4 Two sequences of positive integers a n and b n are defined recursively by taking<br />

a 1 b 1 1 and equating rational and irrational parts in the equation<br />

a n b n 2 a n1 b n1 2 2 for n 2.<br />

Prove that a n2 2b n2 1 for all n 2. Deduce that a n /b n 2 through values 2 ,and<br />

that 2b n /a n 2 through values 2 .<br />

Proof: Note a n b n 2 a n1 b n1 2 2 for n 2, we have<br />

a n a2 n1 2b2 n1 for n 2, and<br />

b n 2a n1 b n1 for n 2<br />

since if A, B, C, andD N with A B 2 C D 2 , then A C, andB D.<br />

Claim that a n2 2b n2 1 for all n 2. We prove the claim by Mathematical<br />

Induction. As n 2, we have by (*)<br />

a 22 2b 22 a 12 2b 12 2 22a 1 b 1 2 1 2 2 22 2 1. Suppose that as n k 2<br />

holds, i.e., a k2 2b k2 1, then as n k 1, we have by (*)<br />

a2 k1 2b2 k1 a k2 2b k2 2 22a k b k 2<br />

a k4 4b k4 4a k2 b2<br />

k<br />

a k2 2b k2 2<br />

1 by induction hypothesis.<br />

Hence, by Mathematical Induction, we have proved the claim. Note that a n2 2b n2 1for<br />

all n 2, we have<br />

*<br />

2 2<br />

a n 1 2 2<br />

b n bn<br />

and<br />

2<br />

2b n<br />

a n 2 2 2.<br />

a2 n<br />

a<br />

Hence, lim n<br />

1<br />

n b n<br />

2 by lim n b n<br />

0 from (*) through values 2 ,and<br />

2b<br />

lim n<br />

1 n a n<br />

2 by lim n an<br />

0 from (*) through values 2 .<br />

Remark: From (*), we know that a n and b n is increasing since a n N and<br />

b n N. That is, we have lim n a n , and lim n b n .<br />

4.5 A real sequence x n satisfies 7x n1 x n3 6forn 1. If x 1 1 , prove that the<br />

2<br />

sequence increases and find its limit. What happens if x 1 3 or if x 2 1 5 2<br />

Proof: Claim that if x 1 1 ,then0 x 2 n 1 for all n N. We prove the claim by<br />

Mathematical Induction. Asn 1, 0 x 1 1 1. Suppose that n k holds, i.e.,<br />

2<br />

0 x k 1, then as n k 1, we have<br />

0 x k1 x k 3 6<br />

<br />

7<br />

1 7<br />

6 1 by induction hypothesis.<br />

Hence, we have prove the claim by Mathematical Induction. Since<br />

x 3 7x 6 x 3x 1x 2, then


x n1 x n x n 3 6<br />

x<br />

7 n<br />

x n 3 7x n 6<br />

7<br />

0since0 x n 1 for all n N.<br />

It means that the sequence x n (strictly) increasing. Since x n is bounded, by<br />

completeness of R, we know that he sequence x n is convergent, denote its limit by x.<br />

Since<br />

x<br />

x lim n<br />

x n1 lim n3 6<br />

n<br />

<br />

7<br />

x3 6 ,<br />

7<br />

we find that x 3, 1, or 2. Since 0 x n 1 for all n N, we finally have x 1.<br />

Claim that if x 1 3 ,then1 x 2 n 2 for all n N. We prove the claim by<br />

Mathematical Induction. Asn 1, there is nothing to prove. Suppose n k holds, i.e.,<br />

1 x k 2, then as n k 1, we have<br />

1 1 6 x k1 x k 3 6<br />

23 6 2.<br />

7<br />

7 7<br />

Hence, we have prove the claim by Mathematical Induction. Since<br />

x 3 7x 6 x 3x 1x 2, then<br />

x n1 x n x n 3 6<br />

x<br />

7 n<br />

x n 3 7x n 6<br />

7<br />

0since1 x n 2 for all n N.<br />

It means that the sequence x n (strictly) decreasing. Since x n is bounded, by<br />

completeness of R, we know that he sequence x n is convergent, denote its limit by x.<br />

Since<br />

x<br />

x lim n<br />

x n1 lim n3 6<br />

n<br />

<br />

7<br />

x3 6 ,<br />

7<br />

we find that x 3, 1, or 2. Since 1 x n 2 for all n N, we finally have x 1.<br />

Claim that if x 1 5 , then x 2 n 5 for all n N. We prove the claim by<br />

2<br />

Mathematical Induction. Asn 1, there is nothing to prove. Suppose n k holds, i.e.,<br />

x k 5 , then as n k 1,<br />

2<br />

x k1 x k 3 6<br />

5 2 3 6<br />

<br />

7 7<br />

173<br />

56 3 5 2 .<br />

Hence, we have proved the claim by Mathematical Induction. Ifx n was convergent,<br />

say its limit x. <strong>The</strong>n the possibilities for x 3, 1, or 2. However, x n 5 for all n N.<br />

2<br />

So, x n diverges if x 1 5 . 2<br />

Remark: Note that in the case x 1 5/2, we can show that x n is increasing by the<br />

same method. So, it implies that x n is unbounded.<br />

4.6 If |a n| 2and|a n2 a n1 | 1 8 |a 2<br />

n1 a n2 | for all n 1, prove that a n <br />

converges.<br />

Proof: Let a n1 a n b n , then we have |b n1 | 1 8 |b n||a n1 a n| 1 2 |b n|, since<br />

|a n| 2 for all n 1. So, we have |b n1 | 1 2 n |b 1 |. Consider (Let m n)


|a m a n| |a m a m1 a m1 a m2 ...a n1 a n |<br />

|b m1 | ...|b n|<br />

|b 1 | 1<br />

2<br />

m2<br />

... 1<br />

2<br />

n1<br />

, *<br />

then a n is a Cauchy sequence since 1 2 k converges. Hence, we know that a n is a<br />

convergent sequence.<br />

Remark: In this exercise, we use the very important theorem, every Cauchy sequence<br />

in the Euclidean space R k is convergent.<br />

4.7 In a metric space S, d, assume that x n x and y n y. Prove that<br />

dx n , y n dx, y.<br />

Proof: Since x n x and y n y, given 0, there exists a positive integer N such<br />

that as n N, wehave<br />

dx n , x /2 and dy n , y /2.<br />

Hence, as n N, wehave<br />

|dx n , y n dx, y| |dx n , x dy n , y|<br />

dx n , x dy n , y /2 /2<br />

.<br />

So, it means that dx n , y n dx, y.<br />

4.8 Prove that in a compact meric space S, d, every sequence in S has a subsequence<br />

which converges in S. This property also implies that S is compact but you are not required<br />

to prove this. (For a proof see either Reference 4.2 or 4.3.)<br />

Proof: Given a sequence x n S, and let T x 1 , x 2 ,.... If the range of T is finite,<br />

there is nothing to prove. So, we assume that the range of T is infinite. Since S is compact,<br />

and T S, wehaveT has a accumulation point x in S. So, there exists a point y n in T such<br />

that By n , x 1 n . It implies that y n x. Hence, we have proved that every sequence in S<br />

has a subsequence which converges in S.<br />

Remark: If every sequence in S has a subsequence which converges in S, then S is<br />

compact. We give a proof as follows.<br />

Proof: In order to show S is compact, it suffices to show that every infinite subset of S<br />

has an accumulation point in S. Given any infinite subset T of S, and thus we choose<br />

x n T (of course in S). By hypothesis, x n has a subsequence x kn which converges<br />

in S, say its limit x. From definition of limit of a sequence, we know that x is an<br />

accumulation of T. So,S is compact.<br />

4.9 Let A beasubsetofametricspaceS. IfA is complete, prove that A is closed. Prove<br />

that the converse also holds if S is complete.<br />

Proof: Let x be an accumulation point of A, then there exists a sequence x n such that<br />

x n x. Sincex n is convergent, we know that x n is a Cauchy sequence. <strong>And</strong> A is<br />

complete, we have x n converges to a point y A. By uniqueness, we know x y A.<br />

So, A contains its all accumulation points. That is, A is closed.<br />

Suppose that S is complete and A is closed in S. Given any Cauchy sequence<br />

x n A, we want to show x n is converges to a point in A. Trivially, x n is also a<br />

Cauchy sequence in S. SinceS is complete, we know that x n is convergent to a point x in<br />

S. By definition of limit of a sequence, it is easy to know that x is an adherent point of A.<br />

So, x A since A is closed. That is, every Cauchy sequence in A is convergent. So, A is


complete.<br />

Supplement<br />

1. Show that the sequence<br />

lim<br />

n<br />

2n!!<br />

2n 1!! 0.<br />

Proof: Let a and b be positive integers satisfying a b 1. <strong>The</strong>n we have<br />

a!b a!b! a b! ab!. *<br />

So, if we let fn 2n!, then we have, by (*)<br />

2n!!<br />

2n 1!! fn!<br />

fn2n 1! 1<br />

2n 1 0.<br />

Hence, we know that lim n<br />

2. Show that<br />

2n!!<br />

2n1!!<br />

0.<br />

a n 1 1 n 1 2 n 1 n n e 1/2 as n ,<br />

where x means Gauss Symbol.<br />

Proof: Since<br />

x 1 2 x2 log1 x x, for all x 1, 1<br />

we have<br />

k n <br />

<br />

k1<br />

k<br />

n 1 2<br />

k<br />

n<br />

k n <br />

2<br />

log an log 1 n<br />

k<br />

k1<br />

k n <br />

<br />

k1<br />

Consider i 2 n i 1 2 , then by Sandwish <strong>The</strong>orem, we know that<br />

lim n<br />

log a n 1/2<br />

which implies that a n e 1/2 as n .<br />

3. Show that n! 1/n n for all n N. ( n! 1/n as n .)<br />

Proof: We prove it by a special method following Gauss’ method. Consider<br />

n! 1 k n<br />

n n k 1 1<br />

and thus let fk : kn k 1, it is easy to show that fk f1 n for all<br />

k 1, 2, . . . , n. So, we have prove that<br />

n! 2 n n<br />

which implies that<br />

n! 1/n n .<br />

Remark: <strong>The</strong>re are many and many method to show n! 1/n as n . We do not<br />

give a detail proofs about it. But We method it as follows as references.<br />

(a) By A. P. G. P. , we have<br />

k1<br />

n 1<br />

k<br />

n 1<br />

n!<br />

1/n<br />

k<br />

n


and use the fact if a n converges to a, thensois<br />

n<br />

a k k1<br />

n .<br />

(b) Use the fact, by Mathematical Induction, n! 1/n n/3 for all n.<br />

(c) Use the fact, A n /n! 0asn for any real A.<br />

(d) Consider pn n!<br />

n<br />

1/n , and thus taking log pn.<br />

n<br />

(e) Use the famuos formula, a n are positive for all n.<br />

lim inf a n1<br />

a n<br />

lim infa n 1/n lim supa n 1/n lim sup a n1<br />

a n<br />

and let a n n!<br />

n<br />

. n x<br />

(f) <strong>The</strong> radius of the power series k<br />

k0<br />

is .<br />

k!<br />

(g) Ue the fact, 1 1/n n e 1 1/n n1 , then en n e n n! en n1 e n .<br />

(h) More.<br />

Limits of functions<br />

Note. In Exercise 4.10 through 4.28, all functions are real valued.<br />

4.10 Let f be defined on an opne interval a, b and assume x a, b. Consider the<br />

two statements<br />

(a) lim h0 |fx h fx| 0;<br />

(b) lim h0 |fx h fx h| 0.<br />

Prove that (a) always implies (b), and give an example in which (b) holds but (a) does<br />

not.<br />

Proof: (a) Since<br />

lim|fx h fx| 0 lim|fx h fx| 0,<br />

h0 h0<br />

we consider<br />

|fx h fx h|<br />

|fx h fx fx fx h|<br />

|fx h fx| |fx fx h| 0ash 0.<br />

So, we have<br />

lim|fx h fx h| 0.<br />

h0<br />

(b) Let<br />

fx <br />

|x| if x 0,<br />

1ifx 0.<br />

<strong>The</strong>n<br />

lim|f0 h f0 h| 0,<br />

h0<br />

but<br />

lim|f0 h f0| lim||h| 1| 1.<br />

h0 h0<br />

So, (b) holds but (a) does not.<br />

Remark: In case (b), there is another example,


1/|x| if x 0,<br />

gx <br />

0ifx 0.<br />

<strong>The</strong> difference of two examples is that the limit of |g0 h g0| does not exist as h<br />

tends to 0.<br />

4.11 Let f be defined on R 2 .If<br />

lim<br />

x,ya,b<br />

fx, y L<br />

and if the one-dimensional lim xa fx, y and lim yb fx, y both exist, prove that<br />

lim xa<br />

lim<br />

yb<br />

fx, y<br />

lim<br />

yb<br />

lim xa<br />

fx, y L.<br />

Proof: Since lim x,ya,b fx, y L, thengiven 0, there exists a 0 such that as<br />

0 |x, y a, b| , wehave<br />

|fx, y L| /2,<br />

which implies<br />

lim|fx, y L| <br />

yb<br />

which implies<br />

limfx, y L<br />

yb<br />

/2 if 0 |x, y a, b| <br />

lim xa<br />

limfx, y L /2 if 0 |x, y a, b| .<br />

yb<br />

Hence, we have proved lim xa|lim yb x, y L| /2 . Since is arbitrary, we have<br />

which implies that<br />

lim xa<br />

limfx, y L 0<br />

yb<br />

lim xa<br />

limfx, y L 0.<br />

yb<br />

So, lim xa lim yb fx, y L. <strong>The</strong> proof of lim yb lim xa fx, y L is similar.<br />

Remark: 1. <strong>The</strong> exercise is much important since in mathematics, we would encounter<br />

many and many similar questions about the interchange of the order of limits. So, we<br />

should keep the exercise in mind.<br />

2. In the proof, we use the concept: |lim xa fx| 0if,andonlyiflim xa fx 0.<br />

3. <strong>The</strong> hypothesis fx, y L as x, y a, b tells us that every approach form these<br />

points x, y to the point a, b, fx, y approaches to L. Use this concept, and consider the<br />

special approach from points x, y to x, b and thus from x, b to a, b. Note that since<br />

lim yb fx, y exists, it means that we can regrad this special approach as one of approaches<br />

from these points x, y to the point a, b. So, it is natural to have the statement.<br />

4. <strong>The</strong> converse of statement is not necessarily true. For example,<br />

x y if x 0ory 0<br />

fx, y <br />

1 otherwise.<br />

Trivially, we have the limit of fx, y does not exist as x, y 0, 0. However,


lim<br />

y0<br />

fx, y <br />

0ifx 0,<br />

1ifx 0.<br />

and lim<br />

x0<br />

fx, y <br />

0ify 0,<br />

1ify 0.<br />

lim<br />

x0<br />

lim<br />

y0<br />

fx, y<br />

lim<br />

y0<br />

limfx, y 1.<br />

x0<br />

In each of the preceding examples, determine whether the following limits exist and<br />

evaluate those limits that do exist:<br />

lim<br />

x0<br />

limfx, y ; lim<br />

y0 y0<br />

limfx, y ;<br />

x0<br />

lim fx, y.<br />

x,y0,0<br />

Now consider the functions f defined on R 2 as follows:<br />

(a) fx, y x2 y 2<br />

if x, y 0, 0, f0, 0 0.<br />

x 2 y 2<br />

Proof: 1. Since(x 0)<br />

we have<br />

2. Since (y 0)<br />

we have<br />

x<br />

limfx, y lim<br />

2 y 2<br />

x0 x0 x 2 y 2<br />

lim<br />

y0<br />

x<br />

limfx, y lim<br />

2 y 2<br />

y0 y0 x 2 y 2<br />

lim<br />

x0<br />

<br />

limfx, y 1.<br />

x0<br />

<br />

limfx, y 1.<br />

y0<br />

y 2<br />

1 ify 0,<br />

y 2 1ify 0,<br />

x 2<br />

1ifx 0,<br />

x 2 1ifx 0,<br />

3. (x, y 0, 0) Letx r cos and y r sin , where 0 2, and note that<br />

x, y 0, 0 r 0. <strong>The</strong>n<br />

lim fx, y lim x 2 y 2<br />

x,y0,0 x,y0,0 x 2 y 2<br />

lim<br />

r0<br />

r 2 cos 2 sin 2 <br />

r 2<br />

cos 2 sin 2 .<br />

So, if we choose and /2, we find the limit of fx, y does not exist as<br />

x, y 0, 0.<br />

Remark: 1. This case shows that<br />

1 lim<br />

x0<br />

lim<br />

y0<br />

fx, y<br />

lim<br />

y0<br />

lim<br />

x0<br />

fx, y<br />

1<br />

2. Obviously, the limit of fx, y does not exist as x, y 0, 0. Since if it was, then<br />

by (*), (**), and the preceding theorem, we know that<br />

which is absurb.<br />

lim<br />

x0<br />

lim<br />

y0<br />

fx, y<br />

lim<br />

y0<br />

lim<br />

x0<br />

fx, y<br />

**


xy<br />

(b) fx, y <br />

2<br />

if x, y 0, 0, f0, 0 0.<br />

xy 2 xy 2<br />

Proof: 1. Since (x 0)<br />

xy 2<br />

limfx, y lim<br />

0 for all y,<br />

x0 x0<br />

xy 2 x y 2<br />

we have<br />

2. Since (y 0)<br />

we have<br />

lim<br />

y0<br />

limfx, y 0.<br />

x0<br />

xy 2<br />

limfx, y lim<br />

0 for all x,<br />

y0 y0<br />

xy 2 x y 2<br />

lim<br />

x0<br />

limfx, y 0.<br />

y0<br />

3. (x, y 0, 0) Letx r cos and y r sin , where 0 2, and note that<br />

x, y 0, 0 r 0. <strong>The</strong>n<br />

xy 2<br />

fx, y <br />

xy 2 x y 2<br />

So,<br />

r 4 cos 2 sin 2 <br />

r 4 cos 2 sin 2 r 2 2r 2 cossin <br />

cos 2 sin 2 .<br />

cos 2 sin 2 12cossin<br />

r 2<br />

0ifr 0<br />

fx, y<br />

1 if /4 or 5/4.<br />

Hence, we know that the limit of fx, y does not exists as x, y 0, 0.<br />

(c) fx, y 1 x sinxy if x 0, f0, y y.<br />

Proof: 1. Since (x 0)<br />

limfx, y lim 1<br />

x0 x0 x sinxy y *<br />

we have<br />

2. Since (y 0)<br />

we have<br />

limfx, y <br />

y0<br />

lim<br />

y0<br />

lim<br />

x0<br />

limfx, y 0.<br />

x0<br />

1<br />

lim y0 x sinxy 0ifx 0,<br />

lim y0 y 0ifx 0,<br />

limfx, y 0.<br />

y0<br />

3. (x, y 0, 0) Letx r cos and y r sin , where 0 2, and note that<br />

x, y 0, 0 r 0. <strong>The</strong>n


fx, y <br />

1<br />

r cos sinr2 cossin if x r cos 0,<br />

r sin if x r cos 0.<br />

0ifr 0,<br />

<br />

0ifr 0.<br />

So, we know that lim x,y0,0 fx, y 0.<br />

Remark: In (*) and (**), we use the famuos limit, that is,<br />

lim sin x<br />

x0 x 1.<br />

<strong>The</strong>re are some similar limits, we write them without proofs.<br />

(a) lim t t sin1/t 1.<br />

(b) lim x0 x sin1/x 0.<br />

(c) lim x0 ,ifb 0.<br />

(d) fx, y <br />

sinax<br />

sinbx<br />

a b<br />

x y sin1/x sin1/y if x 0andy 0,<br />

0 if x 0ory 0.<br />

**<br />

Proof: 1. Since (x 0)<br />

x y sin1/x sin1/y x sin1/x sin1/y y sin1/x sin1/y if y 0<br />

fx, y <br />

0ify 0<br />

we have if y 0, the limit fx, y does not exist as x 0, and if y 0, lim x0 fx, y 0.<br />

Hence, we have (x 0, y 0)<br />

2. Since (y 0)<br />

lim<br />

y0<br />

lim<br />

x0<br />

fx, y<br />

does not exist.<br />

x y sin1/x sin1/y x sin1/x sin1/y y sin1/x sin1/y if x 0<br />

fx, y <br />

0ifx 0<br />

we have if x 0, the limit fx, y does not exist as y 0, and if x 0, lim y0 fx, y 0.<br />

Hence, we have (x 0, y 0)<br />

3. (x, y 0, 0) Consider<br />

we have<br />

lim<br />

x0<br />

|fx, y| <br />

lim<br />

y0<br />

fx, y<br />

does not exist.<br />

|x y| if x 0andy 0,<br />

0ifx 0ory 0.<br />

lim fx, y 0.<br />

x,y0,0<br />

sinxsiny<br />

tan xtan y<br />

,iftanx tan y,<br />

(e) fx, y <br />

cos 3 x if tan x tan y.<br />

Proof: Since we consider the three approaches whose tend to 0, 0, we may assume<br />

that x, y /2, /2. and note that in this assumption, x y tan x tan y. Consider<br />

1. (x 0)


So,<br />

So,<br />

2. (y 0)<br />

lim<br />

x0<br />

fx, y <br />

lim<br />

y0<br />

fx, y <br />

sinxsiny<br />

lim x0 tan xtan y<br />

cosy if x y.<br />

1 if x y.<br />

lim<br />

y0<br />

limfx, y 1.<br />

x0<br />

sinxsiny<br />

lim y0 tan xtan y<br />

cosx if x y.<br />

cos 3 x if x y.<br />

lim<br />

x0<br />

limfx, y 1.<br />

y0<br />

3. Let x r cos and y r sin , where 0 2, and note that<br />

x, y 0, 0 r 0. <strong>The</strong>n<br />

lim<br />

lim fx, y <br />

x,y0,0<br />

sinr cos sinr sin<br />

r0 if cos sin .<br />

tanr cos tanr sin<br />

lim r0 cos 3 r cos if cos sin .<br />

1ifcos sin , byL-Hospital Rule.<br />

<br />

1ifcos sin .<br />

So, we know that lim x,y0,0 fx, y 1.<br />

and<br />

then<br />

So,<br />

Remark: 1. <strong>The</strong>re is another proof about (e)-(3). Consider<br />

x y<br />

sin x sin y 2cos sin<br />

x y<br />

2 2<br />

lim<br />

lim fx, y <br />

x,y0,0<br />

That is, lim x,y0,0 fx, y 1.<br />

tan x tan y sin x cosx sin y<br />

cosy ,<br />

sin x sin y<br />

tan x tan y<br />

xy<br />

cos<br />

2<br />

cosxcosy<br />

cos xy .<br />

2<br />

x,y0,0<br />

cos<br />

xy<br />

cos xcos y<br />

2<br />

cos<br />

xy<br />

2<br />

lim x,y0,0 cos 3 x 1ifx y.<br />

1ifx y,<br />

2. In the process of proof, we use the concept that we write it as follows. Since its proof<br />

is easy, we omit it. If<br />

or<br />

L<br />

lim fx, y <br />

x,ya,b<br />

if x y<br />

L if x y


then we have<br />

lim fx, y L if x 0andy 0,<br />

x,ya,b L if x 0ory 0.<br />

lim fx, y L.<br />

x,ya,b<br />

4.12 If x 0, 1 prove that the following limit exists,<br />

lim<br />

m<br />

lim n cos2n m!x ,<br />

and that its value is 0 or 1, according to whether x is irrational or rational.<br />

Hence,<br />

Proof: If x is rational, say x q/p, whereg. c. d. q, p 1, then p!x N. So,<br />

lim n<br />

cos 2n m!x <br />

lim<br />

m<br />

1ifm p,<br />

0ifm p.<br />

lim n cos2n m!x 1.<br />

If x is irrational, then m!x N for all m N. So, cos 2n m!x 1 for all irrational x.<br />

Hence,<br />

lim n<br />

cos 2n m!x 0 lim m<br />

Continuity of real-valued functions<br />

lim n<br />

cos 2n m!x 0.<br />

4.13 Let f be continuous on a, b and let fx 0 when x is rational. Prove that<br />

fx 0 for every x a, b.<br />

Proof: Given any irrational number x in a, b, and thus choose a sequence x n Q<br />

such that x n x as n . Note that fx n 0 for all n. Hence,<br />

0 lim n<br />

0<br />

lim n<br />

fx n <br />

f lim n<br />

x n by continuity of f at x<br />

fx.<br />

Since x is arbitrary, we have shown fx 0 for all x a, b. Thatis,f is constant 0.<br />

Remark: Here is another good exercise, we write it as a reference. Let f be continuous<br />

on R, andiffx fx 2 , then f is constant.<br />

Proof: Since fx f x 2 fx 2 fx, we know that f is an even function. So,<br />

in order to show f is constant on R, it suffices to show that f is constant on 0, . Given<br />

any x 0, , sincefx 2 fx for all x R, wehavefx 1/2n fx for all n. Hence,<br />

fx lim n<br />

fx<br />

lim n<br />

fx 1/2n <br />

f lim n<br />

x 1/2n by continuity of f at 1<br />

f1 since x 0.<br />

So, we have fx f1 : c for all x 0, . In addition, given a sequence<br />

x n 0, such that x n 0, then we have


c lim n<br />

c<br />

lim n<br />

fx n <br />

f lim n<br />

x n by continuity of f at 0<br />

f0<br />

From the preceding, we have proved that f is constant.<br />

4.14 Let f be continuous at the point a a 1 , a 2 ,...,a n R n . Keep a 2 , a 3 ,...,a n fixed<br />

and define a new function g of one real variable by the equation<br />

gx fx, a 2 ,...,a n .<br />

Prove that g is continuous at the point x a 1 . (This is sometimes stated as follows: A<br />

continuous function of n variables is continuous in each variable separately.)<br />

Proof: Given 0, there exists a 0 such that as y Ba; D, whereD is a<br />

domain of f, wehave<br />

|fy fa| . *<br />

So, as |x a 1 | , which implies |x, a 2 ,...,a n a 1 , a 2 ,...,a n | , wehave<br />

|gx ga 1 | |fx, a 2 ,...,a n fa 1 , a 2 ,...,a n | .<br />

Hence, we have proved g is continuous at x a 1<br />

Remark: Here is an important example like the exercise, we write it as follows. Let<br />

j : R n R n ,and j : x 1 , x 2 ,...,x n 0,.,x j ,..,0. <strong>The</strong>n j is continuous on R n for<br />

1 j n. Note that j is called a projection. Note that a projection P is sometimes<br />

defined as P 2 P.<br />

Proof: Given any point a R n ,and given 0, and choose , then as<br />

x Ba; , wehave<br />

| j x j a| |x j a j | x a for each 1 j n<br />

Hence, we prove that j x is continuous on R n for 1 j n.<br />

4.15 Show by an example that the converse of statement in Exercise 4.14 is not true in<br />

general.<br />

Proof: Let<br />

fx, y <br />

x y if x 0ory 0<br />

1otherwise.<br />

Define g 1 x fx,0 and g 2 y f0, y, then we have<br />

limg 1 x 0 g 1 0<br />

x0<br />

and<br />

limg 2 y 0 g 2 0.<br />

y0<br />

So, g 1 x and g 2 y are continuous at 0. However, f is not continuous at 0, 0 since<br />

limfx, x 1 0 f0, 0.<br />

x0<br />

Remark: 1. For continuity, if f is continuous at x a, then it is NOT necessary for us<br />

to have<br />

lim xa<br />

fx fa


this is because a can be an isolated point. However, if a is an accumulation point, we then<br />

have<br />

f is continuous at a if, and only if, lim xa<br />

fx fa.<br />

4.16 Let f, g, andh be defined on 0, 1 as follows:<br />

fx gx hx 0, whenever x is irrational;<br />

fx 1andgx x, whenever x is rational;<br />

hx 1/n, ifx is the rational number m/n (in lowest terms);<br />

h0 1.<br />

Prove that f is not continuous anywhere in 0, 1, that g is continuous only at x 0, and<br />

that h is continuous only at the irrational points in 0, 1.<br />

Proof: 1. Write<br />

0ifx R Q 0, 1,<br />

fx <br />

1ifx Q 0, 1.<br />

Given any x R Q 0, 1, andy Q 0, 1, and thus choose x n Q 0, 1<br />

such that x n x, andy n R Q 0, 1 such that y n y. Iff is continuous at x,<br />

then<br />

1 lim n<br />

fx n <br />

f lim n<br />

x n by continuity of f at x<br />

fx<br />

0<br />

which is absurb. <strong>And</strong> if f is continuous at y, then<br />

0 lim n<br />

fy n <br />

f lim n<br />

y n by continuity of f at y<br />

fy<br />

1<br />

which is absurb. Hence, f is not continuous on 0, 1.<br />

2. Write<br />

0ifx R Q 0, 1,<br />

gx <br />

x if x Q 0, 1.<br />

Given any x R Q 0, 1, and choose x n Q 0, 1 such that x n x. <strong>The</strong>n<br />

x<br />

lim n<br />

x n<br />

lim n<br />

gx n <br />

limg lim n<br />

x n by continuity of g at x<br />

gx<br />

0<br />

which is absurb since x is irrational. So, f is not continous on R Q 0, 1.<br />

Given any x Q 0, 1, and choose x n R Q 0, 1 such that x n x. Ifg is


continuous at x, then<br />

0<br />

lim n<br />

gx n <br />

g lim n<br />

x n by continuity of f at x<br />

gx<br />

x.<br />

So, the function g may be continuous at 0. In fact, g is continuous at 0 which prove as<br />

follows. Given 0, choose , as|x| , wehave<br />

|gx g0| |gx| |x| . So,g is continuous at 0. Hence, from the preceding,<br />

we know that g is continuous only at x 0.<br />

3. Write<br />

1ifx 0,<br />

hx 0ifx R Q 0, 1,<br />

1/n if x m/n, g. c. d. m, n 1.<br />

Consider a 0, 1 and given 0, there exists the largest positive integer N such that<br />

N 1/. LetT x : hx , then<br />

0, 1 x : hx 1 x : hx 1/2...x : hx 1/N if 1,<br />

T <br />

if 1.<br />

Note that T is at most a finite set, and then we can choose a 0 such that<br />

a , a acontains no points of T and a , a 0, 1. So,if<br />

x a , a a, wehavehx . It menas that<br />

lim xa<br />

hx 0.<br />

Hence, we know that h is continuous at x 0, 1 R Q. For two points x 1, and<br />

y 0, it is clear that h is not continuous at x 1, and not continuous at y 1bythe<br />

method mentioned in the exercise of part 1 and part 2. Hence, we have proved that h is<br />

continuous only at the irrational points in 0, 1.<br />

Remark: 1. Sometimes we call f Dirichlet function.<br />

2. Here is another proof about g, we write it down to make the reader get more.<br />

Proof: Write<br />

0ifx R Q 0, 1,<br />

gx <br />

x if x Q 0, 1.<br />

Given a 0, 1, andifg is continuous at a, then given 0 a, there exists a 0<br />

such that as x a , a 0, 1, wehave<br />

|gx ga| .<br />

If a R Q, choose 0 so that a Q. <strong>The</strong>n a a , a which<br />

implies |ga ga| |ga | a a. But it is impossible.<br />

If a Q, choose 0 so that a R Q. a a , a which<br />

implies |ga ga| |a| a a. But it is impossible.<br />

If a 0, given 0 and choose , thenas0 x , wehave<br />

|gx g0| |gx| |x| x . It means that g is continuous at 0.<br />

4.17 For each x 0, 1, letfx x if x is rational, and let fx 1 x if x is


irrational. Prove that:<br />

(a) ffx x for all x in 0, 1.<br />

Proof: If x is rational, then ffx fx x. <strong>And</strong>ifx is irrarional, so is<br />

1 x 0, 1. <strong>The</strong>n ffx f1 x 1 1 x x. Hence, ffx x for all x in<br />

0, 1.<br />

(b) fx f1 x 1 for all x in 0, 1.<br />

Proof: If x is rational, so is 1 x 0, 1. <strong>The</strong>n fx f1 x x 1 x 1. <strong>And</strong><br />

if x is irrarional, so is 1 x 0, 1. <strong>The</strong>n fx f1 x 1 x 1 1 x 1.<br />

Hence, fx f1 x 1 for all x in 0, 1.<br />

(c) f is continuous only at the point x 1 . 2<br />

Proof: If f is continuous at x, then choose x n Q and y n Q c such that x n x,<br />

and y n x. <strong>The</strong>n we have, by continuity of f at x,<br />

fx f lim n<br />

x n<br />

lim n<br />

fx n lim n<br />

x n x<br />

and<br />

fx f lim n<br />

y n lim n<br />

fy n lim n<br />

1 y n 1 x.<br />

So, x 1/2 is the only possibility for f. Given 0, we want to find a 0 such that as<br />

x 1/2 ,1/2 0, 1, wehave<br />

|fx f1/2| |fx 1/2| .<br />

Choose 0 so that 1/2 ,1/2 0, 1, then as<br />

x 1/2 ,1/2 0, 1, wehave<br />

|fx 1/2| |x 1/2| if x Q,<br />

|fx 1/2| |1 x 1/2| |1/2 x| if x Q c .<br />

Hence, we have proved that f is continuous at x 1/2.<br />

(d) f assumes every value between 0 and 1.<br />

Proof: Given a 0, 1, wewanttofindx 0, 1 such that fx a. Ifa Q, then<br />

choose x a, wehavefx a a. Ifa R Q, then choose x 1 a R Q, we<br />

have fx 1 a 1 1 a a.<br />

Remark: <strong>The</strong> range of f on 0, 1 is 0, 1. In addition, f is an one-to-one mapping<br />

since if fx fy, then x y. (<strong>The</strong> proof is easy, just by definition of 1-1, so we omit it.)<br />

(e) fx y fx fy is rational for all x and y in 0, 1.<br />

Proof: We prove it by four steps.<br />

1. If x Q and y Q, then x y Q. So,<br />

fx y fx fy x y x y 0 Q.<br />

2. If x Q and y Q c , then x y Q c .So,<br />

fx y fx fy 1 x y x 1 y 2x Q.<br />

3. If x Q c and y Q, then x y Q c .So,<br />

fx y fx fy 1 x y 1 x y 2y Q.<br />

4. If x Q c and y Q c , then x y Q c or x y Q. So,


fx y fx fy <br />

1 x y 1 x 1 y 1 Q if x y Q c ,<br />

x y 1 x 1 y 2 Q if x y Q.<br />

Remark: Here is an interesting question about functions. Let f : R 0, 1 R. Iff<br />

satisfies that<br />

fx f x 1<br />

x 1 x,<br />

then fx x3 x 2 1<br />

. 2xx1<br />

Proof: Let x x1<br />

x , then we have 2 x 1<br />

x1 ,and3 x x. So,<br />

fx f x 1<br />

x fx fx 1 x *<br />

which implies that<br />

fx f 2 x 1 x **<br />

and<br />

f 2 x f 3 x f 2 x fx 1 2 x. ***<br />

So, by (*), (**), and (***), we finally have<br />

fx 1 2 1 x x 2 x<br />

x3 x 2 1<br />

2xx 1 .<br />

4.18 Let f be defined on R and assume that there exists at least one x 0 in R at which f<br />

is continuous. Suppose also that, for every x and y in R, f satisfies the equation<br />

fx y fx fy.<br />

Prove that there exists a constant a such that fx ax for all x.<br />

Proof: Let f be defined on R and assume that there exists at least one x 0 in R at which f<br />

is continuous. Suppose also that, for every x and y in R, f satisfies the equation<br />

fx y fx fy.<br />

Prove that there exists a constant a such that fx ax for all x.<br />

Proof: Suppose that f is continuous at x 0 , and given any r R. Since<br />

fx y fx fy for all x, then<br />

fx fy x 0 fr, wherey x r x 0 .<br />

Note that y x 0 x r, then<br />

lim xr<br />

fx lim xr<br />

fy x 0 fr<br />

yx0<br />

lim fy x 0 fr<br />

fr since f is continuous at x 0 .<br />

So, f is continuous at r. Sincer is arbitrary, we have f is continuous on R. Definef1 a,<br />

andthensincefx y fx fy, wehave<br />

f1 f 1 m .. m 1 mtimes<br />

mf<br />

1 m<br />

f 1 m f1<br />

m *


In addition, since f1 f1 by f0 0, we have<br />

f1 f 1 m .. m<br />

1<br />

mf<br />

1 m<br />

f 1 m<br />

mtimes<br />

f1<br />

m *’<br />

Thus we have<br />

f m n f1/m ...1/m ntimes nf1/m m n f1 by (*) and (*’) **<br />

So, given any x R, and thus choose a sequence x n Q with x n x. <strong>The</strong>n<br />

fx f lim n<br />

x n<br />

lim n<br />

fx n by continuity of f on R<br />

lim n<br />

x n f1 by (**)<br />

xf1<br />

ax.<br />

Remark: <strong>The</strong>re is a similar statement. Suppose that fx y fxfy for all real x<br />

and y.<br />

(1) If f is differentiable and non-zero, prove that fx e cx ,wherec is a constant.<br />

Proof: Note that f0 1sincefx y fxfy and f is non-zero. Since f is<br />

differentiable, we define f 0 c. Consider<br />

fx h fx fh f0<br />

fx fxf<br />

h<br />

h<br />

0 cfx as h 0,<br />

we have for every x R, f x cfx. Hence,<br />

fx Ae cx .<br />

Since f0 1, we have A 1. Hence, fx e cx ,wherec is a constant.<br />

Note: (i) If for every x R, f x cfx, then fx Ae cx .<br />

Proof: Since f x cfx for every x, we have for every x,<br />

f x cfxe cx 0 e cx fx 0.<br />

We note that by Elementary Calculus, e cx fx is a constant function A. So,fx Ae cx<br />

for all real x.<br />

(ii) Suppose that fx y fxfy for all real x and y. Iffx 0 0forsomex 0 , then<br />

fx 0 for all x.<br />

Proof: Suppose NOT, then fa 0forsomea. However,<br />

0 fx 0 fx 0 a a fx 0 afa 0.<br />

Hence, fx 0 for all x.<br />

(iii) Suppose that fx y fxfy for all real x and y. Iff is differentiable at x 0 for<br />

some x 0 , then f is differentiable for all x. <strong>And</strong> thus, fx C R.<br />

Proof: Since


fx h fx<br />

h<br />

fx 0 h x x 0 fx 0 x x 0 <br />

h<br />

fx x 0 fx 0 h fx 0 <br />

fx x 0 f x 0 as h 0,<br />

h<br />

we have f x is differentiable and f x fx x 0 f x 0 for all x. <strong>And</strong> thus we have<br />

fx C R.<br />

(iv) Here is another proof by (iii) and Taylor <strong>The</strong>orem with Remainder term R n x.<br />

Proof: Since f is differentiable, by (iii), we have f n x f 0 n fx for all x.<br />

Consider x r, r, then by Taylor <strong>The</strong>orem with Remainder term R n x,<br />

n<br />

fx <br />

k0<br />

<strong>The</strong>n<br />

f k 0<br />

k!<br />

x k R n x, whereR n x : fn1 <br />

n 1! xn1 , 0, x or x,0,<br />

|R n x| fn1 <br />

n 1! xn1<br />

<br />

f 0 n1 f<br />

n 1!<br />

f<br />

<br />

0r n1<br />

n 1!<br />

0asn .<br />

Hence, we have for every x r, r<br />

<br />

fx <br />

k0<br />

f0<br />

x n1<br />

M, whereM max<br />

xr,r |fx|<br />

f<br />

k<br />

0<br />

k!<br />

<br />

<br />

k0<br />

x k<br />

f 0x k<br />

k!<br />

e cx ,wherec : f 0.<br />

Since r is arbitrary, we have proved that fx e cx for all x.<br />

(2) If f is continuous and non-zero, prove that fx e cx ,wherec is a constant.<br />

Proof: Since fx y fxfy, wehave<br />

0 f1 f 1 n ... 1 n<br />

n f 1<br />

ntimes n f 1 n f1 1/n *<br />

and (note that f1 f1 1 by f0 1, )<br />

0 f1 f 1 n ... 1 n f n<br />

1<br />

ntimes n f 1 n f1 1/n *’<br />

f m n f 1 n ... 1 n mtimes<br />

f 1 n<br />

m<br />

f1<br />

m n<br />

by (*) and (*’) **<br />

So, given any x R, and thus choose a sequence x n Q with x n x. <strong>The</strong>n


lim<br />

r n0<br />

fx f lim n<br />

x n<br />

lim n<br />

fx n by continuity of f<br />

lim n<br />

f1 xn by (**)<br />

f1 x<br />

e cx , where log f1 c.<br />

Note: (i) We can prove (2) by the exercise as follows. Note that fx 0 for all x by<br />

the remark (1)-(ii) Consider the composite function gx log fx, then<br />

gx y log fx y log fxfy log fx log fy gx gy. Since log and f<br />

are continuous on R, its composite function g is continuous on R. Use the exercise, we<br />

have gx cx for some c. <strong>The</strong>refore, fx e gx e cx .<br />

(ii) We can prove (2) by the remark (1) as follows. It suffices to show that this f is<br />

differentiable at 0 by remark (1) and (1)-(iii). Sincef m n f1 m n<br />

then for every real r,<br />

fr f1 r a<br />

by continuity of f. Note that lim r b r<br />

r0 r exists. Given any sequence r n <br />

with r n 0, and thus consider<br />

fr n f0<br />

1<br />

1<br />

r n<br />

f1rn r n<br />

f1rn rn<br />

r n<br />

exists,<br />

we have f is differentiable at x 0. So,byremark(1),wehavefx e cx .<br />

(3) Give an example such that f is not continuous on R.<br />

Solution: Consider gx y gx gy for all x, y. <strong>The</strong>n we have gq qg1,<br />

where q Q. ByZorn’s Lemma, we know that every vector space has a basis<br />

v : I. Note that v : I is an uncountable set, so there exists a convergent<br />

sequence s n v : I. Hence, S : v : I s n <br />

n1<br />

sn<br />

n <br />

n1<br />

is a new<br />

basis of R over Q. Givenx, y R, and we can find the same N such that<br />

N<br />

x <br />

k1<br />

N<br />

q k v k and y p k v k ,wherev k S<br />

k1<br />

Define the sume<br />

N<br />

x y : p k q k v k<br />

k1<br />

By uniqueness, we define gx to be the sum of coefficients, i.e.,<br />

N<br />

gx : q k .<br />

k1<br />

Note that<br />

g<br />

s n<br />

1 for all n lim n<br />

g<br />

and<br />

s n 0asn <br />

Hence, g is not continuous at x 0 since if it was, then<br />

s n 1


1 lim n<br />

g<br />

s n<br />

g lim<br />

s n n by continuity of g at 0<br />

g0<br />

0<br />

which is absurb. Hence, g is not continuous on R by the exercise. To find such f, it suffices<br />

to consider fx e gx .<br />

Note: Such g (or f) isnot measurable by Lusin <strong>The</strong>orem.<br />

4.19 Let f be continuous on a, b and define g as follows: ga fa and, for<br />

a x b, letgx be the maximum value of f in the subinterval a, x. Show that g is<br />

continuous on a, b.<br />

Proof: Define gx maxft : t a, x, and choose any point c a, b, wewant<br />

to show that g is continuous at c. Given 0, we want to find a 0 such that as<br />

x c , c a, b, wehave<br />

|gx gc| .<br />

Since f is continuous at x c, then there exists a 0 such that as<br />

x c , c a, b, wehave<br />

fc /2 fx fc /2. *<br />

Consider two cases as follows.<br />

(1) maxft : t a, c a, b fp 1 ,wherep 1 c .<br />

As x c , c a, b, wehavegx fp 1 and gc fp 1 .<br />

Hence, |gx gc| 0.<br />

(2) maxft : t a, c a, b fp 1 ,wherep 1 c .<br />

As x c , c a, b, wehaveby(*)fc /2 gx fc /2.<br />

Hence, |gx gc| .<br />

So, if we choose ,thenforx c , c a, b,<br />

|gx gc| by (1) and (2).<br />

Hence, gx is continuous at c. <strong>And</strong> since c is arbitrary, we have gx is continuous on<br />

a, b.<br />

Remark: It is the same result for minft : t a, x by the preceding method.<br />

4.20 Let f 1 ,...,f m be m real-valued functions defined on R n . Assume that each f k is<br />

continuous at the point a of S. Define a new function f as follows: For each x in S, fx is<br />

the largest of the m numbers f 1 x,...,f m x. Discuss the continuity of f at a.<br />

Proof: Assume that each f k is continuous at the point a of S, then we have f i f j and<br />

|f i f j | are continuous at a, where 1 i, j m. Sincemaxa, b ab|ab| , then<br />

2<br />

maxf 1 , f 2 is continuous at a since both f 1 f 2 and |f 1 f 2 | are continuous at a. Define<br />

fx maxf 1 ,...f m ,useMathematical Induction to show that fx is continuous at<br />

x a as follows. As m 2, we have proved it. Suppose m k holds, i.e., maxf 1 ,...f k is<br />

continuous at x a. <strong>The</strong>n as m k 1, we have<br />

maxf 1 ,...f k1 maxmaxf 1 ,...f k , f k1 <br />

is continuous at x a by induction hypothesis. Hence, by Mathematical Induction, we<br />

have prove that f is continuos at x a.<br />

It is possible that f and g is not continuous on R whihc implies that maxf, g is<br />

continuous on R. For example, let fx 0ifx Q, andfx 1ifx Q c and gx 1


if x Q, andgx 0ifx Q c .<br />

Remark: It is the same rusult for minf 1 ,...f m since maxa, b mina, b a b.<br />

4.21 Let f : S R be continuous on an open set in R n , assume that p S, and assume<br />

that fp 0. Prove that there is an n ball Bp; r such that fx 0 for every x in the<br />

ball.<br />

Proof: Since p S is an open set in R n , there exists a 1 0 such that Bp, 1 S.<br />

Since fp 0, given fp 0, then there exists an n ball Bp; <br />

2 2 such that as<br />

x Bp; 2 S, wehave<br />

fp<br />

fp fx fp 3fp .<br />

2<br />

2<br />

Let min 1 , 2 , then as x Bp; , wehave<br />

fx fp 0.<br />

2<br />

Remark: <strong>The</strong> exercise tells us that under the assumption of continuity at p, we roughly<br />

have the same sign in a neighborhood of p, iffp 0 or fp 0.<br />

4.22 Let f be defined and continuous on a closed set S in R. Let<br />

A x : x S and fx 0 .<br />

Prove that A is a closed subset of R.<br />

Proof: Since A f 1 0, andf is continous on S, wehaveA is closed in S. <strong>And</strong><br />

since S is closed in R, we finally have A is closed in R.<br />

Remark: 1. Roughly speaking, the property of being closed has Transitivity. Thatis,<br />

in M, d let S T M, ifS is closed in T, andT is closed in M, then S is closed in M.<br />

Proof: Let x be an adherent point of S in M, then B M x, r S for every r 0.<br />

Hence, B M x, r T for every r 0. It means that x is also an adherent point of T in<br />

M. SinceT is closed in M, we find that x T. Note that since B M x, r S for every<br />

r 0, we have (S T)<br />

B T x, r S B M x, r T S B M x, r S T B M x, r S .<br />

So, we have x is an adherent point of S in T. <strong>And</strong> since S is closed in T, wehavex S.<br />

Hence, we have proved that if x is an adherent point of S in M, then x S. Thatis,S is<br />

closed in M.<br />

Note: (1) Another proof of remark 1, since S is closed in T, there exists a closed subset<br />

U in Msuch that S U T, and since T is closed in M, wehaveS is closed in M.<br />

(2) <strong>The</strong>re is a similar result, in M, d let S T M, ifS is open in T, andT is open<br />

in M, then S is open in M. (Leave to the reader.)<br />

2. Here is another statement like the exercise, but we should be cautioned. We write it<br />

as follows. Let f and g be continuous on S, d 1 into T, d 2 .LetA x : fx gx,<br />

show that A is closed in S.<br />

Proof: Let x be an accumulation point of A, then there exists a sequence x n A<br />

such that x n x. So, we have fx n gx n for all n. Hence, by continuity of f and g, we<br />

have<br />

fx f lim n<br />

x n lim n<br />

fx n lim n<br />

gx n g lim n<br />

x n gx.<br />

Hence, x A. Thatis,A contains its all adherent point. So, A is closed.


Note: In remark 2, we CANNOT use the relation<br />

fx gx<br />

since the difference "" are not necessarily defined on the metric space T, d 2 .<br />

4.23 Given a function f : R R, define two sets A and B in R 2 as follows:<br />

A x, y : y fx,<br />

B x, y : y fx.<br />

and Prove that f is continuous on R if, and only if, both A and B are open subsets of R 2 .<br />

Proof: () Suppose that f is continuous on R. Leta, b A, then fa b. Sincef is<br />

continuous at a, thengiven fab 0, there exists a 0 such that as<br />

2<br />

|x a| , wehave<br />

fa b<br />

fa fx fa . *<br />

2<br />

Consider x, y Ba, b; , then |x a| 2 |y b| 2 2 which implies that<br />

1. |x a| fx fa b by (*) and<br />

2<br />

2. |y b| y b b fa b .<br />

2<br />

Hence, we have fx y. Thatis,Ba, b; A. So,A is open since every point of A is<br />

interior. Similarly for B.<br />

() Suppose that A and B are open in R 2 . Trivially, a, fa /2 : p 1 A, and<br />

a, fa /2 : p 2 B. SinceA and B are open in R 2 , there exists a /2 0 such<br />

that<br />

Bp 1 , A and Bp 2 , B.<br />

Hence, if x, y Bp 1 , , then<br />

x a 2 y fa /2 2 2 and y fx.<br />

So, it implies that<br />

|x a| , |y fa /2| , andy fx.<br />

Hence, as |x a| , wehave<br />

y fa /2<br />

fa /2 y fx<br />

fa y fx<br />

fa fx. **<br />

<strong>And</strong> if x, y Bp 2 , , then<br />

x a 2 y fa /2 2 2 and y fx.<br />

So, it implies that<br />

|x a| , |y fa /2| , andy fx.<br />

Hence, as |x a| , wehave<br />

fx y fa /2 fa . ***<br />

So, given 0, there exists a 0 such that as |x a| , we have by (**) and (***)<br />

fa fx fa .<br />

That is, f is continuous at a. Sincea is arbitrary, we know that f is continuous on R.<br />

4.24 Let f be defined and bounded on a compact interval S in R. IfT S, the


number<br />

f T supfx fy : x, y T<br />

is called the oscillation (or span) of f on T. Ifx S, the oscillation of f at x is defined to<br />

be the number<br />

f x lim <br />

h0<br />

<br />

fBx; h S.<br />

Prove that this limit always exists and that f x 0if,andonlyif,f is continuous at x.<br />

Proof: 1. Note that since f is bounded, say |fx| M for all x, wehave<br />

|fx fy| 2M for all x, y S. So, f T, the oscillation of f on any subset T of S,<br />

exists. In addition, we define gh f Bx; h S. Note that if T 1 T 2 S, wehave<br />

f T 1 f T 2 . Hence, the oscillation of f at x, f x lim h0<br />

gh g0 since g is<br />

an increasing function. That is, the limit of f Bx; h S always exists as h 0 .<br />

2. Suppose that f x 0, then given 0, there exists a 0 such that as<br />

h 0, , wehave<br />

|gh| gh f Bx; h S /2.<br />

That is, as h 0, , wehave<br />

supft fs : t, s Bx; h S /2<br />

which implies that<br />

/2 ft fx /2 as t x , x S.<br />

So, as t x , x S. wehave<br />

|ft fx| .<br />

That is, f is continuous at x.<br />

Suppose that f is continous at x, thengiven 0, there exists a 0 such that as<br />

t x , x S, wehave<br />

|ft fx| /3.<br />

So, as t, s x , x S, wehave<br />

|ft fs| |ft fx| |fx fs| /3 /3 2/3<br />

which implies that<br />

supt fs : t, s x , x S 2/3 .<br />

So, as h 0, , wehave<br />

f Bx; h S supt fs : t, s x , x S .<br />

Hence, the oscillation of f at x, f x 0.<br />

Remark: 1. <strong>The</strong> compactness of S is not used here, we will see the advantage of the<br />

oscillation of f in text book, <strong>The</strong>orem 7.48, in page 171. (On Lebesgue’s Criterion for<br />

Riemann-Integrability.)<br />

2. One of advantage of the oscillation of f is to show the statement: Let f be defined on<br />

a, b Prove that a bounded f does NOT have the properties:<br />

f is continuous on Q a, b, and discontinuous on R Q a, b.<br />

Proof: Since f x 0if,andonlyif,f is continuous at x, we know that f r 0for<br />

r R Q a, b. DefineJ 1/n r : f r 1/n, then by hypothesis, we know that<br />

<br />

n1 J 1/n R Q a, b. It is easy to show that J 1/n is closed in a, b. Hence,<br />

intclJ 1/n intJ 1/n for all n N. It means that J 1/n is nowhere dense for all<br />

n N. Hence,


a, b <br />

n1 J 1/n Q a, b<br />

is of the firse category which is absurb since every complete metric space is of the second<br />

category. So, this f cannot exist.<br />

Note: 1 <strong>The</strong> Boundedness of f can be removed since we we can accept the concept<br />

0.<br />

2. (J 1/n is closed in a, b) Given an accumulation point x of J 1/n ,ifx J 1/n ,wehave<br />

f x 1/n. So, there exists a 1 ball Bx such that f Bx a, b 1/n. Thus, no<br />

points of Bx can belong to J 1/n , contradicting that x is an accumulation point of J 1/n .<br />

Hence, x J 1/n and J 1/n is closed.<br />

3. (Definition of a nowhere dense set) In a metric space M, d, letA be a subset of M,<br />

we say A is nowhere dense in M if, and only if A contains no balls of M, ( intA ).<br />

4. (Definition of a set of the first category and of the second category) AsetA in a<br />

metric space M is of the first category if, and only if, A is the union of a countable number<br />

of nowhere dense sets. A set B is of the second category if, and only if, B is not of the first<br />

category.<br />

5. (<strong>The</strong>orem) A complete metric space is of the second category.<br />

We write another important theorem about a set of the second category below.<br />

(Baire Category <strong>The</strong>orem) A nonempty open set in a complete metric space is of the<br />

second category.<br />

6. In the notes 3,4 and 5, the reader can see the reference, A First Course in <strong>Real</strong><br />

Analysis written by M. H. Protter and C. B. Morrey, in pages 375-377.<br />

4.25 Let f be continuous on a compact interval a, b. Suppose that f has a local<br />

maximum at x 1 and a local maximum at x 2 . Show that there must be a third point between<br />

x 1 and x 2 where f has a local minimum.<br />

Note.Tosaythatf has a local maximum at x 1 means that there is an 1 ball Bx 1 such<br />

that fx fx 1 for all x in Bx 1 a, b. Local minimum is similarly defined.<br />

Proof: Let x 2 x 1 . Suppose NOT, i.e., no points on x 1 , x 2 can be a local minimum<br />

of f. Sincef is continuous on x 1 , x 2 , then inffx : x x 1 , x 2 fx 1 or fx 2 by<br />

hypothesis. We consider two cases as follows:<br />

(1) If inffx : x x 1 , x 2 fx 1 , then<br />

(i) fx has a local maximum at x 1 and<br />

(ii) fx fx 1 for all x x 1 , x 2 .<br />

By (i), there exists a 0 such that x x 1 , x 1 x 1 , x 2 ,wehave<br />

(iii) fx fx 1 .<br />

So, by (ii) and (iii), as x x 1 , x 1 , wehave<br />

fx fx 1 <br />

which contradicts the hypothesis that no points on x 1 , x 2 can be a local minimum of f.<br />

(2) If inffx : x x 1 , x 2 fx 1 , it is similar, we omit it.<br />

Hence, from (1) and (2), we have there has a third point between x 1 and x 2 where f has<br />

a local minimum.<br />

4.26 Let f be a real-valued function, continuous on 0, 1, with the following property:<br />

For every real y, either there is no x in 0, 1 for which fx y or there is exactly one such<br />

x. Prove that f is strictly monotonic on 0, 1.


Proof: Since the hypothesis says that f is one-to-one, then by <strong>The</strong>orem*, we know that f<br />

is trictly monotonic on 0, 1.<br />

Remark: (<strong>The</strong>orem*) Under assumption of continuity on a compact interval, 1-1 is<br />

equivalent to being strictly monotonic. We will prove it in Exercise 4.62.<br />

4.27 Let f be a function defined on 0, 1 with the following property: For every real<br />

number y, either there is no x in 0, 1 for which fx y or there are exactly two values of<br />

x in 0, 1 for which fx y.<br />

(a) Prove that f cannot be continuous on 0, 1.<br />

Proof: Assume that f is continuous on 0, 1, and thus consider max x0,1 fx and<br />

min x0,1 fx. <strong>The</strong>n by hypothesis, there exist exactly two values a 1 a 2 0, 1 such<br />

that fa 1 fa 2 max x0,1 fx, and there exist exactly two values b 1 b 2 0, 1<br />

such that fb 1 fb 2 min x0,1 fx.<br />

Claim that a 1 0anda 2 1. Suppose NOT, then there exists at least one belonging<br />

to 0, 1. Without loss of generality, say a 1 0, 1. Sincef has maximum at a 1 0, 1<br />

and a 2 0, 1, we can find three points p 1 , p 2 ,andp 3 such that<br />

1. p 1 a 1 p 2 p 3 a 2 ,<br />

2. fp 1 fa 1 , fp 2 fa 1 ,andfp 3 fa 2 .<br />

Since fa 1 fa 2 , we choose a real number r so that<br />

fp 1 r fa 1 r fq 1 ,whereq 1 p 1 , a 1 by continuity of f.<br />

fp 2 r fa 1 r fq 2 ,whereq 2 a 1 , p 2 by continuity of f.<br />

fp 3 r fa 2 r fq 3 ,whereq 3 p 3 , a 2 by continuity of f.<br />

which contradicts the hypothesis that for every real number y, there are exactly two values<br />

of x in 0, 1 for which fx y. Hence, we know that a 1 0anda 2 1. Similarly, we<br />

also have b 1 0andb 2 1.<br />

So, max x0,1 fx min x0,1 fx which implies that f is constant. It is impossible.<br />

Hence, such f does not exist. That is, f is not continuous on 0, 1.<br />

(b) Construct a function f which has the above property.<br />

Proof: Consider 0, 1 Q c 0, 1 Q 0, 1, and write<br />

Q 0, 1 x 1 , x 2 ,...,x n ,.... Define<br />

1. fx 2n1 fx 2n n,<br />

2. fx x if x 0, 1/2 Q c ,<br />

3. fx 1 x if x 1/2, 1 Q c .<br />

<strong>The</strong>n if x y, then it is clear that fx fy. Thatis,f is well-defined. <strong>And</strong> from<br />

construction, we know that the function defined on 0, 1 with the following property: For<br />

every real number y, either there is no x in 0, 1 for which fx y or there are exactly<br />

two values of x in 0, 1 for which fx y.<br />

Remark: x : f is discontinuous at x 0, 1. Givena 0, 1. Note that since<br />

fx N for all x Q 0, 1 and Q is dense in R, forany1ball Ba; r Q 0, 1,<br />

there is always a rational number y Ba; r Q 0, 1 such that |fy fa| 1.<br />

(c) Prove that any function with this property has infinite many discontinuities on<br />

0, 1.<br />

Proof: In order to make the proof clear, property A of f means that


for every real number y, either there is no x in 0, 1 for which fx y or<br />

there are exactly two values of x in 0, 1 for which fx y<br />

Assume that there exist a finite many numbers of discontinuities of f, say these points<br />

x 1 ,...,x n . By property A, there exists a unique y i such that fx i fy i for 1 i n.<br />

Note that the number of the set<br />

S : x 1 ,...,x n y 1 ,...,y n x : fx f0, andfx f1 is even, say 2m<br />

We remove these points from S, and thus we have 2m 1 subintervals, say I j ,<br />

1 j 2m 1. Consider the local extremum in every I j ,1 j 2m 1 and note that<br />

every subinterval I j ,1 j 2m 1, has at most finite many numbers of local extremum,<br />

say # t I j : fx is the local extremum t j j<br />

1 ,..,t pj p j . <strong>And</strong> by property A,<br />

there exists a unique s j j<br />

j<br />

k such that f t k f s k for 1 k p j . We again remove these<br />

points, and thus we have removed even number of points. <strong>And</strong> odd number of open<br />

intervals is left, call the odd number 2q 1. Note that since the function f is monotonic in<br />

every open interval left, R l ,1 l 2q 1, the image of f on these open interval is also an<br />

open interval. If R a R b , sayR a a 1 , a 1 and R b b 1 , b 2 with (without loss of<br />

generality) a 1 b 1 a 2 b 2 , then<br />

R a R b by property A.<br />

(Otherwise, b 1 is only point such that fx fb 1 , which contradicts property A.) Note<br />

that given any R a , there must has one and only one R b such that R a R b . However, we<br />

have 2q 1 open intervals is left, it is impossible. Hence, we know that f has infinite many<br />

discontinuities on 0, 1.<br />

4.28 In each case, give an example of a real-valued function f, continuous on S and<br />

such that fS T, or else explain why there can be no such f :<br />

(a) S 0, 1, T 0, 1.<br />

Solution: Let<br />

fx <br />

2x if x 0, 1/2,<br />

1ifx 1/2, 1.<br />

(b) S 0, 1, T 0, 1 1, 2.<br />

Solution: NO! Since a continuous functions sends a connected set to a connected set.<br />

However, in this case, S is connected and T is not connected.<br />

(c) S R 1 , T the set of rational numbers.<br />

Solution: NO! Since a continuous functions sends a connected set to a connected set.<br />

However, in this case, S is connected and T is not connected.<br />

(d) S 0, 1 2, 3, T 0, 1.<br />

Solution: Let<br />

(e) S 0, 1 0, 1, T R 2 .<br />

fx <br />

0ifx 0, 1,<br />

1ifx 2, 3.<br />

Solution: NO! Since a continuous functions sends a compact set to a compact set.<br />

However, in this case, S is compact and T is not compact.


(f) S 0, 1 0, 1, T 0, 1 0, 1.<br />

Solution: NO! Since a continuous functions sends a compact set to a compact set.<br />

However, in this case, S is compact and T is not compact.<br />

(g) S 0, 1 0, 1, T R 2 .<br />

Solution: Let<br />

fx, y cot x,coty.<br />

Remark: 1. <strong>The</strong>re is some important theorems. We write them as follows.<br />

(<strong>The</strong>orem A) Letf : S, d s T, d T be continuous. If X is a compact subset of S,<br />

then fX is a compact subset of T.<br />

(<strong>The</strong>orem B) Letf : S, d s T, d T be continuous. If X is a connected subset of S,<br />

then fX is a connected subset of T.<br />

2. In (g), the key to the example is to find a continuous function f : 0, 1 R which is<br />

onto.<br />

Supplement on Continuity of real valued functions<br />

Exercise Suppose that fx : 0, R, is continuous with a fx b for all<br />

x 0, , and for any real y, either there is no x in 0, for which fx y or<br />

there are finitely many x in 0, for which fx y. Prove that lim x fx exists.<br />

Proof: For convenience, we say property A, it means that for any real y, either<br />

there is no x in 0, for which fx y or there are finitely many x in 0, for<br />

which fx y.<br />

We partition a, b into n subintervals. <strong>The</strong>n, by continuity and property A, asx<br />

is large enough, fx is lying in one and only one subinterval. Given 0, there<br />

exists N such that 2/N . For this N, we partition a, b into N subintervals, then<br />

there is a M 0 such that as x, y M<br />

|fx fy| 2/N .<br />

So, lim x fx exists.<br />

Exercise Suppose that fx : 0, 1 R is continunous with f0 f1 0. Prove that<br />

(a) there exist two points x 1 and x 2 such that as |x 1 x 2 | 1/n, wehave<br />

fx 1 fx 2 0 for all n. In this case, we call 1/n the length of horizontal strings.<br />

Proof: Define a new function gx fx 1 n fx : 0, 1 1 n . Claim that<br />

there exists p 0, 1 1 n such that gp 0. Suppose NOT, byIntermediate<br />

Value <strong>The</strong>orem, without loss of generality, let gx 0, then<br />

g0 g 1 n ...g 1 1 n f1 0<br />

which is absurb. Hence, we know that there exists p 0, 1 1 n such that<br />

gp 0. That is,<br />

f p 1 n fp.<br />

So,wehave1/n as the length of horizontal strings.<br />

(b) Could you show that there exists 2/3 as the length of horizontal strings<br />

Proof: <strong>The</strong> horizontal strings does not exist, for example,


fx <br />

x, ifx 0, 1 4<br />

x 1 ,ifx 1 , 3 .<br />

2 4 4<br />

x 1, if x 3 ,1 4<br />

Exercise Suppose that fx : a, b R is a continuous and non-constant function. Prove<br />

that the function f cannot have any small periods.<br />

Proof: Say f is continuous at q a, b, and by hypothesis that f is<br />

non-constant, there is a point p a, b such that |fq fp| : M 0. Since f is<br />

continuous at q, thengiven M, there is a 0 such that as<br />

x q , q a, b, wehave<br />

|fx fq| M. *<br />

If f has any small periods, then in the set q , q a, b, there is a point<br />

r q , q a, b such that fr fp. It contradicts to (*). Hence, the<br />

function f cannot have any small periods.<br />

Remark 1. <strong>The</strong>re is a function with any small periods.<br />

Solution:<strong>The</strong> example is Dirichlet function,<br />

0, if x Q<br />

fx <br />

c<br />

1, if x Q .<br />

Since fx q fx, for any rational q, we know that f has any small periods.<br />

2. Prove that there cannot have a non-constant continuous function which has<br />

two period p, andq such that q/p is irrational.<br />

Proof: Since q/p is irrational, there is a sequence qn<br />

p n<br />

Q such that<br />

q n<br />

p n<br />

q p 1 p<br />

<br />

p2 |pq n qp n| <br />

n<br />

p n<br />

0asn .<br />

So, f has any small periods, by this exercise, we know that this f cannot a<br />

non-constant continuous function.<br />

Note: <strong>The</strong> inequality is important; the reader should kepp it in mind. <strong>The</strong>re are<br />

many ways to prove this inequality, we metion two methods without proofs. <strong>The</strong><br />

reader can find the proofs in the following references.<br />

(1) An Introduction To <strong>The</strong> <strong>The</strong>ory Of <strong>Number</strong>s written by G.H. Hardy and<br />

E.M. Wright, charpter X, pp 137-138.<br />

(2) In the text book, exercise 1.15 and 1.16, pp 26.<br />

3. Suppose that fx is differentiable on R prove that if f has any small periods,<br />

then f is constant.<br />

Proof: Givenc R, and consider<br />

fc p n fc<br />

p n<br />

0 for all n.<br />

where p n is a sequence of periods of function such that p n 0. Hence, by<br />

differentiability of f, we know that f c 0. Since c is arbitrary, we know that<br />

f x 0onR. Hence, f is constant.<br />

Continuity in metric spaces


In Exercises 4.29 through 4.33, we assume that f : S T is a function from one metric<br />

space S, d S to another T, d T .<br />

4.29 Prove that f is continuous on S if, and only if,<br />

f 1 intB intf 1 B for every subset B of T.<br />

Proof: () Suppose that f is continuous on S, and let B be a subset of T. Since<br />

intB B, wehavef 1 intB f 1 B. Note that f 1 intB is open since a pull back of<br />

an open set under a continuous function is open. Hence, we have<br />

intf 1 intB f 1 intB intf 1 B.<br />

That is, f 1 intB intf 1 B for every subset B of T.<br />

() Suppose that f 1 intB intf 1 B for every subset B of T. Given an open subset<br />

U T, i.e., intU U, sowehave<br />

f 1 U f 1 intU intf 1 U.<br />

In addition, intf 1 U f 1 U by the fact, for any set A, intA is a subset of A. So,f is<br />

continuous on S.<br />

4.30 Prove that f is continuous on S if, and only if,<br />

fclA clfA for every subset A of S.<br />

Proof: () Suppose that f is continuous on S, and let A be a subset of S. Since<br />

fA clfA, then A f 1 fA f 1 clfA. Note that f 1 clfA is closed<br />

since a pull back of a closed set under a continuous function is closed. Hence, we have<br />

clA clf 1 clfA f 1 clfA<br />

which implies that<br />

fclA ff 1 clfA clfA.<br />

() Suppose that fclA clfA for every subset A of S. Given a closed subset<br />

C T, and consider f 1 C as follows. Define f 1 C A, then<br />

fclf 1 C fclA<br />

clfA clff 1 C<br />

clC C since C is closed.<br />

So,wehaveby(fclA C)<br />

clA f 1 fclA f 1 C A<br />

which implies that A f 1 C is closed set. So, f is continuous on S.<br />

4.31 Prove that f is continuous on S if, and only if, f is continuous on every compact<br />

subset of S. Hint. If x n p in S, the set p, x 1 , x 2 ,... is compact.<br />

Proof: () Suppose that f is continuous on S, then it is clear that f is continuous on<br />

every compact subset of S.<br />

() Suppose that f is continuous on every compact subset of S, Givenp S, we<br />

consider two cases.<br />

(1) p is an isolated point of S, then f is automatically continuous at p.<br />

(2) p is not an isolated point of S, thatis,p is an accumulation point p of S, then there<br />

exists a sequence x n S with x n p. Note that the set p, x 1 , x 2 ,... is compact, so we<br />

know that f is continuous at p. Sincep is arbitrary, we know that f is continuous on S.<br />

Remark: If x n p in S, the set p, x 1 , x 2 ,... is compact. <strong>The</strong> fact is immediately


from the statement that every infinite subset p, x 1 , x 2 ,... of has an accumulation point in<br />

p, x 1 , x 2 ,....<br />

4.32 A function f : S T is called a closed mapping on S if the image fA is closed<br />

in T for every closed subset A of S. Prove that f is continuous and closed on S if, and only<br />

if,<br />

fclA clfA for every subset A of S.<br />

Proof: () Suppose that f is continuous and closed on S, and let A be a subset of S.<br />

Since A clA, wehavefA fclA. So, we have<br />

clfA clfclA fclA since f is closed. *<br />

In addition, since fA clfA, wehaveA f 1 fA f 1 clfA. Note that<br />

f 1 clfA is closed since f is continuous. So, we have<br />

clA clf 1 clfA f 1 clfA<br />

which implies that<br />

fclA ff 1 clfA clfA. **<br />

From (*) and (**), we know that fclA clfA for every subset A of S.<br />

() Suppose that fclA clfA for every subset A of S. Gvien a closed subset C<br />

of S, i.e., clC C, then we have<br />

fC fclC clfC.<br />

So, we have fC is closed. That is, f is closed. Given any closed subset B of T, i.e.,<br />

clB B, we want to show that f 1 B is closed. Since f 1 B : A S, wehave<br />

fclf 1 B fclA clfA clff 1 B clB B<br />

which implies that<br />

fclf 1 B B clf 1 B f 1 fclf 1 B f 1 B.<br />

That is, we have clf 1 B f 1 B. So,f 1 B is closed. Hence, f is continuous on S.<br />

4.33 Give an example of a continuous f and a Cauchy sequence x n in some metric<br />

space S for which fx n is not a Cauchy sequence in T.<br />

Solution: Let S 0, 1, x n 1/n for all n N, andf 1/x : S R. <strong>The</strong>n it is clear<br />

that f is continous on S, andx n is a Cauchy sequence on S. In addition, Trivially,<br />

fx n n is not a Cauchy sequence.<br />

Remark: <strong>The</strong> reader may compare the exercise with the Exercise 4.54.<br />

4.34 Prove that the interval 1, 1 in R 1 is homeomorphic to R 1 . This shows that<br />

neither boundedness nor completeness is a topological property.<br />

Proof: Since fx tan x : 1, 1 R is bijection and continuous, and its<br />

2<br />

converse function f 1 x arctanx : R 1, 1. Hence, we know that f is a Topologic<br />

mapping. (Or say f is a homeomorphism). Hence, 1, 1 is homeomorphic to R 1 .<br />

Remark: A function f is called a bijection if, and only if, f is 1-1 and onto.<br />

4.35 Section 9.7 contains an example of a function f, continuous on 0, 1, with<br />

f0, 1 0, 1 0, 1. Prove that no such f can be one-to-one on 0, 1.<br />

Proof: By section 9.7, let f : 0, 1 0, 1 0, 1 be an onto and continuous function.<br />

If f is 1-1, then so is its converse function f 1 . Note that since f is a 1-1 and continous<br />

function defined on a compact set 0, 1, then its converse function f 1 is also a continous


function. Since f0, 1 0, 1 0, 1, we have the domain of f 1 is 0, 1 0, 1 which<br />

is connected. Choose a special point y 0, 1 0, 1 so that f 1 y : x 0, 1.<br />

Consider a continous function g f 1 | 0,10,1y , then<br />

g : 0, 1 0, 1 y 0, x x,1 which is continous. However, it is impossible<br />

since 0, 1 0, 1 y is connected but 0, x x,1 is not connected. So, such f cannot<br />

exist.<br />

Connectedness<br />

4.36 Prove that a metric space S is disconnected if, and only if there is a nonempty<br />

subset A of S, A S, which is both open and closed in S.<br />

Proof: () Suppose that S is disconnected, then there exist two subset A, B in S such<br />

that<br />

1. A, B are open in S, 2.A and B , 3.A B , and 4. A B S.<br />

Note that since A, B are open in S ,wehaveA S B, B S A are closed in S. So,ifS<br />

is disconnected, then there is a nonempty subset A of S, A S, which is both open and<br />

closed in S.<br />

() Suppose that there is a nonempty subset A of S, A S, which is both open and<br />

closed in S. <strong>The</strong>n we have S A : B is nonempty and B is open in S. Hence, we have two<br />

sets A, B in S such that<br />

1. A, B are open in S, 2.A and B , 3.A B , and 4. A B S.<br />

That is, S is disconnected.<br />

4.37 Prove that a metric space S is connected if, and only if the only subsets of S which<br />

are both open and closed in S are empty set and S itself.<br />

Proof: () Suppose that S is connected. If there exists a subset A of S such that<br />

1. A , 2.A is a proper subset of S, 3.A is open and closed in S,<br />

then let B S A, wehave<br />

1. A, B are open in S, 2.A and B , 3.A B , and 4. A B S.<br />

It is impossible since S is connected. So, this A cannot exist. That is, the only subsets of S<br />

which are both open and closed in S are empty set and S itself.<br />

() Suppose that the only subsets of S which are both open and closed in S are empty<br />

set and S itself. If S is disconnected, then we have two sets A, B in S such that<br />

1. A, B are open in S, 2.A and B , 3.A B , and 4. A B S.<br />

It contradicts the hypothesis that the only subsets of S which are both open and closed in S<br />

are empty set and S itself.<br />

Hence, we have proved that S is connected if, and only if the only subsets of S which<br />

are both open and closed in S are empty set and S itself.<br />

4.38 Prove that the only connected subsets of R are<br />

(a) the empty set,<br />

(b) sets consisting of a single point, and<br />

(c) intervals (open, closed, half-open, or infinite).<br />

Proof: Let S be a connected subset of R. Denote the symbol #A to be the number of<br />

elements in a set A. We consider three cases as follows. (a) #S 0, (b) #S 1, (c)<br />

#S 1.<br />

For case (a), it means that S , and for case (b), it means that S consists of a single<br />

point. It remains to consider the case (c). Note that since #S 1, we have inf S sup S.


Since S R, wehaveS inf S,supS. (Note that we accept that inf S or<br />

sup S .) If S is not an interval, then there exists x inf S,supS such that x S.<br />

(Otherwise, inf S,supS S which implies that S is an interval.) <strong>The</strong>n we have<br />

1. , x S : A is open in S<br />

2. x, S : B is open in S<br />

3. A B S.<br />

Claim that both A and B are not empty. Asume that A is empty, then every s S, wehave<br />

s x inf S. By the definition of infimum, it is impossible. So, A is not empty. Similarly<br />

for B. Hence, we have proved that S is disconnected, a contradiction. That is, S is an<br />

interval.<br />

Remark: 1. We note that any interval in R is connected. It is immediate from Exercise<br />

4.44. But we give another proof as follows. Suppose there exists an interval S is not<br />

connceted, then there exist two subsets A and B such that<br />

1. A, B are open in S, 2.A and B , 3.A B , and 4. A B S.<br />

Since A and B , we choose a A and b B, and let a b. Consider<br />

c : supA a, b.<br />

Note that c clA A implies that c B. Hence, we have a c b. In addition,<br />

c B clB, then there exists a B S c; B . Choose<br />

d B S c; c , c S so that<br />

1. c d b and 2. d B.<br />

<strong>The</strong>n d A. (Otherwise, it contradicts c supA a, b. Note that<br />

d a, b S A B which implies that d A or d B. We reach a contradiction since<br />

d A and d B. Hence, we have proved that any interval in R is connected.<br />

2. Here is an application. Is there a continuous function f : R R such that<br />

fQ Q c ,andfQ c Q <br />

Ans: NO! If such f exists, then both fQ and fQ c are countable. Hence, fR is<br />

countable. In addition, fR is connected. Since fR contains rationals and irrationals, we<br />

know fR is an interval which implies that fR is uncountable, a cotradiction. Hence, such<br />

f does not exist.<br />

4.39 Let X be a connected subset of a metric space S. LetY be a subset of S such that<br />

X Y clX, whereclX is the closure of X. Prove that Y is also connected. In<br />

particular, this shows that clX is connected.<br />

Proof: Given a two valued function f on Y, we know that f is also a two valued<br />

function on X. Hence, f is constant on X, (without loss of generality) say f 0onX.<br />

Consider p Y X, it ,means that p is an accumulation point of X. <strong>The</strong>n there exists a<br />

dequence x n X such that x n p. Note that fx n 0 for all n. So, we have by<br />

continuity of f on Y,<br />

fp f lim n<br />

x n<br />

lim n<br />

fx n 0.<br />

Hence, we have f is constant 0 on Y. Thatis,Y is conneceted. In particular, clX is<br />

connected.<br />

Remark: Of course, we can use definition of a connected set to show the exercise. But,<br />

it is too tedious to write. However, it is a good practice to use definition to show it. <strong>The</strong><br />

reader may give it a try as a challenge.


4.40 If x is a point in a metric space S, letUx be the component of S containing x.<br />

Prove that Ux is closed in S.<br />

Proof: Let p be an accumulation point of Ux. Letf be a two valued function defined<br />

on Ux p, then f is a two valud function defined on Ux. SinceUx is a component<br />

of S containing x, then Ux is connected. That is, f is constant on Ux, (without loss of<br />

generality) say f 0onUx. <strong>And</strong> since p is an accumulation point of Ux, there exists a<br />

sequence x n Ux such that x n p. Note that fx n 0 for all n. So, we have by<br />

continuity of f on Ux p,<br />

fp f lim n<br />

x n<br />

lim n<br />

fx n 0.<br />

So, Ux p is a connected set containing x. SinceUx is a component of S containing<br />

x, wehaveUx p Ux which implies that p Ux. Hence, Ux contains its all<br />

accumulation point. That is, Ux is closed in S.<br />

4.41 Let S be an open subset of R. By <strong>The</strong>orem 3.11, S is the union of a countable<br />

disjoint collection of open intervals in R. Prove that each of these open intervals is a<br />

component of the metric subspace S. Explain why this does not contradict Exercise 4.40.<br />

Proof: By <strong>The</strong>orem 3.11, S <br />

n1 I n ,whereI i is open in R and I i I j if i j.<br />

Assume that there exists a I m such that I m is not a component T of S. <strong>The</strong>n T I m is not<br />

empty. So, there exists x T I m and x I n for some n. Note that the component Ux is<br />

the union of all connected subsets containing x, then we have<br />

T I n Ux. *<br />

In addition,<br />

Ux T **<br />

since T is a component containing x. Hence, by (*) and (**), we have I n T. So,<br />

I m I n T. SinceT is connected in R 1 , T itself is an interval. So, intT is still an interval<br />

which is open and containing I m I n . It contradicts the definition of component interval.<br />

Hence, each of these open intervals is a component of the metric subspace S.<br />

Since these open intervals is open relative to R, not S, this does not contradict Exercise<br />

4.40.<br />

4.42 Given a compact S in R m with the following property: For every pair of points a<br />

and b in S and for every 0 there exists a finite set of points x 0 , x 1 ,...,x n in S with<br />

x 0 a and x n b such that<br />

x k x k1 for k 1,2,..,n.<br />

Prove or disprove: S is connected.<br />

Proof: Suppose that S is disconnected, then there exist two subsets A and B such that<br />

1. A, B are open in S, 2.A and B , 3.A B , and 4. A B S. *<br />

Since A and B , we choose a A, andb A and thus given 1, then by<br />

hypothesis, we can find two points a 1 A, andb 1 B such that a 1 b 1 1. For a 1 ,<br />

and b 1 ,given 1/2, then by hypothesis, we can find two points a 2 A, andb 2 B<br />

such that a 2 b 2 1/2. Continuous the steps, we finally have two sequence a n A<br />

and b n B such that a n b n 1/n for all n. Sincea n A, andb n B, we<br />

have a n S and b n S by S A B. Hence, there exist two subsequence<br />

a nk A and b nk B such that a nk x, andb nk y, wherex, y S since S is<br />

compact. In addition, since A is closed in S, andB is closed in S, wehavex A and<br />

y B. On the other hand, since a n b n 1/n for all n, wehavex y. Thatis,


A B which contradicts (*)-3. Hence, we have prove that S is connected.<br />

Remark: We given another proof by the method of two valued function as follows. Let<br />

f be a two valued function defined on S, and choose any two points a, b S. If we can<br />

show that fa fb, we have proved that f is a constant which implies that S is<br />

connected. Since f is a continuous function defined on a compact set S, then f is uniformly<br />

on S. Thus, given 1 0, there exists a 0 such that as x y , x, y S, we<br />

have |fx fy| 1 fx fy. Hence, for this , there exists a finite set of<br />

points x 0 , x 1 ,...,x n in S with x 0 a and x n b such that<br />

x k x k1 for k 1,2,..,n.<br />

So, we have fa fx 0 fx 1 ... fx n fb.<br />

4.43 Prove that a metric space S is connected if, and only if, every nonempty proper<br />

subset of S has a nonempty boundary.<br />

Proof: () Suppose that S is connected, and if there exists a nonempty proper subset U<br />

of S such that U , thenletB clS U, wehave(defineclU A<br />

1. A . B since S U ,<br />

2. A B clU clS U U S U S<br />

S A B,<br />

3. A B clU clS U U ,<br />

and<br />

4. Both A and B are closed in S Both A and B are open in S.<br />

Hence, S is disconnected. That is, if S is connected, then every nonempty proper subset of<br />

S has a nonempty boundary.<br />

() Suppose that every nonempty proper subset of S has a nonempty boundary. If S is<br />

disconnected, then there exist two subsets A and B such that<br />

1. A, B are closed in S, 2.A and B , 3.A B , and 4. A B S.<br />

<strong>The</strong>n for this A, A is a nonempty proper subset of S with (clA A, andclB B)<br />

A clA clS A clA clB A B <br />

which contradicts the hypothesis that every nonempty proper subset of S has a nonempty<br />

boundary. So, S is connected.<br />

4.44. Prove that every convex subset of R n is connected.<br />

Proof: Given a convex subset S of R n , and since for any pair of points a, b, the set<br />

1 a b :0 1 : T S, i.e., g : 0, 1 T by g 1 a b is a<br />

continuous function such that g0 a, andg1 b. So,S is path-connected. It implies<br />

that S is connected.<br />

Remark: 1. In the exercise, it tells us that every n ball is connected. (In fact, every<br />

n ball is path-connected.) In particular, as n 1, any interval (open, closed, half-open, or<br />

infinite) in R is connected. For n 2, any disk (open, closed, or not) in R 2 is connected.<br />

2. Here is a good exercise on the fact that a path-connected set is connected. Given<br />

0, 1 0, 1 : S, andifT is a countable subset of S. Prove that S T is connected. (In<br />

fact, S T is path-connected.)<br />

Proof: Given any two points a and b in S T, then consider the vertical line L passing<br />

through the middle point a b/2. Let A x : x L S, and consider the lines form<br />

a to A, and from b to A. Note that A is uncountable, and two such lines (form a to A, and


from b to A) are disjoint. So, if every line contains a point of T, then it leads us to get T is<br />

uncountable. However, T is countable. So, it has some line (form a to A, and from b to A)<br />

is in S T. So, it means that S T is path-connected. So, S T is connected.<br />

4.45 Given a function f : R n R m which is 1-1 and continuous on R n .IfA is open<br />

and disconnected in R n , prove that fA is open and disconnected in fR n .<br />

Proof: <strong>The</strong>exerciseiswrong. <strong>The</strong>re is a counter-example. Let f : R R 2<br />

f <br />

cos 2x<br />

1x<br />

cos 2x<br />

1x<br />

2<br />

2<br />

2x<br />

,1 sin if x 0<br />

1x 2<br />

2x<br />

, 1 sin if x 0<br />

1x 2<br />

Remark: If we restrict n, m 1, the conclusion holds. That is, Let f : R R be<br />

continuous and 1-1. If A is open and disconnected, then so is fA.<br />

Proof: In order to show this, it suffices to show that f maps an open interval I to<br />

another open interval. Since f is continuous on I, andI is connected, fI is connected. It<br />

implies that fI is an interval. Trivially, there is no point x in I such that fx equals the<br />

endpoints of fI. Hence, we know that fI is an open interval.<br />

Supplement: Here are two exercises on Homeomorphism to make the reader get more<br />

and feel something.<br />

1. Let f : E R R. Ifx, fx : x E is compact, then f is uniformly continuous<br />

on E.<br />

Proof: Let x, fx : x E S, and thus define gx Ix x, fx : E S.<br />

Claim that g is continuous on E. Consider h : S E by hx, fx x. Trivially, h is 1-1,<br />

continuous on a compact set S. So, its inverse function g is 1-1 and continuous on a<br />

compact set E. <strong>The</strong> claim has proved.<br />

Since g is continuous on E, we know that f is continuous on a compact set E. Hence, f<br />

is uniformly continuous on E.<br />

Note: <strong>The</strong> question in Supplement 1, there has another proof by the method of<br />

contradiction, and use the property of compactness. We omit it.<br />

2. Let f : 0, 1 R. Ifx, fx : x 0, 1 is path-connected, then f is continuous<br />

on 0, 1.<br />

Proof: Let a 0, 1, then there is a compact interval a a 1 , a 2 0, 1. Claim<br />

that the set<br />

x, fx : x a 1 , a 2 : S is compact.<br />

Since S is path-connected, there is a continuous function g : 0, 1 S such that<br />

g0 a 1 , fa 1 and g1 a 2 , fa 2 . If we can show g0, 1 S, wehaveshown<br />

that S is compact. Consider h : S R by hx, fx x; h is clearly continuous on S. So,<br />

the composite function h g : 0, 1 R is also continuous. Note that h g0 a 1 ,and<br />

h g1 a 2 , and the range of h g is connected. So, a 1 , a 2 hg0, 1. Hence,<br />

g0, 1 S. We have proved the claim and by Supplement 1, we know that f is<br />

continuous at a. Sincea is arbitrary, we know that f is continuous on 0, 1.<br />

Note: <strong>The</strong> question in Supplement 2, there has another proof directly by definition of<br />

continuity. We omit the proof.<br />

4.46 Let A x, y :0 x 1, y sin 1/x, B x, y : y 0, 1 x 0,<br />

and let S A B. Prove that S is connected but not arcwise conneceted. (See Fig. 4.5,


Section 4.18.)<br />

Proof: Let f be a two valued function defined on S. SinceA, andB are connected in S,<br />

then we have<br />

fA a, andfB b, wherea, b 0, 1.<br />

Given a sequence x n A with x n 0, 0, then we have<br />

a lim n<br />

fx n f limx n by continuity of f at 0<br />

n0<br />

f0, 0<br />

b.<br />

So, we have f is a constant. That is, S is connected.<br />

Assume that S is arcwise connected, then there exists a continuous function<br />

g : 0, 1 S such that g0 0, 0 and g1 1, sin 1. Given 1/2, there exists a<br />

0 such that as |t| , wehave<br />

gt g0 gt 1/2. *<br />

1<br />

Let N be a positive integer so that<br />

2N<br />

Define two subsets U and V as follows:<br />

U <br />

V <br />

1<br />

, thus let ,0 : p and 1<br />

,0 : q.<br />

2N 2N1<br />

x, y : x p q<br />

2<br />

x, y : x p q<br />

2<br />

gq, p<br />

gq, p<br />

<strong>The</strong>n we have<br />

(1). U V gq, p, (2). U , sincep U and V , sinceq V,<br />

(3). U V by the given set A, and (*)<br />

Since x, y : x pq<br />

2<br />

gq, p. So, we have<br />

and x, y : x <br />

pq<br />

2 are open in R2 , then U and V are open in<br />

(4). U is open in gq, p and V is open in gq, p.<br />

From (1)-(4), we have gq, p is disconnected which is absurb since a connected subset<br />

under a continuous function is connected. So, such g cannot exist. It means that S is not<br />

arcwise connected.<br />

Remark: This exercise gives us an example to say that connectedness does not imply<br />

path-connectedness. <strong>And</strong> it is important example which is worth keeping in mind.<br />

4.47 Let F F 1 , F 2 ,... be a countable collection of connected compact sets in R n<br />

such that F k1 F k for each k 1. Prove that the intersection <br />

k1 F k is connected and<br />

closed.<br />

Proof: Since F k is compact for each k 1, F k is closed for each k 1. Hence,<br />

<br />

k1 F k : F is closed. Note that by <strong>The</strong>orem 3.39, we know that F is compact. Assume<br />

that F is not connected. <strong>The</strong>n there are two subsets A and B with<br />

1.A , B . 2.A B . 3.A B F. 4.A, B are closed in F.<br />

Note that A, B are closed and disjoint in R n . By exercise 4.57, there exist U and V which<br />

are open and disjoint in R n such that A U, andB V. Claim that there exists F k such<br />

that F k U V. Suppose NOT, thenthereexistsx k F k U V. Without loss of<br />

generality, we may assume that x k F k1 . So, we have a sequence x k F 1 which<br />

implies that there exists a convergent subsequence x kn , say lim kn x kn x. Itis<br />

clear that x F k for all k since x is an accumulation point of each F k . So, we have


x F <br />

k1 F k A B U V<br />

which implies that x is an interior point of U V since U and V are open. So,<br />

Bx; U V for some 0, which contradicts to the choice of x k . Hence, we have<br />

proved that there exists F k such that F k U V. LetC U F k ,andD V F k , then<br />

we have<br />

1. C since A U and A F k ,andD since B V and B F k .<br />

2. C D U F k V F k U V .<br />

3. C D U F k V F k F k .<br />

4. C is open in F k and D is open in F k by C, D are open in R n .<br />

Hence, we have F k is disconnected which is absurb. So, we know that F <br />

k1 F k is<br />

connected.<br />

4.48 Let S be an open connected set in R n .LetT be a component of R n S. Prove<br />

that R n T is connected.<br />

Proof: If S is empty, there is nothing to proved. Hence, we assume that S is nonempty.<br />

Write R n S xR n S Ux, whereUx is a component of R n S. So, we have<br />

R n S xR n S Ux.<br />

Say T Up, forsomep. <strong>The</strong>n<br />

R n T S xR n ST Ux.<br />

Claim that clS Ux for all x R n S T. If we can show the claim, given<br />

a, b R n T, and a two valued function on R n T. Note that clS is also connected. We<br />

consider three cases. (1) a S, b Ux for some x. (2)a, b S. (3)a Ux,<br />

b Ux .<br />

For case (1), let c clS Ux, then there are s n S and u n Ux with<br />

s n c and u n c, then we have<br />

fa lim n<br />

fs n f lim n<br />

s n fc f lim n<br />

u n lim n<br />

fu n fb<br />

which implies that fa fb.<br />

For case (2), it is clear fa fb since S itself is connected.<br />

For case (3), we choose s S, and thus use case (1), we know that<br />

fa fs fb.<br />

By case (1)-(3), we have f is constant on R n T. Thatis,R n T is connected.<br />

It remains to show the claim. To show clS Ux for all x R n S T, i.e., to<br />

show that for all x R n S T,<br />

clS Ux S S Ux<br />

S Ux<br />

.<br />

Suppose NOT, i.e., for some x, S Ux which implies that Ux R n clS<br />

which is open. So, there is a component V of R n clS contains Ux, whereV is open by<br />

<strong>The</strong>orem 4.44. However, R n clS R n S, sowehaveV is contained in Ux.<br />

<strong>The</strong>refore, we have Ux V. Note that Ux R n S, andR n S is closed. So,<br />

clUx R n S. By definition of component, we have clUx Ux, whichis<br />

closed. So, we have proved that Ux V is open and closed. It implies that Ux R n or<br />

which is absurb. Hence, the claim has proved.<br />

4.49 Let S, d be a connected metric space which is not bounded. Prove that for every


a in S and every r 0, the set x : dx, a r is nonempty.<br />

Proof: Assume that x : dx, a r is empty. Denote two sets x : dx, a r by A<br />

and x : dx, a r by B. <strong>The</strong>n we have<br />

1. A since a A and B since S is unbounded,<br />

2. A B ,<br />

3. A B S,<br />

4. A Ba; r is open in S,<br />

and consider B as follows. Since x : dx, a r is closed in S, B S x : dx, a r<br />

is open in S. So, we know that S is disconnected which is absurb. Hence, we know that the<br />

set x : dx, a r is nonempty.<br />

Supplement on a connected metric space<br />

Definition Two subsets A and B of a metric space X are said to be separated if both<br />

A clB and clA B .<br />

AsetE X is said to be connected if E is not a union of two nonempty separated<br />

sets.<br />

We now prove the definition of connected metric space is equivalent to this definiton<br />

as follows.<br />

<strong>The</strong>orem A set E in a metric space X is connected if, and only if E is not the union of<br />

two nonempty disjoint subsets, each of which is open in E.<br />

Proof: () Suppose that E is the union of two nonempty disjoint subsets, each<br />

of which is open in E, denote two sets, U and V. Claim that<br />

U clV clU V .<br />

Suppose NOT, i.e., x U clV. Thatis,thereisa 0 such that<br />

B X x, E B E x, U and B X x, V <br />

which implies that<br />

B X x, V B X x, V E<br />

B X x, E V<br />

U V ,<br />

a contradiction. So, we have U clV . Similarly for clU V . So,X is<br />

disconnected. That is, we have shown that if a set E in a metric space X is<br />

connected, then E is not the union of two nonempty disjoint subsets, each of which<br />

is open in E.<br />

() Suppose that E is disconnected, then E is a union of two nonempty<br />

separated sets, denoted E A B, whereA clB clA B . Claim that A<br />

and B are open in E. Suppose NOT, it means that there is a point x A which is<br />

not an interior point of A. So, for any ball B E x, r, there is a correspounding<br />

x r B, wherex r B E x, r. It implies that x clB which is absurb with<br />

A clB . So, we proved that A is open in E. Similarly, B is open in E. Hence,<br />

we have proved that if E is not the union of two nonempty disjoint subsets, each of<br />

whichisopeninE, then E in a metric space X is connected.<br />

Exercise Let A and B be connected sets in a metric space with A B not connected and<br />

suppose A B C 1 C 2 where clC 1 C 2 C 1 clC 2 . Show that<br />

B C 1 is connected.


Proof: Assume that B C 1 is disconnected, and thus we will prove that C 1 is<br />

disconnected. Consider, by clC 1 C 2 C 1 clC 2 ,<br />

C 1 clC 2 A B C 1 clA B C 1 clB *<br />

and<br />

clC 1 C 2 A B clC 1 A B clC 1 B **<br />

we know that at least one of (*) and (**) is nonempty by the hypothesis A is<br />

connected. In addition, by (*) and (**), we know that at leaset one of<br />

C 1 clB<br />

and<br />

clC 1 B<br />

is nonempty. So, we know that C 1 is disconnected by the hypothesis B is connected,<br />

and the concept of two valued function.<br />

From above sayings and hypothesis, we now have<br />

1. B is connected.<br />

2. C 1 is disconnected.<br />

3. B C 1 is disconnected.<br />

Let D be a component of B C 1 so that B D; we have, let<br />

B C 1 D E C 1 ,<br />

D clE clD E <br />

which implies that<br />

clE A E , andclA E E .<br />

So,wehaveprovethatA is disconnected wich is absurb. Hence, we know that<br />

B C 1 is connected.<br />

Remark We prove that clA E E clE A E as follows.<br />

Proof: Since<br />

D clE ,<br />

we obtain that<br />

clE A E<br />

clE D C 2 A B<br />

clE D C 2 B<br />

clE D C 2 since B D<br />

clE C 2 since D clE <br />

<strong>And</strong> since<br />

we obtain that<br />

clC 1 C 2 since E C 1<br />

.<br />

clD E ,


clA E E<br />

clD C 2 A B E<br />

clD C 2 B E<br />

clD C 2 E since B D<br />

clC 2 E since clD E <br />

clC 2 C 1 since E C 1<br />

.<br />

Exercise Prove that every connected metric space with at least two points is uncountable.<br />

Proof: LetX be a connected metric space with two points a and b, where<br />

a b. Define a set A r x : dx, a r and B r x : dx, a r. It is clear<br />

that both of sets are open and disjoint. Assume X is countable. Let<br />

da,b<br />

r , da,b , it guarantee that both of sets are non-empty. Since<br />

4 2<br />

da,b<br />

, da,b is uncountable, we know that there is a 0 such that<br />

4 2<br />

A B X. It implies that X is disconnected. So, we know that such X is<br />

countable.<br />

Uniform continuity<br />

4.50 Prove that a function which is uniformly continuous on S is also continuous on S.<br />

Proof: Let f be uniformly continuous on S, thengiven 0, there exists a 0 such<br />

that as dx, y , x and y in S, then we have<br />

dfx, fy .<br />

Fix y, called a. <strong>The</strong>ngiven 0, there exists a 0 such that as dx, a , x in S,<br />

then we have<br />

dfx, fa .<br />

That is, f is continuous at a. Sincea is arbitrary, we know that f is continuous on S.<br />

4.51 If fx x 2 for x in R, prove that f is not uniformly continuous on R.<br />

Proof: Assume that f is uniformly continuous on R, thengiven 1, there exists a<br />

0 such that as |x y| , wehave<br />

|fx fy| 1.<br />

Choose x y ,(<br />

2<br />

|x y| , then we have<br />

2<br />

|fx fy| y 1. 2<br />

When we choose y 1 , then 1 <br />

2<br />

2<br />

1 <br />

2<br />

1<br />

which is absurb. Hence, we know that f is not uniformly continuous on R.<br />

Remark: <strong>The</strong>re are some similar questions written below.<br />

1. Here is a useful lemma to make sure that a function is uniformly continuous on<br />

a, b, but we need its differentiability.<br />

(Lemma) Letf : a, b R R be differentiable and |f x| M for all x a, b.<br />

<strong>The</strong>n f is uniformly continuous on a, b, wherea, b may be .


Proof: By Mean Value <strong>The</strong>orem, wehave<br />

|fx fy| |f z||x y|, wherez x, y or y, x<br />

M|x y| by hypothesis. *<br />

<strong>The</strong>n given 0, there is a /M such that as |x y| , x, y a, b, wehave<br />

|fx fy| , by (*).<br />

Hence, we know that f is uniformly continuous on a, b.<br />

Note: A standard example is written in Remark 2. But in Remark 2, we still use<br />

definition of uniform continuity to practice what it says.<br />

2. sin x is uniformly continuous on R.<br />

Proof: Given 0, we want to find a 0 such that as |x y| , wehave<br />

Since sin x sin y 2cos xy<br />

2<br />

|sin x sin y| .<br />

xy<br />

sin ,<br />

2<br />

|sin x| |x|, and|cosx| 1, we have<br />

|sin x sin y| |x y|<br />

So, if we choose , then as |x y| , it implies that<br />

|sin x sin y| .<br />

That is, sinx is uniformly continuous on R.<br />

Note: |sin x sin y| |x y| for all x, y R, can be proved by Mean Value <strong>The</strong>orem<br />

as follows.<br />

proof: By Mean Value <strong>The</strong>orem, sin x sin y sin z x y; itimpliesthat<br />

|sin x sin y| |x y|.<br />

3. sinx 2 is NOT uniformly continuous on R.<br />

Proof: Assume that sinx 2 is uniformly continuous on R. <strong>The</strong>ngiven 1, there is a<br />

0 such that as |x y| , wehave<br />

|sinx 2 siny 2 | 1. *<br />

Consider<br />

n <br />

2 n 2<br />

n n 0,<br />

4 n<br />

2<br />

and thus choose N <br />

<br />

<br />

1 <br />

4 2 4 2<br />

N 2<br />

which implies<br />

N .<br />

So, choose x N 2<br />

and y N , thenby(*),wehave<br />

|fx fy| sin N sinN sin<br />

2<br />

1 1<br />

2<br />

which is absurb. So, sinx 2 is not uniformly continuous on R.<br />

4. x is uniformly continuous on 0, .<br />

Proof: Since x y |x y| for all x, y 0, , thengiven 0, there exists<br />

a 2 such that as |x y| , x, y 0, , wehave<br />

So, we know that<br />

x y |x y| .<br />

x is uniformly continuous on 0, .


Note: We have the following interesting results:. Prove that, for x 0, y 0,<br />

|x p y p | <br />

|x y| p if 0 p 1,<br />

p|x y|x p1 y p1 if 1 p .<br />

Proof: (As 0 p 1) Without loss of generality, let x y, consider<br />

fx x y p x p y p , then<br />

f x p x y p1 x p1 0, note that p 1 0.<br />

So, we have f is an increasing function defined on 0, for all given y 0. Hence, we<br />

have fx f0 0. So,<br />

x p y p x y p if x y 0<br />

which implies that<br />

|x p y p | |x y| p<br />

for x 0, y 0.<br />

Ps: <strong>The</strong> inequality, we can prove the case p 1/2 directly. Thus the inequality is not<br />

surprising for us.<br />

(As 1 p Without loss of generality, let x y, consider<br />

x p y p pz p1 x y, wherez y, x, byMean Value <strong>The</strong>orem.<br />

which implies<br />

for x 0, y 0.<br />

px p1 x y, note that p 1 0,<br />

px p1 y p1 x y<br />

|x p y p | p|x y|x p1 y p1 <br />

5. In general, we have<br />

x r is uniformly continuous on 0, , ifr 0, 1,<br />

is NOT uniformly continuous on 0, , ifr 1,<br />

and<br />

sinx r <br />

is uniformly continuous on 0, , ifr 0, 1,<br />

is NOT uniformly continuous on 0, , ifr 1.<br />

Proof: (x r )Asr 0, it means that x r is a constant function. So, it is obviuos. As<br />

r 0, 1, thengiven 0, there is a 1/r 0 such that as |x y| , x, y 0, ,<br />

we have<br />

|x r y r | |x y| r by note in the exercise<br />

r<br />

.<br />

So, x r is uniformly continuous on 0, , ifr 0, 1.<br />

As r 1, assume that x r is uniformly continuous on 0, , thengiven 1 0,<br />

there exists a 0 such that as |x y| , x, y 0, , wehave<br />

|x r y r | 1. *<br />

By Mean Value <strong>The</strong>orem, we have (let x y /2, y 0)


x r y r rz r1 x y<br />

ry r1 /2.<br />

So, if we choose y 2 1<br />

r1<br />

, then we have<br />

r<br />

x r y r 1<br />

which is absurb with (*). Hence, x r is not uniformly continuous on 0, .<br />

Ps: <strong>The</strong> reader should try to realize why x r is not uniformly continuous on 0, , for<br />

r 1. <strong>The</strong> ruin of non-uniform continuity comes from that x is large enough. At the same<br />

time, compare it with theorem that a continuous function defined on a compact set K is<br />

uniflormly continuous on K.<br />

(sin x r )Asr 0, it means that x r is a constant function. So, it is obviuos. As r 0, 1,<br />

given 0, there is a 1/r 0 such that as |x y| , x, y 0, , wehave<br />

|sin x r sin y r x<br />

| 2cos<br />

r y r<br />

sin<br />

xr y r<br />

2<br />

2<br />

|x r y r |<br />

|x y| r by the note in the Remark 4.<br />

r<br />

.<br />

So, sin x r is uniformly continuous on 0, , ifr 0, 1.<br />

As r 1, assume that sin x r is uniformly continuous on 0, , thengiven 1, there<br />

is a 0 such that as |x y| , x, y 0, , wehave<br />

|sin x r sin y r | 1. **<br />

Consider a sequence n 2 1/r n 1/r , it is easy to show that the sequence tends to<br />

0asn . So, there exists a positive integer N such that |x y| , x n 2 1/r ,<br />

y n 1/r . <strong>The</strong>n<br />

sin x r sin y r 1<br />

which contradicts (**). So, we know that sin x r is not uniformly continuous on 0, .<br />

Ps: For n 2 1/r n 1/r : x n 0asn 0, here is a short proof by using<br />

L-Hospital Rule.<br />

Proof: Write<br />

x n n 1/r<br />

n<br />

1/r<br />

2<br />

n 1/r 1 1 2n<br />

1/r<br />

1<br />

and thus consider the following limit<br />

<br />

1 1<br />

2n 1/r 1<br />

n 1/r


lim x<br />

1 1 2x 1/r 1<br />

x 1/r , 0<br />

0<br />

lim 1/r<br />

x 2 x 1 r 1<br />

1 <br />

2x<br />

1 1 r 1<br />

by L-Hospital Rule.<br />

0.<br />

Hence x n 0asn .<br />

6. Here is a useful criterion for a function which is NOT uniformly continuous defined<br />

a subset A in a metric space. We say a function f is not uniformly continuous on a subset A<br />

in a metric space if, and only if, there exists 0 0, and two sequences x n and y n <br />

such that as<br />

lim n<br />

x n y n 0<br />

which implies that<br />

|fx n fy n | 0 for n is large enough.<br />

<strong>The</strong> criterion is directly from the definition on uniform continuity. So, we omit the<br />

proof.<br />

4.52 Assume that f is uniformly continuous on a bounded set S in R n . Prove that f<br />

must be bounded on S.<br />

Proof: Since f is uniformly continuous on a bounded set S in R n ,given 1, then<br />

there exists a 0 such that as x y , x, y S, wehave<br />

dfx, fy 1.<br />

Consider the closure of S, clS is closed and bounded. Hence clS is compact. <strong>The</strong>n for<br />

any open covering of clS, there is a finite subcover. That is,<br />

clS xclS Bx; /2,<br />

clS kn k1 Bx k ; /2, wherex k clS,<br />

S kn k1 Bx k ; /2, wherex k clS.<br />

Note that if Bx k ; /2 S for some k, then we remove this ball. So, we choose<br />

y k Bx k ; /2 S, 1 k n and thus we have<br />

Bx k ; /2 By k ; for 1 k n,<br />

since let z Bx k ; /2,<br />

z y k z x k x k y k /2 /2 .<br />

Hence, we have<br />

S kn k1 By k ; , wherey k S.<br />

Given x S, then there exists By k ; for some k such that x By k ; . So,<br />

dfx, fx k 1 fx Bfy k ;1<br />

Note that kn k1 Bfy k ;1 is bounded since every Bfy k ;1 is bounded. So, let B be a<br />

bounded ball so that kn k1 Bfy k ;1 B. Hence, we have every x S, fx B. Thatis,<br />

f is bounded.<br />

Remark: If we know that the codomain is complete, then we can reduce the above<br />

proof. See Exercise 4.55.<br />

4.53 Let f be a function defined on a set S in R n and assume that fS R m .Letg be<br />

defined on fS with value in R k , and let h denote the composite function defined by


hx gfx if x S. Iff is uniformly continuous on S and if g is uniformly continuous<br />

on fS, show that h is uniformly continuous on S.<br />

Proof: Given 0, we want to find a 0 such that as x y R<br />

n , x, y S, we<br />

have<br />

hx hy gfx gfy .<br />

For the same , sinceg is uniformly continuous on fS, then there exists a 0 such<br />

that as fx fy R<br />

m ,wehave<br />

gfx gfy .<br />

For this ,sincef is uniformly continuous on S, then there exists a 0 such that as<br />

x y R<br />

n , x, y S, wehave<br />

fx fy R<br />

m .<br />

So, given 0, there is a 0 such that as x y R<br />

n , x, y S, wehave<br />

hx hy .<br />

That is, h is uniformly continuous on S.<br />

Remark: It should be noted that (Assume that all functions written are continuous)<br />

(1) uniform continuity uniform continuity uniform continuity.<br />

(2) uniform continuity NOT uniform continuity <br />

(3) NOT uniform continuity uniform continuity <br />

(a) NOT uniform continuity, or<br />

(b) uniform continuity.<br />

(a) NOT uniform continuity, or<br />

(b) uniform continuity.<br />

(4) NOT uniform continuity NOT uniform continuity <br />

(a) NOT uniform continuity, or<br />

(b) uniform continuity.<br />

For (1), it is from the exercise.<br />

For (2), (a) let fx x, andgx x 2 , x R fgx fx 2 x 2 .<br />

(b) let fx x ,andgx x 2 , x 0, fgx fx 2 x.<br />

For (3), (a) let fx x 2 ,andgx x, x R fgx fx x 2 .<br />

(b) let fx x 2 ,andgx x , x 0, fgx f x x.<br />

For (4), (a) let fx x 2 ,andgx x 3 , x R fgx fx 3 x 6 .<br />

(b) let fx 1/x, andgx 1 , x 0, 1 fgx f 1<br />

x .<br />

x x<br />

Note. In (4), we have x r is not uniformly continuous on 0, 1, forr 0. Here is a<br />

proof.<br />

Proof: Let r 0, and assume that x r is not uniformly continuous on 0, 1. Given<br />

1, there is a 0 such that as |x y| , wehave<br />

|x r y r | 1. *<br />

Let x n 2/n, andy n 1/n. <strong>The</strong>n x n y n 1/n. Choose n large enough so that 1/n .<br />

So, we have


|x r y r | 2 n<br />

r<br />

1 n<br />

r<br />

r<br />

1 n |2r 1| , asn since r 0,<br />

which is absurb with (*). Hence, we know that x r is not uniformly continuous on 0, 1, for<br />

r 0.<br />

Ps: <strong>The</strong> reader should try to realize why x r is not uniformly continuous on 0, 1, for<br />

r 0. <strong>The</strong> ruin of non-uniform continuity comes from that x is small enough.<br />

4.54 Assume f : S T is uniformly continuous on S, whereS and T are metric<br />

spaces. If x n is any Cauchy sequence in S, prove that fx n is a Cauchy sequence in T.<br />

(Compare with Exercise 4.33.)<br />

Proof: Given 0, we want to find a positive integer N such that as n, m N, we<br />

have<br />

dfx n , fx m .<br />

For the same , sincef is uniformly continuous on S, then there is a 0 such that as<br />

dx, y , x, y S, wehave<br />

dfx, fy .<br />

For this , sincex n is a Cauchy sequence in S, then there is a positive integer N such<br />

that as n, m N, wehave<br />

dx n , x m .<br />

Hence, given 0, there is a postive integer N such that as n, m N, wehave<br />

dfx n , fx m .<br />

That is, fx n is a Cauchy sequence in T.<br />

Remark: <strong>The</strong> reader should compare with Exercise 4.33 and Exercise 4.55.<br />

4.55 Let f : S T be a function from a metric space S to another metric space T.<br />

Assume that f is uniformly continuous on a subset A of S and let T is complete. Prove that<br />

there is a unique extension of f to clA which is uniformly continuous on clA.<br />

Proof: Since clA A A , it suffices to consider the case x A A. Since<br />

x A A, then there is a sequence x n A with x n x. Note that this sequence is a<br />

Cauchy sequence, so we have by Exercise 4.54, fx n is a Cauchy sequence in T since f<br />

is uniformly on A. In addition, since T is complete, we know that fx n is a convergent<br />

sequence, say its limit L. Note that if there is another sequence x n A with x n x,<br />

then fx n is also a convergent sequence, say its limit L . Note that x n x n is still a<br />

Cauchy sequence. So, we have<br />

dL, L dL, fx n dfx n , fx n dfx n , L 0asn .<br />

So, L L . That is, it is well-defined for g : clA T by the following<br />

gx <br />

fx if x A,<br />

lim n fx n if x A A, wherex n x.<br />

So, the function g is a extension of f to clA.<br />

Claim that this g is uniformly continuous on clA. That is, given 0, we want to<br />

find a 0 such that as dx, y , x, y clA, wehave<br />

dgx, gy .<br />

Since f is uniformly continuous on A, for /3, there is a 0 such that as


dx, y , x, y A, wehave<br />

dfx, fy .<br />

Let x, y clA, and thus we have x n A with x n x, andy n A with y n y.<br />

Choose /3, then we have<br />

dx n , x /3 and dy n , y /3 as n N 1<br />

So, as dx, y /3, wehave(n N 1 )<br />

dx n , y n dx n , x dx, y dy, y n /3 /3 /3 .<br />

Hence, we have as dx, y , (n N 1 )<br />

dgx, gy dgx, fx n dfx n , fy n dfy n , fy<br />

*<br />

dgx, fx n dfy n , gy<br />

<strong>And</strong> since lim n fx n gx, and lim n fy n gy, we can choose N N 1 such that<br />

dgx, fx n and<br />

dfy n , gy .<br />

So, as dx, y , (n N) wehave<br />

dgx, gy 3 by (*).<br />

That is, g is uniformly on clA.<br />

It remains to show that g is a unique extension of f to clA which is uniformly<br />

continuous on clA. If there is another extension h of f to clA which is uniformly<br />

continuous on clA, thengivenx A A, we have, by continuity, (Say x n x)<br />

hx h lim n<br />

x n lim n<br />

hx n lim n<br />

fx n lim n<br />

gx n g lim n<br />

x n gx<br />

which implies that hx gx for all x A A. Hence, we have hx gx for all<br />

x clA. Thatis,g is a unique extension of f to clA which is uniformly continuous on<br />

clA.<br />

Remark: 1. We do not require that A is bounded, in fact, A is any non-empty set in a<br />

metric space.<br />

2. <strong>The</strong> exercise is a criterion for us to check that a given function is NOT uniformly<br />

continuous. For example, let f : 0, 1 R by fx 1/x. Sincef0 does not exist, we<br />

know that f is not uniformly continuous. <strong>The</strong> reader should feel that a uniformly continuous<br />

is sometimes regarded as a smooth function. So, it is not surprising for us to know the<br />

exercise. Similarly to check fx x 2 , x R, and so on.<br />

3. Here is an exercise to make us know that a uniformly continuous is a smooth<br />

function. Let f : R R be uniformly continuous, then there exist , 0 such that<br />

|fx| |x| .<br />

Proof: Since f is uniformly continuous on R, given 1, there is a 0 such that as<br />

|x y| , wehave<br />

|fx fy| 1. *<br />

Given any x R, then there is the positive integer N such that N |x| N 1. If<br />

x 0, we consider<br />

y 0 0, y 1 /2, y 2 ,...,y 2N1 N 2 , y 2N x.<br />

<strong>The</strong>n we have


N<br />

|fx f0| |fy 2k fy 2k1 | |fy 2k1 fy 2k2 |<br />

k1<br />

2N by (*)<br />

which implies that<br />

|fx| 2N |f0|<br />

2 1 |x| |f0| since |x| N 1<br />

<br />

2 |x| 2 |f0|.<br />

<br />

Similarly for x 0. So, we have proved that |fx| |x| for all x.<br />

4.56 In a metric space S, d, letA be a nonempty subset of S. Define a function<br />

f A : S R by the equation<br />

f A x infdx, y : y A<br />

for each x in S. <strong>The</strong> number f A x is called the distance from x to A.<br />

(a) Prove that f A is uniformly continuous on S.<br />

(b) Prove that clA x : x S and f A x 0 .<br />

Proof: (a) Given 0, we want to find a 0 such that as dx 1 , x 2 , x 1 , x 2 S,<br />

we have<br />

|f A x 1 f A x 2 | .<br />

Consider (x 1 , x 2 , y S)<br />

dx 1 , y dx 1 , x 2 dx 2 , y, anddx 2 , y dx 1 , x 2 dx 1 , y<br />

So,<br />

infdx 1 , y : y A dx 1 , x 2 infdx 2 , y : y A and<br />

infdx 2 , y : y A dx 1 , x 2 infdx 1 , y : y A<br />

which implies that<br />

f A x 1 f A x 2 dx 1 , x 2 and f A x 2 f A x 1 dx 1 , x 2 <br />

which implies that<br />

|f A x 1 f A x 2 | dx 1 , x 2 .<br />

Hence, if we choose , thenwehaveasdx 1 , x 2 , x 1 , x 2 S, wehave<br />

|f A x 1 f A x 2 | .<br />

That is, f A is uniformly continuous on S.<br />

(b) Define K x : x S and f A x 0 , we want to show clA K. We prove it<br />

by two steps.<br />

() Letx clA, then Bx; r A for all r 0. Choose y k Bx;1/k A, then<br />

we have<br />

infdx, y : y A dx, y k 0ask .<br />

So, we have f A x infdx, y : y A 0. So, clA K.<br />

() Letx K, then f A x infdx, y : y A 0. That is, given any 0, there<br />

is an element y A such that dx, y . Thatis,y Bx; A. So,x is an adherent<br />

point of A. Thatis,x clA. So, we have K clA.<br />

From above saying, we know that clA x : x S and f A x 0 .<br />

Remark: 1. <strong>The</strong> function f A often appears in Analysis, so it is worth keeping it in mind.


In addition, part (b) comes from intuition. <strong>The</strong> reader may think it twice about distance 0.<br />

2. Here is a good exercise to pratice. <strong>The</strong> statement is that suppose that K and F are<br />

disjoint subsets in a metric space X, K is compact, F is closed. Prove that there exists a<br />

0 such that dp, q if p K, q F. Show that the conclusion is may fail for two<br />

disjoint closed sets if neither is compact.<br />

Proof: Suppose NOT, i.e., for any 0, there exist p K, andq F such that<br />

dp , q . Let 1/n, then there exist two sequence p n K, andq n F such<br />

that dp n , q n 1/n. Note that p n K, andK is compact, then there exists a<br />

subsequence p nk with lim nk p nk p K. Hence, we consider dp nk , q nk n 1 k<br />

to get a<br />

contradiction. Since<br />

dp nk , p dp, q nk dp nk , q nk n 1 , k<br />

then let n k , wehavelim nk q nk p. Thatis,p is an accumulation point of F which<br />

implies that p F. So, we get a contradiction since K F . That is, there exists a<br />

0 such that dp, q if p K, q F.<br />

We give an example to show that the conclusion does not hold. Let<br />

K x,0 : x R and F x,1/x : x 0, then K and F are closd. It is clear that<br />

such cannot be found.<br />

Note: Two disjoint closed sets may has the distance 0, however; if one of closed sets is<br />

compact, then we have a distance 0. <strong>The</strong> reader can think of them in R n , and note that<br />

a bounded and closed subsets in R n is compact. It is why the example is given.<br />

4.57 In a metric space S, d, letA and B be disjoint closed subsets of S. Prove that<br />

there exists disjoint open subsets U and V of S such that A U and B V. Hint. Let<br />

gx f A x f B x, in the notation of Exercise 4.56, and consider g 1 ,0 and<br />

g 1 0, .<br />

Proof: Let gx f A x f B x, then by Exercixe 4.56, we have gx is uniformly<br />

continuous on S. So,gx is continuous on S. Consider g 1 ,0 and g 1 0, , and<br />

note that A, B are disjoint and closed, then we have by part (b) in Exercise 4.56,<br />

gx 0ifx A and<br />

gx 0ifx B.<br />

So, we have A g 1 ,0 : U, andB g 1 0, : V.<br />

Discontinuities<br />

4.58 Locate and classify the discontinuities of the functions f defined on R 1 by the<br />

following equations:<br />

(a) fx sin x/x if x 0, f0 0.<br />

sinx<br />

Solution: f is continuous on R 0, and since lim x0 x 1, we know that f has a<br />

removable discontinuity at 0.<br />

(b) fx e 1/x if x 0, f0 0.<br />

Solution: f is continuous on R 0, and since lim x0<br />

e 1/x and lim x0<br />

e 1/x 0,<br />

we know that f has an irremovable discontinuity at 0.<br />

(c) fx e 1/x sin 1/x if x 0, f0 0.<br />

Solution: f is continuous on R 0, and since the limit fx does not exist as x 0,<br />

we know that f has a irremovable discontinuity at 0.


(d) fx 1/1 e 1/x if x 0, f0 0.<br />

Solution: f is continuous on R 0, and since lim x0<br />

e 1/x and lim x0<br />

e 1/x 0,<br />

we know that f has an irremovabel discontinuity at 0. In addition, f0 0and<br />

f0 1, we know that f has the lefthand jump at 0, f0 f0 1, and f is<br />

continuous from the right at 0.<br />

4.59 Locate the points in R 2 at which each of the functions in Exercise 4.11 is not<br />

continuous.<br />

(a) By Exercise 4.11, we know that fx, y is discontinuous at 0, 0, where<br />

fx, y x2 y 2<br />

if x, y 0, 0, andf0, 0 0.<br />

x 2 y2 Let gx, y x 2 y 2 ,andhx, y x 2 y 2 both defined on R 2 0, 0, we know that g<br />

and h are continuous on R 2 0, 0. Note that h 0onR 2 0, 0. Hence, f g/h is<br />

continuous on R 2 0, 0.<br />

(b) By Exercise 4.11, we know that fx, y is discontinuous at 0, 0, where<br />

xy 2<br />

fx, y <br />

if x, y 0, 0, andf0, 0 0.<br />

xy 2 2<br />

x y<br />

Let gx, y xy 2 ,andhx, y xy 2 x y 2 both defined on R 2 0, 0, we know<br />

that g and h are continuous on R 2 0, 0. Note that h 0onR 2 0, 0. Hence,<br />

f g/h is continuous on R 2 0, 0.<br />

(c) By Exercise 4.11, we know that fx, y is continuous at 0, 0, where<br />

fx, y 1 x sinxy if x 0, and f0, y y,<br />

since lim x,y0,0 fx, y 0 f0, 0. Letgx, y 1/x and hx, y sinxy both defined<br />

on R 2 0, 0, we know that g and h are continuous on R 2 0, 0. Note that h 0<br />

on R 2 0, 0. Hence, f g/h is continuous on R 2 0, 0. Hence, f is continuous on<br />

R 2 .<br />

(d) By Exercise 4.11, we know that fx, y is continuous at 0, 0, where<br />

x y sin1/x sin1/y if x 0andy 0,<br />

fx, y <br />

0 if x 0ory 0.<br />

since lim x,y0,0 fx, y 0 f0, 0. It is the same method as in Exercise 4.11, we know<br />

that f is discontinuous at x,0 for x 0andf is discontinuous at 0, y for y 0. <strong>And</strong> it is<br />

clearly that f is continuous at x, y, wherex 0andy 0.<br />

(e) By Exercise 4.11, Since<br />

we rewrite<br />

fx, y <br />

fx, y <br />

cos<br />

sinxsiny<br />

tan xtan y<br />

,iftanx tan y,<br />

cos 3 x if tan x tan y.<br />

xy<br />

cos xcos y<br />

2<br />

cos<br />

xy<br />

2<br />

if tan x tan y<br />

cos 3 x if tan x tan y.<br />

We consider x, y /2, /2 /2, /2, others are similar. Consider two cases (1)<br />

x y, and (2) x y, wehave<br />

(1) (x y) Since lim x,ya,a fx, y cos 3 a fa, a. Hence, we know that f is<br />

,<br />

.


continuous at a, a.<br />

(2) (x y) Sincex y, itimpliesthattanx tan y. Note that the denominator is not 0<br />

since x, y /2, /2 /2, /2. So, we know that f is continuous at a, b, a b.<br />

So, we know that f is continuous on /2, /2 /2, /2.<br />

Monotonic functions<br />

4.60 Let f be defined in the open interval a, b and assume that for each interior point x<br />

of a, b there exists a 1 ball Bx in which f is increasing. Prove that f is an increasing<br />

function throughout a, b.<br />

Proof: Suppose NOT, i.e., there exist p, q with p q such that fp fq. Consider<br />

p, q a, b, and since for each interior point x of a, b there exists a 1 ball Bx in<br />

which f is increasing. <strong>The</strong>n p, q xp,q Bx; x , (<strong>The</strong> choice of balls comes from the<br />

hypothesis). It implies that p, q n<br />

k1 Bx n ; n : B n . Note that if B i B j , we remove<br />

such B i and make one left. Without loss of generality, we assume that x 1 .. x n .<br />

.fp fx 1 ... fx n fq<br />

which is absurb. So, we know that f is an increasing function throughout a, b.<br />

4.61 Let f be continuous on a compact interval a, b and assume that f does not have a<br />

loacal maximum or a local minimum at any interior point. (See the note following Exercise<br />

4.25.) Prove that f must be monotonic on a, b.<br />

Proof: Since f is continuous on a, b, wehave<br />

max fx fp, wherep a, b and<br />

xa,b<br />

min fx fq, whereq a, b.<br />

xa,b<br />

So, we have p, q a, b by hypothesis that f does not have a local maximum or a local<br />

minimum at any interior point. Without loss of generality, we assume that p a, and<br />

q b. Claim that f is decreasing on a, b as follows.<br />

Suppose NOT, then there exist x, y a, b with x y such that fx fy. Consider<br />

x, y and by hyothesis, we know that f| x,y has the maximum at y, andf| a,y has the<br />

minimum at y. <strong>The</strong>n it implies that there exists By; x, y such that f is constant on<br />

By; x, y, which contradicts to the hypothesis. Hence, we have proved that f is<br />

decreasing on a, b.<br />

4.62 If f is one-to one and continuous on a, b, prove that f must be strictly<br />

monotonic on a, b. That is, prove that every topological mapping of a, b onto an<br />

interval c, d must be strictly monotonic.<br />

Proof: Since f is continuous on a, b, wehave<br />

max fx fp, wherep a, b and<br />

xa,b<br />

min fx fq, whereq a, b.<br />

xa,b<br />

Assume that p a, b, then there exists a 0 such that fy fp for all<br />

y p , p a, b. Choose y 1 x , x and y 2 x, x , thenwehaveby<br />

1-1, fy 1 fx and fy 2 fx. <strong>And</strong> thus choose r so that<br />

fy 1 r fx fz 1 r, wherez 1 y 1 , x by Intermediate Value <strong>The</strong>orem,<br />

fy 2 r fx fz 2 r, wherez 2 x, y 2 by Intermediate Value <strong>The</strong>orem,<br />

which contradicts to 1-1. So, we know that p a, b. Similarly, we have q a, b.


Without loss of generality, we assume that p a and q b. Claim that f is strictly<br />

decreasing on a, b.<br />

Suppose NOT, then there exist x, y a, b, with x y such that fx fy. (""<br />

does not hold since f is 1-1.) Consider x, y and by above method, we know that f| x,y has<br />

the maximum at y, andf| a,y has the minimum at y. <strong>The</strong>n it implies that there exists<br />

By; x, y such that f is constant on By; x, y, which contradicts to 1-1. Hence,<br />

for any x y a, b, wehavefx fy. ("" does not hold since f is 1-1.) So, we<br />

have proved that f is strictly decreasing on a, b.<br />

Reamrk: 1. Here is another proof by Exercise 4.61. It suffices to show that 1-1 and<br />

continuity imply that f does not have a local maximum or a local minimum at any interior<br />

point.<br />

Proof: Suppose NOT, it means that f has a local extremum at some interior point x.<br />

Without loss of generality, we assume that f has a local minimum at the interior point x.<br />

Since x is an interior point of a, b, then there exists an open interval<br />

x , x a, b such that fy fx for all y x , x . Note that f is 1-1, so<br />

we have fy fx for all y x , x x. Choose y 1 x and<br />

y 2 x, x , then we have fy 1 fx and fy 2 fx. <strong>And</strong> thus choose r so that<br />

fy 1 r fx fp r, wherep y 1 , x by Intermediate Value <strong>The</strong>orem,<br />

fy 2 r fx fq r, whereq x, y 2 by Intermediate Value <strong>The</strong>orem,<br />

which contradicts to the hypothesis that f is 1-1. Hence, we have proved that 1-1 and<br />

continuity imply that f does not have a local maximum or a local minimum at any interior<br />

point.<br />

2. Under the assumption of continuity on a compact interval, one-to-one is<br />

equivalent to being strictly monotonic.<br />

Proof: By the exercise, we know that an one-to-one and continuous function defined<br />

on a compact interval implies that a strictly monotonic function. So, it remains to show that<br />

a strictly monotonic function implies that an one-to-one function. Without loss of<br />

generality, let f be increasing on a, b, then as fx fy, we must have x y since if<br />

x y, then fx fy and if x y, then fx fy. So, we have proved that a strictly<br />

monotonic function implies that an one-to-one function. Hence, we get that under the<br />

assumption of continuity on a compact interval, one-to-one is equivalent to being strictly<br />

monotonic.<br />

4.63 Let f be an increasing function defined on a, b and let x 1 ,..,x n be n points in<br />

the interior such that a x 1 x 2 ... x n b.<br />

n<br />

(a) Show that k1<br />

fx k fx k fb fa .<br />

Proof: Let a x 0 and b x n1 ;sincef is an increasing function defined on a, b, we<br />

know that both fx k and fx k exist for 1 k n. Assume that y k x k , x k1 , then<br />

we have fy k fx k and fx k1 fy k1 . Hence,<br />

n<br />

<br />

k1<br />

fx k fx k fy k fy k1 <br />

n<br />

k1<br />

fy n fy 0 <br />

fb fa .<br />

(b) Deduce from part (a) that the set of dicontinuities of f is countable.


Proof: Let D denote the set of dicontinuities of f. Consider<br />

D m x a, b : fx fx m 1 , then D <br />

m1 D m . Note that #D m , so<br />

we have D is countable. That is, the set of dicontinuities of f is countable.<br />

(c) Prove that f has points of continuity in every open subintervals of a, b.<br />

Proof: By (b), f has points of continuity in every open subintervals of a, b, since<br />

every open subinterval is uncountable.<br />

Remark: (1) Here is another proof about (b). Denote Q x 1 ,...,x n ,..., and let x be<br />

a point at which f is not continuous. <strong>The</strong>n we have fx fx 0. (If x is the end<br />

point, we consider fx fx 0orfx fx 0) So, we have an open interval I x<br />

such that I x fa, b fx. <strong>The</strong> interval I x contains infinite many rational numbers,<br />

we choose the smallest index, say m mx. <strong>The</strong>n the number of the set of discontinuities<br />

of f on a, b is a subset of N. Hence, the number of the set of discontinuities of f on a, b<br />

is countable.<br />

(2) <strong>The</strong>re is a similar exercise; we write it as a reference. Let f be a real valued function<br />

defined on 0, 1. Suppose that there is a positive number M having the following<br />

condition: for every choice of a finite number of points x 1 ,..,x n in 0, 1, wehave<br />

n<br />

M i1<br />

x i M. Prove that S : x 0, 1 : fx 0 is countable.<br />

Proof: Consider S n x 0, 1 : |fx| 1/n, then it is clear that every S n is<br />

countable. Since S <br />

n1 S n , we know that S is countable.<br />

4.64 Give an example of a function f, defined and strictly increasing on a set S in R,<br />

such that f 1 is not continuous on fS.<br />

Solution: Let<br />

x if x 0, 1,<br />

fx <br />

1ifx 2.<br />

<strong>The</strong>n it is clear that f is strictly increasing on 0, 1, sof has the incerse function<br />

x if x 0, 1,<br />

f 1 x <br />

2ifx 1.<br />

which is not continuous on fS 0, 1.<br />

Remark: Compare with Exercise 4.65.<br />

4.65 Let f be strictly increasing on a subset S of R. Assume that the image fS has one<br />

of the following properties: (a) fS is open; (b) fS is connected; (c) fS is closed. Prove<br />

that f must be continuous on S.<br />

Proof: (a) Given a S, then fa fS. Given 0, wewantofinda 0 such<br />

that as x Ba; S, wehave|fx fa| . SincefS is open, then there exists<br />

Bfa, fS, where .<br />

Claim that there exists a 0 such that fBa; S Bfa, . Choose<br />

y 1 fa /2 and y 2 fa /2, then y 1 fx 1 and y 2 fx 2 ,wehave<br />

x 1 a x 2 since f is strictly increasing on S. Hence, for x x 1 , x 2 S, wehave<br />

fx 1 fx fx 2 since f is strictly increasing on S. So,fx Bfa, .Let<br />

mina x 1 , x 2 a, then Ba; S a , a S x 1 , x 2 S which implis<br />

that fBa; S Bfa, .( Bfa, )<br />

Hence we have prove the claim, and the claim tells us that f is continuous at a. Sincea<br />

is arbitrary, we know that f is continuous on S.


(b) Note that since fS R, andfS is connected, we know that fS is an interval I.<br />

Given a S, then fa I. We discuss 2 cases as follows. (1) fa is an interior point of I.<br />

(2) fa is the endpoint of I.<br />

For case (1), it is similar to (a). We omit the proof.<br />

For case (2), it is similar to (a). We omit the proof.<br />

So, we have proved that f is continuous on S.<br />

(c) Given a S, then fa fS. SincefS is closed, we consider two cases. (1) fa<br />

is an isolated point and (2) fa is an accumulation point.<br />

For case (1), claim that a is an isolated point. Suppose NOT, there is a sequence<br />

<br />

x n S with x n a. Consider x n n1<br />

x : x n a x : x n a, and thus we<br />

may assume that x : x n a : a n is a infinite subset of x n <br />

n1<br />

.Sincef is<br />

monotonic, we have lim n fx n fa . SincefS is closed, we have fa fS.<br />

<strong>The</strong>refore, there exists b fS such that fa fb fa.<br />

If fb fa, then b a since f is strictly increasing. But is contradicts to that fa is<br />

isolated. On the other hand, if fb fa, then b a since f is strictly increasing. In<br />

addition, fa n fa fb implies that a n b. But is contradicts to that a n a.<br />

Hence, we have proved that a is an isolated point. So, f is sutomatically continuous at<br />

a.<br />

For case (2), suppose that fa is an accumulation point. <strong>The</strong>n Bfa; fS and<br />

Bfa; has infinite many numbers of points in fS. Choose y 1 , y 2 Bfa; fS<br />

with y 1 y 2 , then fx 1 y 1 ,andfx 2 y 2 . <strong>And</strong> thus it is similar to (a), we omit the<br />

proof.<br />

So, we have proved that f is continuous on S by (1) and (2).<br />

Remark: In (b), when we say f is monotonic on a subset of R, its image is also in R.<br />

Supplement.<br />

It should be noted that the discontinuities of a monotonic function need not be isolated.<br />

In fact, given any countable subset E of a, b, which may even be dense, we can<br />

construct a function f, monotonic on a, b, discontinuous at every point of E, and at<br />

no other point of a, b. To show this, let the points of E be arranged in a sequence x n ,<br />

n 1, 2, . . . Let c n be a sequence of positive numbers such that c n converges. Define<br />

fx c n a x b<br />

x nx<br />

Note: <strong>The</strong> summation is to be understood as follows: Sum over those indices n for whcih<br />

x n x. If there are no points x n to the left of x, the sum is empty; following the usual<br />

convention, we define it to be zero. Since absolute convergence, the order in which the<br />

terms are arranged is immaterial.<br />

<strong>The</strong>n fx is desired.<br />

<strong>The</strong> proof that we omit; the reader should see the book, Principles of Mathematical<br />

Analysis written by Walter Rudin, pp97.<br />

Metric space and fixed points<br />

4.66 Let BS denote the set of all real-valued functions which are defined and<br />

bounded on a nonempty set S. Iff BS, let<br />

f sup<br />

xS<br />

<strong>The</strong> number f is called the " sup norm " of f.<br />

|fx|.


(a) Provet that the formula df, g f g defines a metric d on BS.<br />

Proof: We prove that d isametriconBS as follows.<br />

(1) If df, g 0, i.e., f g sup xS |fx gx| 0 |fx gx| for all x S.<br />

So, we have f g on S.<br />

(2) If f g on S, then |fx gx| 0 for all x S. Thatis,f g 0 df, g.<br />

(3) Given f, g BS, then<br />

df, g f g<br />

sup|fx gx|<br />

xS<br />

sup|gx fx|<br />

xS<br />

g f<br />

dg, f.<br />

(4) Given f, g, h BS, thensince<br />

|fx gx| |fx hx| |hx gx|,<br />

we have<br />

|fx gx| <br />

sup<br />

xS<br />

|fx hx| sup|hx gx|<br />

f h h g<br />

which implies that<br />

f g sup|fx gx| f h h g.<br />

xS<br />

So,wehaveprovethatd isametriconBS.<br />

(b) Prove that the metric space BS, d is complete. Hint: If f n is a Cauchy<br />

sequence in BS, show that f n x is a Cauchy sequence of real numbers for each x in S.<br />

Proof: Let f n be a Cauchy sequence on BS, d, That is, given 0, there is a<br />

positive integer N such that as m, n N, wehave<br />

df, g f n f m sup|f n x f m x| . *<br />

xS<br />

So, for every point x S, the sequence f n x R is a Cauchy sequence. Hence, the<br />

sequence f n x is a convergent sequence, say its limit fx. It is clear that the function<br />

fx is well-defined. Let 1 in (*), then there is a positive integer N such that as<br />

m, n N, wehave<br />

|f n x f m x| 1, for all x S. **<br />

Let m , andn N, wehaveby(**)<br />

|f N x fx| 1, for all x S<br />

which implies that<br />

|fx| 1 |f N x|, for all x S.<br />

Since |f N x| BS, say its bound M, and thus we have<br />

|fx| 1 M, for all x S<br />

which implies that fx is bounded. That is, fx BS, d. Hence, we have proved that<br />

BS, d is a complete metric space.<br />

Remark: 1. We do not require that S is bounded.<br />

2. <strong>The</strong> boundedness of a function f cannot be remove since sup norm of f is finite.<br />

xS


3. <strong>The</strong> sup norm of f, often appears and is important; the reader should keep it in mind.<br />

<strong>And</strong> we will encounter it when we discuss on sequences of functions. Also, see Exercise<br />

4.67.<br />

4. Here is an important theorem, the reader can see the definition of uniform<br />

convergence in the text book, page 221.<br />

4.67 Refer to Exercise 4.66 and let CS denote the subset of BS consisting of all<br />

funtions continuous and bounded on S, where now S is a metric space.<br />

(a) Prove that CS is a closed subset of BS.<br />

Proof: Let f be an adherent point of CS, then Bf; r CS for all r 0. So,<br />

there exists a sequence f n x such that f n f as n . So, given 0, there is a<br />

positive integer N such that as n N, wehave<br />

df n , f f n f sup|f n x fx| .<br />

xS<br />

So, we have<br />

|f N x fx| . for all x S. *<br />

Given s S, and note that f N x CS, so for this , there exists a 0 such that as<br />

|x s| , x, s S, wehave<br />

|f N x f N s| . **<br />

We now prove that f is continuous at s as follows. Given 0, and let /3, then there<br />

is a 0 such that as |x s| , x, s S, wehave<br />

|fx fs| |fx f N x| |f N x f N s| |f N s fs|<br />

/3 /3 /3 by (*) and (**)<br />

.<br />

Hence, we know that f is continuous at s, and since s is arbitrary, we know that f is<br />

continuous on S.<br />

(b) Prove that the metric subspace CS is complete.<br />

Proof: By (a), we know that CS is complete since a closed subset of a complete<br />

metric space is complete.<br />

Remark: 1. In (b), we can see Exercise 4.9.<br />

2. <strong>The</strong> reader should see the text book in Charpter 9, and note that <strong>The</strong>orem 9.2 and<br />

<strong>The</strong>orem 9.3.<br />

4.68 Refer to the proof of the fixed points theorem (<strong>The</strong>orem 4.48) for notation.<br />

(a) Prove that dp, p n dx, fx n /1 .<br />

Proof: <strong>The</strong> statement is that a contraction f of a complete metric space S has a unique<br />

fixed point p. Take any point x S, and consider the sequence of iterates:<br />

x, fx, ffx,...<br />

That is, define a sequence p n inductively as follows:<br />

p 0 x, p n1 fp n n 0, 1, 2, . . .<br />

We will prove that p n converges to a fixed point of f. First we show that p n is a<br />

Cauchy sequence. Since f is a contraction (dfx, fy dx, y, 0 1 for all<br />

x, y S), we have<br />

dp n1 , p n dfp n , fp n1 dp n , p n1 ,


so, by induction, we find<br />

dp n1 , p n n dp 1 , p 0 n dx, fx.<br />

Use the triangel inequality we find, for m n,<br />

m1<br />

dp m , p n dp k1 , p k <br />

kn<br />

m1<br />

dx, fx k<br />

kn<br />

dx, fx n m<br />

1 <br />

dx, fx<br />

n<br />

1 . *<br />

Since n 0asn , we know that p n is a Cauchy sequence. <strong>And</strong> since S is<br />

complete, we have p n p S. <strong>The</strong> uniqueness is from the inequality,<br />

dfx, fy dx, y.<br />

From (*), we know that (let m )<br />

dp, p n dx, fx n<br />

1 .<br />

This inequality, which is useful in numberical work, provides an estimate for the<br />

distance from p n to the fixed point p. An example is given in (b)<br />

(b) Take fx 1 x 2/x, S 1, . Prove that f is contraction of S with<br />

2<br />

contraction constant 1/2 and fixed point p 2 . Form the sequence p n starting wth<br />

x p 0 1 and show that p n 2 2 n .<br />

Proof: First, fx fy 1 x 2/x 1 y 2/y 1 yx<br />

x y 2<br />

2 2 2 xy , then we<br />

have<br />

|fx fy| 1 y x<br />

x y 2<br />

2 xy<br />

1 2 x y 1 xy<br />

2<br />

1 2 |x y| since 1 2 xy 1.<br />

So, f is a contraction of S with contraction constant 1/2. By Fixed Point <strong>The</strong>orem, we<br />

know that there is a unique p such that fp p. Thatis,<br />

1<br />

2 p 2 p p p 2 .( 2 is not our choice since S 1, .)<br />

By (a), it is easy to know that<br />

p n 2 2 n .<br />

Remark: Here is a modefied Fixed Point <strong>The</strong>orem: Let f be function defined on a<br />

complete metric space S. If there exists a N such that df N x f N y dx, y for all<br />

x, y S, where 0 1. <strong>The</strong>n f has a unique fixed point p S.<br />

Proof: Since f N is a contraction defined on a complete metric space, with the<br />

contraction constant , with 0 1, by Fixed Point <strong>The</strong>orem, we know that there<br />

exists a unique point p S, such that


f N p p<br />

ff N p fp<br />

f N fp fp.<br />

That is, fp is also a fixed point of f N . By uniqueness, we know that fp p. In addition,<br />

if there is p S such that fp p . <strong>The</strong>n we have<br />

f 2 p fp p ,...,f N p .. p . Hence, we have p p .Thatis,f has a unique<br />

fixed point p S.<br />

4.69 Show by counterexample that the fixed-point theorem for contractions need not<br />

hold if either (a) the underlying metric space is not complete, or (b) the contraction<br />

constant 1.<br />

Solution: (a) Let f 1 1 x : 0, 1 R, then 2 |fx fy| 1 2<br />

|x y|. So,f is a<br />

contraction on 0, 1. However, it has no any fixed point since if it has, say this point p, we<br />

get 1 1 p p p 1 0, 1.<br />

2<br />

(b) Let f 1 x : 0, 1 R, then |fx fy| |x y|. So,f is a contraction with<br />

the contraction constant 1. However, it has no any fixed point since if it has, say this point<br />

p, weget1 p p 1 0, a contradiction.<br />

4.70 Let f : S S be a function from a complete metric space S, d into itself.<br />

Assume there is a real sequence a n which converges to 0 such that<br />

df n x, f n y n dx, y for all n 1 and all x, y in S, wheref n is the nth iterate of f; that<br />

is,<br />

f 1 x fx, f n1 x ff n x for n 1.<br />

Prove that f has a unique point. Hint. Apply the fixed point theorem to f m for a suitable m.<br />

Proof: Since a n 0, given 1/2, then there is a positive integer N such that as<br />

n N, wehave<br />

|a n| 1/2.<br />

Note that a n 0 for all n. Hence, we have<br />

df N x, f N y 1 dx, y for x, y in S.<br />

2<br />

That is, f N x is a contraction defined on a complete metric space, with the contraction<br />

constant 1/2. By Fixed Point <strong>The</strong>orem, we know that there exists a unique point p S,<br />

such that<br />

f N p p<br />

ff N p fp<br />

f N fp fp.<br />

That is, fp is also a fixed point of f N . By uniqueness, we know that fp p. In addition,<br />

if there is p S such that fp p . <strong>The</strong>n we have<br />

f 2 p fp p ,...,f N p .. p . Hence, we have p p .Thatis,f has a unique<br />

fixed point p S.<br />

4.71 Let f : S S be a function from a metric space S, d into itself such that<br />

dfx, fy dx, y<br />

where x y.<br />

(a) Prove that f has at most one fixed point, and give an example of such an f with no<br />

fixed point.


Proof: If p and p are fixed points of f where p p , then by hypothesis, we have<br />

dp, p dfp, fp dp, p <br />

which is absurb. So, f has at most one fixed point.<br />

Let f : 0, 1/2 0, 1/2 by fx x 2 . <strong>The</strong>n we have<br />

|fx fy| |x 2 y 2 | |x y||x y| |x y|.<br />

However, f has no fixed point since if it had, say its fixed point p, then<br />

p 2 p p 1 0, 1/2 or p 0 0, 1/2.<br />

(b) If S is compact, prove that f has exactly one fixed point. Hint. Show that<br />

gx dx, fx attains its minimum on S.<br />

Proof: Let g dx, fx, and thus show that g is continuous on a compact set S as<br />

follows. Since<br />

dx, fx dx, y dy, fy dfy, fx<br />

dx, y dy, fy dx, y<br />

2dx, y dy, fy<br />

dx, fx dy, fy 2dx, y *<br />

and change the roles of x, andy, wehave<br />

dy, fy dx, fx 2dx, y **<br />

Hence, by (*) and (**), we have<br />

|gx gy| |dx, fx dy, fy| 2dx, y for all x, y S. ***<br />

Given 0, there exists a /2 such that as dx, y , x, y S, wehave<br />

|gx gy| 2dx, y by (***).<br />

So, we have proved that g is uniformly continuous on S.<br />

So, consider min xS gx gp, p S. We show that gp 0 dp, fp. Suppose<br />

NOT, i.e., fp p. Consider<br />

df 2 p, fp dfp, p gp<br />

which contradicts to gp is the absolute maximum. Hence, gp 0 p fp. Thatis,<br />

f has a unique fixed point in S by (a).<br />

(c)Give an example with S compact in which f is not a contraction.<br />

Solution: Let S 0, 1/2, andf x 2 : S S. <strong>The</strong>n we have<br />

|x 2 y 2 | |x y||x y| |x y|.<br />

So, this f is not contraction.<br />

Remark: 1. In (b), the Choice of g is natural, since we want to get a fixed point. That<br />

is, fx x. Hence, we consider the function g dx, fx.<br />

2. Here is a exercise that makes us know more about Remark 1. Let f : 0, 1 0, 1<br />

be a continuous function, show that there is a point p such that fp p.<br />

Proof: Consider gx fx x, then g is a continuous function defined on 0, 1.<br />

Assume that there is no point p such that gp 0, that is, no such p so that fp p. So,<br />

by Intermediate Value <strong>The</strong>orem, we know that gx 0 for all x 0, 1, orgx 0<br />

for all x 0, 1. Without loss of generality, suppose that gx 0 for all x 0, 1 which<br />

is absurb since g1 f1 1 0. Hence, we know that there is a point p such that<br />

fp p.


3. Here is another proof on (b).<br />

Proof: Given any point x S, and thus consider f n x S. <strong>The</strong>n there is a<br />

convergent subsequence f nk x, say its limit p, sinceS is compact. Consider<br />

dfp, p d f limf nk x , limf nk x<br />

k k<br />

and<br />

Note that<br />

d limff nk x, limf nk x by continuity of f at p<br />

k k<br />

limdf nk1 x, f nk x 1<br />

k<br />

df nk1 x, f nk x ... df 2 f nk1 x, ff nk1 x. 2<br />

limdf 2 f nk1 x, ff nk1 x<br />

k<br />

d limf 2 f nk1 x, limff nk1 x<br />

k k<br />

d f 2 limf nk1 x , f limf nk1 x by continuity of f 2 and f at p<br />

k k<br />

df 2 p, fp. 3<br />

So, by (1)-(3), we know that<br />

fp, fp df 2 p, fp p fp<br />

by hypothesis<br />

dfx, fy dx, y<br />

where x y. Hence, f has a unique fixed point p by (a) in Exercise.<br />

Note. 1.Ifx n x, andy n y, then dx n , y n dx, y. Thatis,<br />

lim n<br />

dx n , y n d lim n<br />

x n , lim n<br />

y n .<br />

Proof: Consider<br />

dx n , y n dx n , x dx, y dy, y n and<br />

dx, y dx, x n dx n , y n dy n , y,<br />

then<br />

|dx n , y n dx, y| dx, x n dy, y n 0.<br />

So,wehaveproveit.<br />

2. <strong>The</strong> reader should compare the method with Exercise 4.72.<br />

4.72 Assume that f satisfies the condition in Exercise 4.71. If x S, letp 0 x,<br />

p n1 fp n ,andc n dp n , p n1 for n 0.<br />

(a) Prove that c n is a decreasing sequence, and let c limc n .<br />

Proof: Consider<br />

c n1 c n dp n1 , p n2 dp n , p n1 <br />

dfp n , fp n1 dp n , p n1 <br />

dp n , p n1 dp n , p n1 <br />

0,<br />

so c n is a decreasing sequence. <strong>And</strong> c n has a lower bound 0, by Completeness of R,<br />

we know that c n is a convergent sequence, say c limc n .


(b) Assume there is a subsequence p kn which converges to a point q in S. Prove that<br />

c dq, fq dfq, ffq.<br />

Deduce that q is a fixed point of f and that p n q.<br />

Proof: Since lim n p kn q, and lim n c n c, wehavelim n c kn c. So,we<br />

consider<br />

c lim n<br />

c kn<br />

lim n<br />

dp kn , p kn1 <br />

lim n<br />

dp kn , fp kn <br />

dq, fq<br />

and<br />

dp kn , p kn1 dp kn1 , p kn ... dfp kn1 , f 2 p kn1 ,<br />

we have<br />

c dq, fq lim n<br />

dfp kn1 , f 2 p kn1 dfq, f 2 q. *<br />

So, by (*) and hypoethesis<br />

dfx, fy dx, y<br />

where x y, we know that q fq c 0, in fact, this q is a unique fixed point. .<br />

In order to show that p n p, we consider (let m kn)<br />

dp m , q dp m , fq dp m1 , q .. dp kn , q<br />

So, given 0, there exists a positive integer N such that as n N, wehave<br />

dp kn , q .<br />

Hence, as m kN, wehave<br />

dp m , q .<br />

That is, p n p.


Derivatives<br />

<strong>Real</strong>-valued functions<br />

In each following exercise assume, where mecessary, a knowledge of the formulas for<br />

differentiating the elementary trigonometric, exponential, and logarithmic functions.<br />

5.1 Assume that f is said to satisfy a Lipschitz condition of order at c if there<br />

exists a positive number M (which may depend on c) and 1 ball Bc such that<br />

|fx fc| M|x c| <br />

whenever x Bc, x c.<br />

(a) Show that a function which satisfies a Lipschitz condition of order is continuous<br />

at c if 0, and has a derivative at c if 1.<br />

Proof: 1. As 0, given 0, there is a /M 1/ such that as<br />

x c , c Bc, wehave<br />

|fx fc| M|x c| M .<br />

So, we know that f is continuous at c.<br />

2. As 1, consider x Bc, andx c, wehave<br />

fx fc<br />

x c M|x c| 1 0asx c.<br />

So, we know that f has a derivative at c with f c 0.<br />

Remark: It should be note that (a) also holds if we consider the higher dimension.<br />

(b) Given an example of a function satisfying a Lipschitz condition of order 1 at c for<br />

which f c does not exist.<br />

Solution: Consider<br />

||x| |c|| |x c|,<br />

we know that |x| is a function satisfying a Lipschitz condition of order 1 at 0 for which<br />

f 0 does not exist.<br />

5.2 In each of the following cases, determine the intervals in which the function f is<br />

increasing or decreasing and find the maxima and minima (if any) in the set where each f is<br />

defined.<br />

(a) fx x 3 ax b, x R.<br />

Solution: Since f x 3x 2 a on R, we consider two cases: (i) a 0, and (ii) a 0.<br />

(i) As a 0, we know that f is increasig on R by f 0onR. In addition, if f has a<br />

local extremum at some point c, then f c 0. It implies that a 0andc 0. That is,<br />

fx x 3 b has a local extremum at 0. It is impossible since x 3 does not. So, we know<br />

that f has no maximum and minimum.<br />

(ii) As a 0, since f 3x 2 a 3 x a/3 x a/3 , we know that<br />

f x :<br />

, a/3 a/3 , a/3 a/3 , <br />

0 0 0<br />

which implies that<br />

fx :<br />

, a/3 a/3 , a/3 a/3 , <br />

<br />

. *


Hence, f is increasing on , a/3 and a/3 , , and decreasing on<br />

a/3 , a/3 . In addition, if f has a local extremum at some point c, then f c 0.<br />

It implies that c a/3 . With help of (*), we know that fx has a local maximum<br />

f a/3 and a local minimum f a/3 .<br />

(b) fx logx 2 9, |x| 3.<br />

Solution: Since f x <br />

2x , |x| 3, we know that<br />

x 2 9<br />

which implies that<br />

f x :<br />

, 3 3, <br />

0 0<br />

, 3 3, <br />

fx :<br />

.<br />

<br />

Hence, f is increasing on 3, , and decreasing on , 3. It is clear that f cannot have<br />

local extremum.<br />

(c) fx x 2/3 x 1 4 ,0 x 1.<br />

Solution: Since f x 2x13<br />

3x 1/3<br />

which implies that<br />

7x 1, 0 x 1, we know that<br />

f x :<br />

0, 1/7 1/7, 1<br />

0 0<br />

0, 1/7 1/7, 1<br />

fx :<br />

. **<br />

<br />

Hence, we know that f is increasing on 0, 1/7, and decreasing on 1/7, 1. In addition, if f<br />

has a local extremum at some interior point c, then f c 0. It implies that c 1/7. With<br />

help of (**), we know that f has a local maximum f1/7, and two local minima f0, and<br />

f1.<br />

Remark: f has the absolute maximum f1/7, and the absolute minima<br />

f0 f1 0.<br />

(d) fx sin x/x if x 0, f0 1, 0 x /2.<br />

Sulotion: Since f x cosx xtan x as 0 x /2, and f<br />

x 2<br />

0 0, in addition,<br />

f x 0asx 0 by L-Hospital Rule, we know that<br />

which implies that<br />

f x :<br />

0, /2<br />

0<br />

0, /2<br />

fx : . (***)<br />

<br />

Hence, we know that f is decreasing on 0, /2. In addition, note that there is no interior<br />

point c such that f c 0. With help of (***), we know that f has local maximum f0,<br />

and local minimum f/2.<br />

Remark: 1. Here is a proof on f 0 :Since


lim sin x 1<br />

x0 x<br />

<br />

lim<br />

x0<br />

<br />

2sin x/2 2<br />

x 0,<br />

we know that f 0 0.<br />

2. f has the absolute maximum f0, and the absolute minimum f/2.<br />

5.3 Find a polynomial f of lowest possible degree such that<br />

fx 1 a 1 , fx 2 a 2 , f x 1 b 1 , f x 2 b 2<br />

where x 1 x 2 and a 1 , a 2 , b 1 , b 2 are given real numbers.<br />

Proof: It is easy to know that the lowest degree is at most 3 since there are 4 unknows.<br />

<strong>The</strong> degree is depends on the values of a 1 , a 2 , b 1 , b 2 .<br />

5.4 Define f as follows: fx e 1/x2 if x 0, f0 0. Show that<br />

(a) f is continuous for all x.<br />

Proof: In order to show f is continuous on R, it suffices to show f is continuous at 0.<br />

Since<br />

1/x 2<br />

limfx lim<br />

x0 x0<br />

1 e 0 f0,<br />

we know that f is continuous at 0.<br />

(b) f n is continuous for all x, andf n 0 0, (n 1,2,...)<br />

Proof: In order to show f n is continuous on R, it suffices to show f n is continuous at<br />

0. Note that<br />

px<br />

lim<br />

x e x 0, where px is any real polynomial. *<br />

Claim that for x 0, we have f n x e 1/x2 P 3n 1/x, whereP 3n t is a real polynomial of<br />

degree 3n for all n 1,2,.... As n 0, f 0 x fx e 1/x2 e 1/x2 P 0 1/x, where<br />

P 0 1/x is a constant function 1. So, as n 0, it holds. Suppose that n k holds, i.e.,<br />

f k x e 1/x2 P 3k 1/x, whereP 3k t is a real polynomial of degree 3k. Consider<br />

n k 1, we have<br />

f k1 x f k x <br />

**<br />

e 1/x2 P 3k 1/x by induction hypothesis<br />

e 1/x2 2 1 x<br />

3 2<br />

P3k<br />

1 x 1 x P3k<br />

1 x .<br />

Since 2t 3 P 3k t t 2 P 3k t is a real polynomial of degree 3k 3, we define<br />

2t 3 P 3k t t 2 P 3k t P 3k3 t, and thus we have by (**)<br />

f k1 x e 1/x2 P 3k3 1/x.<br />

So, as n k 1, it holds. <strong>The</strong>refore, by Mathematical Induction, we have proved the<br />

claim.<br />

Use the claim to show that f n 0 0, (n 1,2,...) as follows. As n 0, it is trivial<br />

by hypothesis. Suppose that n k holds, i.e., f k 0 0. <strong>The</strong>n as n k 1, we have


f k x f k 0<br />

x 0<br />

fk x<br />

x by induction hypothesis<br />

P<br />

e1/x2 3k 1/x<br />

x<br />

tP 3kt<br />

(let t 1/x)<br />

<br />

e t2<br />

tP 3k t<br />

e t<br />

e t<br />

e t2 0ast ( x 0) by (*).<br />

Hence, f k1 0 0. So, by Mathematical Induction, we have proved that f n 0 0,<br />

(n 1,2,...).<br />

Since<br />

lim<br />

x0<br />

f n x lim<br />

x0<br />

e 1/x2 P 3n 1/x<br />

lim<br />

x0<br />

lim t<br />

P 3n 1/x<br />

e 1/x2<br />

P 3n t<br />

e t<br />

0by(*)<br />

f n 0,<br />

we know that f n x is continuous at 0.<br />

Remark: 1. Here is a proof on (*). Let Px be a real polynomial of degree n, and<br />

choose an even number 2N n. We consider a Taylor Expansion with Remainder as<br />

follows. Since for any x, wehave<br />

2N1<br />

e x 1k! x k e x,0<br />

2N1<br />

2N 2! x2N2 1k! x k ,<br />

k0<br />

k0<br />

then<br />

0 Px Px<br />

e x <br />

0asx <br />

2N1 1<br />

k! xk<br />

k0<br />

2N1<br />

since degPx n deg 1<br />

k0 k! xk 2N 1. By Sandwich <strong>The</strong>orem, wehave<br />

proved<br />

Px<br />

lim<br />

x e x 0.<br />

2. Here is another proof on f n 0 0, (n 1,2,...). By Exercise 5.15, it suffices to<br />

show that lim x0 f n x 0. For the part, we have proved in this exercise. So, we omit the<br />

proof. Exercise 5.15 tells us that we need not make sure that the derivative of f at 0. <strong>The</strong><br />

reader should compare with Exercise 5.15 and Exercise 5.5.<br />

3. In the future, we will encounter the exercise in Charpter 9. <strong>The</strong> Exercises tells us one<br />

important thing that the Taylor’s series about 0 generated by f converges everywhere<br />

on R, but it represents f only at the origin.<br />

5.5 Define f, g, andh as follows: f0 g0 h0 0 and, if x 0,<br />

fx sin1/x, gx x sin1/x, hx x 2 sin1/x. Show that<br />

(a) f x 1/x 2 cos1/x, ifx 0; f 0 does not exist.<br />

e t<br />

e t2


Proof: Trivially, f x 1/x 2 1<br />

cos1/x, ifx 0. Let x n <br />

2n 1 2<br />

consider<br />

fx n f0<br />

sin1/x n<br />

x n 0 x n<br />

2n 1 as n .<br />

2<br />

Hence, we know that f 0 does not exist.<br />

(b) g x sin1/x 1/x cos1/x, ifx 0; g 0 does not exist.<br />

, and thus<br />

Proof: Trivially, g 1<br />

x sin1/x 1/x cos1/x, ifx 0. Let x n ,and<br />

2n 1 2<br />

y n 1 , we know that 2n<br />

gx n g0<br />

sin 1<br />

x n 0 xn<br />

1 for all n<br />

and<br />

gy n g0<br />

sin 1<br />

y n 0 yn<br />

0 for all n.<br />

Hence, we know that g 0 does not exist.<br />

(c) h x 2x sin1/x cos1/x, ifx 0; h 0 0; lim x0 h x does not exist.<br />

Proof: Trivially, h x 2x sin1/x cos1/x, ifx 0. Consider<br />

hx h0<br />

|x sin1/x| |x| 0asx 0,<br />

x 0<br />

so we know that h 1<br />

0 0. In addition, let x n ,andy<br />

2n 1 n 1 ,wehave<br />

2n<br />

2<br />

h x n 2<br />

2n 1 and h y n 1 for all n.<br />

2<br />

Hence, we know that lim x0 h x does not exist.<br />

5.6 Derive Leibnitz’s formula for the nth derivative of the product h of two functions<br />

f and g :<br />

n<br />

h n kn f k g nk x, where kn n!<br />

k!n k! .<br />

k0<br />

Proof: We prove it by mathematical Induction. As n 1, it is clear since<br />

h f g g f. Suppose that n k holds, i.e., h k k<br />

j0<br />

jk f j g kj x. Consider<br />

n k 1, we have


h k1 h k <br />

k<br />

jk f j g kj x <br />

j0<br />

k<br />

jk f j g kj x<br />

j0<br />

k<br />

jk f j1 g kj f j g kj1 <br />

j0<br />

<br />

<br />

<br />

k1 j0<br />

jk f j1 g kj f k1 g 0<br />

k<br />

j1<br />

jk f j g kj1 f 0 g k1<br />

<br />

k1<br />

<br />

j0 jk f j1 g kj f k1 g 0<br />

k1<br />

j0<br />

k<br />

j1<br />

f j1 g kj f 0 g k1<br />

k1<br />

<br />

j0<br />

k1<br />

<br />

j0<br />

jk <br />

k1<br />

j1<br />

k<br />

j1<br />

f j1 g kj f k1 g 0 f 0 g k1<br />

f j1 g kj f k1 g 0 f 0 g k1<br />

k1<br />

jk f j1 g kj .<br />

j0<br />

So, as n k 1, it holds. Hence, by Mathematical Induction, we have proved the<br />

Leibnitz formula.<br />

Remark: We use the famous formula called Pascal <strong>The</strong>orem:n1 k1 kn n k1 ,<br />

where 0 k n.<br />

5.7 Let f and g be two functions defined and having finite third-order derivatives f x<br />

and g x for all x in R. Iffxgx 1 for all x, show that the relations in (a), (b), (c),<br />

and (d) holds at those points where the denominators are not zero:<br />

(a) f x/fx g x/gx 0.<br />

Proof: Since fxgx 1 for all x, wehavef g g f 0 for all x. By hypothesis, we<br />

have<br />

f g g f<br />

0 for those points where the denominators are not zero<br />

fg<br />

which implies that<br />

f x/fx g x/gx 0.<br />

(b) f x/f x 2f x/fx g x/g x 0.<br />

Proof: Since f g g f 0 for all x, wehavef g g f f g 2f g g f 0. By<br />

hypothesis, we have


(c) f x<br />

f x<br />

3 f xg x<br />

fxg x<br />

3 f x<br />

fx<br />

0 f g 2f g g f<br />

f g<br />

f<br />

f <br />

f<br />

f <br />

g x<br />

g x<br />

2 g<br />

g <br />

2 f<br />

f<br />

0.<br />

g<br />

g<br />

g <br />

g <br />

f<br />

f<br />

by (a).<br />

Proof: By (b), we have f g 2f g g f 0 f g 3f g 3f g fg .By<br />

hypothesis, we have<br />

0 f g 3f g 3f g fg <br />

f g<br />

(d) f x<br />

f x<br />

3 2<br />

f x<br />

f x<br />

f<br />

f <br />

3 f g <br />

f g 3 g<br />

g fg<br />

f g<br />

f<br />

f 3f g <br />

f g<br />

f<br />

f <br />

2<br />

<br />

g x<br />

g x<br />

Proof: By(c),wehave f<br />

f <br />

f <br />

f<br />

we know that f<br />

f <br />

f x<br />

f x<br />

3 2<br />

f x<br />

f x<br />

f g <br />

fg <br />

g<br />

<br />

<br />

1 2<br />

1 2<br />

3 f<br />

f<br />

3 2<br />

3 f g <br />

fg <br />

g x<br />

g x<br />

g<br />

3 f<br />

g f<br />

f <br />

f<br />

f <br />

f<br />

3 g 2<br />

2<br />

g x<br />

3 g x 2<br />

f <br />

f <br />

f <br />

f <br />

f <br />

f <br />

<br />

<br />

f 2<br />

<br />

f <br />

<br />

3g 1 g g f<br />

f g<br />

g <br />

g <br />

f 2<br />

<br />

g 2<br />

f g <br />

g x<br />

g x<br />

2<br />

.<br />

g<br />

g <br />

2<br />

.<br />

f g <br />

fg <br />

f <br />

f<br />

g <br />

g <br />

g 2<br />

g <br />

by (a).<br />

.Since<br />

g <br />

g <br />

,<br />

f <br />

f <br />

<br />

g <br />

g <br />

which implies that<br />

by (b)<br />

Note. <strong>The</strong> expression which appears on the left side of (d) is called the Schwarzian<br />

derivative of f at x.<br />

(e) Show that f and g have the same Schwarzian derivative if<br />

gx afx b/cfx d, wheread bc 0.<br />

Hint. If c 0, write af b/cf d a/c bc ad/ccf d, and apply part<br />

(d).<br />

Proof: If c 0, we have g a f b . So, we have<br />

d d


g x<br />

g x 3 2<br />

a<br />

d f x<br />

a<br />

d f x 3 2<br />

f x<br />

f x 3 2<br />

g x<br />

g x<br />

2<br />

a<br />

d f x<br />

a<br />

d f x<br />

f x<br />

f x<br />

So, f and g have the same Schwarzian derivative.<br />

If c 0, write g af b/cf d a/c bc ad/ccf d, then<br />

cg a 1 cf d 1sincead bc 0.<br />

LetG cg a, andF <br />

1<br />

bcad<br />

which implies that<br />

F <br />

F 3 2<br />

bc ad<br />

cf d, then GF 1. It implies that by (d),<br />

F <br />

F 3 2<br />

F <br />

F <br />

2<br />

.<br />

2<br />

G <br />

G 3 2<br />

F 2 c<br />

<br />

bcad f<br />

3 F c<br />

<br />

bcad f 2<br />

f<br />

f 3 2<br />

G<br />

G 3 2<br />

cg<br />

cg 3 2<br />

f 2<br />

f <br />

G 2<br />

G <br />

2<br />

G 2<br />

G <br />

<br />

<br />

cg 2<br />

cg <br />

c<br />

2<br />

bcad f<br />

c<br />

bcad f<br />

g<br />

3 g 2<br />

.<br />

g 2 g <br />

So, f and g have the same Schwarzian derivative.<br />

5.8 Let f 1 , f 2 , g 1 , g 2 be functions having derivatives in a, b. DefineF by means of<br />

the determinant<br />

Fx <br />

f 1 x f 2 x<br />

g 1 x g 2 x<br />

,ifx a, b.<br />

(a) Show that F x exists for each x in a, b and that<br />

F x <br />

f 1 x f 2 x<br />

g 1 x g 2 x<br />

<br />

f 1 x f 2 x<br />

g 1 x g 2 x<br />

.<br />

f 1 x f 2 x<br />

Proof: SinceFx <br />

f 1 g 2 f 2 g 1 ,wehave<br />

g 1 x g 2 x<br />

F f 1 g 2 f 1 g 2 f 2 g 1 f 2 g<br />

1<br />

f 1 g 2 f 2 g 1 f 1 g 2 f 2 g 1 <br />

f<br />

1 x f 2 x f 1 x f 2 x<br />

<br />

g 1 x g 2 x g 1 x g 2 x<br />

.


then<br />

(b) State and prove a more general result for nth order determinants.<br />

Proof: Claim that if<br />

F x <br />

f 11 f 12 ... f<br />

1n<br />

Fx <br />

f 21 f 22 ... f 2n<br />

<br />

... ... ... ...<br />

f n1 f n2 ... f nn<br />

f 11 f 12 ... f 1n<br />

f 21 f 22 ... f 2n<br />

,<br />

... ... ... ...<br />

f n1 f n2 ... f nn<br />

f 11 f 12 ... f 1n<br />

f 21 f 22 ... f<br />

2n<br />

...<br />

... ... ... ...<br />

f n1 f n2 ... f nn<br />

f 11 f 12 ... f 1n<br />

f 21 f 22 ... f 2n<br />

... ... ... ...<br />

f<br />

n1<br />

f n2 ... f<br />

nn<br />

We prove it by Mahematial Induction. As n 2, it has proved in (a). Suppose that n k<br />

holds, consider n k 1,<br />

f k11 f k12 ... f k1k1<br />

<br />

f 11 f 12 ... f 1k1<br />

f 21 f 22 ... f 2k1<br />

... ... ... ...<br />

<br />

f 12 ... f 1k1<br />

f 11 ... f 1k<br />

1 k11 f k11 ... ... ... ..1 k1k1 f k1k1 ... ... ...<br />

f k2 ... f kk1 f k1 ... f kk<br />

. . . . (<strong>The</strong> reader can write it down by induction hypothesis).<br />

Hence, by Mathematical Induction, wehaveprovedit.<br />

Remark: <strong>The</strong> reader should keep it in mind since it is useful in Analysis. For example,<br />

we have the following <strong>The</strong>orem.<br />

(<strong>The</strong>orem) Suppose that f, g, andh are continuous on a, b, and differentiable on<br />

a, b. <strong>The</strong>n there is a a, b such that<br />

Proof: Let<br />

f g h <br />

fa ga ha<br />

fb gb hb<br />

Fx <br />

fx gx hx<br />

fa ga ha<br />

fb gb hb<br />

then it is clear that Fx is continuous on a, b and differentiable on a, b since the<br />

operations on determinant involving addition, substraction, and multiplication without<br />

division. Consider<br />

Fa Fb 0,<br />

then by Rolle’s <strong>The</strong>orem, we know that<br />

0.<br />

,<br />

.


which implies that<br />

F 0, where a, b,<br />

f g h <br />

fa ga ha<br />

fb gb hb<br />

0. *<br />

(Application- Generalized Mean Value <strong>The</strong>orem) Suppose that f and g are<br />

continuous on a, b, and differentiable on a, b. <strong>The</strong>n there is a a, b such that<br />

fb fag f gb ga.<br />

Proof: Let hx 1, and thus by (*), we have<br />

which implies that<br />

which implies that<br />

<br />

f g <br />

fb gb<br />

f g 0<br />

fa ga 1<br />

fb gb 1<br />

<br />

0,<br />

f g <br />

fa ga<br />

fb fag f gb ga.<br />

Note: Use the similar method, we can show Mean Value <strong>The</strong>orem by letting<br />

gx x, andhx 1. <strong>And</strong> from this viewpoint, we know that Rolle’s <strong>The</strong>orem, Mean<br />

Value <strong>The</strong>orem, and Generalized Mean Value <strong>The</strong>orem are equivalent.<br />

5.9 Given n functions f 1 ,...,f n , each having nth order derivatives in a, b. A<br />

function W, called the Wronskian of f 1 ,...,f n , is defined as follows: For each x in a, b,<br />

Wx is the value of the determinant of order n whose element in the kth row and mth<br />

column is f k1 m x, wherek 1, 2, . . , n and m 1,2,...,n. [<strong>The</strong> expression f 0 m x is<br />

written for f m x.]<br />

(a) Show that W x can be obtained by replacing the last row of the determinant<br />

defining Wx by the nth derivatives f n 1 x,...,f n n x.<br />

Proof: Write<br />

Wx <br />

f 1 f 2 ... f n<br />

f 1 f 2 ... f<br />

n<br />

... ... ... ...<br />

f 1<br />

n1<br />

f 2<br />

n1<br />

... f n<br />

n1<br />

and note that if any two rows are the same, its determinant is 0; hence, by Exercise 5.8-(b),<br />

we know that<br />

,<br />

0


W x <br />

f 1 f 2 ... f n<br />

... ... ... ...<br />

f 1<br />

n2<br />

f 1<br />

n<br />

f 2<br />

n2<br />

f 2<br />

n<br />

... f n<br />

n2<br />

... f n<br />

n<br />

.<br />

(b) Assuming the existence of n constants c 1 ,...,c n , not all zero, such that<br />

c 1 f 1 x ...c n f n x 0 for every x in a, b, show that Wx 0 for each x in a, b.<br />

Proof: Since c 1 f 1 x ...c n f n x 0 for every x in a, b, wherec 1 ,...,c n , not all<br />

zero. Without loss of generality, we may assume c 1 0, we know that<br />

c 1 f k 1 x ...c n f k n x 0 for every x in a, b, where 0 k n. Hence, we have<br />

Wx <br />

f 1 f 2 ... f n<br />

f 1 f 2 ... f<br />

n<br />

... ... ... ...<br />

f 1<br />

n1<br />

f 2<br />

n1<br />

... f n<br />

n1<br />

0<br />

since the first column is a linear combination of other columns.<br />

Note. A set of functions satisfing such a relation is said to be a linearly dependent set<br />

on a, b.<br />

(c) <strong>The</strong> vanishing of the Wronskian throughout a, b is necessary, but not sufficient.<br />

for linear dependence of f 1 ,...,f n . Show that in the case of two functions, if the Wronskian<br />

vanishes throughout a, b and if one of the functions does not vanish in a, b, then they<br />

form a linearly dependent set in a, b.<br />

Proof: Let f and g be continuous and differentiable on a, b. Suppose that fx 0for<br />

all x a, b. Since the Wronskian of f and g is 0, for all x a, b, wehave<br />

fg f g 0 for all x a, b. *<br />

Since fx 0 for all x a, b, wehaveby(*),<br />

fg f g<br />

0 g <br />

0 for all x a, b.<br />

f 2 f<br />

Hence, there is a constant c such that g cf for all x a, b. Hence, f, g forms a<br />

linearly dependent set.<br />

Remark: This exercise in (b) is a impotant theorem on O.D.E. We often write (b) in<br />

other form as follows.<br />

(<strong>The</strong>orem) Letf 1 ,..,f n be continuous and differentiable on an interval I. If<br />

Wf 1 ,...,f n t 0 0forsomet 0 I, then f 1 ,..,f n is linearly independent on I<br />

Note: If f 1 ,..,f n is linearly independent on I, ItisNOT necessary that<br />

Wf 1 ,...,f n t 0 0forsomet 0 I. For example, ft t 2 |t|, andgt t 3 . It is easy to<br />

check f, g is linearly independent on 1, 1. <strong>And</strong>Wf, gt 0 for all t 1, 1.<br />

Supplement on Chain Rule and Inverse Function <strong>The</strong>orem.<br />

<strong>The</strong> following theorem is called chain rule, it is well-known that let f be defined on an<br />

open interval S, letg be defined on fS, and consider the composite function g f defined<br />

on S by the equation


g fx gfx.<br />

Assume that there is a point c in S such that fc is an interior point of fS. Iff is<br />

differentiable at c and g is differentiable at fc, then g f is differentiable at c, andwe<br />

have<br />

g f c g fcf c.<br />

We do not give a proof, in fact, the proof can be found in this text book. We will give<br />

another <strong>The</strong>orem called <strong>The</strong> Converse of Chain Rule as follows.<br />

(<strong>The</strong>ConverseofChainRule) Suppose that f, g and u are related so that<br />

fx gux. Ifux is continuous at x 0 , f x 0 exists, g ux 0 exists and not zero.<br />

<strong>The</strong>n u x 0 is defined and we have<br />

f x 0 g ux 0 u x 0 .<br />

Proof: Sincef x 0 exists, and g ux 0 exists, then<br />

fx fx 0 f x 0 x x 0 o|x x 0 | *<br />

and<br />

gux gux 0 g ux 0 ux ux 0 o|ux ux 0 |. **<br />

Since fx gux, andfx 0 gux 0 , by (*) and (**), we know that<br />

f<br />

ux ux 0 <br />

x 0 <br />

g ux 0 x x 0 o|x x 0 | o|ux ux 0 |. ***<br />

Note that since ux is continuous at x 0 , we know that o|ux ux 0 | 0asx x 0 .<br />

So, (***) means that u x 0 is defined and we have<br />

f x 0 g ux 0 u x 0 .<br />

Remark: <strong>The</strong> condition that g ux 0 is not zero is essential, for example, gx 1on<br />

1, 1 and ux |x|, wherex 0 0.<br />

(Inverse Function <strong>The</strong>roem) Suppose that f is continuous, strictly monotonic function<br />

which has an open interval I for domain and has range J. (It implies that<br />

fgx x gfx on its corresponding domain.) Assume that x 0 is a point of J such<br />

that f gx 0 is defined and is different from zero. <strong>The</strong>n g x 0 exists, and we have<br />

g x 0 1<br />

f gx 0 .<br />

Proof: Itisaresultofthe converse of chain rule note that<br />

fgx x.<br />

Mean Value <strong>The</strong>orem<br />

5.10 Given a function defined and having a finite derivative in a, b and such that<br />

lim xb<br />

fx . Prove that lim xb<br />

f x either fails to exist or is infinite.<br />

Proof: Suppose NOT, we have the existence of lim xb<br />

f x, denoted the limit by L.<br />

So, given 1, there is a 0 such that as x b , b we have<br />

|f x| |L| 1. *<br />

Consider x, a b , b with x a, then we have by (*) and Mean Value <strong>The</strong>orem,<br />

|fx fa| |f x a| where a, x<br />

|L| 1|x a|<br />

which implies that


|fx| |fa| |L| 1<br />

which contradicts to lim xb<br />

fx .<br />

Hence, lim xb<br />

f x either fails to exist or is infinite.<br />

5.11 Show that the formula in the Mean Value <strong>The</strong>orem can be written as follows:<br />

fx h fx<br />

f<br />

h<br />

x h,<br />

where 0 1.<br />

Proof: (Mean Value <strong>The</strong>orem) Letf and g be continuous on a, b and differentiable<br />

on a, b. <strong>The</strong>n there exists a a, b such that fb fa f b a. Note that<br />

a b a, where 0 1. So, we have proved the exercise.<br />

Determine as a function of x and h, and keep x 0 fixed, and find lim h0 in each<br />

case.<br />

(a) fx x 2 .<br />

Proof: Consider<br />

fx h fx<br />

h<br />

which implies that<br />

x h2 x 2<br />

h<br />

2x h 2x h f x h<br />

1/2.<br />

Hence, we know that lim h0 1/2.<br />

(b) fx x 3 .<br />

Proof: Consider<br />

fx h fx<br />

x h3 x 3<br />

3x<br />

h<br />

h<br />

2 3xh h 2 3x h 2 f x h<br />

which implies that<br />

3x 9x2 9xh 3h 2<br />

3h<br />

Since 0 1, we consider two cases. (i) x 0, (ii) x 0.<br />

(i) As x 0, since<br />

0 3x 9x2 9xh 3h 2<br />

1,<br />

3h<br />

we have<br />

<br />

3x 9x 2 9xh3h 2<br />

3h<br />

if h 0, and h is sufficiently close to 0,<br />

3x 9x 2 9xh3h 2<br />

if h 0, and h is sufficiently close to 0.<br />

3h<br />

Hence, we know that lim h0 1/2 by L-Hospital Rule.<br />

(ii) As x 0, we have<br />

<br />

3x 9x 2 9xh3h 2<br />

3h<br />

if h 0, and h is sufficiently close to 0,<br />

3x 9x 2 9xh3h 2<br />

if h 0, and h is sufficiently close to 0.<br />

3h<br />

Hence, we know that lim h0 1/2 by L-Hospital Rule.<br />

From (i) and (ii), we know that as x 0, we have lim h0 1/2.


Remark: For x 0, we can show that lim h0 3 3<br />

as follows.<br />

Proof: Since<br />

we have<br />

<br />

0 3h2<br />

3h<br />

1,<br />

3h 2<br />

3h<br />

3 h<br />

3h<br />

3 3<br />

if h 0,<br />

3h 2<br />

3 h 3 if h 0.<br />

3h 3h 3<br />

Hence, we know that lim h0 3 . 3<br />

(c) fx e x .<br />

Proof: Consider<br />

which implies that<br />

fx h fx<br />

h<br />

exh e x<br />

h<br />

e xh f x h<br />

log eh 1<br />

h<br />

.<br />

h<br />

Hence, we know that lim h0 1/2 since<br />

log eh 1<br />

h<br />

lim lim<br />

h0 h0 h<br />

lim e h h e h 1 by L-Hospital Rule.<br />

h0 he h 1<br />

Note that e h 1 h h2<br />

2 oh2 <br />

lim<br />

h0<br />

1<br />

1/2.<br />

(d) fx log x, x 0.<br />

Proof: Consider<br />

fx h fx<br />

h<br />

which implies that<br />

h o1<br />

2<br />

1 h oh<br />

2<br />

<br />

log1 h x <br />

h<br />

h<br />

x log1 h x <br />

h<br />

x log1 h x .<br />

1<br />

x h<br />

Since log1 t t t2 2<br />

ot2 ,wehave


h h<br />

x x 1<br />

lim lim<br />

h 2 x 2 o h x 2<br />

h0 h0 h h<br />

x x 1 h 2 x 2 o h x 2<br />

lim<br />

h0<br />

1<br />

lim<br />

2 h x 2 o h x 2<br />

h x 2 1 2 h x 3 o h x 3<br />

o1<br />

2<br />

1 1 h 2 x o h x <br />

h0<br />

1<br />

1/2.<br />

5.12 Take fx 3x 4 2x 3 x 2 1andgx 4x 3 3x 2 2x in <strong>The</strong>orem 5.20. Show<br />

that f x/g x is never equal to the quotient f1 f0/g1 g0 if 0 x 1. How<br />

do you reconcile this with the equation<br />

fb fa<br />

gb ga <br />

obtainable from <strong>The</strong>orem 5.20 when n 1<br />

Solution: Note that<br />

12x 2 6x 2 12 x 1<br />

4<br />

11<br />

48<br />

f x 1 <br />

g x 1 , a x 1 b,<br />

x 1<br />

4<br />

11<br />

48<br />

, where (0 1 4 11<br />

48 1).<br />

So, when we consider<br />

f1 f0<br />

g1 g0 0<br />

and<br />

f x 12x 3 6x 2 2x xg x x12x 2 6x 2,<br />

we CANNOT write f x/g x x. Otherwise, it leads us to get a contradiction.<br />

Remark: It should be careful when we use Generalized Mean Value <strong>The</strong>orem, we<br />

had better not write the above form unless we know that the denominator is not zero.<br />

5.13 In each of the following special cases of <strong>The</strong>orem 5.20, take n 1, c a, x b,<br />

and show that x 1 a b/2.<br />

(a) fx sin x, gx cosx;<br />

Proof: Since, by <strong>The</strong>orem 5.20,<br />

sin a sin b sinx 1 2cos a b sin<br />

2<br />

a 2<br />

b sinx 1 <br />

cosa cosbcosx 1 <br />

2sin a 2<br />

b sin a 2<br />

b cosx 1 ,<br />

we find that if we choose x 1 a b/2, then both are equal.<br />

(b) fx e x , gx e x .<br />

Proof: Since, by <strong>The</strong>orem 5.20,<br />

e a e b e x 1 e<br />

a<br />

e b e x 1 ,<br />

we find that if we choose x 1 a b/2, then both are equal.<br />

Can you find a general class of such pairs of functions f and g for which x 1 will always<br />

be a b/2 and such that both examples (a) and (b) are in this class


Proof: Look at the Generalized Mean Value <strong>The</strong>orem, we try to get something from<br />

the equality.<br />

fa fbg a 2<br />

b ga gbf a 2<br />

b , *<br />

if fx, andgx satisfy following two conditions,<br />

(i) f x gx and g x fx<br />

and<br />

(ii) fa fb f a 2<br />

b ga gb g a 2<br />

b ,<br />

then we have the equality (*).<br />

5.14 Given a function f defined and having a finite derivative f in the half-open<br />

interval 0 x 1 and such that |f x| 1. Define a n f1/n for n 1, 2, 3, . . . , and<br />

show that lim n a n exists.<br />

Hint. Cauchy condition.<br />

Proof: Consider n m, and by Mean Value <strong>The</strong>orem,<br />

|a n a m| |f1/n f1/m| |f p| 1 n m 1 1 n m<br />

1<br />

then a n is a Cauchy sequence since 1/n is a Cauchy sequence. Hence, we know that<br />

lim n a n exists.<br />

5.15 Assume that f has a finite derivative at each point of the open interval a, b.<br />

Assume also that lim xc f x exists and is finite for some interior point c. Prove that the<br />

value of this limit must be f c.<br />

Proof: It can be proved by Exercise 5.16; we omit it.<br />

5.16 Let f be continuous on a, b with a finite derivative f everywhere in a, b,<br />

expect possibly at c. If lim xc f x exists and has the value A, show that f c must also<br />

exist and has the value A.<br />

Proof: Consider, for x c,<br />

fx fc<br />

x c f where x, c or c, x by Mean Value <strong>The</strong>orem, *<br />

since lim xc f x exists, given 0, there is a 0 such that as x c , c c,<br />

we have<br />

A f x A .<br />

So, if we choose x c , c c in (*), we then have<br />

fx fc<br />

A x c f A .<br />

That is, f c exists and equals A.<br />

Remark: (1) Here is another proof by L-Hospital Rule. Since it is so obvious that we<br />

omit the proof.<br />

(2) We should be noted that Exercise 5.16 implies Exercise 5.15. Both methods<br />

mentioned in Exercise 5.16 are suitable for Exercise 5.15.<br />

5.17 Let f be continuous on 0, 1, f0 0, f x defined for each x in 0, 1. Prove<br />

that if f is an increasing function on 0, 1, then so is too is the function g defined by the<br />

equation gx fx/x.<br />

Proof: Sincef is an increasing function on 0, 1, we know that, for any x 0, 1


f x fx<br />

x<br />

So, let x y, wehave<br />

f x <br />

fx f0<br />

x 0<br />

f x f 0where 0, x. *<br />

gx gy g zx y, wherey z x<br />

f zz fz<br />

x y<br />

0by(*)<br />

which implies that g is an increasing function on 0, 1.<br />

5.18 Assume f has a finite derivative in a, b and is continuous on a, b with<br />

fa fb 0. Prove that for every real there is some c in a, b such that<br />

f c fc.<br />

Hint. Apply Rolle’s <strong>The</strong>orem to gxfx for a suitable g depending on .<br />

Proof: Consider gx fxe x , then by Rolle’s <strong>The</strong>orem,<br />

ga gb g ca b, wherec a, b<br />

0<br />

which implies that<br />

f c fc.<br />

z 2<br />

Remark: (1) <strong>The</strong> finding of an auxiliary function usually comes from the equation that<br />

we consider. We will give some questions around this to get more.<br />

(2)<strong>The</strong>re are some questions about finding auxiliary functions; we write it as follows.<br />

(i) Show that e e .<br />

Proof: (STUDY) Since log x is a strictly increasing on 0, , in order to show<br />

e e , it suffices to show that<br />

log e log e log e e log <br />

which implies that<br />

log e<br />

Consider fx logx<br />

x<br />

e<br />

: e, , wehave<br />

f x 1 log x<br />

x 2<br />

log <br />

.<br />

0wherex e, .<br />

So, we know that fx is strictly decreasing on e, . Hence,<br />

e e .<br />

(ii) Show that e x 1 x for all x R.<br />

loge<br />

e<br />

log<br />

<br />

.Thatis,<br />

Proof: By Taylor <strong>The</strong>orem with Remainder Term, we know that<br />

e x 1 x ec<br />

2 x2 ,forsomec.<br />

So, we finally have e x 1 x for all x R.<br />

Note: (a) <strong>The</strong> method in (ii) tells us one thing, we can give a theorem as follows. Let<br />

f C 2n1 a, b, andf 2n x exists and f 2n x 0ona, b. <strong>The</strong>n we have<br />

2n1<br />

fx f<br />

k<br />

a<br />

.<br />

k!<br />

k0


Proof: By Generalized Mean Value <strong>The</strong>orem, we complete it.<br />

(b) <strong>The</strong>re are many proofs about that e x 1 x for all x R. We list them as a<br />

reference.<br />

(b-1) Let fx e x 1 x, and thus consider the extremum.<br />

(b-2) Use Mean Value <strong>The</strong>orem.<br />

(b-3) Since e x 1 0forx 0ande x 1 0forx 0, we then have<br />

<br />

0<br />

x<br />

et 1dt 0and <br />

x<br />

0<br />

et 1dt 0.<br />

So, e x 1 x for all x R.<br />

(iii) Let f be continuous function on a, b, and differentiabel on a, b. Prove that there<br />

exists a c a, b such that<br />

f fc fa<br />

c .<br />

b c<br />

Proof: (STUDY) Since f c fcfa , we consider f<br />

bc<br />

cb c fc fa.<br />

Hence, we choose gx fx fab x, then by Rolle’s <strong>The</strong>orem,<br />

ga gb g ca b where c a, b<br />

which implies thatf c fcfa .<br />

bc<br />

(iv) Let f be a polynomial of degree n, iff 0onR, then we have<br />

f f ..f n 0onR.<br />

Proof: Let gx f f ..f n , then we have<br />

g g f 0onR since f is a polynomial of degree n. *<br />

Consider hx gxe x , then h x g x gxe x 0onR by (*). It means that h<br />

is a decreasing function on R. Since lim x hx 0bythefactg is still a polynomial,<br />

then hx 0onR. Thatis,gx 0onR.<br />

(v) Suppose that f is continuous on a, b, fa 0 fb, and<br />

x 2 f x 4xf x 2fx 0 for all x a, b. Prove that fx 0ona, b.<br />

Proof: (STUDY) Since x 2 f x 4xf x 2fx x 2 fx by Leibnitz Rule, let<br />

gx x 2 fx, then claim that gx 0ona, b.<br />

Suppose NOT, there is a point p a, b such that gp 0. Note that since fa 0,<br />

and fb 0, So, gx has an absolute maximum at c a, b. Hence, we have g c 0.<br />

By Taylor <strong>The</strong>orem with Remainder term, wehave<br />

gx gc g cx c g <br />

x c 2 ,where x, c or c, x<br />

2!<br />

gc since g c 0, and g x 0 for all x a, b<br />

0sincegc is absolute maximum.<br />

So,<br />

x 2 fx c 2 fc 0 **<br />

which is absurb since let x a in (**).<br />

(vi) Suppose that f is continuous and differentiable on 0, , and<br />

lim x f x fx 0, show that lim x fx 0.<br />

Proof: Since lim x f x fx 0, then given 0, there is M 0 such that as<br />

x M, wehave


f x fx .<br />

So, as x M, wehave<br />

e x e M e M fM e x<br />

e x fx <br />

e x e x e M e M fM .<br />

If we let e x e M e M fM gx, ande x e M e M fM hx, then we have<br />

g x e x fx h x<br />

and<br />

gM e M fM hM.<br />

Hence, for x M,<br />

e x e M e M fM gx<br />

e x fx<br />

hx e x e M e M fM<br />

It implies that, for x M,<br />

e x e M e M fM fx e x e M e M fM<br />

which implies that<br />

lim fx 0since is arbitrary.<br />

x<br />

Note: Intheprocessofproof,weusetheresultonMean Value <strong>The</strong>orem. Letf, g, and<br />

h be continuous on a, b and differentiable on a, b. Suppose fa ga ha and<br />

f x g x h x on a, b. Show that fx gx hx on a, b.<br />

Proof: By Mean Value theorem, wehave<br />

gx fx ga fa gx fx<br />

g c f c, wherec a, x.<br />

0 by hypothesis.<br />

So, fx gx on a, b. Similarly for gx hx on a, b. Hence, fx gx hx<br />

on a, b.<br />

(vii) Let fx a 1 sin x ...a n sin nx, wherea i are real for i 1, 2, . . n. Suppose that<br />

|fx| |x| for all real x. Prove that |a 1 ..na n| 1.<br />

Proof: Let x 0, and by Mean Value <strong>The</strong>orem, we have<br />

|fx f0| |fx| |a 1 sin x ...a n sin nx|<br />

|f cx|, wherec 0, x<br />

|a 1 cosc ...na n cosncx|<br />

|x| by hypothesis.<br />

So,<br />

|a 1 cosc ...na n cosnc| 1<br />

Note that as x 0 ,wehavec 0 ; hence, |a 1 ..na n| 1.<br />

Note: Here are another type:<br />

(a) |sin 2 x sin 2 y| |x y| for all x, y.<br />

(b) |tan x tan y| |x y| for all x, y , . 2 2<br />

(viii) Let f : R R be differentiable with f x c for all x, wherec 0. Show that


there is a point p such that fp 0.<br />

Proof: ByMean Vaule <strong>The</strong>orem, we have<br />

fx f0 f x 1 x f0 cx if x 0<br />

f0 f x 2 x f0 cx if x 0.<br />

So, as x large enough, we have fx 0andasx is smalle enough, we have fx 0. Since<br />

f is differentiable on R, it is continuous on R. Hence, by Intermediate Value <strong>The</strong>orem,<br />

we know that there is a point p such that fp 0.<br />

(3) Here is another type about integral, but it is worth learning. Compare with (2)-(vii).<br />

If<br />

c 0 c 1<br />

... c n<br />

2 n 1 0, where c i are real constants for i 1, 2, . . n.<br />

Prove that c 0 ...c n x n has at least one real root between 0 and 1.<br />

Proof: Suppose NOT, i.e., (i) fx : c 0 ...c n x n 0 for all x 0, 1 or (ii)<br />

fx 0 for all x 0, 1.<br />

In case (i), consider<br />

1<br />

0 fxdx c0 c 1<br />

... c n<br />

0 2 n 1 0<br />

which is absurb. Similarly for case (ii).<br />

So, we know that c 0 ...c n x n has at least one real root between 0 and 1.<br />

5.19. Assume f is continuous on a, b and has a finite second derivative f in the<br />

open interval a, b. Assume that the line segment joining the points A a, fa and<br />

B b, fb intersects the graph of f in a third point P different from A and B. Prove that<br />

f c 0forsomec in a, b.<br />

Proof: Consider a straight line equation, called gx fa fbfa x a. <strong>The</strong>n<br />

ba<br />

hx : fx gx, we knwo that there are three point x a, p and b such that<br />

ha hp hb 0.<br />

So, by Mean Value <strong>The</strong>orem twice, we know that there is a point c a, b such that<br />

h c 0<br />

which implies that f c 0sinceg is a polynomial of degree at least 1.<br />

5.20 If f has a finite third derivative f in a, b and if<br />

fa f a fb f b 0,<br />

prove that f c 0forsomec in a, b.<br />

Proof: Sincefa fb 0, we have f p 0wherep a, b by Rolle’s<br />

<strong>The</strong>orem. Since f a f p 0, we have f q 1 0whereq 1 a, p and since<br />

f p f b 0, we have f q 2 0whereq 2 p, b by Rolle’s <strong>The</strong>orem. Since<br />

f q 1 f q 2 0, we have f c 0wherec q 1 , q 2 by Rolle’s <strong>The</strong>orem.<br />

5.21 Assume f is nonnegative and has a finite third derivative f in the open interval<br />

0, 1. Iffx 0 for at least two values of x in 0, 1, prove that f c 0forsomec in<br />

0, 1.<br />

Proof: Since fx 0 for at least two values of x in 0, 1, sayfa fb 0, where<br />

a, b 0, 1. By Rolle’s <strong>The</strong>orem, we have f p 0wherep a, b. Note that f is<br />

nonnegative and differentiable on 0, 1, so both fa and fb are local minima, where a<br />

and b are interior to a, b. Hence, f a f b 0.


Since f a f p 0, we have f q 1 0whereq 1 a, p and since<br />

f p f b 0, we have f q 2 0whereq 2 p, b by Rolle’s <strong>The</strong>orem. Since<br />

f q 1 f q 2 0, we have f c 0wherec q 1 , q 2 by Rolle’s <strong>The</strong>orem.<br />

5.22 Assume f has a finite derivative in some interval a, .<br />

(a) If fx 1andf x c as x , prove that c 0.<br />

Proof: Consider fx 1 fx f y where y x, x 1 by Mean Value <strong>The</strong>orem,<br />

since<br />

lim fx 1<br />

x<br />

which implies that<br />

limfx 1 fx 0<br />

x<br />

which implies that (x y )<br />

lim<br />

x f y 0 y<br />

lim fy<br />

Since f x c as x , we know that c 0.<br />

Remark: (i) <strong>The</strong>re is a similar exercise; we write it as follows. If fx L and<br />

f x c as x , prove that c 0.<br />

Proof: By the same method metioned in (a), we complete it.<br />

(ii) <strong>The</strong> exercise tells that the function is smooth; its first derivative is smooth too.<br />

(b) If f x 1asx , prove that fx/x 1asx .<br />

Proof: Given 0, we want to find M 0 such that as x M<br />

fx<br />

x 1 .<br />

Since f x 1asx , thengiven 3 ,thereisM 0 such that as x M ,we<br />

have<br />

|f x 1| <br />

3 |f x| 1 <br />

3<br />

<br />

By Taylor <strong>The</strong>orem with Remainder Term,<br />

fx fM f x M <br />

fx x fM f 1x f M ,<br />

then for x M ,<br />

fx<br />

x 1 fM <br />

x |f 1| f M <br />

x<br />

<br />

fM <br />

x 3 1 3<br />

Choose M 0 such that as x M M ,wehave<br />

fM<br />

x <br />

3 M<br />

and x<br />

<br />

M <br />

x by (*)<br />

*<br />

**<br />

/3<br />

1 3 . ***<br />

Combine (**) with (***), we have proved that given 0, there is a M 0 such that as<br />

x M, wehave<br />

fx<br />

x 1 .<br />

That is, lim x<br />

fx<br />

x 1.


Remark: If we can make sure that fx as x , we can use L-Hopital Rule.<br />

We give another proof as follows. It suffices to show that fx as x .<br />

Proof: Since f x 1asx , thengiven 1, there is M 0 such that as<br />

x M, wehave<br />

|f x| 1 1 2.<br />

Consider<br />

fx fM f x M<br />

by Taylor <strong>The</strong>orem with Remainder Term, then<br />

lim fx since x f x is bounded for x M.<br />

(c) If f x 0asx , prove that fx/x 0asx .<br />

Proof: <strong>The</strong> method metioned in (b). We omit the proof.<br />

Remark: (i) <strong>The</strong>re is a similar exercise; we write it as follows. If f x L as x ,<br />

prove that fx/x L as x . <strong>The</strong> proof is mentioned in (b), so we omit it.<br />

(ii) It should be careful that we CANNOT use L-Hospital Rule since we may not have<br />

the fact fx as x . Hence, L-Hospital Rule cannot be used here. For example, f<br />

is a constant function.<br />

5.23 Let h be a fixed positive number. Show that there is no function f satisfying the<br />

following three conditions: f x exists for x 0, f 0 0, f x h for x 0.<br />

Proof: It is called Intermediate Value <strong>The</strong>orem for Derivatives. (Sometimes, we<br />

also call this theorem Darboux.) See the text book in <strong>The</strong>orem 5.16.<br />

(Supplement) 1. Suppose that a R, andf is a twice-differentiable real function on<br />

a, . LetM 0 , M 1 ,andM 2 are the least upper bound of |fx|, |f x|, and|f x|,<br />

respectively, on a, . Prove that M 12 4M 0 M 2 .<br />

Proof: Consider Taylor’s <strong>The</strong>orem with Remainder Term,<br />

fa 2h fa f a2h f <br />

2h 2 ,whereh 0.<br />

2!<br />

then we have<br />

f a <br />

2h 1 fa 2h fa f h<br />

which implies that<br />

Since gh : M 0<br />

h<br />

|f a| M 0<br />

h hM 2 M 1 M 0<br />

h hM 2. *<br />

hM 2 has an absolute maximum at , hence by (*), we know that<br />

M 12 4M 0 M 2 .<br />

Remark:<br />

2. Suppose that f is a twice-differentiable real function on 0, , andf is bounded on<br />

0, , andfx 0asx . Prove that f x 0asx .<br />

Proof: Since M 12 4M 0 M 2 in Supplement 1, wehaveproveit.<br />

3. Suppose that f is real, three times differentiable on 1, 1, such that f1 0,<br />

f0 0, f1 1, and f 0 0. Prove that f 3 x 3forsomex 1, 1.<br />

Proof: Consider Taylor’s <strong>The</strong>orem with Remainder Term,<br />

M 0<br />

M 2


fx f0 f 0x f 0<br />

x<br />

2!<br />

2 f3 c<br />

x<br />

3!<br />

3 ,wherec x,0 or 0, x,<br />

<strong>The</strong>n let x 1, and subtract one from another, we get<br />

f 3 c 1 f 3 c 2 6, where c 1 and c 2 in 1, 1.<br />

So,wehaveprovef 3 x 3forsomex 1, 1.<br />

5.24 If h 0andiff x exists (and is finite) for every x in a h, a h, andiff is<br />

continuous on a h, a h, show that we have:<br />

(a)<br />

fa h fa h<br />

f<br />

h<br />

a h f a h,0 1;<br />

Proof: Let gh fa h fa h, then by Mean Vaule <strong>The</strong>orem, wehave<br />

gh g0 gh<br />

g hh, where 0 1<br />

f a h f a hh<br />

which implies that<br />

fa h fa h<br />

f<br />

h<br />

a h f a h,0 1.<br />

(b)<br />

fa h 2fa fa h<br />

f<br />

h<br />

a h f a h,0 1.<br />

Proof: Let gh fa h 2fa fa h, then by Mean Vaule <strong>The</strong>orem, wehave<br />

gh g0 gh<br />

g hh, where 0 1<br />

f a h f a hh<br />

which implies that<br />

fa h 2fa fa h<br />

f<br />

h<br />

a h f a h,0 1.<br />

(c) If f a exists, show that.<br />

f fa h 2fa fa h<br />

a lim<br />

h0 h 2<br />

Proof: Since<br />

fa h 2fa fa h<br />

lim<br />

h0 h 2<br />

f<br />

lim<br />

a h f a h<br />

by L-Hospital Rule<br />

h0 2h<br />

lim<br />

h0<br />

f a h f a<br />

2h<br />

1 2 2f a since f a exists.<br />

f a.<br />

f a f a h<br />

2h<br />

Remark: <strong>The</strong>re is another proof by using Generalized Mean Value theorem.<br />

Proof: Let g 1 h fa h 2fa fa h and g 2 h h 2 , then by Generalized<br />

Mean Value theorem, we have


which implies that<br />

Hence,<br />

g 1 h g 1 0g 2 h g 1 hg 2 h g 2 0<br />

fa h 2fa fa h<br />

h 2<br />

f a h f a h<br />

2h<br />

fa h 2fa fa h<br />

lim<br />

h0 h 2<br />

f<br />

lim<br />

a h f a h<br />

h0 2h<br />

f a since f a exists.<br />

(d) Give an example where the limit of the quotient in (c) exists but where f a does<br />

not exist.<br />

Solution: (STUDY) Note that in the proof of (c) by using L-Hospital Rule. We know<br />

that |x| is not differentiable at x 0, and |x| satisfies that<br />

|0 h| |0 h|<br />

f<br />

lim<br />

0 lim<br />

0 h f 0 h<br />

h0 2h<br />

h0 2h<br />

So,letustrytofindafunctionf so that f x |x|. So, consider its integral, we know that<br />

x 2<br />

2<br />

fx <br />

if x 0<br />

x2 if x 0<br />

2<br />

Hence, we complete it.<br />

Remark: (i) <strong>The</strong>re is a related statement; we write it as follows. Suppose that f defined<br />

on a, b and has a derivative at c a, b. Ifx n a, c and y n c, b with such<br />

that x n y n 0asn . <strong>The</strong>n we have<br />

f fy<br />

c lim n fx n <br />

n y n x n<br />

.<br />

Proof: Sincef c exists, we have<br />

fy n fc f cy n c oy n c *<br />

and<br />

fx n fc f cx n c ox n c. *’<br />

If we combine (*) and (*’), we have<br />

Note that<br />

we have<br />

fy n fx n <br />

y n x n<br />

y n c<br />

y n x n<br />

lim n<br />

lim n<br />

0.<br />

which implies that, by (**)<br />

f c oy n c<br />

y n x n<br />

1and<br />

oy n c<br />

y n x n<br />

oy n c<br />

y n c<br />

x n c<br />

y n x n<br />

1 for all n,<br />

ox n c<br />

y n x n<br />

y n c<br />

y n x n<br />

ox n c<br />

x n c<br />

ox n c<br />

y n x n<br />

. **<br />

x n c<br />

y n x n


f fy<br />

c lim n fx n <br />

n y n x n<br />

.<br />

(ii) <strong>The</strong>re is a good exercise; we write it as follows. Let f C 2 a, b, andc a, b.<br />

For small |h| such that c h a, b, write<br />

fc h fc f c hhh<br />

where 0 1. Show that if f c 0, then lim h0 h 1/2.<br />

Proof: Since f C 2 a, b, byTaylor <strong>The</strong>orem with Remainder Term, we have<br />

fc h fc f ch f <br />

h<br />

2!<br />

2 ,where c, h or h, c<br />

f c hhh by hypothesis.<br />

So,<br />

f c hh f c<br />

h<br />

h f <br />

2!<br />

and let h 0, we have c by continuity of f at c. Hence,<br />

limh 1/2 since f c 0.<br />

h0<br />

Note: We can modify our statement as follows. Let f be defined on a, b, and<br />

c a, b. For small |h| such that c h a, b, write<br />

fc h fc f c hhh<br />

where 0 1. Show that if f c 0, and x x for x a h, a h, then<br />

lim h0 h 1/2.<br />

Proof: Use the exercise (c), we have<br />

f fc h 2fc fc h<br />

c lim<br />

h0 h 2<br />

f<br />

lim<br />

c hh f c hh<br />

by hypothesis<br />

h0 h<br />

f<br />

lim<br />

c hh f c hh<br />

2h since x x for x a h, a h.<br />

h0 2hh<br />

Since f c 0, we finally have lim h0 h 1/2.<br />

5.25 Let f have a finite derivatiive in a, b and assume that c a, b. Consider the<br />

following condition: For every 0, there exists a 1 ball Bc; , whose radius <br />

depends only on and not on c, such that if x Bc; , andx c, then<br />

fx fc<br />

x c f c .<br />

Show that f is continuous on a, b if this condition holds throughout a, b.<br />

Proof: Given 0, we want to find a 0 such that as dx, y , x, y a, b, we<br />

have<br />

|f x f y| .<br />

Choose any point y a, b, and thus by hypothesis, given /2, there is a 1 ball<br />

By; , whose radius depends only on and not on y, such that if x By; , and<br />

x y, then,<br />

fx fy<br />

x y f y /2 . *<br />

Note that y Bx, , so, we also have<br />

,


fx fy<br />

x y f x /2 *’<br />

Combine (*) with (*’), we have<br />

|f x f y| .<br />

Hence, we have proved f is continuous on a, b.<br />

Remark: (i) <strong>The</strong> open interval can be changed into a closed interval; it just need to<br />

consider its endpoints. That is, f is continuous on a, b if this condition holds throughout<br />

a, b. <strong>The</strong> proof is similar, so we omit it.<br />

(ii) <strong>The</strong> converse of statement in the exercise is alos true. We write it as follows. Let f <br />

be continuous on a, b, and 0. Prove that there exists a 0 such that<br />

fx fc<br />

x c f c <br />

whenever 0 |x c| , a x, c, b.<br />

Proof: Given 0, we want to find a 0 such that<br />

fx fc<br />

x c f c <br />

whenever 0 |x c| , a x, c b. Sincef is continuous on a, b, we know that f is<br />

uniformly continuous on a, b. That is, given 0, there is a 0 such that as<br />

dx, y , wehave<br />

|f x f y| . *<br />

Consider dx, c , x a, b, thenby(*),wehave<br />

fx fc<br />

x c f c |f x f c| by Mean Value <strong>The</strong>orem<br />

where dx , x . So, we complete it.<br />

Note: This could be expressed by saying that f is uniformly differentiable on a, b if f <br />

is continuous on a, b.<br />

5.26 Assume f has a finite derivative in a, b and is continuous on a, b, with<br />

a fx b for all x in a, b and |f x| 1 for all x in a, b. Prove that f has a<br />

unique fixed point in a, b.<br />

Proof: Given any x, y a, b, thus, by Mean Value <strong>The</strong>orem, wehave<br />

|fx fy| |f z||x y| |x y| by hypothesis.<br />

So, we know that f is a contraction on a complete metric space a, b. So,f has a unique<br />

fixed point in a, b.<br />

5.27 Give an example of a pair of functions f and g having a finite derivatives in 0, 1,<br />

such that<br />

but such that lim x0<br />

f x<br />

g x<br />

lim<br />

x0<br />

fx<br />

gx 0,<br />

does not exist, choosing g so that g x is never zero.<br />

Proof: Letfx sin1/x and gx 1/x. <strong>The</strong>n it is trivial for that g x is never zero.<br />

In addition, we have<br />

fx<br />

f<br />

lim 0, and lim<br />

x<br />

does not exist.<br />

x0 gx x0 g x


Remark: In this exercise, it tells us that the converse of L-Hospital Rule is NOT<br />

necessary true. Here is a good exercise very like L-Hospital Rule, but it does not! We<br />

write it as follows.<br />

Suppose that f a and g a exist with g a 0, and fa ga 0. Prove that<br />

Proof: Consider<br />

lim xa<br />

fx<br />

gx lim xa<br />

<br />

lim xa<br />

fx<br />

gx <br />

fx fa<br />

gx ga lim xa<br />

f a<br />

g a .<br />

fx fa/x a<br />

gx ga/x a<br />

f a<br />

g a since f a and g a exist with g a 0.<br />

Note: (i) It should be noticed that we CANNOT use L-Hospital Rule since the<br />

statement tells that f and g have a derivative at a, we do not make sure of the situation of<br />

other points.<br />

(ii) This holds also for complex functions. Let us recall the proof of L-Hospital Rule,<br />

we need use the order field R; however, C is not an order field. Hence, L-Hospital Rule<br />

does not hold for C. Infact,noordercanbedefinedinthecomplexfieldsincei 2 1.<br />

Supplement on L-Hospital Rule<br />

We do not give a proof about the following fact. <strong>The</strong> reader may see the book named A<br />

First Course in <strong>Real</strong> Analysis written by Protter and Morrey, Charpter 4, pp 88-91.<br />

<strong>The</strong>orem ( 0 ) Let f and g be continuous and differentiable on a, b with 0 g 0ona, b.<br />

If<br />

then<br />

lim<br />

xa <br />

lim<br />

xa<br />

fx 0 lim gx 0and<br />

*<br />

<br />

xa f x<br />

g x L,<br />

lim<br />

xa <br />

fx<br />

gx L.<br />

Remark: 1. <strong>The</strong> size of the interval a, b is of no importance; it suffices to<br />

have g 0ona, a , forsome 0.<br />

2. (*) is a sufficient condition, not a necessary condition. For example,<br />

fx x 2 ,andgx sin 1/x both defined on 0, 1.<br />

3. We have some similar results: x a ; x a; x 1/x 0 ;<br />

x 1/x 0 .<br />

<strong>The</strong>orem ( ) Let f and g be continuous and differentiable on a, b with g 0on<br />

a, b. If<br />

then<br />

lim<br />

xa <br />

lim<br />

xa<br />

fx lim gx and<br />

*<br />

<br />

xa f x<br />

g x L,


lim<br />

xa <br />

fx<br />

gx L.<br />

Remark: 1. <strong>The</strong> proof is skilled, and it needs an algebraic identity.<br />

2. We have some similar results: x a ; x a; x 1/x 0 ;<br />

x 1/x 0 .<br />

3. (*) is a sufficient condition, not a necessary condition. For example,<br />

fx x sin x, andgx x.<br />

<strong>The</strong>orem (O. Stolz) Suppose that y n , andy n is increasing. If<br />

lim<br />

x n x n1<br />

n y n y n1<br />

L, (or )<br />

then<br />

lim<br />

x n<br />

n y n<br />

L. (or )<br />

Remark: 1. <strong>The</strong> proof is skilled, and it needs an algebraic identity.<br />

2. <strong>The</strong> difference between <strong>The</strong>orem 2 and <strong>The</strong>orem 3 is that x is a continuous<br />

varibale but x n is not.<br />

<strong>The</strong>orem (Taylor <strong>The</strong>orem with Remainder) Suppose that f is a real function defined on<br />

a, b. Iff n x is continuous on a, b, and differentiable on a, b, then (let<br />

x, c a, b, with x c) there is a x, interior to the interval joining x and c such<br />

that<br />

fx P f x fn1 x<br />

n 1! x cn1 ,<br />

where<br />

n<br />

f<br />

P f x : <br />

k c<br />

x c k .<br />

k!<br />

k0<br />

Remark: 1. As n 1, it is exactly Mean Value <strong>The</strong>orem.<br />

2. <strong>The</strong> part<br />

f n1 x<br />

n 1! x cn1 : R n x<br />

is called the remainder term.<br />

3. <strong>The</strong>re are some types about remainder term. (Lagrange, Cauchy, Berstein,<br />

etc.)<br />

Lagrange<br />

R n x fn1 x<br />

n 1! x cn1 .<br />

Cauchy<br />

Berstein<br />

R n x fn1 c x c<br />

n!<br />

R n x <br />

1 n x c n1 , where 0 1.<br />

1<br />

n! c<br />

x<br />

x t n f n1 tdt


5.28 Prove the following theorem:<br />

Let f and g be two functions having finite nth derivatives in a, b. For some interior<br />

point c in a, b, assume that fc f c ... f n1 c 0, and that<br />

gc g c ... g n1 c 0, but that g n x is never zero in a, b. Show that<br />

lim xc<br />

fx<br />

gx <br />

fn c<br />

g n c .<br />

NOTE. f n and g n are not assumed to be continuous at c.<br />

Hint. Let<br />

Fx fx x cn2 f n c<br />

,<br />

n 2!<br />

define G similarly, and apply <strong>The</strong>orem 5.20 to the functions F and G.<br />

Proof: Let<br />

Fx fx <br />

fn c<br />

x cn2<br />

n 2!<br />

and<br />

Gx gx <br />

gn c<br />

x cn2<br />

n 2!<br />

then inductively,<br />

F k x f k f<br />

x <br />

n c<br />

x cn2k<br />

n 2 k!<br />

and note that<br />

F k c 0 for all k 0, 1, . . , n 3, and F n2 c f n c.<br />

Similarly for G. Hence, by <strong>The</strong>orem 5.20, wehave<br />

n2<br />

Fx <br />

k0<br />

F<br />

k<br />

k! x ck G n1 x 1 F n1 x 1 Gx <br />

k0<br />

n2<br />

G<br />

k<br />

k!<br />

x c k<br />

where x 1 between x and c, which implies that<br />

fxg n1 x 1 f n1 x 1 gx. *<br />

Note that since g n is never zero on a, b; it implies that there exists a 0 such that<br />

every g k is never zero in c , c c, wherek 0, 1, 2. . . , n. Hence, we have, by<br />

(*),<br />

lim xc<br />

fx<br />

gx lim xc<br />

lim<br />

x1 c<br />

lim<br />

x1 c<br />

<br />

f n1 x 1 <br />

g n1 x 1 <br />

f n1 x 1 f n1 c<br />

g n1 x 1 g n1 c since x c x 1 c<br />

f n1 x 1 f n1 c/x 1 c<br />

g n1 x 1 g n1 c/x 1 c<br />

fn c<br />

g n c since fn exists and g n exists 0 on a, b.<br />

Remark: (1) <strong>The</strong> hint is not correct from text book. <strong>The</strong> reader should find the<br />

difference between them.<br />

(2) Here ia another proof by L-Hospital Rule and Remark in Exercise 5.27.<br />

Proof: Since g n is never zero on a, b, it implies that there exists a 0 such that


every g k is never zero in c , c c, wherek 0, 1, 2. . . , n. So, we can apply<br />

n 1 times L-Hospital Rule methoned in Supplement, and thus get<br />

fx<br />

lim xc gx lim f n1 x<br />

xc<br />

g n1 x<br />

lim xc<br />

lim xc<br />

<br />

f n1 x f n1 c<br />

g n1 x g n1 c<br />

f n1 x f n1 c/x c<br />

g n1 x g n1 c/x c<br />

fn c<br />

g n c since fn exists and g n exists 0 on a, b.<br />

5.29 Show that the formula in Taylor’s theorem can also be written as follows:<br />

n1<br />

fx <br />

k0<br />

f<br />

k<br />

c<br />

k!<br />

x c k x cx x 1 n1<br />

n 1!<br />

f n x 1 ,<br />

where x 1 is interior to the interval joining x and c. Let1 x x 1 /x c. Show that<br />

0 1 and deduce the following form of the remainder term (due to Cauchy):<br />

1 n1 x c n<br />

f<br />

n 1!<br />

n x 1 c.<br />

Hint. Take Gt t in the proof of <strong>The</strong>orem 5.20.<br />

Proof: Let<br />

and note that<br />

n1<br />

Ft <br />

k0<br />

f<br />

k<br />

t<br />

k!<br />

x t k ,andGt t,<br />

F t <br />

fn t<br />

x tn1<br />

n 1!<br />

then by Generalized Mean Value <strong>The</strong>orem, wehave<br />

Fx FcG x 1 Gx GcF x 1 <br />

which implies that<br />

n1<br />

fx <br />

k0<br />

f<br />

k<br />

c<br />

k!<br />

So,wehaveprovethat<br />

where<br />

x c k <br />

fn x 1 <br />

n 1! x x 1 n<br />

fn x 1 c<br />

n 1!<br />

n1<br />

fx <br />

k0<br />

f<br />

k<br />

c<br />

k!<br />

x c n 1 n ,wherex 1 x 1 c.<br />

x c k R n1 x,<br />

R n1 x fn x 1 c<br />

x c n 1 n ,wherex<br />

n 1!<br />

1 x 1 c<br />

is called a Cauchy Remainder.<br />

Supplement on some questions.<br />

1. Let f be continuous on 0, 1 and differentiable on 0, 1. Suppose that f0 0and


|f x| |fx| for x 0, 1. Prove that f is constant.<br />

Proof: Given any x 1 0, 1, by Mean Value <strong>The</strong>orem and hypothesis, we know that<br />

|fx 1 f0| x 1 |f x 2 | x 1 |fx 2 |, wherex 2 0, x 1 .<br />

So, we have<br />

|fx 1 | x 1 x n|fx n1 | Mx 1 x n ,wherex n1 0, x n ,andM sup |fx|<br />

xa,b<br />

Since Mx 1 x n 0, as n , we finally have fx 1 0. Since x 1 is arbirary, we find<br />

that fx 0on0, 1.<br />

2. Suppose that g is real function defined on R, with bounded derivative, say |g | M.<br />

Fix 0, and define fx x gx. Show that f is 1-1 if is small enough. (It implies<br />

that f is strictly monotonic.)<br />

Proof: Suppose that fx fy, i.e., x gx y gy which implies that<br />

|y x| |gy gx| M|y x| by Mean Value <strong>The</strong>orem, and hypothesis.<br />

So, as is small enough, we have x y. Thatis,f is 1-1.<br />

Supplement on Convex Function.<br />

Definition(Convex Function) Let f be defined on an interval I, and given 0 1,<br />

we say that f is a convex function if for any two points x, y I,<br />

fx 1 y fx 1 fy.<br />

For example, x 2 is a convex function on R. Sometimes, the reader may see another weak<br />

definition of convex function in case 1/2. We will show that under continuity, two<br />

definitions are equivalent. In addition, it should be noted that a convex function is not<br />

necessarily continuous since we may give a jump on a continuous convex function on its<br />

boundary points, for example, fx x is a continuous convex function on 0, 1, and<br />

define a function g as follows:<br />

gx x, ifx 0, 1 and g1 g0 2.<br />

<strong>The</strong> function g is not continuous but convex. Note that if f is convex, we call f is concave,<br />

vice versa. Note that every increasing convex function of a convex function is convex. (For<br />

example, if f is convex, so is e f . ) It is clear only by definition.<br />

<strong>The</strong>orem(Equivalence) Under continuity, two definitions are equivalent.<br />

Proof: It suffices to consider if<br />

f<br />

x y<br />

2<br />

<br />

fx fy<br />

2<br />

then<br />

fx 1 y fx 1 fy for all 0 1.<br />

Since (*) holds, then by Mathematical Induction, itiseasytoshowthat<br />

f<br />

x 1 ...x 2<br />

n<br />

2 n fx 1 ...fx 2<br />

n <br />

2 n .<br />

Claim that<br />

f<br />

x 1 ...x n<br />

n fx 1 ...fx n <br />

n for all n N. **<br />

Using Reverse Induction, letx n x 1...x n1<br />

, then<br />

n1<br />

*


f<br />

x 1 ...x n<br />

n<br />

f<br />

x 1 ...x n1<br />

n x n<br />

fx n <br />

fx 1 ...fx n <br />

n by induction hypothesis.<br />

So, we have<br />

f<br />

x 1 ...x n1<br />

fx 1 ...fx n1 <br />

.<br />

n 1<br />

n 1<br />

Hence, we have proved (**). Given a rational number m/n 0, 1, where<br />

g. c. d. m, n 1; we choose x : x 1 ... x m ,andy : x m1 ... x n , thenby(**),we<br />

finally have<br />

f mx n my<br />

n n mfx n mfy<br />

n n m n fx 1 m n<br />

fy. ***<br />

Given 0, 1, then there is a sequence q n Q such that q n as n . <strong>The</strong>n<br />

by continuity and (***), we get<br />

fx 1 y fx 1 fy.<br />

Remark: <strong>The</strong> Reverse Induction is that let S N and S has two properties:(1) For<br />

every k 0, 2 k S and (2) k S and k 1 N, then k 1 S. <strong>The</strong>n S N.<br />

(Lemma) Letf be a convex function on a, b, then f is bounded.<br />

Proof: LetM maxfa, fb, then every point z I, write z a 1 b, we<br />

have<br />

fz fa 1 b fa 1 fb M.<br />

In addition, we may write z ab t, wheret is chosen so that z runs through a, b. So,<br />

2<br />

we have<br />

which implies that<br />

2f<br />

f<br />

a b<br />

2<br />

a b<br />

2<br />

1 2 f<br />

f<br />

a b<br />

2<br />

a b<br />

2<br />

t 1 2 f a b<br />

2<br />

t f a b<br />

2<br />

t<br />

t<br />

fz<br />

which implies that<br />

2f a b M : m fz.<br />

2<br />

Hence, we have proved that f is bounded above by M and bounded below by m.<br />

(<strong>The</strong>orem) Iff : I R is convex, then f satisfies a Lipschitz condition on any closed<br />

interval a, b intI. In addition, f is absolutely continuous on a, b and continuous on<br />

intI.<br />

Proof: We choose 0 so that a , b intI. By preceding lemma, we<br />

know that f is bounded, say m fx M on a , b . Given any two points x, andy,<br />

with a x y b We consider an auxiliary point z y , and a suitable <br />

then y z 1 x. So,<br />

fy fz 1 x fz 1 fx fz fx fx<br />

which implies that<br />

fy fx M m y x<br />

Change roles of x and y, we finally have<br />

<br />

M m.<br />

yx<br />

yx ,


|fy fx| K|y x|, whereK M m .<br />

That is, f satisfies a Lipschitz condition on any closed interval a, b.<br />

We call that f is absolutely continuous on a, b if given any 0, there is a 0<br />

n<br />

such that for any collection of a i , b i i1<br />

of disjoint open intervals of a, b with<br />

n<br />

b i a i , wehave<br />

k1<br />

n<br />

|fb i fa i | .<br />

k1<br />

Clearly, the choice /K meets this requirement. Finally, the continuity of f on intI is<br />

obvious.<br />

(<strong>The</strong>orem) Letf be a differentiable real function defined on a, b. Prove that f is<br />

convex if and only if f is monotonically increasing.<br />

Proof: () Suppose f is convex, and given x y, we want to show that f x f y.<br />

Choose s and t such that x u s y, then it is clear that we have<br />

fu fx<br />

u x<br />

fs fu<br />

s u <br />

Let s y , wehaveby(*)<br />

fu fx<br />

u x f y<br />

fy fs<br />

y s . *<br />

which implies that, let u x <br />

f x f y.<br />

() Suppose that f is monotonically increasing, it suffices to consider 1/2, if<br />

x y, then<br />

fx fy<br />

2<br />

f<br />

x y<br />

2<br />

<br />

fx f<br />

xy<br />

2<br />

fy f<br />

xy<br />

2 <br />

2<br />

x y<br />

4 f 1 f 2 , where 1 2<br />

0.<br />

Similarly for x y, and there is nothing to prove x y. Hence, we know that f is convex.<br />

(Corollary 1) Assume next that f x exists for every x a, b, and prove that f is<br />

convex if and only if f x 0 for all x a, b.<br />

Proof: () Suppose that f is convex, we have shown that f is monotonically increasing.<br />

So, we know that f x 0 for all x a, b.<br />

() Suppose that f x 0 for all x a, b, it implies that f is monotonically<br />

increasing. So, we know that f is convex.<br />

(Corollary 2) Let0 , then we have<br />

|y 1 | ...|y n| <br />

n<br />

1/<br />

<br />

|y 1 | ...|y n| <br />

n<br />

Proof: Letp 1, and since x p pp 1x p2 0 for all x 0, we know that<br />

fx x p is convex. So, we have (let p )<br />

by<br />

x 1 ...x n<br />

n<br />

1/<br />

/<br />

<br />

x 1 / ...x n<br />

/<br />

n *


f<br />

x 1 ...x n<br />

n fx 1 ...fx n <br />

n .<br />

Choose x i |y i | ,wherei 1, 2, . . , n. <strong>The</strong>nby(*),wehave<br />

|y 1 | ...|y n| <br />

n<br />

1/<br />

<br />

|y 1 | ...|y n| <br />

n<br />

1/<br />

.<br />

(Corollary 3) Define<br />

M r y <br />

|y 1 | r ...|y n| r<br />

n<br />

1/r<br />

,wherer 0.<br />

<strong>The</strong>n M r y is a monotonic function of r on 0, . In particular, we have<br />

M 1 y M 2 y,<br />

that is,<br />

|y 1 | ...|y n|<br />

n<br />

<br />

|y 1 | 2 ...|y n| 2<br />

n<br />

Proof: It is clear by Corollary 2.<br />

(Corollary 4) By definition of M r y in Corollary 3, wehave<br />

lim M ry |y 1 | |y<br />

r0 n| 1/n : M 0 y<br />

<br />

and<br />

lim r<br />

M r y max|y 1 |,...,|y n| : M y<br />

Proof: 1. Since M r y <br />

<strong>The</strong>orem, wehave<br />

log<br />

So,<br />

2. As r 0, we have<br />

|y 1| r ...|y n| r<br />

n 0<br />

r 0<br />

|y 1 | r ...|y n| r<br />

n<br />

max|y 1 |,...,|y n| r<br />

n<br />

1/2<br />

.<br />

1/r<br />

, taking log and thus by Mean Value<br />

1 n<br />

n |yi i1<br />

| r log|y i |<br />

1 n<br />

, where 0 r r.<br />

n |yi i1<br />

| r<br />

log<br />

lim M ry lim e<br />

r0 r0<br />

<br />

1 n<br />

lim e<br />

r0<br />

<br />

i1<br />

e n<br />

1/r<br />

n<br />

y<br />

r<br />

1 ...|yn| r<br />

n<br />

r<br />

n<br />

i1<br />

1<br />

n<br />

log y i<br />

n<br />

i1<br />

|y 1 | |y n| 1/n .<br />

y i<br />

r log y i<br />

y i<br />

r <br />

M r y max|y 1 |,...,|y n| r 1/r<br />

which implies that, by Sandwich <strong>The</strong>orem,<br />

lim r<br />

M r y max|y 1 |,...,|y n|<br />

since lim r 1 n 1/r 1.<br />

(Inequality 1) Letf be convex on a, b, and let c a, b. Define


lx fa <br />

then fx lx for all x c, b.<br />

fc fa<br />

c a<br />

x a,<br />

Proof: Consider x c, d, then c xa xc a ca<br />

xa x, wehave<br />

fc x a c fa c x a fx<br />

which implies that<br />

fc fa<br />

fx fa c a x a lx.<br />

(Inequality 2) Letf be a convex function defined on a, b. Leta s t u b,<br />

then we have<br />

ft fs<br />

t s<br />

<br />

fu fs<br />

u s<br />

<br />

fu ft<br />

u t<br />

Proof: By definition of convex, we know that<br />

fu fs<br />

fx fs u s x s, x s, u *<br />

and by inequality 1, we know that<br />

ft fs<br />

fs <br />

t s<br />

x s fx, x t, u. **<br />

So, as x t, u, by (*) and (**), we finally have<br />

ft fs fu fs<br />

t s<br />

u s .<br />

Similarly, we have<br />

Hence, we have<br />

ft fs<br />

t s<br />

fu fs<br />

u s<br />

<br />

<br />

fu fs<br />

u s<br />

fu ft<br />

u t<br />

<br />

.<br />

fu ft<br />

u t<br />

Remark: Using abvoe method, it is easy to verify that if f is a convex function on a, b,<br />

then f x and f x exist for all x a, b. In addition, if x y, wherex, y a, b, then<br />

we have<br />

f x f x f y f y.<br />

That is, f x and f x are increasing on a, b. We omit the proof.<br />

(Exercise 1) Letfx be convex on a, b, and assume that f is differentiable at<br />

c a, b, wehave<br />

lx fc f cx c fx.<br />

That is, the equation of tangent line is below fx if the equation of tangent line exists.<br />

Proof: Sincef is differentiable at c a, b, we write the equation of tangent line at c,<br />

lx fc f cx c.<br />

Define<br />

fs fc<br />

ms s c where a s c and mt <br />

then it is clear that<br />

ms f c mt<br />

which implies that<br />

.<br />

.<br />

ft fc<br />

t c<br />

where b t c,


lx fc f cx c fx.<br />

(Exercise 2) Letf : R R be convex. If f is bounded above, then f is a constant<br />

function.<br />

Proof: Suppose that f is not constant, say fa fb, wherea b. Iffb fa, we<br />

consider<br />

fx fb fb fa<br />

,wherex b<br />

x b b a<br />

which implies that as x b,<br />

fb fa<br />

fx x b fb as x <br />

b a<br />

<strong>And</strong> if fb fa, we consider<br />

fx fa fb fa<br />

x a ,wherex a<br />

b a<br />

which implies that as x a,<br />

fb fa<br />

fx x a fa as x .<br />

b a<br />

So, we obtain that f is not bouded above. So, f must be a constant function.<br />

(Exercise 3) Note that e x is convex on R. Use this to show that A. P. G. P.<br />

Proof: Sincee x e x 0onR, we know that e x is convex. So,<br />

ex 1 ...e<br />

x n<br />

n ,wherex i R, i 1, 2, . . . , n.<br />

So, let e x i y i 0, for i 1, 2, . . . , n. <strong>The</strong>n<br />

e x 1 ...xn<br />

n<br />

Vector-Valued functions<br />

y 1 y n 1/n y 1 ...y n<br />

n .<br />

5.30 If a vector valued function f is differentiable at c, prove that<br />

f c lim 1 fc h fc.<br />

h0 h<br />

Conversely, if this limit exists, prove that f is differentiable at c.<br />

Proof: Write f f 1 ,...,f n : S R R n , and let c be an interior point of S. <strong>The</strong>n if<br />

f is differentiable at c, each f k is differentiable at c. Hence,<br />

lim 1 fc h fc<br />

h0 h<br />

f<br />

lim 1 c h f 1 c<br />

,..., f nc h f n c<br />

h0 h<br />

h<br />

f<br />

lim 1 c h f 1 c f<br />

,...,lim n c h f n c<br />

h0 h<br />

h0 h<br />

f 1 c,...,f n c<br />

f c.<br />

Conversly, it is obvious by above.<br />

Remark: We give a summary about this. Let f be a vector valued function defined on<br />

S. Write f : S R n R m , c is a interior point.<br />

f f 1 ,...,f n is differentiable at c each f k is differentiable at c, *<br />

and


f f 1 ,...,f n is continuous at c each f k is continuous at c.<br />

Note: <strong>The</strong> set S can be a subset in R n , the definition of differentiation in higher<br />

dimensional space makes (*) holds. <strong>The</strong> reader can see textbook, Charpter 12.<br />

5.31 A vector-valued function f is differentiable at each point of a, b and has constant<br />

norm f. Prove that ft f t 0ona, b.<br />

Proof: Sincef, f f 2 is constant on a, b, wehavef, f 0ona, b. It implies<br />

that 2f, f 0ona, b. Thatis,ft f t 0ona, b.<br />

Remark: <strong>The</strong> proof of f, g f , g f, g is easy from definition of differentiation.<br />

So, we omit it.<br />

5.32 A vector-valued function f is never zero and has a derivative f which exists and is<br />

continuous on R. If there is a real function such that f t tft for all t, prove that<br />

there is a positive real function u and a constant vector c such that ft utc for all t.<br />

Proof: Since f t tft for all t, wehave<br />

f 1 t,...,f n t f t tft tf 1 t,...,tf n t<br />

which implies that<br />

f i t<br />

t since f is never zero. *<br />

fit<br />

Note that f i t<br />

f i<br />

is a continuous function from R to R for each i 1,2,...,n, sincef is<br />

t<br />

continuous on R, we have, by (*)<br />

f<br />

1 t<br />

ax f 1 t dt x<br />

tdt fi t f ia<br />

e t for i 1, 2, . . . , n.<br />

a<br />

e a<br />

So, we finally have<br />

ft f 1 t,...,f n t<br />

where ut e t and c <br />

e t<br />

utc<br />

f 1 a<br />

e a<br />

f 1 a fna<br />

,..., .<br />

a<br />

e e a<br />

,..., f na<br />

e a<br />

Supplement on Mean Value <strong>The</strong>orem in higher dimensional space.<br />

In the future, we will learn so called Mean Value <strong>The</strong>orem in higher dimensional<br />

space from the text book in Charpter 12. We give a similar result as supplement.<br />

Suppose that f is continuous mapping of a, b into R n and f is differentiable in a, b.<br />

<strong>The</strong>n there exists x a, b such that<br />

fb fa b af x.<br />

Proof: Letz fb fa, and define x fx z which is a real valued function<br />

defined on a, b. It is clear that x is continuous on a, b and differentiable on a, b.<br />

So, by Mean Value <strong>The</strong>orem, we know that<br />

b a xb a, wherex a, b<br />

which implies that<br />

|b a| | xb a|<br />

fb fa xb a by Cauchy-Schwarz inequality.


So, we have<br />

Partial derivatives<br />

fb fa b af x.<br />

5.33 Consider the function f defined on R 2 by the following formulas:<br />

xy<br />

fx, y if x, y 0, 0 f0, 0 0.<br />

x 2 y2 Prove that the partial derivatives D 1 fx, y and D 2 fx, y exist for every x, y in R 2 and<br />

evaluate these derivatives explicitly in terms of x and y. Also, show that f is not continuous<br />

at 0, 0.<br />

Proof: It is clear that for all x, y 0, 0, wehave<br />

D 1 fx, y y y2 x 2<br />

x 2 y 2 and D 2fx, y x x2 y 2<br />

2 x 2 y 2 . 2<br />

For x, y 0, 0, wehave<br />

fx,0 f0, 0<br />

D 1 f0, 0 lim<br />

0.<br />

x0 x 0<br />

Similarly, we have<br />

D 2 f0, 0 0.<br />

In addition, let y x and y 2x, wehave<br />

limfx, x 1/2 limfx,2x 2/5.<br />

x0 x0<br />

Hence, f is not continuous at 0, 0.<br />

Remark: <strong>The</strong> existence of all partial derivatives does not make sure the continuity of f.<br />

<strong>The</strong> trouble with partial derivatives is that they treat a function of several variables as a<br />

function of one variable at a time.<br />

5.34 Let f be defined on R 2 as follows:<br />

fx, y y x2 y 2<br />

if x,,y 0, 0, f0, 0 0.<br />

x 2 y2 Compute the first- and second-order partial derivatives of f at the origin, when they exist.<br />

Proof: For x, y 0, 0, it is clear that we have<br />

4xy<br />

D 1 fx, y <br />

3<br />

x 2 y 2 and D 2fx, y x4 4x 2 y 2 y 4<br />

2 x 2 y 2 2<br />

and for x, y 0, 0, wehave<br />

fx,0 f0, 0<br />

f0, y f0, 0<br />

D 1 f0, 0 lim<br />

0, D<br />

x0 x 0<br />

2 f0, 0 lim<br />

1.<br />

y0 y 0<br />

Hence,<br />

and<br />

D 1,2 f0, 0 lim<br />

x0<br />

D 1 fx,0 D 1 f0, 0<br />

0,<br />

x 0<br />

D 2 fx,0 D 2 f0, 0<br />

lim 2<br />

x 0<br />

x0 x does not exist,<br />

D 1 f0, y D 1 f0, 0<br />

0,<br />

y 0<br />

D 1,1 f0, 0 lim<br />

x0<br />

D 2,1 f0, 0 lim<br />

y0


D 2,2 f0, 0 lim<br />

y0<br />

D 2 f0, y D 2 f0, 0<br />

y 0<br />

lim<br />

y0<br />

0 y 0.<br />

Remark: We do not give a detail computation, but here are answers. Leave to the<br />

reader as a practice. For x, y 0, 0, wehave<br />

D 1,1 fx, y 4y3 y 2 3x 2 <br />

x 2 y 2 3 ,<br />

and<br />

complex-valued functions<br />

D 1,2 fx, y 4xy2 3x 2 y 2 <br />

x 2 y 2 3 ,<br />

D 2,1 fx, y 4xy2 3x 2 y 2 <br />

x 2 y 2 3 ,<br />

D 2,2 fx, y 4x2 yy 2 3x 2 <br />

x 2 y 2 3 .<br />

5.35 Let S be an open set in C and let S be the set of complex conjugates z, where<br />

z S. Iff is defined on S, defineg on S as follows: gz fz, the complex conjugate<br />

of fz. Iff is differentiable at c, prove that g is differentiable at c and that g c f c.<br />

Proof: Sincec S, we know that c is an interior point. Thus, it is clear that c is also an<br />

interior point of S . Note that we have<br />

the conjugate of<br />

fz fc<br />

z c<br />

z<br />

f fc<br />

z c<br />

gz gc<br />

<br />

by gz fz.<br />

z c<br />

Note that z c z c, so we know that if f is differentiable at c, prove that g is<br />

differentiable at c and that g c f c.<br />

5.36 (i) In each of the following examples write f u iv and find explicit formulas<br />

for ux, y and vx, y : ( <strong>The</strong>se functions are to be defined as indicated in Charpter 1.)<br />

(a) fz sin z,<br />

Solution: Since e iz cosz i sin z, we know that<br />

sin z 1 2 ey e y sin x ie y e y cosx<br />

from sin z eiz e iz<br />

2i<br />

and<br />

. So, we have<br />

ux, y ey e y sin x<br />

2<br />

vx, y ey e y cosx<br />

.<br />

2<br />

(b) fz cosz,<br />

Solution: Sincee iz cosz i sin z, we know that<br />

cosz 1 2 ey e y cosx e y e y sin x<br />

from cos z eiz e iz<br />

2<br />

. So, we have


ux, y ey e y cosx<br />

2<br />

and<br />

vx, y ey e y sin x<br />

.<br />

2<br />

(c) fz |z|,<br />

Solution: Since|z| x 2 y 2 1/2 , we know that<br />

ux, y x 2 y 2 1/2<br />

and<br />

vx, y 0.<br />

(d) fz z,<br />

Solution: Since z x iy, we know that<br />

ux, y x<br />

and<br />

vx, y y.<br />

(e) fz arg z, (z 0),<br />

Solution: Since argz R, we know that<br />

ux, y argx iy<br />

and<br />

vx, y 0.<br />

(f) fz Log z, (z 0),<br />

Solution: Since Log z log|z| i argz, we know that<br />

ux, y logx 2 y 2 1/2<br />

and<br />

vx, y argx iy.<br />

and<br />

(g) fz e z2 ,<br />

Solution: Since e z2<br />

e x2 y 2 i2xy<br />

, we know that<br />

ux, y e x2 y 2 cos2xy<br />

vx, y e x2 y 2 sin2xy.<br />

(h) fz z ,( complex, z 0).<br />

Solution: Since z e Log z , thenwehave(let 1 i 2 )<br />

z e 1i 2 log|z|i arg z<br />

e 1 log|z| 2 arg zi 2 log|z| 1 arg z<br />

.<br />

So, we know that<br />

ux, y e 1 log|z| 2 arg z<br />

cos 2 log|z| 1 arg z<br />

e 1 logx 2 y 2 1/2 2 argxiy<br />

cos 2 logx 2 y 2 1/2 1 argx iy


and<br />

vx, y e 1 log|z| 2 arg z<br />

sin 2 log|z| 1 arg z<br />

e 1 logx 2 y 2 1/2 2 argxiy<br />

sin 2 logx 2 y 2 1/2 1 argx iy .<br />

(ii) Show that u and v satisfy the Cauchy -Riemanns equation for the following values<br />

of z :Allz in (a), (b), (g); no z in (c), (d), (e); all z except real z 0in(f),(h).<br />

So,<br />

and<br />

Proof: (a)sinz u iv, where<br />

ux, y ey e y sin x<br />

2<br />

u x v y ey e y cosx<br />

2<br />

and vx, y ey e y cosx<br />

2<br />

for all z x iy<br />

u y v x ey e y sin x<br />

for all z x iy.<br />

2<br />

(b) cos z u iv, where<br />

ux, y ey e y cosx<br />

and vx, y ey e y sin x<br />

2<br />

2<br />

So,<br />

u x v y ey e y sin x<br />

for all z x iy.<br />

2<br />

and<br />

u y v x ey e y cosx<br />

for all z x iy.<br />

2<br />

(c) |z| u iv, where<br />

ux, y x 2 y 2 1/2 and vx, y 0.<br />

So,<br />

u x xx 2 y 2 1/2 v y 0ifx 0, y 0.<br />

and<br />

u y yx 2 y 2 1/2 v x 0ifx 0, y 0.<br />

So, we know that no z makes Cauchy-Riemann equations hold.<br />

(d) z u iv, where<br />

ux, y x and vx, y y.<br />

So,<br />

u x 1 1 v y .<br />

So, we know that no z makes Cauchy-Riemann equations hold.<br />

(e) arg z u iv, where<br />

ux, y argx 2 y 2 1/2 and vx, y 0.<br />

Note that<br />

.<br />

.


ux, y <br />

(1) arctany/x, ifx 0, y R<br />

(2) /2, if x 0, y 0<br />

(3) arctany/x , ifx 0, y 0<br />

(4) arctany/x , ifx 0, y 0<br />

(5) /2, if x 0, y 0.<br />

and<br />

v x v y 0.<br />

So, we know that by (1)-(5), for x, y 0, 0<br />

y<br />

u x <br />

x 2 y 2<br />

and for x, y x, y : x 0, y 0, wehave<br />

u y x<br />

x 2 y . 2<br />

Hence, we know that no z makes Cauchy-Riemann equations hold.<br />

Remark: We can give the conclusion as follows:<br />

y<br />

arg z x<br />

for x, y 0, 0<br />

x 2 y 2<br />

and<br />

arg z y<br />

x for x, y x, y : x 0, y 0.<br />

x 2 y2 (f) Log z u iv, where<br />

ux, y logx 2 y 2 1/2 and vx, y argx 2 y 2 1/2 .<br />

Since<br />

u x x<br />

x 2 y and u y<br />

2 y <br />

x 2 y 2<br />

and<br />

y<br />

v x <br />

x 2 y for x, y 0, 0 and v 2 y x for x, y x, y : x 0, y 0,<br />

x 2 y2 we know that all z except real z 0 make Cauchy-Riemann equations hold.<br />

Remark: Log z is differentiable on C x, y : x 0, y 0 since Cauchy-Riemann<br />

equations along with continuity of u x iv x ,andu y iv y .<br />

(g) e z2 u iv, where<br />

ux, y e x2 y 2 cos2xy and vx, y e x2 y 2 sin2xy.<br />

So,<br />

u x v y 2e x2 y 2 xcos2xy ysin 2xy for all z x iy.<br />

and<br />

u y v x 2e x2 y 2 ycos2xy xsin 2xy for all z x iy.<br />

Hence, we know that all z make Cauchy-Riemann equations hold.<br />

(h) Since z e Log z ,ande z is differentiable on C, we know that, by the remark of (f),<br />

we know that z is differentiable for all z except real z 0. So, we know that all z except<br />

real z 0 make Cauchy-Riemann equations hold.<br />

( In part (h), the Cauchy-Riemann equations hold for all z if is a nonnegative integer,


and they hold for all z 0if is a negative integer.)<br />

Solution: It is clear from definition of differentiability.<br />

(iii) Compute the derivative f z in (a), (b), (f), (g), (h), assuming it exists.<br />

Solution: Sincef z u x iv x , if it exists. So, we know all results by (ii).<br />

5.37 Write f u iv and assume that f has a derivative at each point of an open disk D<br />

centered at 0, 0. Ifau 2 bv 2 is constant on D for some real a and b, not both 0. Prove<br />

that f is constant on D.<br />

Proof: Letau 2 bv 2 be constant on D. We consider three cases as follows.<br />

1. As a 0, b 0, then we have<br />

v 2 is constant on D<br />

which implies that<br />

vv x 0.<br />

If v 0onD, it is clear that f is constant.<br />

If v 0onD, that is v x 0onD. So, we still have f is contant.<br />

2. As a 0, b 0, then it is similar. We omit it.<br />

3. As a 0, b 0, Taking partial derivatives we find<br />

auu x bvv x 0onD. 1<br />

and<br />

auu y bvv y 0onD.<br />

By Cauchy-Riemann equations the second equation can be written as we have<br />

auv x bvu x 0onD. 2<br />

We consider 1v x 2u x and 1u x 2v x , then we have<br />

bvv x2 u x2 0 3<br />

and<br />

auv x2 u x2 0 4<br />

which imply that<br />

au 2 bv 2 v x2 u x2 0. 5<br />

If au 2 bv 2 c, constant on D, wherec 0, then v x2 u x2 0. So, f is constant.<br />

If au 2 bv 2 c, constant on D, wherec 0, then if there exists x, y such that<br />

v x2 u x2 0, then by (3) and (4), ux, y vx, y 0. By continuity of v x2 u x2 , we know<br />

that there exists an open region S D such that u v 0onS. Hence, by Uniqueness<br />

<strong>The</strong>orem, we know that f is constant.<br />

Remark: In complex theory, the Uniqueness theorem is fundamental and important.<br />

<strong>The</strong> reader can see this from the book named <strong>Complex</strong> Analysis by Joseph Bak and<br />

Donald J. Newman.


Functions of Bounded Variation and Rectifiable Curves<br />

Functions of bounded variation<br />

6.1 Determine which of the follwoing functions are of bounded variation on 0, 1.<br />

(a) fx x 2 sin1/x if x 0, f0 0.<br />

(b) fx x sin1/x if x 0, f0 0.<br />

Proof: (a) Since<br />

f x 2x sin1/x cos1/x for x 0, 1 and f 0 0,<br />

we know that f x is bounded on 0, 1, in fact, |f x| 3on0, 1. Hence, f is of<br />

bounded variation on 0, 1.<br />

1<br />

(b) First, we choose n 1beanevenintegersothat 1, and thus consider a<br />

2 n1<br />

partition P 0 x 0 , x 1 1 <br />

we have<br />

2<br />

n2<br />

<br />

k1<br />

, x 2 1<br />

2 2<br />

,..., x n 1<br />

n 2<br />

|f k | 2 2 <br />

n<br />

k1<br />

, x n1 <br />

1<br />

n1 2<br />

1/k .<br />

, x n2 1 , then<br />

Since 1/k diverges to , we know that f is not of bounded variation on 0, 1.<br />

6.2 A function f, defined on a, b, is said to satisfy a uniform Lipschitz condition of<br />

order 0ona, b if there exists a constant M 0 such that |fx fy| M|x y| for<br />

all x and y in a, b. (Compare with Exercise 5.1.)<br />

(a) If f is such a function, show that 1 implies f is constant on a, b, whereas<br />

1 implies f is of bounded variation a, b.<br />

Proof: As 1, we consider, for x y, wherex, y a, b,<br />

|fx fy|<br />

0 M|x y| 1 .<br />

|x y|<br />

Hence, f x exists on a, b, and we have f x 0ona, b. So, we know that f is<br />

constant.<br />

As 1, consider any partition P a x 0 , x 1 ,..., x n b, wehave<br />

n<br />

<br />

k1<br />

n<br />

|f k | M |x k1 x k | Mb a.<br />

k1<br />

That is, f is of bounded variation on a, b.<br />

(b) Give an example of a function f satisfying a uniform Lipschitz condition of order<br />

1ona, b such that f is not of bounded variation on a, b.<br />

Proof: First, note that x satisfies uniform Lipschitz condition of order , where<br />

1<br />

0 1. Choosing 1 such that 1 and let M k1<br />

since the series<br />

k <br />

converges. So, we have 1 <br />

1 1<br />

.<br />

M k1 k <br />

Define a function f as follows. We partition 0, 1 into infinitely many subsintervals.<br />

Consider<br />

x 0 0, x 1 x 0 <br />

M 1 1 1 , x 2 x 1 <br />

M 1 1 2 ,..., x n x n1 <br />

M 1 1 n ,.... <br />

<strong>And</strong> in every subinterval x i , x i1 ,wherei 0,1,...., we define


fx x x i x i1<br />

2<br />

then f is a continuous function and is not bounded variation on 0, 1 since k1<br />

<br />

,<br />

1<br />

2M<br />

diverges.<br />

In order to show that f satisfies uniform Lipschitz condition of order , we consider<br />

three cases.<br />

(1) If x, y x i , x i1 ,andx, y x i , x ix i1<br />

2<br />

or x, y x ix i1<br />

2<br />

, x i1 , then<br />

|fx fy| |x y | |x y| .<br />

(2) If x, y x i , x i1 ,andx x i , x ix i1<br />

or y x ix i1<br />

, x<br />

2 2 i1 , then there is a<br />

z x i , x ix i1<br />

such that fy fz. So,<br />

2<br />

|fx fy| |fx fz| |x z | |x z| |x y| .<br />

(3) If x x i , x i1 and y x j , x j1 ,wherei j.<br />

If x x i , x ix i1<br />

, then there is a z x<br />

2 i , x ix i1<br />

such that fy fz. So,<br />

2<br />

|fx fy| |fx fz| |x z | |x z| |x y| .<br />

Similarly for x x ix i1<br />

, x<br />

2 i1 .<br />

Remark: Here is another example. Since it will use Fourier <strong>The</strong>ory, we do not give a<br />

proof. We just write it down as a reference.<br />

<br />

ft <br />

k1<br />

cos3k t<br />

3 k .<br />

(c) Give an example of a function f which is of bounded variation on a, b but which<br />

satisfies no uniform Lipschitz condition on a, b.<br />

Proof: Since a function satisfies uniform Lipschitz condition of order 0, it must be<br />

continuous. So, we consider<br />

x if x a, b<br />

fx <br />

b 1ifx b.<br />

Trivially, f is not continuous but increasing. So, the function is desired.<br />

Remark: Here is a good problem, we write it as follows. If f satisfies<br />

|fx fy| K|x y| 1/2 for x 0, 1, wheref0 0.<br />

define<br />

fx<br />

x 1/3<br />

if x 0, 1<br />

gx <br />

0ifx 0.<br />

<strong>The</strong>n g satisfies uniform Lipschitz condition of order 1/6.<br />

Proof: Note that if one of x, andy is zero, the result is trivial. So, we may consider<br />

0 y x 1 as follows. Consider<br />

|gx gy| <br />

fx<br />

x 1/3<br />

<br />

<br />

fx<br />

x 1/3<br />

fx<br />

x 1/3<br />

fy<br />

y 1/3<br />

fy<br />

x 1/3<br />

fy<br />

x 1/3<br />

fy fy<br />

x 1/3 x 1/3<br />

fy<br />

y 1/3<br />

1<br />

k <br />

fy<br />

y 1/3 . *


For the part<br />

fx<br />

x fy 1 |fx fy|<br />

1/3 x 1/3 x1/3 K<br />

x |x 1/3 y|1/2 by hypothesis<br />

K|x y| 1/2 |x y| 1/3 since x x y 0<br />

K|x y| 1/6 . A<br />

fy<br />

For another part fy , we consider two cases.<br />

x 1/3 y 1/3<br />

(1) x 2y which implies that x x y y 0,<br />

fy<br />

x 1/3<br />

fy<br />

y 1/3<br />

|fy|<br />

|fy|<br />

x 1/3 y 1/3<br />

xy 1/3<br />

x y 1/3<br />

xy 1/3 since |x 1/3 y 1/3 | |x y| 1/3 for all x, y 0<br />

|fy| x 1/3<br />

xy 1/3 since x y 1/3 x 1/3<br />

|fy| 1<br />

y 1/3<br />

K |y|1/2<br />

|y| 1/3<br />

K|y| 1/6<br />

by hypothesis<br />

K|x y| 1/6 since y x y. B<br />

(2) x 2y which implies that x y x y 0,<br />

fy<br />

x 1/3<br />

fy<br />

y 1/3<br />

|fy|<br />

|fy|<br />

|fy|<br />

x 1/3 y 1/3<br />

xy 1/3<br />

x y 1/3<br />

xy 1/3 since |x 1/3 y 1/3 | |x y| 1/3 for all x, y 0<br />

x y 1/3<br />

y 2/3<br />

K|y| 1/2 x y 1/3<br />

y 2/3<br />

K|y| 1/6 |x y| 1/3<br />

since x y<br />

by hypothesis<br />

K|x y| 1/6 |x y| 1/3 since y x y<br />

K|x y| 1/6 . C<br />

So, by (A)-(C), (*) tells that g satisfies uniform Lipschitz condition of order 1/6.<br />

Note: Hereisageneralresult.Let0 2. Iff satisfies<br />

|fx fy| K|x y| for x 0, 1, wheref0 0.<br />

define


fx<br />

x <br />

if x 0, 1<br />

gx <br />

0ifx 0.<br />

<strong>The</strong>n g satisfies uniform Lipschitz condition of order . <strong>The</strong> proof is similar, so we<br />

omit it.<br />

6.3 Show that a polynomial f is of bounded variation on every compact interval a, b.<br />

Describe a method for finding the total variation of f on a, b if the zeros of the derivative<br />

f are known.<br />

Proof: If f is a constant, then the total variation of f on a, b is zero. So, we may<br />

assume that f is a polynomial of degree n 1, and consider f x 0 by two cases as<br />

follows.<br />

(1) If there is no point such that f x 0, then by Intermediate Value <strong>The</strong>orem of<br />

Differentiability, we know that f x 0ona, b, orf x 0ona, b. So, it implies<br />

that f is monotonic. Hence, the total variation of f on a, b is |fb fa|.<br />

(2) If there are m points such that f x 0, say<br />

a x 0 x 1 x 2 ... x m b x m1 , where 1 m n, then we know the monotone<br />

property of function f. So, the total variation of f on a, b is<br />

m1<br />

|fx i fx i1 |.<br />

i1<br />

Remark: Here is another proof. Let f be a polynomial on a, b, then we know that f is<br />

bounded on a, b since f is also polynomial which implies that it is continuous. Hence, we<br />

know that f is of bounded variation on a, b.<br />

6.4 A nonempty set S of real-valued functions defined on an interval a, b is called a<br />

linear space of functions if it has the following two properties:<br />

(a) If f S, then cf S for every real number c.<br />

(b) If f S and g S, then f g S.<br />

<strong>The</strong>orem 6.9 shows that the set V of all functions of bounded variation on a, b is a<br />

linear space. If S is any linear space which contains all monotonic functions on a, b,<br />

prove that V S. This can be described by saying that the functions of bounded<br />

variation form the samllest linear space containing all monotonic functions.<br />

Proof: It is directlt from <strong>The</strong>orem 6.9 and some facts in Linear Algebra. We omit the<br />

detail.<br />

6.5 Let f be a real-valued function defined on 0, 1 such that f0 0, fx x for all<br />

x, andfx fy whenever x y. LetA x : fx x. Prove that sup A A, andthat<br />

f1 1.<br />

Proof: Note that since f0 0, A is not empty. Suppose that sup A : a A, i.e.,<br />

fa a since fx x for all x. So, given any n 0, then there is a b n A such that<br />

a n b n . *<br />

In addition,<br />

b n fb n since b n A. **<br />

So,by(*)and(**),wehave(let n 0 ),<br />

a fa fa since f is monotonic increasing.<br />

which contradicts to fa a. Hence, we know that sup A A.


Claim that 1 sup A. Suppose NOT, thatis,a 1. <strong>The</strong>n we have<br />

a fa f1 1.<br />

Since a sup A, consider x a, fa, then<br />

fx x<br />

which implies that<br />

fa a<br />

which contradicts to a fa. So, we know that sup A 1. Hence, we have proved that<br />

f1 1.<br />

Remark: <strong>The</strong> reader should keep the method in mind if we ask how to show that<br />

f1 1 directly. <strong>The</strong> set A is helpful to do this. Or equivalently, let f be strictly increasing<br />

on 0, 1 with f0 0. If f1 1, then there exists a point x 0, 1 such that fx x.<br />

6.6 If f is defined everywhere in R 1 , then f is said to be of bounded variation on<br />

, if f is of bounded variation on every finite interval and if there exists a positive<br />

number M such that V f a, b M for all compact interval a, b. <strong>The</strong> total variation of f on<br />

, is then defined to be the sup of all numbers V f a, b, a b , and<br />

denoted by V f , . Similar definitions apply to half open infinite intervals a, and<br />

, b.<br />

(a) State and prove theorems for the inifnite interval , analogous to the<br />

<strong>The</strong>orems 6.7, 6.9, 6.10, 6.11, and 6.12.<br />

(<strong>The</strong>orem 6.7*) Letf : R R be of bounded variaton, then f is bounded on R.<br />

Proof: Given any x R, then x 0, a or x a,0. Ifx 0, a, then f is bounded<br />

on 0, a with<br />

|fx| |f0| V f 0, a |f0| V f , .<br />

Similarly for x a,0.<br />

(<strong>The</strong>orem 6.9*) Assume that f, andg be of bounded variaton on R, thensoarethier<br />

sum, difference, and product. Also, we have<br />

V fg , V f , V g , <br />

and<br />

V fg , AV f , BV g , ,<br />

where A sup xR |gx| and B sup xR |fx|.<br />

Proof: For sum and difference, given any compact interval a, b, wehave<br />

V fg a, b V f a, b V g a, b,<br />

V f , V g , <br />

which implies that<br />

V fg , V f , V g , .<br />

For product, given any compact interval a, b, wehave(letAa, b sup xa,b |gx|,<br />

and Ba, b sup xa,b |fx|),<br />

V fg a, b Aa, bV f a, b Ba, bV g a, b<br />

AV f , BV g , <br />

which implies that


V fg , AV f , BV g , .<br />

(<strong>The</strong>orem 6.10*) Letf be of bounded variation on R, and assume that f is bounded<br />

away from zero; that is, suppose that there exists a positive number m such that<br />

0 m |fx| for all x R. <strong>The</strong>n g 1/f is also of bounded variation on R, and<br />

V g , V f, <br />

m 2 .<br />

Proof: Given any compacgt interval a, b, wehave<br />

which implies that<br />

V g a, b V fa, b<br />

m 2<br />

V f, <br />

m 2<br />

V g , V f, <br />

.<br />

m 2<br />

(<strong>The</strong>orem 6.11*) Letf be of bounded variation on R, and assume that c R. <strong>The</strong>n f is<br />

of bounded variation on , c and on c, and we have<br />

V f , V f , c V f c, .<br />

Proof: Given any a compact interval a, b such that c a, b. <strong>The</strong>n we have<br />

V f a, b V f a, c V f c, b.<br />

Since<br />

V f a, b V f , <br />

which implies that<br />

V f a, c V f , and V f c, b V f , ,<br />

we know that the existence of V f , c and V f c, . Thatis,f is of bounded variation on<br />

, c and on c, .<br />

Since<br />

V f a, c V f c, b V f a, b V f , <br />

which implies that<br />

V f , c V f c, V f , , *<br />

and<br />

V f a, b V f a, c V f c, b V f , c V f c, <br />

which implies that<br />

V f , V f , c V f c, , **<br />

we know that<br />

V f , V f , c V f c, .<br />

(<strong>The</strong>orem 6.12*) Letf be of bounded variation on R. LetVx be defined on , x as<br />

follows:<br />

Vx V f , x if x R, andV 0.<br />

<strong>The</strong>n (i) V is an increasing function on , and (ii) V f is an increasing function on<br />

, .<br />

Proof: (i)Letx y, then we have Vy Vx V f x, y 0. So, we know that V is<br />

an increasing function on , .<br />

(ii) Let x y, then we have V fy V fx V f x, y fy fx 0. So,


we know that V f is an increasing function on , .<br />

(b) Show that <strong>The</strong>orem 6.5 is true for , if ”monotonic” is replaced by ”bounded<br />

and monotonic.” State and prove a similar modefication of <strong>The</strong>orem 6.13.<br />

(<strong>The</strong>orem 6.5*) Iff is bounded and monotonic on , , then f is of bounded<br />

variation on , .<br />

Proof: Given any compact interval a, b, then we have V f a, b exists, and we have<br />

V f a, b |fb fa|, sincef is monotonic. In addition, since f is bounded on R, say<br />

|fx| M for all x, we know that 2M is a upper bounded of V f a, b for all a, b. Hence,<br />

V f , exists. That is, f is of bounded variation on R.<br />

(<strong>The</strong>orem 6.13*) Letf be defined on , , then f is of bounded variation on<br />

, if, and only if, f can be expressed as the difference of two increasing and<br />

bounded functions.<br />

Proof: Suppose that f is of bounded variation on , , then by <strong>The</strong>orem 6.12*, we<br />

know that<br />

f V V f,<br />

where V and V f are increasing on , . In addition, since f is of bounded variation<br />

on R, we know that V and f is bounded on R which implies that V f is bounded on R. So,<br />

we have proved that if f is of bounded variation on , then f can be expressed as the<br />

difference of two increasing and bounded functions.<br />

Suppose that f can be expressed as the difference of two increasing and bounded<br />

functions, say f f 1 f 2 , <strong>The</strong>n by <strong>The</strong>orem 6.9*, and<strong>The</strong>orem 6.5*, we know that f is of<br />

bounded variaton on R.<br />

Remark: <strong>The</strong> representation of a function of bounded variation as a difference of two<br />

increasing and bounded functions is by no mean unique. It is clear that <strong>The</strong>orem 6.13*<br />

also holds if ”increasing” is replaced by ”strictly increasing.” For example,<br />

f f 1 g f 2 g, whereg is any strictly increasing and bounded function on R. One<br />

of such g is arctan x.<br />

6.7 Assume that f is of bounded variation on a, b and let<br />

P x 0 , x 1 ,...,x n þa, b.<br />

As usual, write f k fx k fx k1 , k 1,2,...,n. Define<br />

AP k : f k 0, BP k : f k 0.<br />

<strong>The</strong> numbers<br />

and<br />

p f a, b sup<br />

f k : P þa, b<br />

kAP<br />

n f a, b sup<br />

|f k | : P þa, b<br />

kBP<br />

are called respectively, the positive and negative variations of f on a, b. For each x in<br />

a, b. LetVx V f a, x, px p f a, x, nx n f a, x, and let<br />

Va pa na 0. Show that we have:<br />

(a) Vx px nx.


Proof: Given a partition P on a, x, then we have<br />

n<br />

<br />

k1<br />

|f k | |f k | |f k |<br />

kAP<br />

<br />

kAP<br />

kBP<br />

f k |f k |,<br />

kBP<br />

which implies that (taking supermum)<br />

Vx px nx. *<br />

Remark: <strong>The</strong> existence of px and qx is clear, so we know that (*) holds by<br />

<strong>The</strong>orem 1.15.<br />

(b) 0 px Vx and 0 nx Vx.<br />

Proof: Consider a, x, and since<br />

n<br />

Vx |f k | |f k |,<br />

k1 kAP<br />

we know that 0 px Vx. Similarly for 0 nx Vx.<br />

(c) p and n are increasing on a, b.<br />

Proof: Letx, y in a, b with x y, and consider py px as follows. Since<br />

py f k f k ,<br />

kAP, a,y kAP, a,x<br />

we know that<br />

py px.<br />

That is, p is increasing on a, b. Similarly for n.<br />

(d) fx fa px nx. Part (d) gives an alternative proof of <strong>The</strong>orem 6.13.<br />

Proof: Consider a, x, and since<br />

which implies that<br />

fx fa <br />

k1<br />

n<br />

f k f k f k<br />

kAP kBP<br />

fx fa |f k | f k<br />

kBP kAP<br />

which implies that fx fa px nx.<br />

(e) 2px Vx fx fa, 2nx Vx fx fa.<br />

Proof: By (d) and (a), the statement is obvious.<br />

(f) Every point of continuity of f is also a point of continuity of p and of n.<br />

Proof: By (e) and <strong>The</strong>orem 6.14, the statement is obvious.<br />

Curves<br />

6.8 Let f and g be complex-valued functions defined as follows:<br />

ft e 2it if t 0, 1, gt e 2it if t 0, 2.<br />

(a) Prove that f and g have the same graph but are not equivalent according to defintion


in Section 6.12.<br />

Proof: Sinceft : t 0, 1 gt : t 0, 2 the circle of unit disk, we know<br />

that f and g have the same graph.<br />

If f and g are equivalent, then there is an 1-1 and onto function : 0, 2 0, 1 such<br />

that<br />

ft gt.<br />

That is,<br />

e 2it cos2t i sin 2t e 2it cos2t i sin 2t.<br />

In paticular, 1 : c 0, 1. However,<br />

fc cos2c i sin 2c g1 1<br />

which implies that c Z, a contradiction.<br />

(b) Prove that the length of g is twice that of f.<br />

Proof: Since<br />

and<br />

the length of g <br />

0<br />

2<br />

|g t|dt 4<br />

the length of f <br />

0<br />

1<br />

|f t|dt 2,<br />

we know that the length of g is twice that of f.<br />

6.9 Let f be rectifiable path of length L defined on a, b, and assume that f is not<br />

constant on any subinterval of a, b. Lets denote the arc length function given by<br />

sx f a, x if a x b, sa 0.<br />

(a) Prove that s 1 exists and is continuous on 0, L.<br />

Proof: By<strong>The</strong>orem 6.19, we know that sx is continuous and strictly increasing on<br />

0, L. So, the inverse function s 1 exists since s is an 1-1 and onto function, and by<br />

<strong>The</strong>orem 4.29, we know that s 1 is continuous on 0, L.<br />

(b) Define gt fs 1 t if t 0, L and show that g is equivalent to f. Since<br />

ft gst, the function g is said to provide a representation of the graph of f with arc<br />

length as parameter.<br />

Proof: tisclearby<strong>The</strong>orem 6.20.<br />

6.10 Let f and g be two real-valued continuous functions of bounded variation defined<br />

on a, b, with 0 fx gx for each x in a, b, fa ga, fb gb. Leth be the<br />

complex-valued function defined on the interval a,2b a as follows:<br />

ht t ift, ifa t b<br />

2b t ig2b t, ifb t 2b a.<br />

(a) Show that h describes a rectifiable curve .<br />

Proof: It is clear that h is continuous on a,2b a. Note that t, f and g are of bounded<br />

variation on a, b, so h a,2b a exists. That is, h is rectifiable on a,2b a.<br />

(b) Explain, by means of a sketch, the geometric relationship between f, g, andh.<br />

Solution: <strong>The</strong> reader can give it a draw and see the graph lying on x y plane is a


closed region.<br />

(c) Show that the set of points<br />

S x, y : a x b, fx y gx<br />

in a region in R 2 whose boundary is the curve .<br />

Proof: It can be answered by (b), so we omit it.<br />

(d) Let H be the complex-valued function defined on a,2b a as follows:<br />

Ht t 1 igt ft, ifa t b<br />

2<br />

2b t 1 ig2b t f2b t, ifb t 2b a.<br />

2<br />

Show that H describes a rectifiable curve 0 which is the boundary of the region<br />

S 0 x, y : a x b, fx gx 2y gx fx.<br />

Proof: LetFt 1 gt ft and Gt 1 gt ft defined on a, b. Itis<br />

2 2<br />

clear that Ft and Gt are of bounded variation and continuous on a, b with<br />

0 Fx Gx for each x a, b, Fb Gb 0, and Fb Gb 0. In<br />

addition, we have<br />

Ht t iFt, ifa t b<br />

2b t iG2b t, ifb t 2b a.<br />

So, by preceding (a)-(c), we have prove it.<br />

(e) Show that, S 0 has the x axis as a line of symmetry. (<strong>The</strong> region S 0 is called the<br />

symmetrization of S with respect to x axis.)<br />

Proof: Itisclearsincex, y S 0 x, y S 0 by the fact<br />

fx gx 2y gx fx.<br />

(f) Show that the length of 0 does not exceed the length of .<br />

Proof: By (e), the symmetrization of S with respect to x axis tells that<br />

H a, b H b,2b a. So, it suffices to show that h a,2b a 2 H a, b.<br />

Choosing a partition P 1 x 0 a,...,x n b on a, b such that<br />

2 H a, b 2 H P 1 <br />

n<br />

2 x i x i1 2 1 2 f gx i 1 2 f gx i1 2 1/2<br />

i1<br />

n<br />

<br />

i1<br />

4x i x i1 2 f gx i f gx i1 2 1/2<br />

and note that b a 2b a b, we use this P 1 to produce a partition<br />

P 2 P 1 x n b, x n1 b x n x n1 ,...,x 2n 2b a on a,2b a. <strong>The</strong>n we have<br />

*


2n<br />

h P 2 hx i hx i1 <br />

i1<br />

n<br />

2n<br />

hx i hx i1 hx i hx i1 <br />

i1<br />

in1<br />

n<br />

<br />

i1<br />

n<br />

<br />

i1<br />

x i x i1 2 fx i fx i1 2<br />

2n<br />

1/2<br />

<br />

in1<br />

x i x i1 2 gx i gx i1 2 1/2<br />

x i x i1 2 fx i fx i1 2 1/2<br />

xi x i1 2 gx i gx i1 2 1/2<br />

From (*) and (**), we know that<br />

2 H a, b 2 H P 1 h P 2 ***<br />

which implies that<br />

H a,2b a 2 H a, b h a,2b a.<br />

So, we know that the length of 0 does not exceed the length of .<br />

Remark: Definex i x i1 a i , fx i fx i1 b i ,andgx i gx i1 c i , then we<br />

have<br />

4a i2 b i c i 2 1/2<br />

ai2 b i2 1/2 a i2 c i2 1/2 .<br />

Hence we have the result (***).<br />

Proof: It suffices to square both side. We leave it to the reader.<br />

Absolutely continuous functions<br />

A real-valued function f defined on a, b is said to be absolutely continuous on a, b<br />

if for every 0, there is a 0 such that<br />

n<br />

<br />

k1<br />

|fb k fa k | <br />

for every n disjoint open subintervals a k , b k of a, b, n 1, 2, . . . , the sum of whose<br />

n<br />

lengths k1<br />

b k a k is less than .<br />

Absolutely continuous functions occur in the Lebesgue theory of integration and<br />

differentiation. <strong>The</strong> following exercises give some of their elementary properties.<br />

6.11 Prove that every absolutely continuous function on a, b is continuous and of<br />

bounded variation on a, b.<br />

Proof: Letf be absolutely continuous on a, b. <strong>The</strong>n 0, there is a 0 such that<br />

n<br />

<br />

k1<br />

|fb k fa k | <br />

for every n disjoint open subintervals a k , b k of a, b, n 1, 2, . . . , the sum of whose<br />

n<br />

lengths k1<br />

b k a k is less than . So,as|x y| , wherex, y a, b, wehave<br />

|fx fy| .<br />

That is, f is uniformly continuous on a, b. So,f is continuous on a, b.<br />

n<br />

In addition, given any 1, there exists a 0 such that as k1<br />

b k a k ,<br />

where a k , b k s are disjoint open intervals in a, b, wehave<br />

**


n<br />

|fb k fa k | 1.<br />

k1<br />

For this , and let K be the smallest positive integer such that K/2 b a. So,we<br />

partition a, b into K closed subintervals, i.e.,<br />

P y 0 a, y 1 a /2,....,y K1 a K 1/2, y K b. So, it is clear that f is<br />

of bounded variation y i , y i1 ,wherei 0, 1, . . . , K. It implies that f is of bounded<br />

variation on a, b.<br />

Note: <strong>The</strong>re exists functions which are continuous and of bounded variation but<br />

not absolutely continuous.<br />

Remark: 1. <strong>The</strong> standard example is called Cantor-Lebesgue function. <strong>The</strong> reader<br />

can see this in the book, Measure and Integral, An Introduction to <strong>Real</strong> Analysis by<br />

Richard L. Wheeden and Antoni Zygmund, pp 35 and pp 115.<br />

2. If we wrtie ”absolutely continuous” by ABC, ”continuous” by C, and ”bounded<br />

variation” by B, then it is clear that by preceding result, ABC implies B and C, andB and<br />

C do NOT imply ABC.<br />

6.12 Prove that f is absolutely continuous if it satisfies a uniform Lipschitz condition<br />

of order 1 on a, b. (See Exercise 6.2)<br />

Proof: Letf satisfy a uniform Lipschitz condition of order 1 on a, b, i.e.,<br />

|fx fy| M|x y| where x, y a, b. <strong>The</strong>ngiven 0, there is a /M such that<br />

n<br />

as k1<br />

b k a k , wherea k , b k s are disjoint open subintervals on a, b, k 1, . . , n,<br />

we have<br />

n<br />

<br />

k1<br />

|fb k fa k | M|b k a k |<br />

n<br />

k1<br />

n<br />

<br />

k1<br />

Mb k a k <br />

M<br />

.<br />

Hence, f is absolutely continuous on a, b.<br />

6.13 If f and g are absolutely continunous on a, b, prove that each of the following<br />

is also: |f|, cf (c constant), f g, f g; alsof/g if g is bounded away from zero.<br />

Proof: (1) (|f| is absolutely continuous on a, b): Given 0, we want to find a<br />

n<br />

0, such that as k1<br />

b k a k , wherea k , b k s are disjoint open intervals on<br />

a, b, wehave<br />

n<br />

||fb k | |fa k || . 1*<br />

k1<br />

Since f is absolutely continunous on a, b, for this , there is a 0 such that as<br />

n<br />

k1<br />

b k a k , wherea k , b k s are disjoint open intervals on a, b, wehave<br />

n<br />

|fb k fa k | <br />

k1<br />

which implies that (1*) holds by the following


n<br />

<br />

k1<br />

n<br />

||fb k | |fa k || |fb k fa k | .<br />

So, we know that |f| is absolutely continuous on a, b.<br />

(2) (cf is absolutely continuous on a, b): If c 0, it is clear. So, we may assume that<br />

n<br />

c 0. Given 0, we want to find a 0, such that as k1<br />

b k a k , where<br />

a k , b k s are disjoint open intervals on a, b, wehave<br />

n<br />

<br />

k1<br />

k1<br />

|cfb k cfa k | . 2*<br />

Since f is absolutely continunous on a, b, for this , there is a 0 such that as<br />

n<br />

k1<br />

b k a k , wherea k , b k s are disjoint open intervals on a, b, wehave<br />

n<br />

|fb k fa k | /|c|<br />

k1<br />

which implies that (2*) holds by the following<br />

n<br />

<br />

k1<br />

|cfb k cfa k | |c| |fb k fa k | .<br />

So, we know that cf is absolutely continuous on a, b.<br />

(3) (f g is absolutely continuous on a, b): Given 0, we want to find a 0,<br />

n<br />

such that as k1<br />

b k a k , wherea k , b k s are disjoint open intervals on a, b, we<br />

have<br />

n<br />

|f gb k f ga k | . 3*<br />

k1<br />

Since f and g are absolutely continunous on a, b, for this , there is a 0 such that as<br />

n<br />

b k a k , wherea k , b k s are disjoint open intervals on a, b, wehave<br />

k1<br />

n<br />

<br />

k1<br />

|fb k fa k | /2 and |gb k ga k | /2<br />

which implies that (3*) holds by the following<br />

n<br />

<br />

k1<br />

n<br />

n<br />

k1<br />

n<br />

k1<br />

|f gb k f ga k |<br />

|fb k fa k gb k ga k |<br />

k1<br />

n<br />

<br />

k1<br />

n<br />

|fb k fa k | <br />

k1<br />

|gb k ga k |<br />

.<br />

So, we know that f g is absolutely continuous on a, b.<br />

(4) (f g is absolutely continuous on a, b.): Let M f sup xa,b |fx| and<br />

M g sup xa,b |gx|. Given 0, we want to find a 0, such that as<br />

n<br />

b k a k , wherea k , b k s are disjoint open intervals on a, b, wehave<br />

k1


n<br />

|f gb k f ga k | . 4*<br />

k1<br />

Since f and g are absolutely continunous on a, b, for this , there is a 0 such that as<br />

n<br />

b k a k , wherea k , b k s are disjoint open intervals on a, b, wehave<br />

k1<br />

n<br />

n<br />

|fb k fa k | <br />

2M g 1 and |gb k ga k | <br />

k1<br />

k1<br />

which implies that (4*) holds by the following<br />

n<br />

<br />

k1<br />

n<br />

<br />

k1<br />

M f <br />

k1<br />

<br />

.<br />

|f gb k f ga k |<br />

|fb k gb k ga k ga k fb k fa k |<br />

n<br />

n<br />

|gb k ga k | M g <br />

k1<br />

M f<br />

2M f 1 <br />

M g<br />

2M g 1<br />

|fb k fa k |<br />

<br />

2M f 1<br />

Remark: <strong>The</strong> part shows that f n is absolutely continuous on a, b, wheren N, iff is<br />

absolutely continuous on a, b.<br />

(5) (f/g is absolutely continuous on a, b): By (4) it suffices to show that 1/g is<br />

absolutely continuous on a, b. Sinceg is bounded away from zero, say 0 m gx for<br />

n<br />

all x a, b. Given 0, we want to find a 0, such that as k1<br />

b k a k ,<br />

where a k , b k s are disjoint open intervals on a, b, wehave<br />

n<br />

<br />

k1<br />

|1/gb k 1/ga k | . 5*<br />

Since g is absolutely continunous on a, b, for this , there is a 0 such that as<br />

n<br />

k1<br />

b k a k , wherea k , b k s are disjoint open intervals on a, b, wehave<br />

n<br />

|gb k ga k | m 2 <br />

k1<br />

which implies that (4*) holds by the following<br />

n<br />

<br />

k1<br />

n<br />

<br />

k1<br />

|1/gb k 1/ga k |<br />

gb k ga k <br />

gb k ga k <br />

n<br />

1 |gb m 2 k ga k |<br />

k1<br />

.


Supplement on lim sup and lim inf<br />

Introduction<br />

In order to make us understand the information more on approaches of a given real<br />

sequence a n <br />

n1<br />

, we give two definitions, thier names are upper limit and lower limit. It<br />

is fundamental but important tools in analysis. We do NOT give them proofs. <strong>The</strong> reader<br />

can see the book, Infinite Series by Chao Wen-Min, pp 84-103. (Chinese Version)<br />

Definition<br />

Definition of limit sup and limit inf<br />

and<br />

Example<br />

Example<br />

Example<br />

Given a real sequence a n <br />

n1<br />

,wedefine<br />

b n supa m : m ≥ n<br />

1 −1 n <br />

n1<br />

−1 n <br />

n n1<br />

<br />

−n n1<br />

c n infa m : m ≥ n.<br />

0,2,0,2,..., sowehave<br />

b n 2andc n 0 for all n.<br />

−1, 2, −3,4,..., sowehave<br />

b n and c n − for all n.<br />

−1, −2, −3, . . . , sowehave<br />

b n −n and c n − for all n.<br />

Proposition Given a real sequence a n <br />

n1<br />

, and thus define b n and c n as the same as<br />

before.<br />

1 b n ≠−,andc n ≠∀n ∈ N.<br />

2 If there is a positive integer p such that b p , then b n ∀n ∈ N.<br />

If there is a positive integer q such that c q −, then c n − ∀n ∈ N.<br />

3 b n is decreasing and c n is increasing.<br />

By property 3, we can give definitions on the upper limit and the lower limit of a given<br />

sequence as follows.<br />

Definition Given a real sequence a n and let b n and c n as the same as before.<br />

(1) If every b n ∈ R, then<br />

infb n : n ∈ N<br />

is called the upper limit of a n , denoted by<br />

lim n→<br />

sup a n .<br />

That is,<br />

lim n→<br />

sup a n inf b n.<br />

n<br />

If every b n , then we define<br />

lim n→<br />

sup a n .<br />

(2) If every c n ∈ R, then<br />

supc n : n ∈ N<br />

is called the lower limit of a n , denoted by


lim n→<br />

inf a n .<br />

That is,<br />

lim n→<br />

inf a n sup c n .<br />

n<br />

If every c n −, then we define<br />

lim n→<br />

inf a n −.<br />

Remark <strong>The</strong> concept of lower limit and upper limit first appear in the book (Analyse<br />

Alge’brique) written by Cauchy in 1821. But until 1882, Paul du Bois-Reymond<br />

gave explanations on them, it becomes well-known.<br />

Example 1 −1 n <br />

n1<br />

0,2,0,2,..., sowehave<br />

b n 2andc n 0 for all n<br />

which implies that<br />

lim sup a n 2 and lim inf a n 0.<br />

Example −1 n <br />

n n1<br />

−1, 2, −3,4,..., sowehave<br />

b n and c n − for all n<br />

which implies that<br />

lim sup a n and lim inf a n −.<br />

<br />

−n n1<br />

Example −1, −2, −3, . . . , sowehave<br />

b n −n and c n − for all n<br />

which implies that<br />

lim sup a n − and lim inf a n −.<br />

Relations with convergence and divergence for upper (lower) limit<br />

<strong>The</strong>orem Let a n be a real sequence, then a n converges if, and only if, the upper<br />

limit and the lower limit are real with<br />

lim n→<br />

sup a n lim n→<br />

inf a n lim n→<br />

a n .<br />

<strong>The</strong>orem Let a n be a real sequence, then we have<br />

(1) lim n→ sup a n a n has no upper bound.<br />

(2) lim n→ sup a n − for any M 0, there is a positive integer n 0 such<br />

that as n ≥ n 0 ,wehave<br />

a n ≤−M.<br />

(3) lim n→ sup a n a if, and only if, (a) given any 0, there are infinite<br />

many numbers n such that<br />

a − a n<br />

and (b) given any 0, there is a positive integer n 0 such that as n ≥ n 0 ,wehave<br />

a n a .<br />

Similarly, we also have<br />

<strong>The</strong>orem Let a n be a real sequence, then we have


(1) lim n→ inf a n − a n has no lower bound.<br />

(2) lim n→ inf a n for any M 0, there is a positive integer n 0 such<br />

that as n ≥ n 0 ,wehave<br />

a n ≥ M.<br />

(3) lim n→ inf a n a if, and only if, (a) given any 0, there are infinite<br />

many numbers n such that<br />

a a n<br />

and (b) given any 0, there is a positive integer n 0 such that as n ≥ n 0 ,wehave<br />

a n a − .<br />

From <strong>The</strong>orem 2 an <strong>The</strong>orem 3, the sequence is divergent, we give the following<br />

definitios.<br />

Definition Let a n be a real sequence, then we have<br />

(1) If lim n→ sup a n −, then we call the sequence a n diverges to −,<br />

denoted by<br />

lim n→<br />

a n −.<br />

<strong>The</strong>orem<br />

<strong>The</strong>orem<br />

<strong>The</strong>orem<br />

(2) If lim n→ inf a n , then we call the sequence a n diverges to ,<br />

denoted by<br />

lim n→<br />

a n .<br />

Let a n be a real sequence. If a is a limit point of a n , then we have<br />

lim n→<br />

inf a n ≤ a ≤ lim n→<br />

sup a n .<br />

Some useful results<br />

Let a n be a real sequence, then<br />

(1) lim n→ inf a n ≤ lim n→ sup a n .<br />

(2) lim n→ inf−a n − lim n→ sup a n and lim n→ sup−a n − lim n→ inf a n<br />

(3) If every a n 0, and 0 lim n→ inf a n ≤ lim n→ sup a n , then we<br />

have<br />

lim n→<br />

sup a 1 n<br />

1<br />

lim n→ inf a n<br />

Let a n and b n be two real sequences.<br />

(1) If there is a positive integer n 0 such that a n ≤ b n , then we have<br />

lim n→<br />

inf a n ≤ lim n→<br />

inf b n and lim n→<br />

sup a n ≤ lim n→<br />

sup b n .<br />

(2) Suppose that − lim n→ inf a n , lim n→ inf b n , lim n→ sup a n ,<br />

lim n→ sup b n , then<br />

lim n→<br />

inf a n lim n→<br />

inf b n<br />

and lim n→<br />

inf 1 a n<br />

1<br />

lim n→ sup a n<br />

.<br />

≤ lim n→<br />

infa n b n <br />

≤ lim n→<br />

inf a n lim n→<br />

sup b n (or lim n→<br />

sup a n lim n→<br />

inf b n )<br />

≤ lim n→<br />

supa n b n <br />

≤ lim n→<br />

sup a n lim n→<br />

sup b n .


In particular, if a n converges, we have<br />

lim n→<br />

supa n b n lim n→<br />

a n lim n→<br />

sup b n<br />

and<br />

lim n→<br />

infa n b n lim n→<br />

a n lim n→<br />

inf b n .<br />

(3) Suppose that − lim n→ inf a n , lim n→ inf b n , lim n→ sup a n ,<br />

lim n→ sup b n , anda n 0, b n 0 ∀n, then<br />

lim n→<br />

inf a n<br />

lim n→<br />

inf b n<br />

≤ lim n→<br />

infa n b n <br />

≤ lim n→<br />

inf a n lim n→<br />

sup b n (or lim n→<br />

inf b n<br />

≤ lim n→<br />

supa n b n <br />

≤ lim n→<br />

sup a n lim n→<br />

sup b n .<br />

In particular, if a n converges, we have<br />

and<br />

lim n→<br />

supa n b n <br />

lim n→<br />

infa n b n <br />

lim n→<br />

a n<br />

lim n→<br />

a n<br />

lim n→<br />

sup b n<br />

lim n→<br />

inf b n .<br />

lim n→<br />

sup a n )<br />

<strong>The</strong>orem Let a n be a positive real sequence, then<br />

lim n→<br />

inf a n1<br />

a n<br />

≤ lim n→<br />

infa n 1/n ≤ lim n→<br />

supa n 1/n ≤ lim n→<br />

sup a n1<br />

a n<br />

.<br />

Remark We can use the inequalities to show<br />

n! 1/n<br />

lim n→ n 1/e.<br />

<strong>The</strong>orem Let a n be a real sequence, then<br />

lim n→<br />

inf a n ≤ lim n→<br />

inf a 1 ...a n<br />

n<br />

≤ lim n→<br />

sup a 1 ...a n<br />

n<br />

≤ lim n→<br />

sup a n .<br />

Exercise Let f : a, d → R be a continuous function, and a n is a real sequence. If f is<br />

increasing and for every n, lim n→ inf a n , lim n→ sup a n ∈ a, d, then<br />

lim n→<br />

sup fa n f lim n→<br />

sup a n<br />

and lim n→<br />

inf fa n f lim n→<br />

inf a n .<br />

Remark: (1) <strong>The</strong> condition that f is increasing cannot be removed. For<br />

example,<br />

fx |x|,<br />

and<br />

1/k if k is even<br />

a k <br />

−1 − 1/k if k is odd.<br />

(2) <strong>The</strong> proof is easy if we list the definition of limit sup and limit inf. So, we<br />

omit it.<br />

Exercise Let a n be a real sequence satisfying a np ≤ a n a p for all n, p. Show that<br />

an<br />

n converges.<br />

Hint: Consider its limit inf.


Remark: <strong>The</strong> exercise is useful in the theory of Topological Entorpy.<br />

Infinite Series <strong>And</strong> Infinite Products<br />

Sequences<br />

8.1 (a) Given a real-valed sequence a n bounded above, let u n supa k : k ≥ n.<br />

<strong>The</strong>n u n ↘ and hence U lim n→ u n is either finite or −. Prove that<br />

U lim n→<br />

sup a n lim n→<br />

supa k : k ≥ n.<br />

Proof: It is clear that u n ↘ and hence U lim n→ u n is either finite or −.<br />

If U −, then given any M 0, there exists a positive integer N such that as n ≥ N,<br />

we have<br />

u n ≤−M<br />

which implies that, as n ≥ N, a n ≤−M. So, lim n→ a n −. Thatis,a n is not bounded<br />

below. In addition, if a n has a finite limit supreior, say a. <strong>The</strong>ngiven 0, and given<br />

m 0, there exists an integer n m such that<br />

a n a − <br />

which contradicts to lim n→ a n −. From above results, we obtain<br />

U lim n→<br />

sup a n<br />

in the case of U −.<br />

If U is finite, then given 0, there exists a positive integer N such that as n ≥ N, we<br />

have<br />

U ≤ u n U .<br />

So, as n ≥ N, u n U which implies that, as n ≥ N, a n U . In addition, given<br />

′ 0, and m 0, there exists an integer n m,<br />

U − ′ a n<br />

by U ≤ u n supa k : k ≥ n if n ≥ N. From above results, we obtain<br />

U lim n→<br />

sup a n<br />

in the case of U is finite.<br />

(b)Similarly, if a n is bounded below, prove that<br />

V lim n→<br />

inf a n lim n→<br />

infa k : k ≥ n.<br />

Proof: Since the proof is similar to (a), we omit it.<br />

If U and V are finite, show that:<br />

(c) <strong>The</strong>re exists a subsequence of a n which converges to U and a subsequence which<br />

converges to V.<br />

Proof: SinceU lim sup n→ a n by (a), then<br />

(i) Given 0, there exists a positive integer N such that as n ≥ N, wehave<br />

a n U .<br />

(ii) Given 0, and m 0, there exists an integer Pm m,<br />

U − a Pm .<br />

Hence, a Pm is a convergent subsequence of a n with limit U.<br />

Similarly for the case of V.


(d) If U V, every subsequnce of a n converges to U.<br />

Proof: By(a)and(b),given 0, then there exists a positive integer N 1 such that as<br />

n ≥ N 1 ,wehave<br />

a n U <br />

and there exists a positive integer N 2 such that as n ≥ N 2 ,wehave<br />

U − a n .<br />

Hence, as n ≥ maxN 1 , N 2 ,wehave<br />

U − a n U .<br />

That is, a n is a convergent sequence with limit U. So, every subsequnce of a n <br />

converges to U.<br />

8.2 Given two real-valed sequence a n and b n bounded below. Prove hat<br />

(a) lim sup n→ a n b n ≤ lim sup n→ a n lim sup n→ b n .<br />

Proof: Note that a n and b n bounded below, we have lim sup n→ a n or is<br />

finite. <strong>And</strong> lim sup n→ b n or is finite. It is clear if one of these limit superior is ,<br />

sowemayassumethatbotharefinite.Leta lim sup n→ a n and b lim sup n→ b n . <strong>The</strong>n<br />

given 0, there exists a positive integer N such that as n ≥ N, wehave<br />

a n b n a b /2. *<br />

In addition, let c lim sup n→ a n b n ,wherec by (*). So, for the same 0, and<br />

given m N there exists a positive integer K such that as K ≥ N, wehave<br />

c − /2 a K b K . **<br />

By (*) and (**), we obtain that<br />

c − /2 a K b K a b /2<br />

which implies that<br />

c ≤ a b<br />

since is arbitrary. So,<br />

lim sup a n b n ≤ lim sup a n lim sup b n.<br />

n→ n→ n→<br />

Remark: (1) <strong>The</strong> equality may NOT hold. For example,<br />

a n −1 n and b n −1 n1 .<br />

(2) <strong>The</strong> reader should noted that the finitely many terms does NOT change the relation<br />

of order. <strong>The</strong> fact is based on process of proof.<br />

(b) lim sup n→ a n b n ≤ lim sup n→ a n lim sup n→ b n if a n 0, b n 0 for all n, and<br />

if both lim sup n→ a n and lim sup n→ b n are finite or both are infinite.<br />

Proof: Let lim sup n→ a n a and lim sup n→ b n b. It is clear that we may assume<br />

that a and b are finite. Given 0, there exists a positive integer N such that as n ≥ N,<br />

we have<br />

a n b n a b ab a b . *<br />

In addition, let c lim sup n→ a n b n ,wherec by (*). So, for the same 0, and<br />

given m N there exists a positive integer K such that as K ≥ N, wehave<br />

c − a K b K . **<br />

By (*) and (**), we obtain that<br />

c − a K b K a b a b


which implies that<br />

c ≤ a b<br />

since is arbitrary. So,<br />

lim sup a nb n ≤ lim sup a n lim sup b n .<br />

n→ n→ n→<br />

Remark: (1) <strong>The</strong> equality may NOT hold. For example,<br />

a n 1/n if n is odd and a n 1ifn is even.<br />

and<br />

b n 1ifn is odd and b n 1/n if n is even.<br />

(2) <strong>The</strong> reader should noted that the finitely many terms does NOT change the relation<br />

of order. <strong>The</strong> fact is based on the process of the proof.<br />

(3) <strong>The</strong> reader should be noted that if letting A n log a n and B n log b n , thenby(a)<br />

and log x is an increasing function on 0, , we have proved (b).<br />

8.3 Prove that <strong>The</strong>orem 8.3 and 8.4.<br />

(<strong>The</strong>orem 8.3) Leta n be a sequence of real numbers. <strong>The</strong>n we have:<br />

(a) lim inf n→ a n ≤ lim sup n→ a n .<br />

Proof: If lim sup n→ a n , then it is clear. We may assume that<br />

lim sup n→ a n . Hence, a n is bounded above. We consider two cases: (i)<br />

lim sup n→ a n a, wherea is finite and (ii) lim sup n→ a n −.<br />

For case (i), if lim inf n→ a n −, then there is nothing to prove it. We may assume<br />

that lim inf n→ a n a ′ ,wherea ′ is finite. By definition of limit superior and limit inferior,<br />

given 0, there exists a positive integer N such that as n ≥ N, wehave<br />

a ′ − /2 a n a /2<br />

which implies that a ′ ≤ a since is arbitrary.<br />

For case (ii), since lim sup n→ a n −, wehavea n is not bounded below. If<br />

lim inf n→ a n −, then there is nothing to prove it. We may assume that<br />

lim inf n→ a n a ′ ,wherea ′ is finite. By definition of limit inferior, given 0, there<br />

exists a positive integer N such that as n ≥ N, wehave<br />

a ′ − /2 a n<br />

which contradicts that a n is not bounded below.<br />

So, from above results, we have proved it.<br />

(b) <strong>The</strong> sequence converges if and only if, lim sup n→ a n and lim inf n→ a n are both<br />

finite and equal, in which case lim n→ a n lim inf n→ a n lim sup n→ a n .<br />

Proof: ()Given a n a convergent sequence with limit a. So, given 0, there<br />

exists a positive integer N such that as n ≥ N, wehave<br />

a − a n a .<br />

By definition of limit superior and limit inferior, a lim inf n→ a n lim sup n→ a n .<br />

()By definition of limit superior, given 0, there exists a positive integer N 1 such<br />

that as n ≥ N 1 ,wehave<br />

a n a <br />

and by definition of limit superior, given 0, there exists a positive integer N 2 such that<br />

as n ≥ N 2 ,wehave


a − a n .<br />

So, as n ≥ maxN 1 , N 2 ,wehave<br />

a − a n a .<br />

That is, lim n→ a n a.<br />

(c) <strong>The</strong> sequence diverges to if and only if, lim inf n→ a n lim sup n→ a n .<br />

Proof: ()Given a sequence a n with lim n→ a n . So, given M 0, there is a<br />

positive integer N such that as n ≥ N, wehave<br />

M ≤ a n . *<br />

It implies that a n is not bounded above. So, lim sup n→ a n . In order to show that<br />

lim inf n→ a n . We first note that a n is bounded below. Hence, lim inf n→ a n ≠−.<br />

So, it suffices to consider that lim inf n→ a n is not finite. (So, we have<br />

lim inf n→ a n . ). Assume that lim inf n→ a n a, wherea is finite. <strong>The</strong>n given 1,<br />

and an integer m, there exists a positive Km m such that<br />

a Km a 1<br />

which contradicts to (*) if we choose M a 1. So, lim inf n→ a n is not finite.<br />

(d) <strong>The</strong> sequence diverges to − if and only if, lim inf n→ a n lim sup n→ a n −.<br />

Proof: Note that, lim sup n→ −a n − lim inf n→ a n . So, by (c), we have proved it.<br />

(<strong>The</strong>orem 8.4)Assume that a n ≤ b n for each n 1,2,.... <strong>The</strong>n we have:<br />

lim n→<br />

inf a n ≤ lim n→<br />

inf b n and lim n→<br />

sup a n ≤ lim n→<br />

sup b n .<br />

Proof: If lim inf n→ b n , there is nothing to prove it. So, we may assume that<br />

lim inf n→ b n . That is, lim inf n→ b n − or b, whereb is finite.<br />

For the case, lim inf n→ b n −, it means that the sequence a n is not bounded<br />

below. So, b n is also not bounded below. Hence, we also have lim inf n→ a n −.<br />

For the case, lim inf n→ b n b, whereb is finite. We consider three cases as follows.<br />

(i) if lim inf n→ a n −, then there is nothing to prove it.<br />

(ii) if lim inf n→ a n a, wherea is finite. Given 0, then there exists a positive<br />

integer N such that as n ≥ N<br />

a − /2 a n ≤ b n b /2<br />

which implies that a ≤ b since is arbitrary.<br />

(iii) if lim inf n→ a n , then by <strong>The</strong>orem 8.3 (a) and (c), we know that<br />

lim n→ a n which implies that lim n→ b n . Also, by <strong>The</strong>orem 8.3 (c), wehave<br />

lim inf n→ b n which is absurb.<br />

So, by above results, we have proved that lim inf n→ a n ≤ lim inf n→ b n .<br />

Similarly, we have lim sup n→ a n ≤ lim sup n→ b n .<br />

8.4 If each a n 0, prove that<br />

lim n→<br />

inf a n1<br />

a n<br />

≤ lim n→<br />

infa n 1/n ≤ lim n→<br />

supa n 1/n ≤ lim n→<br />

sup a n1<br />

a n<br />

.<br />

Proof: By<strong>The</strong>orem 8.3 (a), it suffices to show that<br />

lim n→<br />

inf a n1<br />

a n<br />

We first prove<br />

≤ lim n→<br />

infa n 1/n and lim n→<br />

supa n 1/n ≤ lim n→<br />

sup a n1<br />

a n<br />

.<br />

lim n→<br />

supa n 1/n ≤ lim n→<br />

sup a n1<br />

a n<br />

.


If lim sup n→<br />

a n1<br />

a n<br />

, then it is clear. In addition, since a n1<br />

a n<br />

is positive,<br />

a<br />

≠−. So, we may assume that lim sup n1<br />

n→ a n<br />

a, wherea is finite.<br />

Given 0, then there exists a positive integer N such that as n ≥ N, wehave<br />

a n1<br />

a n<br />

a <br />

lim sup n→<br />

a n1<br />

a n<br />

which implies that<br />

So,<br />

which implies that<br />

a Nk a N a k ,wherek 1,2,....<br />

a Nk 1<br />

Nk<br />

a N 1<br />

Nk a <br />

k<br />

Nk<br />

lim supa Nk 1<br />

Nk<br />

k→<br />

≤ lim supa N 1<br />

k<br />

Nk a <br />

Nk<br />

k→<br />

a .<br />

So,<br />

lim supa Nk 1<br />

Nk<br />

≤ a<br />

k→<br />

since is arbitrary. Note that the finitely many terms do NOT change the value of limit<br />

superiror of a given sequence. So, we finally have<br />

lim n→<br />

supa n 1/n ≤ a lim n→<br />

sup a n1<br />

a n<br />

.<br />

Similarly for<br />

lim n→<br />

inf a n1<br />

a n<br />

≤ lim n→<br />

infa n 1/n .<br />

Remark: <strong>The</strong>se ineqaulities is much important; we suggest that the reader keep it mind.<br />

At the same time, these inequalities tells us that the root test is more powerful than the<br />

ratio test. We give an example to say this point. Given a series<br />

1<br />

2 1 3 1 2 1 2 3 ... 1 2 2 n 3 1 n ...<br />

where<br />

n n<br />

a 2n−1 1 ,anda2n 1 , n 1, 2, . . .<br />

2 3<br />

with<br />

and<br />

lim n→<br />

inf a n1<br />

a n<br />

lim n→<br />

supa n 1/n 1<br />

2<br />

1<br />

0, lim n→<br />

sup a n1<br />

a n<br />

.<br />

8.5 Let a n n n /n!. Show that lim n→ a n1 /a n e and use Exercise 8.4 to deduce that<br />

lim n<br />

n→<br />

e.<br />

n! 1/n<br />

Proof: Since<br />

a n1<br />

a n<br />

n 1n1 n!<br />

n 1!n n 1 1 n<br />

n → e,<br />

by Exercise 8.4, wehave<br />

lim n→<br />

a n 1/n lim n<br />

n→<br />

e.<br />

n! 1/n


Remark: <strong>The</strong>re are many methods to show this. We do NOT give the detailed proof.<br />

But there are hints.<br />

(1) Taking log on n!<br />

n n 1/n , and thus consider<br />

1<br />

n log 1 n ... log n n → <br />

0<br />

1<br />

log xdx −1.<br />

(2) Stirling’s Formula:<br />

n! n n e −n 2n e <br />

12n ,where ∈ 0, 1.<br />

Note: In general, we have<br />

lim<br />

x→<br />

Γx 1<br />

x x e −x 2x 1,<br />

where Γx is the Gamma Function. <strong>The</strong> reader can see the book, Principles of<br />

Mathematical Analysis by Walter Rudin, pp 192-195.<br />

(3) Note that 1 1 x x ↗ e and 1 1 x x1 ↘ e on 0, . So,<br />

1 1 n n1<br />

n e 1 1 n<br />

which implies that<br />

en n e −n n! en n1 e −n .<br />

(4) Using O-Stolz’s <strong>The</strong>orem: Let lim n→ y n and y n ↗.If<br />

lim<br />

x n1 − x n<br />

n→ y n1 − y n<br />

a, wherea is finite or ,<br />

then<br />

lim<br />

x n<br />

n→ y n<br />

a.<br />

Let x n log 1 n ... log n n and y n n.<br />

Note: For the proof of O-Stolz’s <strong>The</strong>orem, the reader can see the book, An<br />

Introduction to Mathematical Analysis by Loo-Keng Hua, pp 195. (Chinese Version)<br />

(5) Note that, if a n is a positive sequence with lim n→ a n a, then<br />

a 1 a n 1/n → a as n → .<br />

Taking a n 1 1 n n , then<br />

a 1 a n 1/n n n<br />

1 <br />

n!<br />

1 n → e.<br />

Note: For the proof, it is easy from the Exercise 8.6. We give it a proof as follows. Say<br />

lim n→ a n a. Ifa 0, then by A. P. ≥ G. P. , we have<br />

a 1 a n 1/n ≤ a 1 ...a n<br />

n → 0byExercise 8.6.<br />

So, we consider a ≠ 0 as follows. Note that log a n → log a. So, by Exercise 8.6,<br />

log a 1 ... log a n<br />

n → log a<br />

which implies that a 1 a n 1/n → a.<br />

8.6 Let a n be real-valued sequence and let n a 1 ...a n /n. Show that<br />

lim n→<br />

inf a n ≤ lim n→<br />

inf n ≤ lim n→<br />

sup n ≤ lim n→<br />

sup a n .<br />

Proof: By<strong>The</strong>orem 8.3 (a), it suffices to show that<br />

lim n→<br />

inf a n ≤ lim n→<br />

inf n and lim n→<br />

sup n ≤ lim n→<br />

sup a n .<br />

1/n


We first prove<br />

lim n→<br />

sup n ≤ lim n→<br />

sup a n .<br />

If lim sup n→ a n , there is nothing to prove it. We may assume that<br />

lim sup n→ a n − or a, wherea is finite.<br />

For the case, lim sup n→ a n −, then by <strong>The</strong>orem 8.3 (d), wehave<br />

lim n→<br />

a n −.<br />

So, given M 0, there exists a positive integer N such that as n ≥ N, wehave<br />

a n ≤−M. *<br />

Let n N, wehave<br />

n a 1 ...a N ..a n<br />

n<br />

a 1 ...a N<br />

n<br />

a N1 ...a n<br />

n<br />

≤ a 1 ...a N<br />

n n − N<br />

n −M<br />

which implies that<br />

lim n→<br />

sup n ≤−M.<br />

Since M is arbitrary, we finally have<br />

lim n→<br />

sup n −.<br />

For the case, lim sup n→ a n a, wherea is finite. Given 0, there exists a positive<br />

integer N such that as n ≥ N, wehave<br />

a n a .<br />

Let n N, wehave<br />

n a 1 ...a N ..a n<br />

n<br />

a 1 ...a N<br />

n<br />

a N1 ...a n<br />

n<br />

≤ a 1 ...a N<br />

n n − N<br />

n<br />

a <br />

which implies that<br />

lim n→<br />

sup n ≤ a <br />

which implies that<br />

lim n→<br />

sup n ≤ a<br />

since is arbitrary.<br />

Hence, from above results, we have proved that lim sup n→ n ≤ lim sup n→ a n .<br />

Similarly for lim inf n→ a n ≤ lim inf n→ n .<br />

Remark: We suggest that the reader keep it in mind since it is the fundamental and<br />

useful in the theory of Fourier Series.<br />

8.7 Find lim sup n→ a n and lim inf n→ a n if a n is given by<br />

(a) cosn<br />

Proof: Note that, a b : a, b ∈ Z is dense in R. Bycosn cosn 2k, we<br />

know that<br />

lim n→<br />

sup cos n 1 and lim n→<br />

inf cos n −1.


Remark: <strong>The</strong> reader may give it a try to show that<br />

lim n→<br />

sup sin n 1 and lim n→<br />

inf sin n −1.<br />

(b) 1 1 n cosn<br />

Proof: Note that<br />

So, it is clear that<br />

lim n→<br />

sup 1 1 n<br />

1 1 n cosn <br />

1ifn 2k<br />

−1 ifn 2k − 1 .<br />

cosn 1 and lim n→<br />

inf 1 1 n cosn −1.<br />

(c) n sin n 3<br />

Proof: Note that as n 1 6k, n sin n 1 6k sin ,andasn 4 6k,<br />

3 3<br />

n −4 6k sin . So, it is clear that<br />

3<br />

lim n→<br />

sup n sin n and lim<br />

3<br />

inf n sin n n→ 3<br />

−.<br />

(d) sin n cos n 2 2<br />

Proof: Note that sin n cos n 1 sin n 0, we have<br />

2 2 2<br />

lim n→<br />

sup sin n 2 cos n 2<br />

lim inf sin n n→ 2 cos n 2 0.<br />

(e) −1 n n/1 n n<br />

Proof: Note that<br />

we know that<br />

(f) n 3<br />

− n 3 <br />

Proof: Note that<br />

So, it is clear that<br />

lim n→<br />

−1 n n/1 n n 0,<br />

lim n→<br />

sup−1 n n/1 n n lim n→<br />

inf−1 n n/1 n n 0.<br />

n<br />

3 − n 3<br />

<br />

1<br />

3<br />

2<br />

3<br />

if n 3k 1<br />

if n 3k 2<br />

0ifn 3k<br />

,wherek 0, 1, 2, . . . .<br />

lim n→<br />

sup n 3 − n <br />

3<br />

2 and lim<br />

3 inf n n→ 3 − n 3<br />

Note.In(f),x denoted the largest integer ≤ x.<br />

n<br />

8.8 Let a n 2 n − ∑ k1<br />

1/ k . Prove that the sequence a n converges to a limit p<br />

in the interval 1 p 2.<br />

n<br />

n<br />

Proof: Consider ∑ k1<br />

1/ k : S n and x<br />

−1/2<br />

dx : T n , then<br />

1<br />

lim n→<br />

d n exists, where d n S n − T n<br />

by Integral Test. We denote the limit by d, then<br />

0.


0 ≤ d 1 *<br />

by <strong>The</strong>orem 8.23 (i). Note that d n − fn is a positive increasing sequence, so we have<br />

d 0. **<br />

Since<br />

T n 2 n − 2<br />

which implies that<br />

lim n→<br />

n<br />

2 n − ∑ 1/ k lim n→<br />

a n 2 − d p.<br />

k1<br />

By (*) and (**), we have proved that 1 p 1.<br />

Remark: (1) <strong>The</strong> use of Integral Test is very useful since we can know the behavior<br />

of a given series by integral. However, in many cases, the integrand may be so complicated<br />

that it is not easy to calculate. For example: Prove that the convergence of<br />

<br />

∑<br />

n2<br />

1<br />

nlog n p ,wherep 1.<br />

Of course, it can be checked by Integral Test. But there is the <strong>The</strong>orem called Cauchy<br />

Condensation <strong>The</strong>orem much powerful than Integral Test in this sense. In addition, the<br />

reader can think it twice that in fact, Cauchy condensation <strong>The</strong>orem is equivalent to<br />

Integral Test.<br />

(Cauchy Condensation <strong>The</strong>orem)Let a n be a positive decreasing sequence. <strong>The</strong>n<br />

<br />

∑<br />

n1<br />

a n converges if, and only if, ∑<br />

k0<br />

<br />

2 k a 2<br />

k converges.<br />

Note: (1) <strong>The</strong> proof is not hard; the reader can see the book, Principles of<br />

Mathematical Analysis by Walter Rudin, pp 61-63.<br />

(2) <strong>The</strong>re is an extension of Cauchy Condensation <strong>The</strong>orem (Oskar Schlomilch):<br />

Suppose that a k be a positive and decreasing sequence and m k ⊆ N is a sequence. If<br />

there exists a c 0 such that<br />

0 m k2 − m k1 ≤ cm k1 − m k for all k,<br />

then<br />

<br />

∑<br />

k1<br />

<br />

a k converges if, and only if, ∑m k1 − m k a mk .<br />

k0<br />

Note: <strong>The</strong> proof is similar with Cauchy Condensation <strong>The</strong>orem, soweomitit.<br />

(2) <strong>The</strong>re is a similar <strong>The</strong>orem, we write it as a reference. If t ≥ a, ft is a<br />

non-negative increasing function, then as x ≥ a, wehave<br />

x<br />

∑ fn − ftdt<br />

a≤n≤x<br />

a<br />

≤ fx.<br />

Proof: <strong>The</strong> proof is easy by drawing a graph. So, we omit it.<br />

P.S.: <strong>The</strong> theorem is useful when we deal with some sums. For example,<br />

ft log t.<br />

<strong>The</strong>n


∑ log n − x log x x − 1 ≤ log x.<br />

1≤n≤x<br />

In particular, as x ∈ N, we thus have<br />

n log n − n 1 − log n ≤ log n! ≤ n log n − n 1 log n<br />

which implies that<br />

n n−1 e −n1 ≤ n! ≤ n n1 e −n1 .<br />

In each of Exercise 8.9. through 8.14, show that the real-valed sequence a n is<br />

convergent. <strong>The</strong> given conditions are assumed to hold for all n ≥ 1. In Exercise 8.10<br />

through 8.14, show that a n has the limit L indicated.<br />

8.9 |a n| 2, |a n2 − a n1 | ≤ 1 8 |a 2<br />

n1 − a n2 |.<br />

Proof: Since<br />

|a n2 − a n1 | ≤ 1 8 |a 2<br />

n1 − a n2 |<br />

1 8 |a n1 − a n||a n1 a n|<br />

we know that<br />

So,<br />

≤ 1 2 |a n1 − a n| since |a n| 2<br />

|a n1 − a n| ≤ 1<br />

2<br />

n−1<br />

|a2 − a 1 | ≤ 1<br />

2<br />

n−3<br />

.<br />

k<br />

|a nk − a n| ≤ ∑|a nj − a nj−1 |<br />

j1<br />

k<br />

≤ ∑<br />

j1<br />

1<br />

2<br />

nj−4<br />

≤ 1<br />

2<br />

n−2<br />

→ as n → .<br />

Hence, a n is a Cauchy sequence. So, a n is a convergent sequence.<br />

Remark: (1) If |a n1 − a n| ≤ b n for all n ∈ N, and∑ b n converges, then ∑ a n<br />

converges.<br />

Proof: Since the proof is similar with the Exercise, we omit it.<br />

(2) In (1), the condition ∑ b n converges CANNOT omit. For example,<br />

n 1<br />

(i) Let a n sin ∑ k1<br />

Or<br />

k<br />

(ii) a n is defined as follows:<br />

a 1 1, a 2 1/2, a 3 0, a 4 1/4, a 5 1/2, a 6 3/4, a 7 1, and so on.<br />

8.10 a 1 ≥ 0, a 2 ≥ 0, a n2 a n a n1 1/2 , L a 1 a 22 1/3 .<br />

Proof: If one of a 1 or a 2 is 0, then a n 0 for all n ≥ 2. So, we may assume that<br />

a 1 ≠ 0anda 2 ≠ 0. So, we have a n ≠ 0 for all n. Letb n a n1<br />

a n<br />

, then<br />

b n1 1/ b n for all n<br />

which implies that


Consider<br />

which implies that<br />

which implies that<br />

b n1 b 1 −1<br />

2<br />

n<br />

→ 1asn → .<br />

n1 j2 b j n<br />

j1 b j −1/2<br />

a 1 1/2 a 2<br />

−2/3<br />

an1 1<br />

b n1<br />

2/3<br />

lim n→<br />

a n1 a 1 a 22 1/3 .<br />

Remark: <strong>The</strong>re is another proof. We write it as a reference.<br />

Proof: If one of a 1 or a 2 is 0, then a n 0 for all n ≥ 2. So, we may assume that<br />

a 1 ≠ 0anda 2 ≠ 0. So, we have a n ≠ 0 for all n. Leta 2 ≥ a 1 .Sincea n2 a n a n1 1/2 ,<br />

then inductively, we have<br />

a 1 ≤ a 3 ≤...≤ a 2n−1 ≤...≤ a 2n ≤...≤ a 4 ≤ a 2 .<br />

So, both of a 2n and a 2n−1 converge. Say<br />

lim n→<br />

a 2n x and lim n→<br />

a 2n−1 y.<br />

Note that a 1 ≠ 0anda 2 ≠ 0, so x ≠ 0, and y ≠ 0. In addition, x y by<br />

a n2 a n a n1 1/2 . Hence, a n converges to x.<br />

By a n2 a n a n1 1/2 , and thus<br />

n<br />

j1 a2 j2 n<br />

j1 a j a j1 a 1 a 22 a n1 n−2 j1 a2<br />

j2<br />

which implies that<br />

which implies that<br />

a n1 a2 n2 a 1 a2<br />

2<br />

lim n→<br />

a n x a 1 a 22 1/3 .<br />

8.11 a 1 2, a 2 8, a 2n1 1 2 a 2n a 2n−1 , a 2n2 a 2na 2n−1<br />

a 2n1<br />

, L 4.<br />

Proof: First, we note that<br />

a 2n1 a 2n a 2n−1<br />

2<br />

≥ a 2n a 2n−1 by A. P. ≥ G. P. *<br />

for n ∈ N. So, by a 2n2 a 2na 2n−1<br />

a 2n1<br />

and (*),<br />

a 2n2 a 2na 2n−1<br />

a 2n1<br />

≤ a 2n a 2n−1 ≤ a 2n1 for all n ∈ N.<br />

Hence, by Mathematical Induction, itiseasytoshowthat<br />

a 4 ≤ a 6 ≤...≤ a 2n2 ≤...≤ a 2n1 ≤...≤ a 5 ≤ a 3<br />

for all n ∈ N. It implies that both of a 2n and a 2n−1 converge, say<br />

lim n→<br />

a 2n x and lim n→<br />

a 2n−1 y.<br />

With help of a 2n1 1 a 2 2n a 2n−1 , we know that x y. In addition, by a 2n2 a 2na 2n−1<br />

a 2n1<br />

,<br />

a 1 2, and a 2 8, we know that x 4.<br />

8.12 a 1 −3 2 n1 2 a n3 , L 1. Modify a 1 to make L −2.<br />

Proof: ByMathematical Induction, itiseasytoshowthat<br />

− 2 ≤ a n ≤ 1 for all n. *


So,<br />

3a n1 − a n a n3 − 3a n 2 ≥ 0<br />

by (*) and fx x 3 − 3x 2 x − 1 2 x 2 ≥ 0on−2, 1. Hence, a n is an<br />

increasing sequence with a upper bound 1. So, a n is a convergent sequence with limit L.<br />

So,by3a n1 2 a n3 ,<br />

L 3 − 3L 2 0<br />

which implies that<br />

L 1or − 2.<br />

So, L 1sinca n ↗ and a 1 −3/2.<br />

In order to make L −2, it suffices to let a 1 −2, then a n −2 for all n.<br />

8.13 a 1 3, a n1 31an<br />

3a n<br />

, L 3 .<br />

Proof: By Mathematical Induction, itiseasytoshowthat<br />

a n ≥ 3 for all n. *<br />

So,<br />

a n1 − a n 3 − a n 2<br />

≤ 0<br />

3 a n<br />

which implies that a n is a decreasing sequence. So, a n is a convergent sequence with<br />

limit L by (*). Hence,<br />

31 L<br />

L <br />

3 L<br />

which implies that<br />

L 3 .<br />

So, L 3 since a n ≥ 3 for all n.<br />

and<br />

8.14 a n b n1<br />

b n<br />

,whereb 1 b 2 1, b n2 b n b n1 , L 1 5<br />

Hint. Show that b n2 b n − b2 n1 −1 n1 and deduce that |a n − a n1 | n −2 ,ifn 4.<br />

Proof: ByMathematical Induction, itiseasytoshowthat<br />

Thus, (Note that b n ≠ 0 for all n)<br />

|a n1 − a n| b n2<br />

b n1<br />

b n2 b n − b2 n1 −1 n1 for all n<br />

b n ≥ n if n 4<br />

2<br />

.<br />

− b n1<br />

−1n1 ≤ 1<br />

b n b n b n1 nn 1 1 if n 4.<br />

n2 So, a n is a Cauchy sequence. In other words, a n is a convergent sequence, say<br />

lim n→ b n L. <strong>The</strong>n by b n2 b n b n1 ,wehave<br />

b n2<br />

b n<br />

1<br />

b n1 b n1<br />

which implies that (Note that 0 ≠L ≥ 1sincea n ≥ 1 for all n)<br />

L <br />

L 1 1<br />

which implies that<br />

L 1 5<br />

2<br />

.


So, L 1 5 since L ≥ 1.<br />

2<br />

Remark: (1) <strong>The</strong> sequence b n is the famous sequence named Fabonacci sequence.<br />

<strong>The</strong>re are many researches around it. Also, it is related with so called Golden Section,<br />

5 −1<br />

0.618....<br />

2<br />

(2) <strong>The</strong> reader can see the book, An Introduction To <strong>The</strong> <strong>The</strong>ory Of <strong>Number</strong>s by G.<br />

H. Hardy and E. M. Wright, Chapter X. <strong>The</strong>n it is clear by continued fractions.<br />

(3) <strong>The</strong>re is another proof. We write it as a reference.<br />

Proof: (STUDY) Sinceb n2 b n b n1 , we may think<br />

x n2 x n x n1 ,<br />

and thus consider x 2 x 1. Say and are roots of x 2 x 1, with . <strong>The</strong>nlet<br />

F n n − n<br />

− ,<br />

we have<br />

F n b n .<br />

So,itiseasytoshowthatL 1 5 . We omit the details.<br />

2<br />

Note: <strong>The</strong> reader should be noted that there are many methods to find the formula of<br />

Fabonacci sequence like F n . For example, using the concept of Eigenvalues if we can<br />

find a suitable matrix.<br />

Series<br />

8.15 Test for convergence (p and q denote fixed rela numbers).<br />

(a) ∑ <br />

n1<br />

n 3 e −n<br />

Proof: ByRoot Test, wehave<br />

So, the series converges.<br />

(b) ∑ <br />

n2<br />

log n p<br />

lim n→<br />

sup<br />

n3 1/n<br />

e 1/e 1.<br />

n<br />

Proof: We consider 2 cases: (i) p ≥ 0, and (ii) p 0.<br />

For case (i), the series diverges since log n p does not converge to zero.<br />

For case (ii), the series diverges by Cauchy Condensation <strong>The</strong>orem (or Integral<br />

Test.)<br />

(c) ∑ n1<br />

p n n p (p 0)<br />

Proof: ByRoot Test, wehave<br />

lim n→<br />

sup<br />

pn 1/n<br />

n p.<br />

p<br />

So, as p 1, the series diverges, and as p 1, the series converges. For p 1, it is clear<br />

that the series ∑ n diverges. Hence,<br />

and<br />

<br />

∑<br />

n1<br />

p n n p converges if p ∈ 0, 1


∑ p n n p diverges if p ∈ 1, .<br />

n1<br />

1<br />

(d) ∑ n2 n p −n<br />

(0 q p)<br />

q<br />

1<br />

Proof: Note that<br />

n p −n<br />

1 1<br />

q n<br />

. We consider 2 cases: (i) p 1 and (ii) p ≤ 1.<br />

p 1−n q−p<br />

For case (i), by Limit Comparison Test with 1 n<br />

, p<br />

1<br />

n<br />

lim<br />

p −n q<br />

n→ 1<br />

1,<br />

n p<br />

the series converges.<br />

For case (ii), by Limit Comparison Test with 1 n<br />

, p<br />

the series diverges.<br />

(e) ∑ <br />

n1<br />

n −1−1/n<br />

1<br />

n<br />

lim<br />

p −n q<br />

n→ 1<br />

n p 1,<br />

Proof: Sincen −1−1/n ≥ n −1 for all n, the series diverges.<br />

1<br />

(f) ∑ n1 p n −q<br />

(0 q p)<br />

n<br />

1<br />

Proof: Note that<br />

p n −q<br />

1 1<br />

n p n 1−<br />

q n . We consider 2 cases: (i) p 1 and (ii) p ≤ 1.<br />

p<br />

For case (i), by Limit Comparison Test with 1 p<br />

, n<br />

1<br />

p<br />

lim<br />

n −q n<br />

n→<br />

1,<br />

1<br />

p n<br />

the series converges.<br />

For case (ii), by Limit Comparison Test with 1 p n ,<br />

the series diverges.<br />

1<br />

(g) ∑ n1 nlog11/n<br />

1<br />

p<br />

lim<br />

n −q n<br />

n→<br />

1,<br />

1<br />

p n<br />

Proof: Since<br />

lim 1<br />

n→ n log1 1/n 1,<br />

we know that the series diverges.<br />

1<br />

(h) ∑ n2 logn log n<br />

Proof: Since the identity a logb b loga ,wehave<br />

log n logn nlog logn<br />

So, the series converges.<br />

1<br />

(i) ∑ n3 nlognlog logn p<br />

≥ n 2 as n ≥ n 0 .<br />

Proof: We consider 3 cases: (i) p ≤ 0, (ii) 0 p 1 and (iii) p 1.


For case (i), since<br />

1<br />

n log nlog log n p ≥ 1 for n ≥ 3,<br />

n log n<br />

1<br />

we know that the series diverges by the divergence of ∑ .<br />

n3 n logn<br />

For case (ii), we consider (choose n 0 large enough)<br />

<br />

∑<br />

jn 0<br />

2 j<br />

2 j log 2 j log log 2 j p 1<br />

log 2 ∑ jn 0<br />

<br />

≥ ∑<br />

jn 0<br />

<br />

1<br />

jlog j p ,<br />

1<br />

jlog j log 2 p<br />

then, by Cauchy Condensation <strong>The</strong>orem, the series diverges since ∑ jn0<br />

by using Cauchy Condensation <strong>The</strong>orem again.<br />

For case (iii), we consider (choose n 0 large enough)<br />

<br />

∑<br />

jn 0<br />

2 j<br />

2 j log 2 j log log 2 j p 1<br />

log 2 ∑ jn 0<br />

<br />

≤ 2 ∑<br />

jn 0<br />

<br />

≤ 4 ∑<br />

jn 0<br />

<br />

1<br />

jlog j log 2 p<br />

1<br />

jlog j log 2 p<br />

1<br />

jlog j p ,<br />

1<br />

jlogj p<br />

1<br />

jlogj p<br />

diverges<br />

then, by Cauchy Condensation <strong>The</strong>orem, the series converges since ∑ jn0<br />

converges by using Cauchy Condensation <strong>The</strong>orem again.<br />

Remark: <strong>The</strong>re is another proof by Integral Test. We write it as a reference.<br />

1<br />

Proof: It is easy to check that fx is continous, positive, and<br />

xlogxlog logx p<br />

decreasing to zero on a, where a 0 for each fixed p. Consider<br />

<br />

<br />

<br />

dx<br />

dy<br />

a x log xlog log x p <br />

log loga y p<br />

which implies that the series converges if p 1 and diverges if p ≤ 1byIntegral Test.<br />

1<br />

(j) ∑ n3 log logn<br />

log logn<br />

log logn<br />

1<br />

Proof: Leta n <br />

log logn for n ≥ 3andbn 1/n, then<br />

log logn<br />

a n<br />

n 1<br />

b n log log n<br />

e −ylogy−ey <br />

→.<br />

So, by Limit Comparison Test, the series diverges.<br />

<br />

(k) ∑ n1<br />

1 n 2 − n<br />

Proof: Note that<br />

1 n 2 − n 1<br />

1 n 2 n ≥ 1 for all n.<br />

1 2 n<br />

So, the series diverges.


(l) ∑ n2<br />

n p 1<br />

n−1 − 1 n<br />

Proof: Note that<br />

n p 1<br />

n − 1 −<br />

1 n<br />

1<br />

n 3 2 −p<br />

n<br />

n − 1<br />

1<br />

1 <br />

n−1<br />

n<br />

So, as p 1/2, the series converges and as p ≥ 1/2, the series diverges by Limit<br />

Comparison Test.<br />

<br />

(m) ∑ n1<br />

n 1/n − 1 n<br />

Proof: With help of Root Test,<br />

lim n→<br />

sup n 1/n − 1 n 1/n 0 1,<br />

the series converges.<br />

(n) ∑ n1<br />

n p n 1 − 2 n n − 1<br />

Proof: Note that<br />

n p n 1 − 2 n n − 1<br />

1<br />

n 3 2<br />

n 3 2 −p n n 1 n n − 1 n − 1 n 1<br />

.<br />

So, as p 1/2, the series converges and as p ≥ 1/2, the series diverges by Limit<br />

Comparison Test.<br />

8.16 Let S n 1 , n 2 ,... denote the collection of those positive integers that do not<br />

involve the digit 0 is their decimal representation. (For example, 7 ∈ S but 101 ∉ S.)<br />

Show that ∑ k1<br />

1/n k converges and has a sum less than 90.<br />

Proof: DefineS j the j − digit number ⊆ S. <strong>The</strong>n#S j 9 j and S <br />

j1 S j . Note<br />

that<br />

∑ 1/n k 9 j<br />

10 . j−1 k∈S j<br />

So,<br />

<br />

∑<br />

k1<br />

<br />

1/n k ≤ ∑<br />

<br />

j1<br />

9 j<br />

10 j−1 90.<br />

In addition, it is easy to know that ∑ k1<br />

1/n k ≠ 90. Hence, we have proved that ∑ k1<br />

1/n k<br />

converges and has a sum less than 90.<br />

8.17 Given integers a 1 , a 2 ,...such that 1 ≤ a n ≤ n − 1, n 2, 3, . . . Show that the<br />

<br />

sum of the series ∑ n1<br />

a n /n! is rational if and only if there exists an integer N such that<br />

a n n − 1 for all n ≥ N. Hint: For sufficency, show that ∑ <br />

n2<br />

n − 1/n! is a telescoping<br />

series with sum 1.<br />

Proof: ()Assume that there exists an integer N such that a n n − 1 for all n ≥ N.<br />

<strong>The</strong>n<br />

.


()Assume that ∑ <br />

n1<br />

<br />

That is, p! ∑ np1<br />

a n<br />

p! ∑<br />

np1<br />

<br />

∑<br />

n1<br />

an<br />

n!<br />

N−1<br />

∑<br />

n1<br />

N−1<br />

∑<br />

n1<br />

N−1<br />

∑<br />

n1<br />

<br />

an<br />

n! ∑<br />

nN<br />

an<br />

n!<br />

<br />

an<br />

n! ∑ n − 1<br />

n!<br />

nN<br />

<br />

an<br />

n! ∑<br />

nN<br />

1<br />

n − 1! − 1 n!<br />

N−1<br />

∑<br />

an<br />

n! 1<br />

N − 1! ∈ Q.<br />

n1<br />

a n /n! is rational, say q p ,whereg. c. d. p, q 1. <strong>The</strong>n<br />

n!<br />

∈ Z. Note that<br />

<br />

a n<br />

n!<br />

<br />

≤ p! ∑<br />

np1<br />

<br />

p! ∑<br />

n1<br />

n − 1<br />

n!<br />

an<br />

n!<br />

∈ Z.<br />

p!<br />

p! 1since1 ≤ a n ≤ n − 1.<br />

So, a n n − 1 for all n ≥ p 1. That is, there exists an integer N such that a n n − 1for<br />

all n ≥ N.<br />

Remark: From this, we have proved that e is irrational. <strong>The</strong> reader should be noted that<br />

we can use <strong>The</strong>orem 8.16 to show that e is irrational by considering e −1 . Since it is easy,<br />

we omit the proof.<br />

8.18 Let p and q be fixed integers, p ≥ q ≥ 1, and let<br />

pn<br />

x n ∑<br />

kqn1<br />

n<br />

1<br />

k<br />

, s n ∑<br />

k1<br />

−1 k1<br />

k<br />

(a) Use formula (8) to prove that lim n→ x n logp/q.<br />

Proof: Since<br />

we know that<br />

n<br />

∑<br />

k1<br />

1<br />

k<br />

log n r O 1 n ,<br />

pn<br />

x n ∑<br />

k1<br />

qn<br />

1 − k<br />

∑<br />

k1<br />

logp/q O 1 n<br />

which implies that lim n→ x n logp/q.<br />

(b) When q 1, p 2, show that s 2n x n and deduce that<br />

<br />

∑<br />

n1<br />

1<br />

k<br />

−1<br />

n1<br />

n log 2.<br />

Proof: We prove it by Mathematical Induction as follows. As n 1, it holds<br />

trivially. Assume that n m holds, i.e.,<br />

.


2m<br />

s 2m ∑<br />

k1<br />

consider n m 1 as follows.<br />

x m1 <br />

2m1<br />

∑<br />

km11<br />

−1<br />

k1<br />

k<br />

1<br />

k<br />

2m<br />

∑<br />

km1<br />

1<br />

k<br />

x m<br />

x m − 1<br />

m 1 1<br />

2m 1 1<br />

2m 2<br />

s 2m 1<br />

2m 1 − 1<br />

2m 2<br />

s 2m1 .<br />

So, by Mathematical Induction, we have proved that s 2n x n for all n.<br />

By s 2n x n for all n, wehave<br />

<br />

lim n→<br />

s 2n ∑<br />

k1<br />

−1<br />

k1<br />

k<br />

log 2 lim n→<br />

x n .<br />

(c) rearrange the series in (b), writing alternately p positive terms followed by q<br />

negative terms and use (a) to show that this rearrangement has sum<br />

log 2 1 2 logp/q.<br />

∑ <br />

k1<br />

<strong>The</strong>n<br />

Proof: We prove it by using <strong>The</strong>orem 8.13. So, we can consider the new series<br />

a k as follows:<br />

a k 1<br />

2k − 1p 1 ... 1<br />

2kp − 1<br />

n<br />

S n ∑ a k<br />

k1<br />

2np<br />

∑<br />

k1<br />

np<br />

1k − ∑<br />

k1<br />

nq<br />

1<br />

2k − ∑<br />

k1<br />

1<br />

2k<br />

− 1<br />

2k − 1q ... 1<br />

2kq<br />

log 2np O 1 n − 1 2 log np − 2 O 1 n − 1 2 log nq − 2 O 1 n<br />

log 2np − log n pq O<br />

1 n<br />

log 2 p q O 1 n .<br />

So,<br />

lim n→<br />

S n log 2 1 2 logp/q<br />

by <strong>The</strong>orem 8.13.<br />

Remark: <strong>The</strong>re is a reference around rearrangement of series. <strong>The</strong> reader can see the<br />

book, Infinite Series by Chao Wen-Min, pp 216-220. (Chinese Version)<br />

(d) Find the sum of ∑ <br />

n1<br />

−1 n1 1/3n − 2 − 1/3n − 1.<br />

Proof: Write


n<br />

S n ∑−1 k1 1<br />

3k − 2 − 1<br />

3k − 1<br />

k1<br />

n<br />

∑−1 k 1<br />

k1<br />

n<br />

−∑<br />

k1<br />

−<br />

−<br />

−<br />

3n<br />

∑<br />

k1<br />

n<br />

3k − 1 ∑<br />

k1<br />

−1 3k−1 1<br />

−1 k1 1<br />

3k − 2<br />

n<br />

3k − 1 − ∑−1 3k−2 1<br />

3k − 2<br />

k1<br />

n<br />

n<br />

∑−1 3k−1 1<br />

3k − 1 ∑−1 3k−2 1<br />

3k − 2<br />

k1<br />

k1<br />

3n<br />

∑<br />

k1<br />

3n<br />

∑<br />

k1<br />

−1 k<br />

k<br />

−1 k<br />

k<br />

−1 k1<br />

k<br />

n<br />

− ∑<br />

k1<br />

− 1 3 ∑ k1<br />

n<br />

−1 3k<br />

3k<br />

n<br />

−1 k<br />

k<br />

−1 k1<br />

k<br />

− 1 3 ∑ k1<br />

→ 2 log 2.<br />

3<br />

So, the series has the sum 2 log 2.<br />

3<br />

Remark: <strong>The</strong>re is a refernece around rearrangement of series. <strong>The</strong> reader can see the<br />

book, An Introduction to Mathematical Analysis by Loo-Keng Hua, pp 323-325.<br />

(Chinese Version)<br />

8.19 Let c n a n ib n ,wherea n −1 n / n , b n 1/n 2 . Show that ∑ c n is<br />

conditioinally convergent.<br />

Proof: It is clear that ∑ c n converges. Consider<br />

∑|c n| ∑ 1 n 2 1 n 4 ∑ 1 n 1 1 n 2 ≥ ∑ 1 n<br />

Hence, ∑|c n| diverges. That is, ∑ c n is conditioinally convergent.<br />

Remark: Wesay∑ c n converges if, and only if, the real part ∑ a n converges and the<br />

imaginary part ∑ b n converges, where c n a n ib n .<br />

8.20 Use <strong>The</strong>orem 8.23 to derive the following formulas:<br />

n<br />

(a) ∑ k1<br />

logk<br />

1 k 2 log2 n A O logn<br />

n<br />

(A is constant)<br />

Proof: Letfx logx<br />

x define on 3, , then f ′ x 1−logx 0on3, . So, it is<br />

x 2<br />

clear that fx is a positive and continuous function on 3, , with<br />

log x<br />

lim x→<br />

fx lim x→ x<br />

So, by <strong>The</strong>orem 8.23, wehave<br />

lim x→<br />

1 x 0byL-Hospital Rule.


n<br />

∑<br />

k3<br />

log k<br />

k<br />

which implies that<br />

<br />

3 n log x<br />

x dx C O log n<br />

n<br />

1 2 log2 n − 1 2 log2 3 C O<br />

n<br />

∑<br />

k1<br />

log k<br />

k<br />

,whereC is a constant<br />

log n<br />

n<br />

1 2 log2 n A O<br />

,whereC is a constant<br />

log n<br />

n ,<br />

where A C log2 − 1 2 2 log2 3 is a constant.<br />

n 1<br />

1<br />

(b) ∑ k2<br />

loglog n B O (B is constant)<br />

klogk n logn<br />

1<br />

Proof: Letfx defined on 2, , then 2<br />

xlogx f′ 1<br />

x −<br />

xlogx 1 log x 0on<br />

2, . So, it is clear that fx is a positive and continuous function on 3, , with<br />

lim x→<br />

fx lim 1<br />

x→ x log x 0.<br />

So, by <strong>The</strong>orem 8.23, wehave<br />

n<br />

∑<br />

k2<br />

1<br />

k log k n<br />

2<br />

dx<br />

x log x C O 1<br />

n log n<br />

log log n B O 1<br />

n log n<br />

where B C − log log 2 is a constant.<br />

8.21 If 0 a ≤ 1, s 1, define s, a ∑ n0<br />

n a −s .<br />

,whereC is a constant<br />

,whereC is a constant<br />

(a) Show that this series converges absolutely for s 1 and prove that<br />

k<br />

∑ s, h k<br />

k<br />

s s if k 1, 2, . . .<br />

h1<br />

where s s,1 is the Riemann zeta function.<br />

Proof: First, it is clear that s, a converges absolutely for s 1. Consider<br />

k<br />

∑ s, h k<br />

h1<br />

k<br />

∑<br />

h1<br />

k<br />

∑<br />

h1<br />

<br />

∑<br />

n0<br />

<br />

∑<br />

n0<br />

k<br />

∑ ∑<br />

n0 h1<br />

<br />

k s ∑<br />

n0<br />

<br />

k s ∑<br />

n0<br />

k s s.<br />

k<br />

∑<br />

h1<br />

1<br />

n h k s<br />

k s<br />

kn h s<br />

k s<br />

kn h s<br />

1<br />

kn h s<br />

1<br />

n 1 s


(b) Prove that ∑ n1<br />

−1 n−1 /n s 1 − 2 1−s s if s 1.<br />

Proof: Let<br />

n<br />

S n ∑ j1<br />

−1 j−1<br />

j s<br />

, and thus consider its subsequence S 2n as follows:<br />

2n<br />

S 2n ∑<br />

j1<br />

2n<br />

∑<br />

j1<br />

1<br />

j s<br />

1<br />

j s<br />

n<br />

− 2 ∑<br />

j1<br />

n<br />

− 2 1−s ∑<br />

j1<br />

1<br />

2j s<br />

which implies that<br />

lim n→<br />

S 2n 1 − 2 1−s s.<br />

Since S n converges, we know that S 2n also converges and has the same value. Hence,<br />

<br />

∑<br />

n1<br />

1<br />

j s<br />

−1 n−1 /n s 1 − 2 1−s s.<br />

8.22 Given a convergent series ∑ a n , where each a n ≥ 0. Prove that ∑ a n n −p<br />

converges if p 1/2. Give a counterexample for p 1/2.<br />

Proof: Since<br />

a n n −2p<br />

2<br />

≥ a n n −2p a n n −p ,<br />

we have ∑ a n n −p converges if p 1/2 since<br />

∑ a n converges and ∑ n −2p converges if p 1/2.<br />

and<br />

For p 1/2, we consider a n <br />

1<br />

nlogn 2 , then<br />

∑ a n converges by Cauchy Condensation <strong>The</strong>orem<br />

∑ a n n −1/2 ∑ 1<br />

n log n<br />

diverges by Cauchy Condensation <strong>The</strong>orem.<br />

8.23 Given that ∑ a n diverges. Prove that ∑ na n also diverges.<br />

n<br />

Proof: Assume ∑ na n converges, then its partial sum ∑ k1<br />

ka k is bounded. <strong>The</strong>n by<br />

Dirichlet Test, we would obtain<br />

∑ka k 1<br />

k<br />

∑ a k converges<br />

which contradicts to ∑ a n diverges. Hence, ∑ na n diverges.<br />

8.24 Given that ∑ a n converges, where each a n 0. Prove that<br />

∑a n a n1 1/2<br />

also converges. Show that the converse is also true if a n is monotonic.<br />

Proof: Since<br />

a n a n1<br />

≥ a<br />

2<br />

n a n1 1/2 ,<br />

we know that<br />

∑a n a n1 1/2


converges by ∑ a n converges.<br />

Conversly, since a n is monotonic, it must be decreasing since ∑ a n converges. So,<br />

a n ≥ a n1 for all n. Hence,<br />

a n a n1 1/2 ≥ a n1 for all n.<br />

So, ∑ a n converges since ∑a n a n1 1/2 converges.<br />

8.25 Given that ∑ a n converges absolutely. Show that each of the following series<br />

also converges absolutely:<br />

(a) ∑ a n<br />

2<br />

Proof: Since∑ a n converges, then a n → 0asn → . So, given 1, there exists a<br />

positive integer N such that as n ≥ N, wehave<br />

|a n| 1<br />

which implies that<br />

a n2 |a n| for n ≥ N.<br />

So, ∑ a n2 converges if ∑|a n| converges. Of course, ∑ a n2 converges absolutely.<br />

(b) ∑ an<br />

1a n<br />

(if no a n −1)<br />

Proof: Since∑|a n| converges, we have lim n→ a n 0. So, there exists a positive<br />

integer N such that as n ≥ N, wehave<br />

1/2 |1 a n|.<br />

Hence, as n ≥ N,<br />

a n<br />

2|a<br />

1 a n|<br />

n<br />

which implies that ∑<br />

(c) ∑ a n 2<br />

1a n<br />

2<br />

Proof: It is clear that<br />

an<br />

1a n<br />

By (a), we have proved that ∑ a n 2<br />

converges. So, ∑ an<br />

1a n<br />

1a n<br />

2<br />

a2<br />

n<br />

≤ a<br />

1 a2 n2 .<br />

n<br />

converges absolutely.<br />

converges absolutely.<br />

8.26 Determine all real values of x for which the following series converges.<br />

<br />

∑<br />

n1<br />

Proof: Consider its partial sum<br />

n<br />

∑<br />

k1<br />

1 1 2 ... 1 n<br />

1 1 2<br />

... 1 k <br />

k<br />

sin nx<br />

n .<br />

sin kx<br />

as follows.<br />

As x 2m, the series converges to zero. So it remains to consider x ≠ 2m as<br />

follows. Define<br />

and<br />

a k 1 1 2<br />

... 1 k<br />

k


then<br />

b k sin kx,<br />

a k1 − a k 1 1 2 ... 1 k 1<br />

k1<br />

k 1<br />

− 1 1 2<br />

... 1 k<br />

k<br />

k1 1 ... 1 1 − k 11 1 2 k k1 2<br />

kk 1<br />

k<br />

k1<br />

<br />

− 1 1 ... 1 <br />

2 k<br />

0<br />

kk 1<br />

and<br />

n<br />

∑ b k ≤ 1<br />

k1<br />

sin x .<br />

2<br />

So, by Dirichlet Test, we know that<br />

<br />

∑<br />

k1<br />

a k b k ∑<br />

k1<br />

1 1 ... 1 <br />

2 k<br />

sin kx<br />

k<br />

... 1 k <br />

converges.<br />

From above results, we have shown that the series converges for all x ∈ R.<br />

8.27. Prove that following statements:<br />

(a) ∑ a n b n converges if ∑ a n converges and if ∑b n − b n1 converges absolutely.<br />

Proof: Consider summation by parts, i.e., <strong>The</strong>orem 8.27, then<br />

n<br />

∑<br />

k1<br />

a k b k A n b n1 − ∑<br />

k1<br />

n<br />

A k b k1 − b k .<br />

Since ∑ a n converges, then |A n| ≤ M for all n. In addition, by <strong>The</strong>orem 8.10, lim n→ b n<br />

exists. So, we obtain that<br />

(1). lim n→<br />

A n b n1 exists<br />

and<br />

n<br />

(2). ∑<br />

k1<br />

n<br />

|A k b k1 − b k | ≤ M ∑<br />

k1<br />

<br />

|b k1 − b k | ≤ M ∑<br />

k1<br />

|b k1 − b k |.<br />

(2) implies that<br />

n<br />

(3). ∑ A k b k1 − b k converges.<br />

k1<br />

n<br />

By (1) and (3), we have shown that ∑ k1<br />

a k b k converges.<br />

Remark: In 1871, Paul du Bois Reymond (1831-1889) gave the result.<br />

(b) ∑ a n b n converges if ∑ a n has bounded partial sums and if ∑b n − b n1 converges<br />

absolutely, provided that b n → 0asn → .<br />

Proof: Bysummation by parts, wehave<br />

n<br />

∑<br />

k1<br />

a k b k A n b n1 − ∑ A k b k1 − b k .<br />

k1<br />

Since b n → 0asn → and ∑ a n has bounded partial sums, say |A n| ≤ M for all n. <strong>The</strong>n<br />

n


In addition,<br />

n<br />

(2). ∑<br />

k1<br />

(1). lim n→<br />

A n b n1 exists.<br />

n<br />

|A k b k1 − b k | ≤ M ∑<br />

k1<br />

<br />

|b k1 − b k | ≤ M ∑<br />

k1<br />

|b k1 − b k |.<br />

(2) implies that<br />

n<br />

(3). ∑ A k b k1 − b k converges.<br />

k1<br />

n<br />

By (1) and (3), we have shown that ∑ k1<br />

a k b k converges.<br />

Remark: (1) <strong>The</strong> result is first discovered by Richard Dedekind (1831-1916).<br />

(2) <strong>The</strong>re is an exercise by (b), we write it as a reference. Show the convergence of the<br />

<br />

k<br />

−1<br />

series ∑ k1<br />

.<br />

k<br />

Proof: Leta k −1 k<br />

, then in order to show the convergence of<br />

1/3<br />

<br />

∑ k1<br />

−1<br />

k<br />

and b<br />

k 2/3 k 1<br />

k<br />

, it suffices to show that ∑<br />

k k1<br />

a k : S n<br />

n ∈ N, there exists j ∈ N such that j 2 ≤ N j 1 2 . Consider<br />

S n a 1 a 2 a 3 a 4 ...a 8 a 9 ....a 15 ...a j<br />

2 ...a n<br />

n<br />

is bounded sequence. Given<br />

≤ 3a 3 5a 4 7a 15 9a 16 ..4k − 1a 2k 2 −1 4k 1a 2<br />

2k<br />

if j 2k, k ≥ 2<br />

3a 3 5a 4 7a 15 9a 16 ..4k − 3a 2k−2<br />

2 if j 2k − 1, k ≥ 3<br />

then as n large enough,<br />

S n ≤ −3a 4 5a 4 −7a 16 9a 16 ... −4k − 1a 2k<br />

2 4k 1a 2k<br />

2<br />

−3a 4 5a 4 −7a 16 9a 16 ... −4k − 5a 2k−2<br />

2 4k − 3a 2k−2<br />

2<br />

which implies that as n large enough,<br />

<br />

<br />

S n ≤ 2 ∑ a 2j<br />

2 2 ∑<br />

1<br />

j2<br />

j2 2j 4/3 : M 1 *<br />

Similarly, we have<br />

M 2 ≤ S n for all n **<br />

By (*) and (**), we have shown that<br />

n<br />

a k : S n is bounded sequence.<br />

∑ k1<br />

Note: (1) By above method, it is easy to show that<br />

<br />

−1<br />

k<br />

∑<br />

k1<br />

converges for p 1/2. For 0 p ≤ 1/2, the series diverges by<br />

1<br />

n 2 p ... 1<br />

n 2 2n p ≥ 2n 1<br />

n 2 n p ≥ 2n 1<br />

n 2 n p ≥ 2n 1<br />

n 1 2p ≥ 2n n 1 1 1.<br />

k p<br />

log k<br />

−1<br />

.<br />

k<br />

<br />

(2) <strong>The</strong>re is a similar question, show the divergence of the series ∑ k1<br />

Proof: Weuse<strong>The</strong>orem 8.13 to show it by inserting parentheses as follows. We insert<br />

k<br />

−1log<br />

parentheses such that the series ∑ forms ∑−1 k b k . If we can show ∑−1 k b k<br />

k


diverges, then ∑<br />

where<br />

By (2) and (4),<br />

By (1) and (3),<br />

−1log<br />

k<br />

k<br />

also diverges. Consider<br />

b k 1 m ... 1 m r , *<br />

(1). log m N<br />

(2). logm − 1 N − 1 log em − 1 N<br />

(3). logm r N<br />

(4). logm r 1 N 1 log mr1<br />

e N<br />

m r 1<br />

e<br />

m − 1 r 1 ≥ m if m is large enough.<br />

2m ≥ r.<br />

So, as k large enough ( m is large enough),<br />

b k ≥ m r 1 r ≥ 3m m 1 by (*).<br />

3<br />

It implies that ∑−1 k b k diverges since b k does NOT tends to zero as k goes infinity.So,<br />

k<br />

−1log<br />

we have proved that the series ∑ diverges.<br />

k<br />

(3) <strong>The</strong>re is a good exercise by summation by parts, we write it as a reference.<br />

Assume that ∑ <br />

k1<br />

a k b k converges and b n ↗ with lim n→ b n . Show that b n ∑ <br />

kn<br />

a k<br />

converges.<br />

Proof: First, we show that the convergence of ∑ <br />

k1<br />

a k by Dirichlet Test as follows.<br />

Since b n ↗ , there exists a positive integer n 0 such that as n n 0 ,wehaveb n 0. So,<br />

<br />

1<br />

we have<br />

b nn0<br />

is decreasing to zero. So<br />

n1<br />

<br />

∑ a kn0<br />

k1<br />

converges by Dirichlet Test.<br />

<br />

<br />

∑a kn0 b kn0 1<br />

b kn0<br />

k1<br />

For the convergence of b n ∑ kn<br />

a k ,letn n 0 , then<br />

<br />

b n ∑ a k ∑ a k b<br />

b n k<br />

b k<br />

kn kn<br />

and define c k a k b k and d k bn<br />

b k<br />

. Note that d k is decreasing to zero. Define<br />

k<br />

C k ∑ j1<br />

c j and thus we have<br />

m m<br />

b n ∑ a k ∑ a k b<br />

b n k<br />

b k<br />

kn kn<br />

So,<br />

m<br />

∑C k − C k−1 d k<br />

kn<br />

m−1<br />

∑ C k d k − d k1 C m d m − C n−1 d n .<br />

kn<br />

.


n ∑<br />

kn<br />

<br />

a k ∑<br />

kn<br />

<br />

a k b k<br />

b n<br />

b k<br />

∑ C k d k − d k1 C d − C n−1 d n<br />

kn<br />

<br />

∑<br />

kn<br />

C k d k − d k1 − C n−1 d n<br />

by C lim k→ C k and lim k→ d k 0. In order to show the existence of lim n→ b n ∑ kn<br />

a k ,<br />

it suffices to show the existence of lim n→ ∑ <br />

kn<br />

C k d k − d k1 . Since the series<br />

∑ <br />

kn<br />

C k d k − d k1 exists, lim n→ ∑ <br />

kn<br />

C k d k − d k1 0. From above results, we have<br />

proved the convergence of lim n→ b n ∑ kn<br />

a k .<br />

Note: We also show that lim n→ b n ∑ <br />

kn<br />

a k 0 by preceding sayings.<br />

Supplement on the convergence of series.<br />

A Show the divergence of ∑ 1/k. We will give some methods listed below. If the<br />

proof is easy, we will omit the details.<br />

(1) Use Cauchy Criterion for series. Since it is easy, we omit the proof.<br />

(2) Just consider<br />

1 1 2 1 3 1 4 ... 2 1 n ≥ 1 1 2 2 1 4 1 ...2n−1 2 n<br />

1 n 2 → .<br />

Remark: We can consider<br />

1 1 ... 1 1 ... 2 10<br />

1 ...≥ 1 11 100<br />

10 9 100 90 ...<br />

Note: <strong>The</strong> proof comes from Jing Yu.<br />

(3) Use Mathematical Induction to show that<br />

1<br />

k − 1 1 k 1<br />

k 1 ≥ 3 if k ≥ 3.<br />

k<br />

<strong>The</strong>n<br />

1 1 2 1 3 1 4 1 5 1 6 ....≥ 1 3 3 3 6 3 9 ...<br />

Remark: <strong>The</strong> proof comes from Bernoulli.<br />

(4) Use Integral Test. Since the proof is easy, we omit it.<br />

(5) Use Cauchy condensation <strong>The</strong>orem. Since the proof is easy, we omit it.<br />

(6) Euler Summation Formula, the reader can give it a try. We omit the proof.<br />

(7) <strong>The</strong> reader can see the book, Princilpes of Mathematical Analysis by Walter<br />

Rudin, Exercise 11-(b) pp 79.<br />

Suppose a n 0, S n a 1 ...a n ,and∑ a n diverges.<br />

(a) Prove that ∑ an<br />

1a n<br />

diverges.<br />

Proof: Ifa n → 0asn → , then by Limit Comparison <strong>The</strong>orem, we know that<br />

diverges. If a n does not tend to zero. Claim that does not tend to zero.<br />

∑ an<br />

1a n<br />

a n<br />

1a n


Suppose NOT, it means that lim n→<br />

a n<br />

1a n<br />

0. That is,<br />

lim 1<br />

n→<br />

0 lim<br />

1 a 1 n→<br />

a n 0<br />

n<br />

which contradicts our assumption. So, ∑ an<br />

1a n<br />

(b) Prove that<br />

and deduce that ∑ an<br />

S n<br />

Proof: Consider<br />

diverges.<br />

a N1<br />

S N1<br />

a N1<br />

S N1<br />

... a Nk<br />

S Nk<br />

diverges by claim.<br />

... a Nk<br />

S Nk<br />

≥ 1 − S N<br />

S Nk<br />

then ∑ an<br />

S n<br />

diverges by Cauchy Criterion with (*).<br />

Remark: Leta n 1, then ∑ an<br />

S n<br />

(c) Prove that<br />

and deduce that ∑ an<br />

S2<br />

n<br />

Proof: Consider<br />

and<br />

So, ∑ an<br />

S n<br />

2<br />

converges.<br />

≥ a N1 ...a Nk<br />

S Nk<br />

1 − S N<br />

S Nk<br />

, *<br />

∑ 1/n diverges.<br />

a n<br />

S n<br />

2<br />

≤ 1<br />

S n−1<br />

1<br />

S n−1<br />

− 1 S n<br />

<br />

− 1 S n<br />

a n<br />

S n−1 S n<br />

≥ a n<br />

S n<br />

2 ,<br />

∑<br />

S 1<br />

n−1<br />

−<br />

S 1 n<br />

converges by telescoping series with<br />

S 1 n<br />

→ 0.<br />

converges.<br />

(d) What can be said about<br />

∑<br />

Proof: For∑<br />

the series ∑<br />

For ∑<br />

so ∑<br />

an<br />

1n 2 a n<br />

a n<br />

1 na n<br />

and ∑ a n<br />

1 n 2 a n<br />

<br />

an<br />

1na n<br />

:asa n 1 for all n, theseries∑ an<br />

1na n<br />

∑ 1<br />

1n<br />

an<br />

1na n<br />

∑ 1<br />

an<br />

1n 2 a n<br />

1k 2<br />

: Consider<br />

converges.<br />

(8) Consider ∑ sin 1 n diverges.<br />

Proof: Since<br />

a n <br />

converges.<br />

a n<br />

0ifn ≠ k2<br />

,<br />

1ifn k 2<br />

1 ≤ 1 1 n 2 a<br />

1 n a n<br />

n 2 n , 2<br />

lim<br />

sin 1 n<br />

n→ 1<br />

n<br />

1,<br />

the series ∑ 1 n diverges by Limit Comparison <strong>The</strong>orem.<br />

diverges. As


Remark: In order to show the series ∑ sin 1 n diverges, we consider Cauchy Criterion<br />

as follows.<br />

n sin 1 ≤ sin 1 ... sin 1<br />

2n n 1<br />

n n<br />

and given x ∈ R, forn 0,1,2,..., we have<br />

|sin nx| ≤ n|sin x|.<br />

So,<br />

sin 1 2 ≤ sin 1 ... sin 1<br />

n 1<br />

n n<br />

for all n. Hence, ∑ sin 1 n diverges.<br />

Note: <strong>The</strong>re are many methods to show the divergence of the series ∑ sin 1 n . We can<br />

use Cauchy Condensation <strong>The</strong>orem to prove it. Besides, by (11), it also owrks.<br />

(9) O-Stolz’s <strong>The</strong>orem.<br />

n 1<br />

Proof: LetS n ∑ j1 j<br />

and X n log n. <strong>The</strong>n by O-Stolz’s <strong>The</strong>orem, it is easy to see<br />

lim n→<br />

S n .<br />

(10) Since n k1 1 1 k<br />

diverges, the series ∑ 1/k diverges by <strong>The</strong>orem 8.52.<br />

(11) Lemma: If a n is a decreasing sequence and ∑ a n converges. <strong>The</strong>n<br />

lim n→ na n 0.<br />

Proof: Sincea n → 0anda n is a decreasing sequence, we conclude that a n ≥ 0.<br />

Since ∑ a n converges, given 0, there exists a positive integer N such that as n ≥ N,<br />

we have<br />

a n ..a nk /2 for all k ∈ N<br />

which implies that<br />

k 1a nk /2 since a n ↘.<br />

Let k n, then as n ≥ N, wehave<br />

n 1a 2n /2<br />

which implies that as n ≥ N<br />

2n 1a 2n <br />

which implies that<br />

lim n→<br />

2na 2n 0 since lim n→<br />

a n 0. *<br />

Similarly, we can show that<br />

lim n→<br />

2n 1a 2n1 0. **<br />

So,by(*)adn(**),wehaveprovedthatlim n→ na n 0.<br />

Remark: From this, it is clear that ∑ 1 n diverges. In addition, we have the convergence<br />

of ∑ na n − a n1 . We give it a proof as follows.<br />

Proof: Write


then<br />

since<br />

n<br />

lim n→<br />

∑<br />

k1<br />

n<br />

S n ∑ ka k − a k1 <br />

k1<br />

n<br />

∑<br />

k1<br />

a k − na n1 ,<br />

lim n→<br />

S n exists<br />

a k exists and lim n→<br />

na n 0.<br />

B Prove that ∑ 1 p diverges, where p is a prime.<br />

Proof: GivenN, letp 1 ,...,p k be the primes that divide at least one integer≤ N. <strong>The</strong>n<br />

N<br />

∑<br />

n1<br />

k<br />

1<br />

n ≤ <br />

j1<br />

k<br />

<br />

j1<br />

≤ exp<br />

1 1 p j<br />

1 p j<br />

2 ...<br />

1<br />

1 − p 1 j<br />

by 1 − x −1 ≤ e 2x if 0 ≤ x ≤ 1/2. Hence, ∑ 1 p diverges since ∑ 1 n diverges.<br />

Remark: <strong>The</strong>re are many proofs about it. <strong>The</strong> reader can see the book, An<br />

Introduction To <strong>The</strong> <strong>The</strong>ory Of <strong>Number</strong>s by Loo-Keng Hua, pp 91-93. (Chinese<br />

Version)<br />

C Discuss some series related with ∑ sink .<br />

k<br />

STUDY: (1) We have shown that the series ∑ sin 1 diverges.<br />

k<br />

(2) <strong>The</strong> series ∑ sinna b diverges where a ≠ n for all n ∈ Z and b ∈ R.<br />

Proof: Suppose that ∑ sinna b converges, then lim n→ sinna b 0. Hence,<br />

lim n→|sinn 1a b − sinna b| 0. Consider<br />

|sinn 1a b − sinna b|<br />

k<br />

∑<br />

j1<br />

2<br />

p j<br />

2cos na b a 2<br />

sin a 2<br />

which implies that<br />

2 cosna b cos a<br />

2<br />

− sinna b sin a 2<br />

sin a 2


lim n→<br />

|sinn 1a b − sinna b|<br />

lim n→<br />

sinn 1a b − sinna b<br />

lim n→<br />

sup 2 cosna b cos a<br />

2<br />

− sinna b sin a 2<br />

sin a 2<br />

lim n→<br />

sup 2 cosna b cos a sin a 2 2<br />

|sin a| ≠ 0<br />

which is impossible. So, ∑ sinna b diverges.<br />

Remark: (1) By the same method, we can show the divergence of ∑ cosna b if<br />

a ≠ n for all n ∈ Z and b ∈ R.<br />

(2) <strong>The</strong> reader may give it a try to show that,<br />

and<br />

p<br />

by considering ∑ n0<br />

(**).<br />

(3) <strong>The</strong> series ∑ sink<br />

k<br />

p<br />

∑<br />

n0<br />

cosna b <br />

Proof: First, it is clear that ∑ sink<br />

k<br />

|∑ sin k| ≤<br />

1<br />

sin 1 2<br />

partial sums as follows: Since<br />

sin<br />

p1<br />

2 b<br />

sin b 2<br />

sin a p 2 b *<br />

p<br />

p1<br />

sin<br />

∑ sinna b b 2<br />

cos a p sin b 2 b **<br />

n0<br />

2<br />

e inab . However, it is not easy to show the divergence by (*) and<br />

converges conditionally.<br />

converges by Dirichlet’s Test since<br />

, we consider its<br />

. In order to show that the divergence of ∑ sink<br />

k<br />

3n3<br />

n<br />

∑ sin k ∑<br />

sin 3k 1 sin 3k 2 sin 3k 3<br />

k<br />

3k 1 3k 2 3k 3<br />

k1<br />

k0<br />

and note that there is one value is bigger than 1/2 among three values |sin 3k 1|,<br />

|sin 3k 2|, and|sin 3k 3|. So,<br />

3n3<br />

∑<br />

k1<br />

sin k<br />

k<br />

n 1<br />

2<br />

≥ ∑<br />

3k 3<br />

k0<br />

which implies the divergence of ∑ sink .<br />

k<br />

<br />

Remark: <strong>The</strong> series is like Dirichlet Integral sinx<br />

0<br />

x<br />

Integral converges conditionally.<br />

(4) <strong>The</strong> series ∑ |sink|r diverges for any r ∈ R.<br />

k<br />

Proof: We prove it by three cases as follows.<br />

(a) As r ≤ 0, we have<br />

So, ∑ |sink|r diverges in this case.<br />

k<br />

(b) As 0 r ≤ 1, we have<br />

∑<br />

|sin k|r<br />

k<br />

≥ ∑ 1 k .<br />

dx. Also, we know that Dirichlet


|sin k|r<br />

∑<br />

k<br />

So, ∑ |sink|r diverges in this case by (3).<br />

k<br />

(c) As r 1, we have<br />

3n3<br />

∑ |sin k|<br />

r<br />

k<br />

k1<br />

n<br />

∑<br />

k0<br />

n<br />

≥ ∑<br />

k0<br />

|sin 3k 1| r<br />

3k 1<br />

1 2 r<br />

3k 3 .<br />

≥ ∑<br />

<br />

|sin k|<br />

k<br />

.<br />

|sin 3k 2|r<br />

3k 2<br />

<br />

|sin 3k 3|r<br />

3k 3<br />

So, ∑ |sink|r diverges in this case.<br />

k<br />

(5) <strong>The</strong> series ∑ sin2p−1 k<br />

,wherep ∈ N, converges.<br />

k<br />

Proof: We will prove that there is a positive integer Mp such that<br />

n<br />

∑<br />

k1<br />

sin 2p−1 k ≤ Mp for all n. *<br />

So, if we can show (*), then by Dirichlet’s Test, we have proved it. In order to show (*),<br />

we claim that sin 2p−1 k can be written as a linear combination of sink, sin3k,...,<br />

sin2p − 1k. So,<br />

n<br />

∑<br />

k1<br />

n<br />

sin 2p−1 k ∑<br />

k1<br />

n<br />

≤ |a 1 | ∑<br />

k1<br />

|a 1 |<br />

a 1 sin k a 2 sin 3k ...a p sin2p − 1k<br />

n<br />

sin k ...|a p| ∑ sin2p − 1k<br />

k1<br />

|a<br />

≤ ... p|<br />

: Mp by <strong>The</strong>orem 8.30.<br />

sin 1 2 sin 2p−1<br />

2<br />

We show the claim by Mathematical Induction as follows. As p 1, it trivially holds.<br />

Assume that as p s holds, i.e.,<br />

then as p s 1, we have<br />

sin 2s−1 k ∑<br />

j1<br />

s<br />

a j sin2j − 1k


sin 2s1 k sin 2 ksin k 2s−1<br />

sin 2 k<br />

s<br />

∑ a j sin2j − 1k<br />

j1<br />

s<br />

∑ a j sin 2 k sin2j − 1k<br />

j1<br />

s<br />

∑ a j<br />

j1<br />

1 2<br />

1 2<br />

s<br />

∑<br />

j1<br />

1 − cos2k<br />

2<br />

sin2j − 1k<br />

by induction hypothesis<br />

s<br />

a j sin2j − 1k − ∑ a j cos2k sin2j − 1k<br />

j1<br />

s<br />

∑ a j sin2j − 1k − 1 2 ∑ s<br />

j1<br />

j1<br />

a j sin2j 1k sin2j − 3k<br />

which is a linear combination of sink,...,sin2s 1k. Hence, we have proved the claim<br />

by Mathematical Induction.<br />

Remark: By the same argument, the series<br />

n<br />

∑<br />

k1<br />

cos 2p−1 k<br />

is also bounded, i.e., there exists a positive number Mp such that<br />

n<br />

(6) Define ∑ k1<br />

n<br />

∑<br />

k1<br />

|cos 2p−1 k| ≤ Mp.<br />

sinkx<br />

: F<br />

k n x, then F n x is boundedly convergent on R.<br />

Proof: SinceF n x is a periodic function with period 2, andF n x is an odd function.<br />

So, it suffices to consider F n x is defined on 0, . In addition, F n 0 0 for all n.<br />

Hence, the domain I that we consider is 0, . Note that sinkx x<br />

cosktdt. So,<br />

0<br />

n<br />

F n x ∑<br />

k1<br />

<br />

0<br />

x<br />

∑<br />

k1<br />

<br />

0<br />

x<br />

<br />

0<br />

x<br />

sin kx<br />

k<br />

n<br />

cosktdt<br />

sinn 1 t − sin 1 t<br />

2 2<br />

2sin 1 t dt<br />

2<br />

sinn 1 2 t<br />

t<br />

dt <br />

0<br />

x<br />

1<br />

2sin t 2<br />

− 1 t<br />

k<br />

sin n 1 2 t dt − x 2<br />

which implies that<br />

<br />

0<br />

n 1 2<br />

x sin t<br />

t<br />

|F n x| ≤ <br />

0<br />

n 1 2 x sin t<br />

t<br />

dt <br />

0<br />

x t − 2sin t 2<br />

2t sin t 2<br />

dt <br />

0<br />

x t − 2sin t 2<br />

2t sin t 2<br />

sin n 1 2 t dt − x 2<br />

sin n 1 2 t dt 2 .


n<br />

For the part 1 2<br />

0<br />

that<br />

x sint<br />

<br />

t<br />

dt :Since sint<br />

0 t<br />

dt converges, there exists a positive M 1 such<br />

<br />

0<br />

n 1 2<br />

x sin t<br />

t<br />

dt ≤ M 1 for all x ∈ I and for all n.<br />

For the part 0<br />

x t−2sin t 2<br />

2t sin t 2<br />

sinn 1 2 tdt<br />

: Consider<br />

<br />

0<br />

x t − 2sin t 2<br />

2t sin t 2<br />

sin n 1 2<br />

t dt<br />

≤ <br />

0<br />

x t − 2sin t 2<br />

2t sin t 2<br />

dt since t − 2sin t 2 0onI<br />

≤ <br />

0<br />

t − 2sin t 2<br />

2t sin t 2<br />

dt : M 2 since lim<br />

t→0<br />

<br />

t − 2sin t 2<br />

2t sin t 2<br />

Hence,<br />

|F n x| ≤ M 1 M 2 for all x ∈ I and for all n.<br />

2<br />

So, F n x is uniformly bounded on I. It means that F n x is uniformly bounded on R.<br />

In addition, since<br />

F n x <br />

0<br />

n 1 2<br />

fixed x ∈ I, wehave<br />

x sin t<br />

t<br />

dt <br />

0<br />

x t − 2sin t 2<br />

2t sin t 2<br />

sin t<br />

0 t<br />

dt exists.<br />

and by Riemann-Lebesgue Lemma, in the text book, pp 313,<br />

0.<br />

sin n 1 2 t dt − x 2 ,<br />

x t − 2sin<br />

lim n→<br />

t 2<br />

0 2t sin t sin n 1 t dt 0.<br />

2<br />

2<br />

So, we have proved that<br />

lim n→<br />

F n x sin t<br />

0 t<br />

dt −<br />

2 x where x ∈ 0, .<br />

Hence, F n x is pointwise convergent on I. It means that F n x is pointwise<br />

convergent on R.<br />

Remark: (1) For definition of being boundedly convergent on a set S, the reader can<br />

see the text book, pp 227.<br />

(2) In the proof, we also shown the value of Dirichlet Integral<br />

sin t<br />

0 t<br />

dt 2<br />

by letting x .<br />

(3) <strong>The</strong>re is another proof on uniform bound. We write it as a reference.<br />

Proof: <strong>The</strong> domain that we consider is still 0, . Let 0, and consider two cases as<br />

follows.<br />

(a) x ≥ 0 : Using summation by parts,


n<br />

∑<br />

k1<br />

sin kx<br />

k<br />

≤ 1<br />

n 1 ∑ n<br />

k1<br />

≤ 1<br />

n 1<br />

sin kx<br />

k<br />

n<br />

∑<br />

k1<br />

1<br />

sin 2 1<br />

sin 2 1 − 1<br />

n 1<br />

1<br />

sin .<br />

2<br />

(b) 0 x ≤ :LetN 1 x , consider two cases as follows.<br />

As n N, then<br />

and as n ≥ N, then<br />

≤<br />

n<br />

∑<br />

k1<br />

N−1<br />

∑<br />

k1<br />

sin kx<br />

k<br />

sin kx<br />

k<br />

n<br />

≤ 1 ∑<br />

kN<br />

sin kx<br />

k<br />

≤ 1 1<br />

n 1 ∑ n<br />

k1<br />

n<br />

∑<br />

kN<br />

by (*)<br />

sin kx<br />

k<br />

by summation by parts<br />

≤ 1 1<br />

n 1 sin x 2<br />

1 2 .<br />

1 x sin x 2<br />

2<br />

Note that lim x→0<br />

<br />

1 x sin x 2<br />

n<br />

∑<br />

k1<br />

sin kx<br />

k<br />

<br />

1<br />

N sin x 2<br />

sin kx<br />

k<br />

N−1<br />

1<br />

N ∑ k1<br />

k<br />

∑ sin jx 1<br />

k 1 − 1 k<br />

j1<br />

≤ n|x| N|x| ≤ 1 *<br />

sin kx<br />

k<br />

1<br />

N<br />

− 1<br />

n 1<br />

n<br />

∑<br />

kN<br />

1<br />

sin x 2<br />

4. So, we may choose a ′ such that<br />

k<br />

∑ sin jx 1<br />

k 1 − 1 k<br />

j1<br />

2 ≤ 5 for all x ∈ 0, <br />

1 x sin x ′ .<br />

2<br />

By preceding sayings, we have proved that F n x is uniformly bounded on I. It means<br />

that F n x is uniformly bounded on R.<br />

D In 1911, Otto Toeplitz proves the following. Let a n and x n be two sequences<br />

1<br />

such that a n 0 for all n with lim n→ a 1 ...a n<br />

0 and lim n→ x n x. <strong>The</strong>n<br />

lim<br />

a 1 x 1 ...a n x n<br />

n→ a 1 ...a n<br />

x.<br />

n<br />

Proof: LetS n ∑ k1<br />

n<br />

a k and T n ∑ k1<br />

a k x k , then<br />

lim<br />

T n1 − T n<br />

n→ S n1 − S n<br />

lim n→<br />

a n1 x n1<br />

a n1<br />

lim n→<br />

x n1 x.<br />

So, by O-Stolz’s <strong>The</strong>orem, wehaveproveit.<br />

Remark: (1) Let a n 1, then it is an extension of <strong>The</strong>orem 8.48.<br />

(2) Show that


lim n→<br />

sin ... sin n<br />

1 ... 1 n<br />

.<br />

Proof: Write<br />

sin ... sin n<br />

1 1sin ... 1 1 n n sin n<br />

,<br />

1 ... 1 n<br />

1 ... 1 n<br />

the by Toeplitz’s <strong>The</strong>orem, we have proved it.<br />

E <strong>The</strong>orem 8.16 emphasizes the decrease of the sequence a n , we may ask if we<br />

remove the condition of decrease, is it true <strong>The</strong> answer is NOT necessary. For example,<br />

let<br />

a n 1 n −1n1 . 0<br />

2n<br />

F Some questions on series.<br />

(1) Show the convergence of the series ∑ n1<br />

log n sin 1 n .<br />

Proof: Since n sin 1 n 1 for all n, logn sin 1 n 0 for all n. Hence, we consider the<br />

new series<br />

<br />

∑<br />

n1<br />

− log n sin 1 <br />

n ∑ log<br />

n1<br />

sin 1/n<br />

1/n<br />

as follows. Let a n log sin1/n and b<br />

1/n n log 1 1 , then<br />

n 2<br />

lim<br />

a n<br />

n→<br />

<br />

b 1 n 6 .<br />

In addition,<br />

∑ b n ≤ ∑ 1 n 2<br />

by e x ≥ 1 x for all x ∈ R. From the convergence of ∑ b n , we have proved that the<br />

convergence of ∑ a n by Limit Comparison Test.<br />

<br />

∑ n1<br />

(2) Suppose that a n ∈ R, and the series ∑ <br />

n1<br />

converges absolutely.<br />

a n<br />

n<br />

Proof: By A. P. ≥ G. P. , we have<br />

a n2 1 n 2<br />

2<br />

<br />

which implies that ∑ n1<br />

a n<br />

n<br />

≥<br />

converges absolutely.<br />

a n2 converges. Prove that the series<br />

Remark: We metion that there is another proof by using Cauchy-Schwarz inequality.<br />

the difference of two proofs is that one considers a n , and another considers the partial<br />

sums S n .<br />

Proof: ByCauchy-Schwarz inequality,<br />

<br />

which implies that ∑ n1<br />

a n<br />

n<br />

n<br />

∑<br />

k1<br />

|a n|<br />

k<br />

2<br />

≤<br />

a n<br />

n<br />

∑ a2<br />

k<br />

k1<br />

converges absolutely.<br />

Double sequences and series<br />

8.28 Investigate the existence of the two iterated limits and the double limit of the<br />

n<br />

∑<br />

k1<br />

1<br />

k 2


double sequence f defined by the followings. Answer. Double limit exists in (a), (d), (e),<br />

(g). Both iterated limits exists in (a), (b), (h). Only one iterated limit exists in (c), (e).<br />

Neither iterated limit exists in (d), (f).<br />

(a) fp, q pq<br />

1<br />

Proof: It is easy to know that the double limit exists with lim p,q→ fp, q 0by<br />

definition. We omit it. In addition, lim p→ fp, q 0. So, lim q→ lim p→ fp, q 0.<br />

Similarly, lim p→ lim q→ fp, q 0. Hence, we also have the existence of two iterated<br />

limits.<br />

(b) fp, q <br />

p<br />

pq<br />

Proof: Letq np, then fp, q 1 . It implies that the double limit does not exist.<br />

n1<br />

However, lim p→ fp, q 1, and lim q→ fp, q 0. So, lim q→ lim p→ fp, q 1, and<br />

lim p→ lim q→ fp, q 0.<br />

(c) fp, q −1p p<br />

pq<br />

Proof: Letq np, then fp, q −1p<br />

n1<br />

. It implies that the double limit does not exist.<br />

In addition, lim q→ fp, q 0. So, lim p→ lim q→ fp, q 0. However, since<br />

lim p→ fp, q does not exist, lim q→ lim p→ fp, q does not exist.<br />

(d) fp, q −1 pq 1 p 1 q <br />

Proof: It is easy to know lim p,q→ fp, q 0. However, lim q→ fp, q and lim p→ fp, q<br />

do not exist. So, neither iterated limit exists.<br />

(e) fp, q −1p<br />

q<br />

Proof: It is easy to know lim p,q→ fp, q 0. In addition, lim q→ fp, q 0. So,<br />

lim p→ lim q→ fp, q 0. However, since lim p→ fp, q does not exist,<br />

lim q→ lim p→ fp, q does not exist.<br />

(f) fp, q −1 pq<br />

Proof: Let p nq, then fp, q −1 n1q . It means that the double limit does not<br />

exist. Also, since lim p→ fp, q and lim q→ fp, q do not exist, lim q→ lim p→ fp, q and<br />

lim p→ lim q→ fp, q do not exist.<br />

(g) fp, q cos p<br />

q<br />

Proof: Since|fp, q| ≤ 1 q , then lim p,q→ fp, q 0, and lim p→ lim q→ fp, q 0.<br />

However, since cosp : p ∈ N dense in −1, 1, we know that lim q→ lim p→ fp, q does<br />

not exist.<br />

(h) fp, q p q<br />

∑<br />

q 2 n1<br />

sin n p<br />

Proof: Rewrite<br />

and thus let p nq, fp, q sin 1 2n sin<br />

fp, q p sin q<br />

2p<br />

q1<br />

2nq<br />

nqsin 1<br />

2nq<br />

sin<br />

q1<br />

2p<br />

q 2 sin 1<br />

2p<br />

. It means that the double limit does not exist.<br />

However, lim p→ fp, q q1<br />

2q<br />

since sin x~x as x → 0. So, lim q→ lim p→ fp, q 1 2 .


Also, lim q→ fp, q lim q→ p sin 1 2p<br />

lim p→ lim q→ fp, q 0.<br />

8.29 Prove the following statements:<br />

sin q 2p<br />

sin<br />

q1<br />

2p<br />

q 2<br />

0since|sin x| ≤ 1. So,<br />

(a) A double series of positive terms converges if, and only if, the set of partial sums is<br />

bounded.<br />

Proof: ()Suppose that ∑ m,n<br />

fm, n converges, say ∑ m,n<br />

fm, n A 1 , then it means<br />

that lim p,q→ sp, q A 1 . Hence, given 1, there exists a positive integer N such that as<br />

p, q ≥ N, wehave<br />

|sp, q| ≤ |A 1 | 1.<br />

So, let A 2 maxsp, q :1≤ p, q N, wehave|sp, q| ≤ maxA 1 , A 2 for all p, q.<br />

Hence, we have proved the set of partial sums is bounded.<br />

()Suppose that the set of partial sums is bounded by M, i.e., if<br />

S sp, q : p, q ∈ N, then sup S : A ≤ M. Hence, given 0, then there exists a<br />

sp 1 , q 1 ∈ S such that<br />

A − sp 1 , q 1 ≤ A.<br />

Choose N maxp 1 , q 1 , then<br />

A − sp, q ≤ A for all p, q ≥ N<br />

since every term is positive. Hence, we have proved lim p,q→ sp, q A. Thatis,<br />

∑ m,n<br />

fm, n converges.<br />

(b) A double series converges if it converges absolutely.<br />

p<br />

Proof: Lets 1 p, q ∑ m1<br />

q<br />

∑ n1<br />

p<br />

|fm, n| and s 2 p, q ∑ m1<br />

q<br />

∑ n1<br />

fm, n, wewant<br />

to show that the existence of lim p,q→ s 2 p, q by the existence of lim p,q→ s 1 p, q as<br />

follows.<br />

Since lim p,q→ s 1 p, q exists, say its limit a. <strong>The</strong>n lim p→ s 1 p, p a. It implies that<br />

lim p→ s 2 p, p converges, say its limit b. So, given 0, there exists a positive integer N<br />

such that as p, q ≥ N<br />

|s 1 p, p − s 1 q, q| /2<br />

and<br />

|s 2 N, N − b| /2.<br />

So, as p ≥ q ≥ N,<br />

|s 2 p, q − b| |s 2 N, N − b s 2 p, q − s 2 N, N|<br />

/2 |s 2 p, q − s 2 N, N|<br />

/2 s 1 p, p − s 1 N, N<br />

/2 /2<br />

.<br />

Similarly for q ≥ p ≥ N. Hence, we have shown that<br />

lim<br />

p,q→ s 2p, q b.<br />

That is, we have prove that a double series converges if it converges absolutely.<br />

(c) ∑ m,n<br />

e −m2 n 2 <br />

converges.<br />

Proof: Letfm, n e −m2 n 2 <br />

, then by <strong>The</strong>orem 8.44, we have proved that


∑ m,n<br />

e −m2 n 2 <br />

converges since ∑ m,n<br />

e −m2 n 2 <br />

∑ m<br />

e −m2 ∑ n<br />

e −n2 .<br />

<br />

Remark: ∑ m,n1<br />

e −m2 n 2 <br />

∑ <br />

m1<br />

e −m2 ∑ n1<br />

e −n2 e<br />

<br />

e 2 −1<br />

8.30 Asume that the double series ∑ m,n<br />

anx mn converges absolutely for |x| 1. Call<br />

its sum Sx. Show that each of the following series also converges absolutely for |x| 1<br />

and has sum Sx :<br />

<br />

∑<br />

n1<br />

Proof: By<strong>The</strong>orem 8.42,<br />

So, ∑ <br />

n1<br />

∑ <br />

n1<br />

that<br />

∑<br />

m,n<br />

<br />

an x n<br />

1 − x n , ∑<br />

<br />

anx mn ∑<br />

n1<br />

n1<br />

<br />

an ∑<br />

m1<br />

Anx n ,whereAn ∑ ad.<br />

d|n<br />

<br />

x mn ∑ an<br />

n1<br />

2<br />

.<br />

x n<br />

1 − xn if |x| 1.<br />

an xn converges absolutely for<br />

1−x<br />

|x| 1 and has sum Sx.<br />

n<br />

Since every term in ∑ m,n<br />

anx mn , the term appears once and only once in<br />

Anx n . <strong>The</strong> converse also true. So, by <strong>The</strong>orem 8.42 and <strong>The</strong>orem 8.13, we know<br />

<br />

∑<br />

n1<br />

Anx n ∑ anx mn Sx.<br />

m,n<br />

8.31 If is real, show that the double series ∑ m,n<br />

m in − converges absolutely if,<br />

p q<br />

and only if, 2. Hint. Let sp, q ∑ m1<br />

∑ n1<br />

|m in| − . <strong>The</strong> set<br />

m in : m 1,2,...p, n 1, 2, . . . , p<br />

consists of p 2 complex numbers of which one has absolute value 2 , three satisfy<br />

|1 2i| ≤ |m in| ≤ 2 2, five satisfy |1 3i| ≤ |m in| ≤ 3 2, etc. Verify this<br />

geometricall and deduce the inequlity<br />

p<br />

p<br />

2 −/2 ∑<br />

2n − 1<br />

n ≤ sp, p ≤ ∑<br />

2n − 1<br />

n1<br />

n1 n 2 1 . /2<br />

Proof: Since the hint is trivial, we omit the proof of hint. From the hint, we have<br />

p<br />

∑<br />

n1<br />

2n − 1<br />

n 2 ≤ sp, p ∑<br />

m1<br />

p<br />

p<br />

∑<br />

n1<br />

p<br />

|m in| − ≤ ∑<br />

n1<br />

2n − 1<br />

1 n 2 /2 .<br />

Thus, it is clear that the double series ∑ m,n<br />

m in − converges absolutely if, and only if,<br />

2.<br />

8.32 (a) Show that the Cauchy product of ∑ <br />

n0<br />

−1 n1 / n 1 with itself is a<br />

divergent series.<br />

Proof: Since


n<br />

c n ∑ a k b n−k<br />

k0<br />

n<br />

∑<br />

k0<br />

−1 k<br />

k 1<br />

n<br />

−1 n ∑<br />

k0<br />

−1 n−k<br />

n − k 1<br />

1<br />

k 1 n − k 1<br />

and let fk n − k 1k 1 −k − n 2 2 n2<br />

2 2 ≤ n2 for k 0, 1, . . . , n.<br />

2<br />

Hence,<br />

n<br />

|c n| ∑<br />

k0<br />

1<br />

k 1 n − k 1<br />

2n 1<br />

≥ → 2asn → .<br />

n 2<br />

That is, the Cauchy product of ∑ <br />

n0<br />

−1 n1 / n 1 with itself is a divergent series.<br />

(b) Show that the Cauchy product of ∑ <br />

n0<br />

−1 n1 /n 1 with itself is the series<br />

<br />

2 ∑ −1<br />

n1<br />

1 <br />

n 1<br />

1 2 ... 1 n .<br />

n1<br />

Proof: Since<br />

n<br />

c n ∑ a k b n−k<br />

we have<br />

<br />

∑<br />

n0<br />

k0<br />

n<br />

∑<br />

k0<br />

−1 n ∑<br />

k0<br />

2−1n<br />

n 2<br />

−1 n<br />

n − k 1k 1<br />

<br />

c n ∑<br />

n<br />

n0<br />

<br />

2 ∑<br />

n0<br />

<br />

2 ∑<br />

n1<br />

∑ k0<br />

1<br />

n 2<br />

n<br />

2−1<br />

n<br />

n 2<br />

1<br />

k 1 ,<br />

∑ n<br />

k0<br />

1<br />

k 1 1<br />

n − k 1<br />

1<br />

k 1<br />

−1<br />

n<br />

n 2 1 1 2 ... 1<br />

n 1<br />

−1<br />

n1<br />

n 1<br />

(c) Does this converge Why<br />

Proof: Yes by the same argument in Exercise 8.26.<br />

1 1 2 ... 1 n .<br />

8.33 Given two absolutely convergent power series, say ∑ <br />

n0<br />

a n x n and ∑ n0<br />

b n x n ,<br />

having sums Ax and Bx, respectively, show that ∑ n0<br />

c n x n AxBx where


n<br />

c n ∑ a k b n−k .<br />

k0<br />

Proof: By<strong>The</strong>orem 8.44 and <strong>The</strong>orem 8.13, it is clear.<br />

Remark: We can use Mertens’ <strong>The</strong>orem, thenitisclear.<br />

8.34 Aseriesoftheform∑ <br />

n1<br />

a n /n s is called a Dirichlet series. Given two absolutely<br />

convergent Dirichlet series, say ∑ <br />

n1<br />

a n /n s and ∑ <br />

n1<br />

b n /n s , having sums As and Bs,<br />

respectively, show that ∑ n1<br />

c n /n s AsBs, wherec n ∑ d|n<br />

a d b n/d .<br />

Proof: By<strong>The</strong>orem 8.44 and <strong>The</strong>orem 8.13, wehave<br />

<br />

∑<br />

n1<br />

a n /n s<br />

<br />

∑<br />

n1<br />

<br />

b n /n s ∑<br />

n1<br />

where<br />

C n ∑ a d d −s b n/d n/d −s<br />

d|n<br />

n −s ∑ a d b n/d<br />

d|n<br />

c n /n s .<br />

So,wehaveprovedit.<br />

8.35 s ∑ <br />

n1<br />

1/n s , s 1, show that 2 s ∑ <br />

n1<br />

dn/n s ,wheredn is the<br />

number of positive divisors of n (including 1 and n).<br />

Proof: ItisclearbyExercise 8.34. So, we omit the proof.<br />

Ces’aro summability<br />

8.36 Show that each of the following series has C,1 sum 0 :<br />

(a) 1 − 1 − 1 1 1 − 1 − 1 1 1 −−.<br />

Proof: It is clear that |s 1 ...s n| ≤ 1 for all n, wheres n means that the nth partial sum<br />

of given series. So,<br />

s 1 ...s n<br />

n ≤ 1 n<br />

which implies that the given series has C,1 sum 0.<br />

(b) 1 − 1 1 1 − 1 1 1 − 1 − .<br />

2 2 2 2 2<br />

Proof: It is clear that |s 1 ...s n| ≤ 1 for all n, wheres 2<br />

n means that the nth partial<br />

sum of given series. So,<br />

s 1 ...s n<br />

n ≤<br />

2n<br />

1<br />

which implies that the given series has C,1 sum 0.<br />

(c) cos x cos3x cos5x (x real, x ≠ m).<br />

Proof: Lets n cosx ... cos2n − 1x, then<br />

C n


So,<br />

n<br />

s n ∑ cos2k − 1x<br />

j1<br />

sin 2nx<br />

2sinx .<br />

n<br />

∑ j1<br />

s ∑ n<br />

j sin 2jx<br />

j1<br />

n <br />

2n sin x<br />

sin nx sinn 1x<br />

<br />

2n sin x sin x<br />

≤ 1 → 0<br />

2nsin x 2<br />

which implies that the given series has C,1 sum 0.<br />

8.37 Given a series ∑ a n ,let<br />

Prove that:<br />

(a) t n n 1s n − n n<br />

n<br />

s n ∑<br />

k1<br />

Proof: DefineS 0 0, and thus<br />

n<br />

t n ∑ ka k<br />

k1<br />

n<br />

n<br />

a k , t n ∑<br />

k1<br />

∑ ks k − s k−1 <br />

k1<br />

n<br />

∑<br />

k1<br />

n<br />

n<br />

ks k − ∑ ks k−1<br />

k1<br />

∑ ks k − ∑k 1s k<br />

k1<br />

n<br />

∑<br />

k1<br />

n−1<br />

k1<br />

n<br />

ks k − ∑<br />

k1<br />

n 1s n − ∑<br />

k1<br />

n 1s n − n n .<br />

ka k , n 1 n<br />

n ∑ s k .<br />

k1<br />

k 1s k n 1s n<br />

(b) If ∑ a n is C,1 summable, then ∑ a n converges if, and only if, t n on as<br />

n → .<br />

Proof: Assume that ∑ a n converges. <strong>The</strong>n lim n→ s n exists, say its limit a. By(a),we<br />

have<br />

t nn n 1 n s n − n .<br />

<strong>The</strong>n by <strong>The</strong>orem 8.48, we also have lim n→ n a. Hence,<br />

n<br />

s k


lim<br />

t nn<br />

n→<br />

lim n 1<br />

n→ n s n − n<br />

lim n 1<br />

n→ n lim n→<br />

s n − lim n→<br />

n<br />

1 a − a<br />

0<br />

which is t n on as n → .<br />

Conversely, assume that t n on as n → , thenby(a),wehave<br />

n t nn n<br />

n 1 n 1 n s n<br />

which implies that (note that lim n→ n exists by hypothesis)<br />

lim n→<br />

s n lim n t nn<br />

n→<br />

n<br />

n 1 n 1 n<br />

lim n<br />

n→ n 1 lim t nn lim n<br />

n→ n→ n 1 lim<br />

n→<br />

n<br />

1 0 1 lim n→<br />

n<br />

lim n→<br />

n<br />

That is, ∑ a n converges.<br />

(c) ∑ a n is C,1 summable if, and only if, ∑ t n /nn 1 converges.<br />

Proof: Consider<br />

which implies that<br />

t n<br />

nn 1 s n −<br />

n<br />

∑<br />

k1<br />

n<br />

n 1<br />

n n − n − 1 n−1<br />

n − n<br />

n 1<br />

n<br />

n 1 n − n − n<br />

1 n−1<br />

t k<br />

kk 1 <br />

n<br />

n 1 n. *<br />

()Suppose that ∑ a n is C,1 summable, i.e., lim n→ n exists. <strong>The</strong>n<br />

n t<br />

lim n→ ∑ k<br />

k1<br />

exists by (*).<br />

kk1<br />

n t<br />

()Suppose that lim n→ ∑ k<br />

k1<br />

exists. <strong>The</strong>n lim<br />

kk1<br />

n→ n exists by (*). Hence, ∑ a n<br />

is C,1 summable.<br />

8.38 Given a monotonic a n of positive terms, such that lim n→ a n 0. Let<br />

n<br />

s n ∑<br />

k1<br />

Prove that:<br />

(a) v n 1 u 2 n −1 n s n /2.<br />

n<br />

a k , u n ∑<br />

k1<br />

Proof: Defines 0 0, and thus consider<br />

−1 k a k , v n ∑<br />

k1<br />

n<br />

−1 k s k .


n<br />

u n ∑−1 k a k<br />

k1<br />

n<br />

∑−1 k s k − s k−1 <br />

k1<br />

n<br />

∑−1 k s k ∑−1 k1 s k−1<br />

k1<br />

n<br />

∑<br />

k1<br />

n<br />

k1<br />

n<br />

−1 k s k ∑<br />

k1<br />

−1 k s k −1 n1 s n<br />

which implies that<br />

2v n −1 n1 s n<br />

v n 1 2 u n −1 n s n /2.<br />

(b) ∑ <br />

n1<br />

−1 n s n is C,1 summable and has Ces’aro sum 1 ∑ −1 n a<br />

2 n1 n .<br />

Proof: First, lim n→ u n exists since it is an alternating series. In addition, since<br />

lim n→ a n 0, we know that lim n→ s n /n 0by<strong>The</strong>orem 8.48. Hence,<br />

Consider by (a),<br />

v n<br />

n<br />

∑ k1<br />

v k<br />

n<br />

u n<br />

2n −1n s n<br />

2n<br />

<br />

1 n<br />

2<br />

∑ k1<br />

∑ n<br />

u<br />

k1 k<br />

2n<br />

→ 1 2 lim n→ u k<br />

<br />

→ 0asn → .<br />

u k 1 n<br />

2<br />

∑ k1<br />

−1 k s k<br />

n<br />

v n<br />

2n<br />

1 2 ∑ −1 n a n<br />

n1<br />

by <strong>The</strong>orem 8.48.<br />

(c) ∑ n1<br />

−1 n 1 1 2 ... 1 n − log 2 C,1.<br />

<br />

Proof: By (b) and ∑ n1<br />

Infinite products<br />

−1 n<br />

n<br />

− log 2, it is clear.<br />

8.39 Determine whether or not the following infinite products converges. Find the<br />

value of each convergent product.<br />

<br />

(a) n2<br />

1 −<br />

2<br />

nn1<br />

Proof: Consider<br />

we have<br />

1 − 2<br />

nn 1<br />

<br />

n − 1n 2<br />

nn 1<br />

,


n<br />

<br />

n2<br />

which implies that<br />

(b) <br />

n2<br />

1 − n −2 <br />

Proof: Consider<br />

we have<br />

which implies that<br />

<br />

(c) n2<br />

n 3 −1<br />

n 3 1<br />

Proof: Consider<br />

1 − 2<br />

kk 1<br />

n<br />

<br />

k2<br />

<br />

<br />

n2<br />

n<br />

<br />

n2<br />

k − 1k 2<br />

kk 1<br />

1 2 4<br />

3 2 3 5<br />

4 3 4 6<br />

5<br />

n 3n<br />

2<br />

1 − n −2 <br />

1 − 2<br />

nn 1<br />

1 − k −2 <br />

we have (let fk k − 1 2 k − 1 1),<br />

n<br />

<br />

k2<br />

1 3 .<br />

n − 1n 1<br />

nn ,<br />

n<br />

k2<br />

n 1<br />

2n<br />

k − 1k 1<br />

kk<br />

<br />

1 − n −2 1/2.<br />

n2<br />

n 3 − 1<br />

n 3 1 n − 1n2 n 1<br />

n 1n 2 − n 1<br />

n − 1n<br />

<br />

2 n 1<br />

n 1 n − 1 2 n − 1 1<br />

n<br />

k 3 − 1<br />

k 3 1 <br />

k2<br />

n<br />

− 1n 2<br />

nn 1<br />

k − 1k 2 k 1<br />

k 1 k − 1 2 k − 1 1<br />

2 3 n2 n 1<br />

nn 1<br />

which implies that<br />

<br />

n3 − 1<br />

n 3 1 2 3 .<br />

n2<br />

(d) n0<br />

1 z 2n <br />

if |z| 1.<br />

Proof: Consider<br />

n<br />

1 z 2k <br />

1 z1 z 2 1 z 2n <br />

<br />

k0


which implies that<br />

which implies that (if |z| 1)<br />

So,<br />

n<br />

<br />

k0<br />

n<br />

1 − z 1 z 2k <br />

1 − z 2n1 <br />

k0<br />

1 z 2k <br />

1 − z2n1 <br />

1 − z<br />

→ 1<br />

1 − z<br />

<br />

1 z 2n <br />

1<br />

1 − z .<br />

n0<br />

as n → .<br />

8.40 If each partial sum s n of the convergent series ∑ a n is not zero and if the sum<br />

itself is not zero, show that the infinite product a 1 n2<br />

1 − a n /s n−1 converges and has the<br />

value ∑ n1<br />

a n .<br />

Proof: Consider<br />

n<br />

n<br />

s<br />

a 1 1 a k /s k−1 a 1 k−1 a k<br />

s k−1<br />

k2<br />

k2<br />

n<br />

<br />

a 1 <br />

k2<br />

s k<br />

s k−1<br />

s n → ∑ a n ≠ 0.<br />

So, the infinite product a 1 <br />

n2<br />

1 − a n /s n−1 converges and has the value ∑ n1<br />

a n .<br />

8.41 Find the values of the following products by establishing the following identities<br />

and summing the series:<br />

(a) n2<br />

1 − 1 2 ∑ 2<br />

2 n −2 −n .<br />

n1<br />

Proof: Consider<br />

we have<br />

1 − 1<br />

2 n − 2 2n − 1<br />

2 n − 2 1 2 2n − 1<br />

2 n−1 − 1 ,<br />

n<br />

<br />

k2<br />

1 − 1<br />

2 k − 2<br />

n<br />

<br />

k2<br />

1<br />

2 2k − 1<br />

2 k−1 − 1<br />

n<br />

2 −n−1 <br />

k2<br />

2 k − 1<br />

2 k−1 − 1<br />

2 −n−1 2 n − 1<br />

2 −n−1 2 n−1 ...1<br />

1 ... 1<br />

n<br />

∑<br />

k1<br />

n<br />

2 ∑<br />

k1<br />

1<br />

2 k−1<br />

2 n−1<br />

1<br />

2 k .


So,<br />

<br />

(b) n2<br />

1 1<br />

n 2 −1<br />

Proof: Consider<br />

we have<br />

So,<br />

<br />

<br />

n2<br />

1 − 1<br />

2 n − 2<br />

1<br />

2 ∑ .<br />

n1 nn1<br />

1 1<br />

n 2 − 1 n 2<br />

n 2 − 1 <br />

n<br />

<br />

k2<br />

<br />

<br />

n2<br />

1 1<br />

k 2 − 1<br />

1 1<br />

n 2 − 1<br />

<br />

2 ∑ 2 −n<br />

n1<br />

2.<br />

nn<br />

n − 1n 1 ,<br />

n<br />

<br />

kk<br />

k − 1k 1<br />

k2<br />

2 n<br />

n 1<br />

2 1 − 1<br />

n<br />

2 ∑<br />

k1<br />

<br />

2 ∑<br />

n1<br />

2.<br />

n 1<br />

1<br />

kk 1 .<br />

1<br />

nn 1<br />

8.42 Determine all real x for which the product <br />

n1<br />

cosx/2 n converges and find the<br />

value of the product when it does converge.<br />

Proof: Ifx ≠ m, wherem ∈ Z, thensin x<br />

n<br />

<br />

k1<br />

cosx/2 k 2n sin x<br />

2 n<br />

2 n sin x<br />

n<br />

<br />

2 n k1<br />

2 n ≠ 0 for all n ∈ N. Hence,<br />

cosx/2 k <br />

sin x<br />

2 n sin x → sin x x .<br />

2 n<br />

If x m, wherem ∈ Z. <strong>The</strong>n as m 0, it is clear that the product converges to 1. So,<br />

we consider m ≠ 0 as follows. Since x m, choosing n large enough, i.e., as n ≥ N so<br />

that sin x ≠ 0. Hence,<br />

2 n<br />

and note that<br />

Hence,<br />

n<br />

<br />

k1<br />

N−1<br />

cosx/2 k <br />

k1<br />

lim n→<br />

n<br />

cosx/2 k cosx/2 k <br />

kN<br />

N−1<br />

cosx/2 k sinx/2<br />

<br />

N−1 <br />

2 n−N1 sinx/2 n <br />

k1<br />

sinx/2 N−1 <br />

2 n−N1 sinx/2 n sinx/2N−1 <br />

x/2 N−1 .<br />

<br />

cosx/2 k sinx/2N−1 N−1<br />

<br />

cosx/2<br />

x/2 N−1 k .<br />

k1<br />

k1


So, by above sayings, we have prove that the convergence of the product for all x ∈ R.<br />

8.43 (a) Let a n −1 n / n for n 1,2,... Show that1 a n diverges but that<br />

∑ a n converges.<br />

Proof: Clearly, ∑ a n converges since it is alternating series. Consider<br />

2n<br />

<br />

k2<br />

2n<br />

1 a k 1 −1k<br />

k2<br />

k<br />

1 1 2<br />

≤ 1 1 2<br />

1 − 1 3<br />

1 − 1 4<br />

1 1 4<br />

1 1 4<br />

1 − 1<br />

2n − 1<br />

1 − 1<br />

2n<br />

1 1<br />

2n<br />

1 1<br />

2n<br />

1 1 2<br />

1 − 1 4<br />

1 −<br />

1 2n<br />

*<br />

and note that<br />

n<br />

1 −<br />

2k 1 : p n<br />

k2<br />

is decreasing. From the divergence of ∑ 1 , we know that p 2k n → 0. So,<br />

That is, <br />

k2<br />

1 a k diverges to zero.<br />

<br />

1 a k 0.<br />

k2<br />

(b) Let a 2n−1 −1/ n , a 2n 1/ n 1/n for n 1, 2, . . . Show that 1 a n <br />

converges but ∑ a n diverges.<br />

Proof: Clearly, ∑ a n diverges. Consider<br />

and<br />

2n<br />

1 a k 1 a 2 1 a 3 1 a 4 1 a 2n <br />

k2<br />

31 a 3 1 a 4 1 a 2n <br />

3 1 − 1<br />

2 2<br />

1 − 1<br />

n n<br />

2n1<br />

1 a k 1 a 2 1 a 3 1 a 4 1 a 2n 1 a 2n1 <br />

k2<br />

3 1 − 1 1 − 1<br />

2 2<br />

n n<br />

By (*) and (**), we know that<br />

1 a n converges<br />

n<br />

since k2<br />

1 − 1<br />

k k<br />

converges.<br />

1 − 1<br />

n 1<br />

8.44 Assume that a n ≥ 0 for each n 1, 2, . . . Assume further that<br />

*<br />

**


Show that k1<br />

a 2n2<br />

1 a 2n2<br />

a 2n1 <br />

a 2n<br />

1 a 2n<br />

for n 1, 2, . . .<br />

1 −1 k a k converges if, and only if, ∑ k1<br />

−1 k a k converges.<br />

a<br />

1c<br />

Proof: First, we note that if b, then 1 a1 − b 1, and if b <br />

1a<br />

c , then<br />

1 1 − b1 c. Hence, by hypothesis, we have<br />

1 1 a 2n 1 − a 2n1 *<br />

and<br />

1 1 a 2n2 1 − a 2n1 . **<br />

()Suppose that ∑ <br />

k1<br />

−1 k a k converges, then lim k→ a k 0. Consider Cauchy<br />

Condition for product,<br />

1 −1 p1 a p1 1 −1 p2 a p2 1 −1 pq a pq − 1 for q 1, 2, 3, . . . .<br />

If p 1 2m, andq 2l, then<br />

1 −1 p1 a p1 1 −1 p2 a p2 1 −1 pq a pq − 1<br />

|1 a 2m 1 − a 2m1 1 a 2m2l − 1|<br />

≤ 1 a 2m − 1by(*)and(**)<br />

a 2m → 0.<br />

<br />

Similarly for other cases, so we have proved that k1<br />

1 −1 k a k converges by<br />

Cauchy Condition for product.<br />

()This is a counterexample as follows. Let a n −1 n<br />

n, thenitiseasytoshowthat<br />

a 2n2<br />

a<br />

1 a 2n1 <br />

2n2<br />

In addition,<br />

n<br />

<br />

k1<br />

1 −1 k a k <br />

k1<br />

However, consider<br />

n<br />

∑<br />

k1<br />

n<br />

∑<br />

k1<br />

n<br />

∑<br />

k1<br />

n<br />

n<br />

a 2k − a 2k−1 <br />

exp 1<br />

2k<br />

exp −1k<br />

k<br />

− exp<br />

exp<br />

expb k 1<br />

2k 1<br />

2k − 1<br />

<br />

exp −1n<br />

n<br />

a 2n<br />

1 a 2n<br />

for n 1, 2, . . .<br />

−1<br />

2k − 1<br />

n<br />

∑<br />

k1<br />

−1 k<br />

k<br />

,whereb k ∈<br />

≥ ∑ exp−1 1<br />

k1<br />

2k 1<br />

2k − 1<br />

So, by <strong>The</strong>orem 8.13, we proved the divergence of ∑ k1<br />

→ as n → .<br />

<br />

−1 k a k .<br />

− 1<br />

≥ 0 for all<br />

→ exp− log 2 as n → .<br />

−1<br />

2k − 1 , 1<br />

2k<br />

8.45 A complex-valued sequence fn is called multiplicative if f1 1andif<br />

fmn fmfn whenever m and n are relatively prime. (See Section 1.7) It is called<br />

completely multiplicative if<br />

f1 1andiffmn fmfn for all m and n.


(a) If fn is multiplicative and if the series ∑ fn converges absolutely, prove that<br />

<br />

∑<br />

n1<br />

fn 1 fp k fp k2 ...,<br />

k1<br />

where p k denote the kth prime, the product being absolutely convergent.<br />

<br />

Proof: We consider the partial product P m m<br />

k1<br />

1 fp k fp k2 ... and show<br />

that P m → ∑ <br />

n1<br />

fn as m → . Writing each factor as a geometric series we have<br />

m<br />

P m 1 fp k fp k2 ...,<br />

k1<br />

a product of a finite number of absolutely convergent series. When we multiple these series<br />

together and rearrange the terms such that a typical term of the new absolutely convergent<br />

series is<br />

fn fp 1<br />

a 1<br />

fp m<br />

a m<br />

,wheren p 1<br />

a 1<br />

p m<br />

a m<br />

,<br />

and each a i ≥ 0. <strong>The</strong>refore, we have<br />

P m ∑ fn,<br />

1<br />

where ∑ 1<br />

is summed over those n having all their prime factors ≤ p m .Bytheunique<br />

factorization theorem (<strong>The</strong>orem 1.9), each such n occors once and only once in ∑ 1<br />

.<br />

Substracting P m from ∑ <br />

n1<br />

fn, weget<br />

<br />

<br />

∑ fn − P m ∑ fn − ∑ fn ∑ fn<br />

n1<br />

n1<br />

1<br />

2<br />

where ∑ 2<br />

is summed over those n having at least one prime factor p m . Since these n<br />

occors among the integers p m ,wehave<br />

<br />

∑<br />

n1<br />

fn − P m<br />

<br />

≤ ∑<br />

np m<br />

|fn|.<br />

As m → the last sum tends to 0 because ∑ n1<br />

fn converges, so P m → ∑ n1<br />

fn.<br />

To prove that the product converges absolutely we use <strong>The</strong>orem 8.52. <strong>The</strong> product has<br />

the form 1 a k ,where<br />

a k fp k fp k2 ....<br />

<br />

<strong>The</strong> series ∑|a k | converges since it is dominated by ∑ n1|fn|.<br />

<strong>The</strong>reofore, 1 ak <br />

also converges absolutely.<br />

Remark: <strong>The</strong> method comes from Euler. By the same method, it also shows that there<br />

are infinitely many primes. <strong>The</strong> reader can see the book, An Introduction To <strong>The</strong> <strong>The</strong>ory<br />

Of <strong>Number</strong>s by Loo-Keng Hua, pp 91-93. (Chinese Version)<br />

(b) If, in addition, fn is completely multiplicative, prove that the formula in (a)<br />

becomes<br />

<br />

<br />

∑ fn <br />

1<br />

1 − fp<br />

n1<br />

k1<br />

k .<br />

Note that Euler’s product for s (<strong>The</strong>orem 8.56) is the special case in which fn n −s .<br />

Proof: By(a),iffn is completely multiplicative, then rewrite


1 fp k fp k2 ... ∑fp k n<br />

n0<br />

1<br />

1 − fp k <br />

since |fp k | 1 for all p k . (Suppose NOT, then |fp k | ≥ 1 |fp kn | |fp k | n ≥ 1<br />

contradicts to lim n→ fn 0. ).<br />

Hence,<br />

<br />

∑<br />

n1<br />

<br />

fn <br />

k1<br />

1<br />

1 − fp k .<br />

8.46 This exercise outlines a simple proof of the formula 2 2 /6. Start with the<br />

inequality sinx x tan x, valid for 0 x /2, taking recipocals, and square each<br />

member to obtain<br />

cot 2 x 1 1 cot<br />

x 2 x.<br />

2<br />

Now put x k/2m 1, wherek and m are integers, with 1 ≤ k ≤ m, and sum on k to<br />

obtain<br />

m<br />

∑ cot 2 k<br />

m<br />

m<br />

2m 12<br />

<br />

2m 1<br />

∑<br />

1 m ∑ cot<br />

2 k 2<br />

2<br />

2m k<br />

1 .<br />

k1<br />

k1<br />

k1<br />

Use the formula of Exercise 1.49(c) to deduce the ineqaulity<br />

m2m − 1 2<br />

32m 1 2<br />

m<br />

∑<br />

k1<br />

1<br />

k 2<br />

<br />

2mm 12<br />

32m 1 2<br />

Now let m → to obtain 2 2 /6.<br />

Proof: <strong>The</strong> proof is clear if we follow the hint and Exercise 1.49 (c), soweomitit.<br />

8.47 Use an argument similar to that outlined in Exercise 8.46 to prove that<br />

4 4 /90.<br />

Proof: <strong>The</strong> proof is clear if we follow the Exercise 8.46 and Exercise 1.49 (c), sowe<br />

omit it.<br />

Remark: (1) From this, it is easy to compute the value of 2s, where<br />

s ∈ n : n ∈ N. In addition, we will learn some new method such as Fourier series and<br />

so on, to find the value of Riemann zeta function.<br />

(2) <strong>The</strong>r is an open problem that 2s − 1, wheres ∈ n ∈ N : n 1.


Sequences of Functions<br />

Uniform convergence<br />

9.1 Assume that f n → f uniformly on S and that each f n is bounded on<br />

S. Prove that {f n } is uniformly bounded on S.<br />

Proof: Since f n → f uniformly on S, then given ε = 1, there exists a<br />

positive integer n 0 such that as n ≥ n 0 , we have<br />

|f n (x) − f (x)| ≤ 1 for all x ∈ S. (*)<br />

Hence, f (x) is bounded on S by the following<br />

|f (x)| ≤ |f n0 (x)| + 1 ≤ M (n 0 ) + 1 for all x ∈ S. (**)<br />

where |f n0 (x)| ≤ M (n 0 ) for all x ∈ S.<br />

Let |f 1 (x)| ≤ M (1) , ..., |f n0 −1 (x)| ≤ M (n 0 − 1) for all x ∈ S, then by<br />

(*) and (**),<br />

So,<br />

|f n (x)| ≤ 1 + |f (x)| ≤ M (n 0 ) + 2 for all n ≥ n 0 .<br />

|f n (x)| ≤ M for all x ∈ S and for all n<br />

where M = max (M (1) , ..., M (n 0 − 1) , M (n 0 ) + 2) .<br />

Remark: (1) In the proof, we also shows that the limit function f is<br />

bounded on S.<br />

(2) <strong>The</strong>re is another proof. We give it as a reference.<br />

Proof: Since Since f n → f uniformly on S, then given ε = 1, there exists<br />

a positive integer n 0 such that as n ≥ n 0 , we have<br />

|f n (x) − f n+k (x)| ≤ 1 for all x ∈ S and k = 1, 2, ...<br />

So, for all x ∈ S, and k = 1, 2, ...<br />

|f n0 +k (x)| ≤ 1 + |f n0 (x)| ≤ M (n 0 ) + 1 (*)<br />

where |f n0 (x)| ≤ M (n 0 ) for all x ∈ S.<br />

Let |f 1 (x)| ≤ M (1) , ..., |f n0 −1 (x)| ≤ M (n 0 − 1) for all x ∈ S, then by<br />

(*),<br />

|f n (x)| ≤ M for all x ∈ S and for all n<br />

1


where M = max (M (1) , ..., M (n 0 − 1) , M (n 0 ) + 1) .<br />

9.2 Define two sequences {f n } and {g n } as follows:<br />

(<br />

f n (x) = x 1 + 1 )<br />

if x ∈ R, n = 1, 2, ...,<br />

n<br />

g n (x) =<br />

{ 1<br />

n<br />

Let h n (x) = f n (x) g n (x) .<br />

if x = 0 or if x is irrational,<br />

b + 1 n if x is rational, say x = a b , b > 0.<br />

(a) Prove that both {f n } and {g n } converges uniformly on every bounded<br />

interval.<br />

and<br />

Proof: Note that it is clear that<br />

lim f n (x) = f (x) = x, for all x ∈ R<br />

n→∞<br />

{ 0 if x = 0 or if x is irrational,<br />

lim g n (x) = g (x) =<br />

n→∞ b if x is ratonal, say x = a, b > 0.<br />

b<br />

In addition, in order to show that {f n } and {g n } converges uniformly<br />

on every bounded interval, it suffices to consider the case of any compact<br />

interval [−M, M] , M > 0.<br />

Given ε > 0, there exists a positive integer N such that as n ≥ N, we<br />

have<br />

M<br />

n < ε and 1 n < ε.<br />

Hence, for this ε, we have as n ≥ N<br />

|f n (x) − f (x)| = ∣ x ∣ ≤ M < ε for all x ∈ [−M, M]<br />

n n<br />

and<br />

|g n (x) − g (x)| ≤ 1 n<br />

< ε for all x ∈ [−M, M] .<br />

That is, we have proved that {f n } and {g n } converges uniformly on every<br />

bounded interval.<br />

Remark: In the proof, we use the easy result directly from definition<br />

of uniform convergence as follows. If f n → f uniformly on S, then f n → f<br />

uniformly on T for every subset T of S.<br />

2


(b) Prove that h n (x) does not converges uniformly on any bounded interval.<br />

Proof: Write<br />

{<br />

h n (x) =<br />

a + a n<br />

x<br />

n<br />

(<br />

1 +<br />

1<br />

( n)<br />

if x = 0 or x is irrational<br />

1 +<br />

1<br />

+ ) 1<br />

b bn if x is rational, say x =<br />

a<br />

b<br />

<strong>The</strong>n<br />

{ 0 if x = 0 or x is irrational<br />

lim h n (x) = h (x) =<br />

n→∞ a if x is rational, say x = a .<br />

b<br />

Hence, if h n (x) converges uniformly on any bounded interval I, then h n (x)<br />

converges uniformly on [c, d] ⊆ I. So, given ε = max (|c| , |d|) > 0, there is a<br />

positive integer N such that as n ≥ N, we have<br />

max (|c| , |d|) > |h n (x) − h (x)|<br />

{ ∣ ( )∣ x<br />

= n 1 +<br />

1 ∣<br />

n =<br />

|x| ∣<br />

n 1 +<br />

1 ∣ ( n if x ∈ Q c ∩ [c, d] or x = 0<br />

∣ a<br />

n 1 +<br />

1<br />

+ )∣ 1 ∣<br />

b bn if x ∈ Q ∩ [c, d] , x =<br />

a<br />

b<br />

which implies that (x ∈ [c, d] ∩ Q c or x = 0)<br />

max (|c| , |d|) > |x|<br />

n ∣ 1 + 1 n∣ ≥ |x|<br />

n<br />

≥<br />

max (|c| , |d|)<br />

n<br />

which is absurb. So, h n (x) does not converges uniformly on any bounded<br />

interval.<br />

9.3 Assume that f n → f uniformly on S, g n → f uniformly on S.<br />

(a) Prove that f n + g n → f + g uniformly on S.<br />

Proof: Since f n → f uniformly on S, and g n → f uniformly on S, then<br />

given ε > 0, there is a positive integer N such that as n ≥ N, we have<br />

.<br />

and<br />

|f n (x) − f (x)| < ε 2<br />

|g n (x) − g (x)| < ε 2<br />

for all x ∈ S<br />

for all x ∈ S.<br />

Hence, for this ε, we have as n ≥ N,<br />

|f n (x) + g n (x) − f (x) − g (x)| ≤ |f n (x) − f (x)| + |g n (x) − g (x)|<br />

< ε for all x ∈ S.<br />

3


That is, f n + g n → f + g uniformly on S.<br />

Remark: <strong>The</strong>re is a similar result. We write it as follows. If f n → f<br />

uniformly on S, then cf n → cf uniformly on S for any real c. Since the proof<br />

is easy, we omit the proof.<br />

(b) Let h n (x) = f n (x) g n (x) , h (x) = f (x) g (x) , if x ∈ S. Exercise 9.2<br />

shows that the assertion h n → h uniformly on S is, in general, incorrect.<br />

Prove that it is correct if each f n and each g n is bounded on S.<br />

Proof: Since f n → f uniformly on S and each f n is bounded on S, then<br />

f is bounded on S by Remark (1) in the Exercise 9.1. In addition, since<br />

g n → g uniformly on S and each g n is bounded on S, then g n is uniformly<br />

bounded on S by Exercise 9.1.<br />

Say |f (x)| ≤ M 1 for all x ∈ S, and |g n (x)| ≤ M 2 for all x and all n. <strong>The</strong>n<br />

given ε > 0, there exists a positive integer N such that as n ≥ N, we have<br />

and<br />

|f n (x) − f (x)| <<br />

|g n (x) − g (x)| <<br />

which implies that as n ≥ N, we have<br />

ε<br />

2 (M 2 + 1)<br />

ε<br />

2 (M 1 + 1)<br />

|h n (x) − h (x)| = |f n (x) g n (x) − f (x) g (x)|<br />

for all x ∈ S<br />

for all x ∈ S<br />

= |[f n (x) − f (x)] [g n (x)] + [f (x)] [g n (x) − g (x)]|<br />

≤ |f n (x) − f (x)| |g n (x)| + |f (x)| |g n (x) − g (x)|<br />

ε<br />

<<br />

2 (M 2 + 1) M ε<br />

2 + M 1<br />

2 (M 1 + 1)<br />

< ε 2 + ε 2<br />

= ε<br />

for all x ∈ S. So, h n → h uniformly on S.<br />

9.4 Assume that f n → f uniformly on S and suppose there is a constant<br />

M > 0 such that |f n (x)| ≤ M for all x in S and all n. Let g be continuous<br />

on the closure of the disk B (0; M) and define h n (x) = g [f n (x)] , h (x) =<br />

g [f (x)] , if x ∈ S. Prove that h n → h uniformly on S.<br />

4


Proof: Since g is continuous on a compact disk B (0; M) , g is uniformly<br />

continuous on B (0; M) . Given ε > 0, there exists a δ > 0 such that as<br />

|x − y| < δ, where x, y ∈ S, we have<br />

|g (x) − g (y)| < ε. (*)<br />

For this δ > 0, since f n → f uniformly on S, then there exists a positive<br />

integer N such that as n ≥ N, we have<br />

|f n (x) − f (x)| < δ for all x ∈ S. (**)<br />

Hence, by (*) and (**), we conclude that given ε > 0, there exists a positive<br />

integer N such that as n ≥ N, we have<br />

Hence, h n → h uniformly on S.<br />

|g (f n (x)) − g (f (x))| < ε for all x ∈ S.<br />

9.5 (a) Let f n (x) = 1/ (nx + 1) if 0 < x < 1, n = 1, 2, ... Prove that {f n }<br />

converges pointwise but not uniformly on (0, 1) .<br />

Proof: First, it is clear that lim n→∞ f n (x) = 0 for all x ∈ (0, 1) . Supppos<br />

that {f n } converges uniformly on (0, 1) . <strong>The</strong>n given ε = 1/2, there exists a<br />

positive integer N such that as n ≥ N, we have<br />

|f n (x) − f (x)| =<br />

1<br />

∣1 + nx∣ < 1/2 for all x ∈ (0, 1) .<br />

So, the inequality holds for all x ∈ (0, 1) . It leads us to get a contradiction<br />

since<br />

1<br />

1 + Nx < 1 2<br />

for all x ∈ (0, 1) ⇒ lim<br />

x→0 + 1<br />

1 + Nx = 1 < 1/2.<br />

That is, {f n } converges NOT uniformly on (0, 1) .<br />

(b) Let g n (x) = x/ (nx + 1) if 0 < x < 1, n = 1, 2, ... Prove that g n → 0<br />

uniformly on (0, 1) .<br />

Proof: First, it is clear that lim n→∞ g n (x) = 0 for all x ∈ (0, 1) . Given<br />

ε > 0, there exists a positive integer N such that as n ≥ N, we have<br />

1/n < ε<br />

5


which implies that<br />

∣ ∣ |g n (x) − g| =<br />

x<br />

∣∣∣ ∣1 + nx∣ = 1 ∣∣∣<br />

1<br />

+ n < 1 n < ε.<br />

x<br />

So, g n → 0 uniformly on (0, 1) .<br />

9.6 Let f n (x) = x n . <strong>The</strong> sequence {f n (x)} converges pointwise but not<br />

uniformly on [0, 1] . Let g be continuous on [0, 1] with g (1) = 0. Prove that<br />

the sequence {g (x) x n } converges uniformly on [0, 1] .<br />

Proof: It is clear that f n (x) = x n converges NOT uniformly on [0, 1]<br />

since each term of {f n (x)} is continuous on [0, 1] and its limit function<br />

{ 0 if x ∈ [0, 1)<br />

f =<br />

1 if x = 1.<br />

is not a continuous function on [0, 1] by <strong>The</strong>orem 9.2.<br />

In order to show {g (x) x n } converges uniformly on [0, 1] , it suffices to<br />

shows that {g (x) x n } converges uniformly on [0, 1). Note that<br />

lim g (x)<br />

n→∞ xn = 0 for all x ∈ [0, 1).<br />

We partition the interval [0, 1) into two subintervals: [0, 1 − δ] and (1 − δ, 1) .<br />

As x ∈ [0, 1 − δ] : Let M = max x∈[0,1] |g (x)| , then given ε > 0, there is a<br />

positive integer N such that as n ≥ N, we have<br />

M (1 − δ) n < ε<br />

which implies that for all x ∈ [0, 1 − δ] ,<br />

|g (x) x n − 0| ≤ M |x n | ≤ M (1 − δ) n < ε.<br />

Hence, {g (x) x n } converges uniformly on [0, 1 − δ] .<br />

As x ∈ (1 − δ, 1) : Since g is continuous at 1, given ε > 0, there exists a<br />

δ > 0 such that as |x − 1| < δ, where x ∈ [0, 1] , we have<br />

|g (x) − g (1)| = |g (x) − 0| = |g (x)| < ε<br />

which implies that for all x ∈ (1 − δ, 1) ,<br />

|g (x) x n − 0| ≤ |g (x)| < ε.<br />

6


Hence, {g (x) x n } converges uniformly on (1 − δ, 1) .<br />

So, from above sayings, we have proved that the sequence of functions<br />

{g (x) x n } converges uniformly on [0, 1] .<br />

Remark: It is easy to show the followings by definition. So, we omit the<br />

proof.<br />

(1) Suppose that for all x ∈ S, the limit function f exists. If f n → f<br />

uniformly on S 1 (⊆ S) , then f n → f uniformly on S, where # (S − S 1 ) <<br />

+∞.<br />

(2) Suppose that f n → f uniformly on S and on T. <strong>The</strong>n f n → f uniformly<br />

on S ∪ T.<br />

9.7 Assume that f n → f uniformly on S and each f n is continuous on S.<br />

If x ∈ S, let {x n } be a sequence of points in S such that x n → x. Prove that<br />

f n (x n ) → f (x) .<br />

Proof: Since f n → f uniformly on S and each f n is continuous on S, by<br />

<strong>The</strong>orem 9.2, the limit function f is also continuous on S. So, given ε > 0,<br />

there is a δ > 0 such that as |y − x| < δ, where y ∈ S, we have<br />

|f (y) − f (x)| < ε 2 .<br />

For this δ > 0, there exists a positive integer N 1 such that as n ≥ N 1 , we<br />

have<br />

|x n − x| < δ.<br />

Hence, as n ≥ N 1 , we have<br />

|f (x n ) − f (x)| < ε 2 . (*)<br />

In addition, since f n → f uniformly on S, given ε > 0, there exists a<br />

positive integer N ≥ N 1 such that as n ≥ N, we have<br />

|f n (x) − f (x)| < ε 2<br />

for all x ∈ S<br />

which implies that<br />

|f n (x n ) − f (x n )| < ε 2 . (**)<br />

7


By (*) and (**), we obtain that given ε > 0, there exists a positie integer N<br />

such that as n ≥ N, we have<br />

|f n (x n ) − f (x)| = |f n (x n ) − f (x n )| + |f (x n ) − f (x)|<br />

< ε 2 + ε 2<br />

= ε.<br />

That is, we have proved that f n (x n ) → f (x) .<br />

9.8 Let {f n } be a seuqnece of continuous functions defined on a compact<br />

set S and assume that {f n } converges pointwise on S to a limit function f.<br />

Prove that f n → f uniformly on S if, and only if, the following two conditions<br />

hold.:<br />

(i) <strong>The</strong> limit function f is continuous on S.<br />

(ii) For every ε > 0, there exists an m > 0 and a δ > 0, such that n > m<br />

and |f k (x) − f (x)| < δ implies |f k+n (x) − f (x)| < ε for all x in S and all<br />

k = 1, 2, ...<br />

Hint. To prove the sufficiency of (i) and (ii), show that for each x 0 in S<br />

there is a neighborhood of B (x 0 ) and an integer k (depending on x 0 ) such<br />

that<br />

|f k (x) − f (x)| < δ if x ∈ B (x 0 ) .<br />

By compactness, a finite set of integers, say A = {k 1 , ..., k r } , has the property<br />

that, for each x in S, some k in A satisfies |f k (x) − f (x)| < δ. Uniform<br />

convergence is an easy consequences of this fact.<br />

Proof: (⇒) Suppose that f n → f uniformly on S, then by <strong>The</strong>orem<br />

9.2, the limit function f is continuous on S. In addition, given ε > 0, there<br />

exists a positive integer N such that as n ≥ N, we have<br />

|f n (x) − f (x)| < ε for all x ∈ S<br />

Let m = N, and δ = ε, then (ii) holds.<br />

(⇐) Suppose that (i) and (ii) holds. We prove f k → f uniformly on S as<br />

follows. By (ii), given ε > 0, there exists an m > 0 and a δ > 0, such that<br />

n > m and |f k (x) − f (x)| < δ implies |f k+n (x) − f (x)| < ε for all x in S<br />

and all k = 1, 2, ...<br />

Consider ∣ ∣f k(x0 ) (x 0 ) − f (x 0 ) ∣ ∣ < δ, then there exists a B (x 0 ) such that as<br />

x ∈ B (x 0 ) ∩ S, we have<br />

∣ fk(x0 ) (x) − f (x) ∣ ∣ < δ<br />

8


y continuity of f k(x0 ) (x) − f (x) . Hence, by (ii) as n > m<br />

∣<br />

∣f k(x0 )+n (x) − f (x) ∣ ∣ < ε if x ∈ B (x 0 ) ∩ S. (*)<br />

Note that S is compact and S = ∪ x∈S (B (x) ∩ S) , then S = ∪ p k=1 (B (x k) ∩ S) .<br />

So, let N = max p i=1 (k (x p) + m) , as n > N, we have<br />

|f n (x) − f (x)| < ε for all x ∈ S<br />

with help of (*). That is, f n → f uniformly on S.<br />

9.9 (a) Use Exercise 9.8 to prove the following theorem of Dini: If<br />

{f n } is a sequence of real-valued continuous functions converginf<br />

pointwise to a continuous limit function f on a compact set S, and<br />

if f n (x) ≥ f n+1 (x) for each x in S and every n = 1, 2, ..., then f n → f<br />

uniformly on S.<br />

Proof: By Exercise 9.8, in order to show that f n → f uniformly on S,<br />

it suffices to show that (ii) holds. Since f n (x) → f (x) and f n+1 (x) ≤ f n (x)<br />

on S, then fixed x ∈ S, and given ε > 0, there exists a positive integer<br />

N (x) = N such that as n ≥ N, we have<br />

0 ≤ f n (x) − f (x) < ε.<br />

Choose m = 1 and δ = ε, then by f n+1 (x) ≤ f n (x) , then (ii) holds. We<br />

complete it.<br />

Remark: (1) Dini’s <strong>The</strong>orem is important in Analysis; we suggest the<br />

reader to keep it in mind.<br />

(2) <strong>The</strong>re is another proof by using Cantor Intersection <strong>The</strong>orem.<br />

We give it as follows.<br />

Proof: Let g n = f n − f, then g n is continuous on S, g n → 0 pointwise on<br />

S, and g n (x) ≥ g n+1 (x) on S. If we can show g n → 0 uniformly on S, then<br />

we have proved that f n → f uniformly on S.<br />

Given ε > 0, and consider S n := {x : g n (x) ≥ ε} . Since each g n (x) is<br />

continuous on a compact set S, we obtain that S n is compact. In addition,<br />

S n+1 ⊆ S n since g n (x) ≥ g n+1 (x) on S. <strong>The</strong>n<br />

∩S n ≠ φ (*)<br />

9


if each S n is non-empty by Cantor Intersection <strong>The</strong>orem. However (*)<br />

contradicts to g n → 0 pointwise on S. Hence, we know that there exists a<br />

positive integer N such that as n ≥ N,<br />

S n = φ.<br />

That is, given ε > 0, there exists a positive integer N such that as n ≥ N,<br />

we have<br />

|g n (x) − 0| < ε.<br />

So, g n → 0 uniformly on S.<br />

(b) Use he sequence in Exercise 9.5(a) to show that compactness of S is<br />

essential in Dini’s <strong>The</strong>orem.<br />

Proof: Let f n (x) = 1 , where x ∈ (0, 1) . <strong>The</strong>n it is clear that each<br />

1+nx<br />

f n (x) is continuous on (0, 1) , the limit function f (x) = 0 is continuous on<br />

(0, 1) , and f n+1 (x) ≤ f n (x) for all x ∈ (0, 1) . However, f n → f not uniformly<br />

on (0, 1) by Exercise 9.5 (a). Hence, compactness of S is essential in Dini’s<br />

<strong>The</strong>orem.<br />

9.10 Let f n (x) = n c x (1 − x 2 ) n for x real and n ≥ 1. Prove that {f n }<br />

converges pointwsie on [0, 1] for every real c. Determine those c for which the<br />

convergence is uniform on [0, 1] and those for which term-by-term integration<br />

on [0, 1] leads to a correct result.<br />

Proof: It is clear that f n (0) → 0 and f n (1) → 0. Consider x ∈ (0, 1) ,<br />

then |1 − x 2 | := r < 1, then<br />

lim f n (x) = lim n c r n x = 0 for any real c.<br />

n→∞ n→∞<br />

Hence, f n → 0 pointwise on [0, 1] .<br />

Consider<br />

f n ′ (x) = n ( c 1 − x 2) ( )<br />

n−1 1<br />

(2n − 1)<br />

2n − 1 − x2 ,<br />

then each f n has the absolute maximum at x n = 1 √ 2n−1<br />

.<br />

As c < 1/2, we obtain that<br />

|f n (x)| ≤ |f n (x n )|<br />

(<br />

n c<br />

= √ 1 − 1 ) n<br />

2n − 1 2n − 1<br />

[√ (<br />

= n c− 1 n<br />

2<br />

1 − 1 ) n ]<br />

2n − 1 2n − 1<br />

10<br />

→ 0 as n → ∞. (*)


In addition, as c ≥ 1/2, if f n → 0 uniformly on [0, 1] , then given ε > 0, there<br />

exists a positive integer N such that as n ≥ N, we have<br />

which implies that as n ≥ N,<br />

which contradicts to<br />

|f n (x)| < ε for all x ∈ [0, 1]<br />

|f n (x n )| < ε<br />

{<br />

lim f √ 1<br />

n (x n ) = 2e<br />

if c = 1/2<br />

n→∞ ∞ if c > 1/2 . (**)<br />

From (*) and (**), we conclude that only as c < 1/2, the seqences of<br />

functions converges uniformly on [0, 1] .<br />

In order to determine those c for which term-by-term integration on [0, 1] ,<br />

we consider ∫ 1<br />

n c<br />

f n (x) dx =<br />

2 (n + 1)<br />

and ∫ 1<br />

f (x) dx =<br />

0<br />

0<br />

∫ 1<br />

0<br />

0dx = 0.<br />

Hence, only as c < 1, we can integrate it term-by-term.<br />

9.11 Prove that ∑ x n (1 − x) converges pointwise but not uniformly on<br />

[0, 1] , whereas ∑ (−1) n x n (1 − x) converges uniformly on [0, 1] . This illustrates<br />

that uniform convergence of ∑ f n (x) along with pointwise convergence<br />

of ∑ |f n (x)| does not necessarily imply uniform convergence<br />

of ∑ |f n (x)| .<br />

Proof: Let s n (x) = ∑ n<br />

k=0 xk (1 − x) = 1 − x n+1 , then<br />

{ 1 if x ∈ [0, 1)<br />

s n (x) →<br />

.<br />

0 if x = 1<br />

Hence, ∑ x n (1 − x) converges pointwise but not uniformly on [0, 1] by <strong>The</strong>orem<br />

9.2 since each s n is continuous on [0, 1] .<br />

Let g n (x) = x n (1 − x) , then it is clear that g n (x) ≥ g n+1 (x) for all x ∈<br />

[0, 1] , and g n (x) → 0 uniformly on [0, 1] by Exercise 9.6. Hence, by Dirichlet’s<br />

Test for uniform convergence, we have proved that ∑ (−1) n x n (1 − x)<br />

converges uniformly on [0, 1] .<br />

11


9.12 Assume that g n+1 (x) ≤ g n (x) for each x in T and each n = 1, 2, ...,<br />

and suppose that g n → 0 uniformly on T. Prove that ∑ (−1) n+1 g n (x) converges<br />

uniformly on T.<br />

Proof: It is clear by Dirichlet’s Test for uniform convergence.<br />

9.13 Prove Abel’s test for uniform convergence: Let {g n } be a sequence<br />

of real-valued functions such that g n+1 (x) ≤ g n (x) for each x in T and for<br />

every n = 1, 2, ... If {g n } is uniformly bounded on T and if ∑ f n (x) converges<br />

uniformly on T, then ∑ f n (x) g n (x) also converges uniformly on T.<br />

Proof: Let F n (x) = ∑ n<br />

k=1 f k (x) . <strong>The</strong>n<br />

s n (x) =<br />

n∑<br />

f k (x) g k (x) = F n g 1 (x)+<br />

k=1<br />

and hence if n > m, we can write<br />

n∑<br />

(F n (x) − F k (x)) (g k+1 (x) − g k (x))<br />

k=1<br />

s n (x)−s m (x) = (F n (x) − F m (x)) g m+1 (x)+<br />

n∑<br />

k=m+1<br />

Hence, if M is an uniform bound for {g n } , we have<br />

|s n (x) − s m (x)| ≤ M |F n (x) − F m (x)| + 2M<br />

(F n (x) − F k (x)) (g k+1 (x) − g k (x))<br />

n∑<br />

k=m+1<br />

|F n (x) − F k (x)| . (*)<br />

Since ∑ f n (x) converges uniformly on T, given ε > 0, there exists a positive<br />

integer N such that as n > m ≥ N, we have<br />

|F n (x) − F m (x)| < ε<br />

M + 1<br />

By (*) and (**), we have proved that as n > m ≥ N,<br />

|s n (x) − s m (x)| < ε for all x ∈ T.<br />

Hence, ∑ f n (x) g n (x) also converges uniformly on T.<br />

for all x ∈ T (**)<br />

Remark: In the proof, we establish the lemma as follows. We write it<br />

as a reference.<br />

12


(Lemma) If {a n } and {b n } are two sequences of complex numbers, define<br />

A n =<br />

n∑<br />

a k .<br />

k=1<br />

<strong>The</strong>n we have the identity<br />

n∑<br />

n∑<br />

a k b k = A n b n+1 − A k (b k+1 − b k )<br />

(i)<br />

k=1<br />

= A n b 1 +<br />

k=1<br />

n∑<br />

(A n − A k ) (b k+1 − b k ) . (ii)<br />

k=1<br />

Proof: <strong>The</strong> identity (i) comes from <strong>The</strong>orem 8.27. In order to show<br />

(ii), it suffices to consider<br />

b n+1 = b 1 +<br />

n∑<br />

b k+1 − b k .<br />

k=1<br />

9.14 Let f n (x) = x/ (1 + nx 2 ) if x ∈ R, n = 1, 2, ... Find the limit function<br />

f of the sequence {f n } and the limit function g of the sequence {f ′ n} .<br />

(a) Prove that f ′ (x) exists for every x but that f ′ (0) ≠ g (0) . For what<br />

values of x is f ′ (x) = g (x)<br />

Proof: It is easy to show that the limit function f = 0, and by f ′ n (x) =<br />

1−nx 2<br />

(1+nx 2 ) 2 , we have<br />

{ 1 if x = 0<br />

lim f n ′ (x) = g (x) =<br />

n→∞ 0 if x ≠ 0 .<br />

Hence, f ′ (x) exists for every x and f ′ (0) = 0 ≠ g (0) = 1. In addition, it is<br />

clear that as x ≠ 0, we have f ′ (x) = g (x) .<br />

(b) In what subintervals of R does f n → f uniformly<br />

Proof: Note that<br />

1 + nx 2<br />

2<br />

≥ √ n |x|<br />

13


y A.P. ≥ G.P. for all real x. Hence,<br />

∣ x ∣∣∣<br />

∣ ≤ 1<br />

1 + nx 2 2 √ n<br />

which implies that f n → f uniformly on R.<br />

(c) In what subintervals of R does f ′ n → g uniformly<br />

Proof: Since each f n ′ =<br />

1−nx2 is continuous on R, and the limit function<br />

(1+nx 2 ) 2<br />

g is continuous on R − {0} , then by <strong>The</strong>orem 9.2, the interval I that we<br />

consider does not contains 0. Claim that f n ′ → g uniformly on such interval<br />

I = [a, b] which does not contain 0 as follows.<br />

Consider ∣ ∣ ∣∣∣ 1 − nx 2 ∣∣∣ 1<br />

(1 + nx 2 ) 2 ≤<br />

1 + nx ≤ 1<br />

2 na , 2<br />

so we know that f ′ n → g uniformly on such interval I = [a, b] which does not<br />

contain 0.<br />

9.15 Let f n (x) = (1/n) e −n2 x 2 if x ∈ R, n = 1, 2, ... Prove that f n → 0<br />

uniformly on R, that f ′ n → 0 pointwise on R, but that the convergence of<br />

{f ′ n} is not uniform on any interval containing the origin.<br />

Proof: It is clear that f n → 0 uniformly on R, that f n ′ → 0 pointwise<br />

on R. Assume that f n ′ → 0 uniformly on [a, b] that contains 0. We will prove<br />

that it is impossible as follows.<br />

We may assume that 0 ∈ (a, b) since other cases are similar. Given ε = 1,<br />

e<br />

then there exists a positive integer N ′ such that as n ≥ max ( )<br />

N ′ , 1 b := N<br />

(⇒ 1 ≤ b), we have N<br />

|f n ′ (x) − 0| < 1 for all x ∈ [a, b]<br />

e<br />

which implies that<br />

∣ ∣∣∣<br />

2 Nx<br />

e (Nx)2 ∣ ∣∣∣<br />

< 1 e<br />

for all x ∈ [a, b]<br />

which implies that, let x = 1 N , 2<br />

e < 1 e<br />

which is absurb. So, the convergence of {f ′ n} is not uniform on any interval<br />

containing the origin.<br />

14


9.16 Let {f n } be a sequence of real-valued continuous functions defined<br />

on [0, 1] and assume that f n → f uniformly on [0, 1] . Prove or disprove<br />

∫ 1−1/n<br />

lim<br />

n→∞<br />

0<br />

f n (x) dx =<br />

∫ 1<br />

0<br />

f (x) dx.<br />

Proof: By <strong>The</strong>orem 9.8, we have<br />

∫ 1<br />

lim<br />

n→∞<br />

0<br />

f n (x) dx =<br />

∫ 1<br />

0<br />

f (x) dx. (*)<br />

Note that {f n } is uniform bound, say |f n (x)| ≤ M for all x ∈ [0, 1] and all<br />

n by Exercise 9.1. Hence,<br />

∫ 1<br />

∣ f n (x) dx<br />

∣ ≤ M → 0. (**)<br />

n<br />

1−1/n<br />

Hence, by (*) and (**), we have<br />

∫ 1−1/n<br />

lim<br />

n→∞<br />

0<br />

f n (x) dx =<br />

∫ 1<br />

0<br />

f (x) dx.<br />

9.17 Mathematicinas from Slobbovia decided that the Riemann integral<br />

was too complicated so that they replaced it by Slobbovian integral, defined<br />

as follows: If f is a function defined on the set Q of rational numbers<br />

in [0, 1] , the Slobbovian integral of f, denoted by S (f) , is defined to be the<br />

limit<br />

1<br />

S (f) = lim<br />

n→∞ n<br />

n∑<br />

f<br />

k=1<br />

( k<br />

n)<br />

,<br />

whenever the limit exists. Let {f n } be a sequence of functions such that<br />

S (f n ) exists for each n and such that f n → f uniformly on Q. Prove that<br />

{S (f n )} converges, that S (f) exists, and S (f n ) → S (f) as n → ∞.<br />

Proof: f n → f uniformly on Q, then given ε > 0, there exists a positive<br />

integer N such that as n > m ≥ N, we have<br />

|f n (x) − f (x)| < ε/3 (1)<br />

15


and<br />

So, if n > m ≥ N,<br />

|S (f n ) − S (f m )| =<br />

∣ lim 1<br />

k∑<br />

k→∞ k<br />

j=1<br />

1<br />

k∑<br />

= lim<br />

k→∞ k ∣<br />

|f n (x) − f m (x)| < ε/2. (2)<br />

1<br />

≤ lim<br />

k→∞ k<br />

= ε/2<br />

< ε<br />

j=1<br />

( ( ) ( )) ∣ j j ∣∣∣<br />

f n − f m<br />

k k<br />

( ( ) ( )) ∣ j j ∣∣∣<br />

f n − f m<br />

k k<br />

k∑<br />

ε/2 by (2)<br />

j=1<br />

which implies that {S (f n )} converges since it is a Cauchy sequence. Say its<br />

limit S.<br />

Consider, by (1) as n ≥ N,<br />

1<br />

k<br />

k∑<br />

j=1<br />

[ ( ) ] j<br />

f n − ε/3 ≤ 1 k<br />

k<br />

k∑<br />

( ) j<br />

f ≤ 1 k k<br />

j=1<br />

k∑<br />

j=1<br />

[ ( ) ] j<br />

f n + ε/3<br />

k<br />

which implies that<br />

[<br />

1<br />

k∑<br />

( ) ] j<br />

f n − ε/3 ≤ 1 k k<br />

k<br />

j=1<br />

k∑<br />

( ) j<br />

f ≤<br />

k<br />

j=1<br />

[<br />

1<br />

k<br />

k∑<br />

( ) ] j<br />

f n + ε/3<br />

k<br />

j=1<br />

which implies that, let k → ∞<br />

S (f n ) − ε/3 ≤ lim<br />

k→∞<br />

sup 1 k<br />

k∑<br />

( ) j<br />

f ≤ S (f n ) + ε/3 (3)<br />

k<br />

j=1<br />

and<br />

S (f n ) − ε/3 ≤ lim<br />

k→∞<br />

inf 1 k<br />

k∑<br />

( ) j<br />

f ≤ S (f n ) + ε/3 (4)<br />

k<br />

j=1<br />

16


which implies that<br />

∣ lim sup 1 k∑<br />

( ) j<br />

f − lim inf 1 k∑<br />

( ) ∣ j ∣∣∣<br />

f<br />

k→∞ k k k→∞ k k<br />

j=1<br />

j=1<br />

≤<br />

∣ lim sup 1 k∑<br />

( )<br />

∣<br />

j ∣∣∣∣ f − S (f n )<br />

k→∞ k k ∣ + lim inf 1<br />

k→∞ k<br />

j=1<br />

k∑<br />

( )<br />

j f − S (f n )<br />

k ∣<br />

≤ 2ε by (3) and (4)<br />

3<br />

< ε. (5)<br />

Note that (3)-(5) imply that the existence of S (f) . Also, (3) or (4) implies<br />

that S (f) = S. So, we complete the proof.<br />

9.18 Let f n (x) = 1/ (1 + n 2 x 2 ) if 0 ≤ x ≤ 1, n = 1, 2, ... Prove that {f n }<br />

converges pointwise but not uniformly on [0, 1] . Is term-by term integration<br />

permissible<br />

Proof: It is clear that<br />

lim f n (x) = 0<br />

n→∞<br />

for all x ∈ [0, 1] . If {f n } converges uniformly on [0, 1] , then given ε = 1/3,<br />

there exists a positive integer N such that as n ≥ N, we have<br />

which implies that<br />

|f n (x)| < 1/3 for all x ∈ [0, 1]<br />

∣ ∣∣∣<br />

f N<br />

( 1<br />

N<br />

)∣ ∣∣∣<br />

= 1 2 < 1 3<br />

j=1<br />

which is impossible. So, {f n } converges pointwise but not uniformly on [0, 1] .<br />

Since {f n (x)} is clearly uniformly bounded on [0, 1] , i.e., |f n (x)| ≤ 1<br />

for all x ∈ [0, 1] and n. Hence, by Arzela’s <strong>The</strong>orem, we know that the<br />

sequence of functions can be integrated term by term.<br />

9.19 Prove that ∑ ∞<br />

n=1 x/nα (1 + nx 2 ) converges uniformly on every finite<br />

interval in R if α > 1/2. Is the convergence uniform on R<br />

Proof: By A.P. ≥ G.P., we have<br />

x<br />

∣n α (1 + nx 2 ) ∣ ≤ 1<br />

17<br />

2n α+ 1 2<br />

for all x.


So, by Weierstrass M-test, we have proved that ∑ ∞<br />

n=1 x/nα (1 + nx 2 ) converges<br />

uniformly on R if α > 1/2. Hence, ∑ ∞<br />

n=1 x/nα (1 + nx 2 ) converges<br />

uniformly on every finite interval in R if α > 1/2.<br />

9.20 Prove that the series ∑ ∞<br />

n=1 ((−1)n / √ n) sin (1 + (x/n)) converges<br />

uniformly on every compact subset of R.<br />

Proof: It suffices to show that the series ∑ ∞<br />

n=1 ((−1)n / √ n) sin (1 + (x/n))<br />

converges uniformly on [0, a] . Choose n large enough so that a/n ≤ 1/2, and<br />

therefore sin ( 1 + ( )) (<br />

x<br />

n+1 ≤ sin 1 +<br />

x<br />

n)<br />

for all x ∈ [0, a] . So, if we let fn (x) =<br />

(−1) n / √ n and g n (x) = sin ( 1 + n) x , then by Abel’s test for uniform convergence,<br />

we have proved that the series ∑ ∞<br />

n=1 ((−1)n / √ n) sin (1 + (x/n))<br />

converges uniformly on [0, a] .<br />

Remark: In the proof, we metion something to make the reader get<br />

more. (1) since a compact set K is a bounded set, say K ⊆ [−a, a] , if we can<br />

show the series converges uniformly on [−a, a] , then we have proved it. (2)<br />

<strong>The</strong> interval that we consider is [0, a] since [−a, 0] is similar. (3) Abel’s test<br />

for uniform convergence holds for n ≥ N, where N is a fixed positive<br />

integer.<br />

9.21 Prove that the series ∑ ∞<br />

n=0 (x2n+1 / (2n + 1) − x n+1 / (2n + 2)) converges<br />

pointwise but not uniformly on [0, 1] .<br />

Proof: We show that the series converges pointwise on [0, 1] by considering<br />

two cases: (1) x ∈ [0, 1) and (2) x = 1. Hence, it is trivial. Define<br />

f (x) = ∑ ∞<br />

n=0 (x2n+1 / (2n + 1) − x n+1 / (2n + 2)) , if the series converges<br />

uniformly on [0, 1] , then by <strong>The</strong>orem 9.2, f (x) is continuous on [0, 1] .<br />

However,<br />

f (x) =<br />

{ 1<br />

2<br />

log (1 + x) if x ∈ [0, 1)<br />

log 2 if x = 1<br />

Hence, the series converges not uniformly on [0, 1] .<br />

Remark: <strong>The</strong> function f (x) is found by the following. Given x ∈ [0, 1),<br />

then both<br />

∞∑<br />

t 2n = 1<br />

1 − t and 1 ∞∑<br />

t n 1<br />

=<br />

2 2 2 (1 − t)<br />

n=0<br />

converges uniformly on [0, x] by <strong>The</strong>orem 9.14. So, by <strong>The</strong>orem 9.8, we<br />

n=0<br />

.<br />

18


have<br />

∫ x<br />

0<br />

∞∑<br />

t 2n − 1 2<br />

n=0<br />

∞∑<br />

t n =<br />

n=0<br />

=<br />

∫ x<br />

0<br />

∫ x<br />

0<br />

1<br />

1 − t − 1<br />

2 2 (1 − t) dt<br />

(<br />

1 1<br />

2 1 − t + 1 )<br />

1 + t<br />

= 1 log (1 + x) .<br />

2<br />

− 1 2<br />

( ) 1<br />

dt<br />

1 − t<br />

<strong>And</strong> as x = 1,<br />

∞∑ (<br />

x 2n+1 / (2n + 1) − x n+1 / (2n + 2) )<br />

n=0<br />

=<br />

=<br />

∞∑<br />

n=0<br />

∞∑<br />

n=0<br />

1<br />

2n + 1 − 1<br />

2n<br />

(−1) n+1<br />

n + 1<br />

by <strong>The</strong>orem8.14.<br />

= log 2 by Abel’s Limit <strong>The</strong>orem.<br />

9.22 Prove that ∑ a n sin nx and ∑ b n cos nx are uniformly convergent on<br />

R if ∑ |a n | converges.<br />

Proof: It is trivial by Weierstrass M-test.<br />

9.23 Let {a n } be a decreasing sequence of positive terms. Prove that<br />

the series ∑ a n sin nx converges uniformly on R if, and only if, na n → 0 as<br />

n → ∞.<br />

Proof: (⇒) Suppose that the series ∑ a n sin nx converges uniformly on<br />

R, then given ε > 0, there exists a positive integer N such that as n ≥ N,<br />

we have<br />

∣ ∣∣∣∣ 2n−1<br />

∑<br />

a k sin kx<br />

< ε. (*)<br />

∣<br />

Choose x = 1<br />

2n , then sin 1 2<br />

k=n<br />

≤ sin kx ≤ sin 1. Hence, as n ≥ N, we always<br />

19


have, by (*)<br />

2n−1<br />

∑<br />

2n−1<br />

(ε >)<br />

a k sin kx<br />

∣<br />

∣ = ∑<br />

a k sin kx<br />

k=n<br />

≥<br />

=<br />

k=n<br />

2n−1<br />

∑<br />

a 2n sin 1 2 since a k > 0 and a k ↘<br />

k=n<br />

( 1<br />

2 sin 1 2)<br />

(2na 2n ) .<br />

That is, we have proved that 2na 2n → 0 as n → ∞. Similarly, we also have<br />

(2n − 1) a 2n−1 → 0 as n → ∞. So, we have proved that na n → 0 as n → ∞.<br />

(⇐) Suppose that na n → 0 as n → ∞, then given ε > 0, there exists a<br />

positive integer n 0 such that as n ≥ n 0 , we have<br />

ε<br />

|na n | = na n <<br />

2 (π + 1) . (*)<br />

In order to show the uniform convergence of ∑ ∞<br />

n=1 a n sin nx on R, it suffices<br />

to show the uniform convergence of ∑ ∞<br />

n=1 a n sin nx on [0, π] . So, if we can<br />

show that as n ≥ n 0<br />

n+p<br />

∑<br />

a k sin kx<br />

< ε for all x ∈ [0, π] , and all p ∈ N<br />

∣<br />

∣<br />

k=n+1<br />

then we complete [ ] it. We consider two cases as follows. (n ≥ n 0 )<br />

π<br />

As x ∈ 0, , then<br />

n+p<br />

∣<br />

∣<br />

∣<br />

n+p<br />

∑<br />

k=n+1<br />

n+p<br />

a k sin kx<br />

∣ = ∑<br />

≤<br />

=<br />

k=n+1<br />

n+p<br />

∑<br />

k=n+1<br />

n+p<br />

∑<br />

k=n+1<br />

a k sin kx<br />

a k kx by sin kx ≤ kx if x ≥ 0<br />

(ka k ) x<br />

ε pπ<br />

≤<br />

by (*)<br />

2 (π + 1) n + p<br />

< ε.<br />

20


[ ]<br />

π<br />

<strong>And</strong> as x ∈ , π , then<br />

n+p<br />

∣<br />

n+p<br />

∑<br />

k=n+1<br />

a k sin kx<br />

∣ ≤<br />

≤<br />

≤<br />

≤<br />

≤<br />

m<br />

∑<br />

k=n+1<br />

a k sin kx +<br />

∣<br />

n+p<br />

∑<br />

k=m+1<br />

[ π<br />

]<br />

a k sin kx<br />

∣ , where m = x<br />

m∑<br />

a k kx + 2a m+1<br />

sin x by Summation by parts<br />

k=n+1<br />

2<br />

ε<br />

2 (π + 1) (m − n) x + 2a m+1<br />

sin x 2<br />

ε<br />

π<br />

2 (π + 1) mx + 2a m+1<br />

x by 2x [0,<br />

π ≤ sin x if x ∈ π 2<br />

ε<br />

2 (π + 1) π + 2a m+1 (m + 1)<br />

< ε 2 + 2 ε<br />

2 (π + 1)<br />

< ε.<br />

Hence, ∑ ∞<br />

n=1 a n sin nx converges uniformly on R.<br />

Remark: (1) In the proof (⇐), if we can make sure that na n ↘ 0, then<br />

we can use the supplement on the convergnce of series in Ch8, (C)-<br />

(6) to show the uniform convergence of ∑ ∞<br />

n=1 a n sin nx = ∑ ∞<br />

n=1 (na n) ( )<br />

sin nx<br />

n<br />

by Dirichlet’s test for uniform convergence.<br />

(2)<strong>The</strong>re are similar results; we write it as references.<br />

(a) Suppose a n ↘ 0, then for each α ∈ ( 0, π 2<br />

)<br />

,<br />

∑ ∞<br />

n=1 a n cos nx and<br />

∑ ∞<br />

n=1 a n sin nx converges uniformly on [α, 2π − α] .<br />

Proof: <strong>The</strong> proof follows from (12) and (13) in <strong>The</strong>orem 8.30 and<br />

Dirichlet’s test for uniform convergence. So, we omit it. <strong>The</strong> reader<br />

can see the textbook, example in pp 231.<br />

(b) Let {a n } be a decreasing sequence of positive terms. ∑ ∞<br />

n=1 a n cos nx<br />

uniformly converges on R if and only if ∑ ∞<br />

n=1 a n converges.<br />

Proof: (⇒) Suppose that ∑ ∞<br />

n=1 a n cos nx uniformly converges on R, then<br />

let x = 0, then we have ∑ ∞<br />

n=1 a n converges.<br />

(⇐) Suppose that ∑ ∞<br />

n=1 a n converges, then by Weierstrass M-test, we<br />

have proved that ∑ ∞<br />

n=1 a n cos nx uniformly converges on R.<br />

]<br />

21


9.24 Given a convergent series ∑ ∞<br />

∑ n=1 a n. Prove that the Dirichlet series<br />

∞<br />

n=1 a nn −s converges uniformly<br />

∑<br />

on the half-infinite interval 0 ≤ s < +∞.<br />

Use this to prove that lim ∞<br />

s→0 +<br />

n=1 a nn −s = ∑ ∞<br />

n=1 a n.<br />

Proof: Let f n (s) = ∑ n<br />

k=1 a k and g n (s) = n −s , then by Abel’s test for<br />

uniform convergence, we have proved that the Dirichlet series ∑ ∞<br />

n=1 a nn −s<br />

converges uniformly on the half-infinite<br />

∑<br />

interval 0 ≤ s < +∞. <strong>The</strong>n by<br />

<strong>The</strong>orem 9.2, we know that lim ∞<br />

s→0 +<br />

n=1 a nn −s = ∑ ∞<br />

n=1 a n.<br />

9.25 Prove that the series ζ (s) = ∑ ∞<br />

n=1 n−s converges uniformly on every<br />

half-infinite interval 1 + h ≤ s < +∞, where h > 0. Show that the equation<br />

ζ ′ (s) = −<br />

∞∑<br />

n=1<br />

log n<br />

n s<br />

is valid for each s > 1 and obtain a similar formula for the kth derivative<br />

ζ (k) (s) .<br />

Proof: Since n −s ≤ n −(1+h) for all s ∈ [1 + h, ∞), we know that ζ (s) =<br />

∑ ∞<br />

n=1 n−s converges uniformly on every half-infinite interval 1+h ≤ s < +∞<br />

by Weierstrass M-test. Define T n (s) = ∑ n<br />

k=1 k−s , then it is clear that<br />

<strong>And</strong><br />

1. For each n, T n (s) is differentiable on [1 + h, ∞),<br />

3. T ′ n (s) = −<br />

n∑<br />

k=1<br />

2. lim<br />

n→∞<br />

T n (2) = π2<br />

6 .<br />

log k<br />

k s converges uniformly on [1 + h, ∞)<br />

by Weierstrass M-test. Hence, we have proved that<br />

ζ ′ (s) = −<br />

∞∑<br />

n=1<br />

log n<br />

n s<br />

by <strong>The</strong>orem 9.13. By Mathematical Induction, we know that<br />

ζ (k) (s) = (−1) k<br />

∞<br />

∑<br />

n=1<br />

(log n) k<br />

n s .<br />

22


0.1 Supplement on some results on Weierstrass M-<br />

test.<br />

1. In the textbook, pp 224-223, there is a surprising result called Spacefilling<br />

curve. In addition, note the proof is related with Cantor set in<br />

exercise 7. 32 in the textbook.<br />

2. <strong>The</strong>re exists a continuous function defined on R which is nowhere<br />

differentiable. <strong>The</strong> reader can see the book, Principles of Mathematical<br />

Analysis by Walter Rudin, pp 154.<br />

Remark: <strong>The</strong> first example comes from Bolzano in 1834, however, he<br />

did NOT give a proof. In fact, he only found the function f : D → R that<br />

he constructed is not differentiable on D ′ (⊆ D) where D ′ is countable and<br />

dense in D. Although the function f is the example, but he did not find the<br />

fact.<br />

In 1861, Riemann gave<br />

g (x) =<br />

∞∑<br />

n=1<br />

sin (n 2 πx)<br />

n 2<br />

as an example. However, Reimann did NOT give a proof in his life until<br />

1916, the proof is given by G. Hardy.<br />

In 1860, Weierstrass gave<br />

h (x) =<br />

∞∑<br />

a n cos (b n πx) , b is odd, 0 < a < 1, and ab > 1 + 3π 2 ,<br />

n=1<br />

until 1875, he gave the proof. <strong>The</strong> fact surprises the world of Math, and<br />

produces many examples. <strong>The</strong>re are many researches related with it until<br />

now 2003.<br />

Mean Convergence<br />

9.26 Let f n (x) = n 3/2 xe −n2 x 2 . Prove that {f n } converges pointwise to 0<br />

on [−1, 1] but that l.i.m. n→∞ f n ≠ 0 on [−1, 1] .<br />

Proof: It is clear that {f n } converges pointwise to 0 on [−1, 1] , so it<br />

23


emains to show that l.i.m. n→∞ f n ≠ 0 on [−1, 1] . Consider<br />

∫ 1<br />

−1<br />

f 2 n (x) dx = 2<br />

∫ 1<br />

0<br />

n 3 x 2 e −2n2 x 2 dx since f 2 n (x) is an even function on [−1, 1]<br />

= √ 1 ∫ √ 2n<br />

y 2 e −y2 dy by Change of Variable, let y = √ 2nx<br />

2<br />

= 1<br />

−2 √ 2<br />

0<br />

∫ √ 2n<br />

0<br />

yd<br />

(e −y2)<br />

[<br />

= 1 ∣√ ∣∣ 2n<br />

∫ √ ]<br />

2n<br />

−2 √ ye −y2 − e −y2 dy<br />

2<br />

0 0<br />

√ ∫ π<br />

∞<br />

√ π<br />

→<br />

4 √ 2 since e −x2 dx = by Exercise 7. 19.<br />

2<br />

0<br />

So, l.i.m. n→∞ f n ≠ 0 on [−1, 1] .<br />

9.27 Assume that {f n } converges pointwise to f on [a, b] and that<br />

l.i.m. n→∞ f n = g on [a, b] . Prove that f = g if both f and g are continuous<br />

on [a, b] .<br />

Proof: Since l.i.m. n→∞ f n = g on [a, b] , given ε k = 1<br />

2 k , there exists a n k<br />

such that<br />

Define<br />

then<br />

∫ b<br />

a<br />

|f nk (x) − g (x)| p dx ≤ 1 2 k , where p > 0<br />

h m (x) =<br />

m∑<br />

k=1<br />

∫ x<br />

a. h m (x) ↗ as x ↗<br />

b. h m (x) ≤ h m+1 (x)<br />

a<br />

|f nk (t) − g (t)| p dt,<br />

c. h m (x) ≤ 1 for all m and all x.<br />

So, we obtain h m (x) → h (x) as m → ∞, h (x) ↗ as x ↗, and<br />

h (x) − h m (x) =<br />

∞∑<br />

k=m+1<br />

∫ x<br />

a<br />

|f nk (t) − g (t)| p dt ↗ as x ↗<br />

24


which implies that<br />

h (x + t) − h (x)<br />

t<br />

≥ h m (x + t) − h m (x)<br />

t<br />

for all m. (*)<br />

Since h and h m are increasing, we have h ′ and h ′ m exists a.e. on [a, b] . Hence,<br />

by (*)<br />

m∑<br />

h ′ m (x) = |f nk (t) − g (t)| p ≤ h ′ (x) a.e. on [a, b]<br />

k=1<br />

which implies that<br />

∞∑<br />

|f nk (t) − g (t)| p exists a.e. on [a, b] .<br />

k=1<br />

So, f nk (t) → g (t) a.e. on [a, b] . In addition, f n → f on [a, b] . <strong>The</strong>n we<br />

conclude that f = g a.e. on [a, b] . Since f and g are continuous on [a, b] , we<br />

have<br />

∫ b<br />

a<br />

|f − g| dx = 0<br />

which implies that f = g on [a, b] . In particular, as p = 2, we have f = g.<br />

Remark: (1) A property is said to hold almost everywhere on a set<br />

S (written: a.e. on S) if it holds everywhere on S except for a set of measurer<br />

zero. Also, see the textbook, pp 254.<br />

(2) In this proof, we use the theorem which states: A monotonic function<br />

h defined on [a, b] , then h is differentiable a.e. on [a, b] . <strong>The</strong> reader can<br />

see the book, <strong>The</strong> reader can see the book, Measure and Integral (An<br />

Introduction to <strong>Real</strong> Analysis) written by Richard L. Wheeden and<br />

Antoni Zygmund, pp 113.<br />

(3) <strong>The</strong>re is another proof by using Fatou’s lemma: Let {f k } be a<br />

measruable function defined on a measure set E. If f k ≥ φ a.e. on E and<br />

φ ∈ L (E) , then ∫<br />

∫<br />

lim k ≤ lim inf f k .<br />

E<br />

k→∞ k→∞<br />

E<br />

Proof: It suffices to show that f nk (t) → g (t) a.e. on [a, b] . Since<br />

l.i.m. n→∞ f n = g on [a, b] , and given ε > 0, there exists a n k such that<br />

∫ b<br />

a<br />

|f nk − g| 2 dx < 1 2 k<br />

25


which implies that<br />

∫ b m∑<br />

a<br />

k=1<br />

|f nk − g| 2 dx <<br />

which implies that, by Fatou’s lemma,<br />

m∑<br />

k=1<br />

1<br />

2 k<br />

∫ b<br />

a<br />

lim inf ∑ m ∫ b m∑<br />

|f nk − g| 2 dx ≤ lim inf |f nk − g| 2 dx<br />

m→∞ m→∞<br />

k=1<br />

a<br />

k=1<br />

∞∑<br />

∫ b<br />

= |f nk − g| 2 dx < 1.<br />

k=1<br />

a<br />

That is,<br />

which implies that<br />

∫ b ∞∑<br />

a<br />

k=1<br />

|f nk − g| 2 dx < 1<br />

∞∑<br />

|f nk − g| 2 < ∞ a.e. on [a, b]<br />

k=1<br />

which implies that f nk → g a.e. on [a, b] .<br />

Note: <strong>The</strong> reader can see the book, Measure and Integral (An Introduction<br />

to <strong>Real</strong> Analysis) written by Richard L. Wheeden and<br />

Antoni Zygmund, pp 75.<br />

(4) <strong>The</strong>re is another proof by using Egorov’s <strong>The</strong>orem: Let {f k } be a<br />

measurable functions defined on a finite measurable set E with finite limit<br />

function f. <strong>The</strong>n given ε > 0, there exists a closed set F (⊆ E) , where<br />

|E − F | < ε such that<br />

f k → f uniformly on F.<br />

Proof: If f ≠ g on [a, b] , then h := |f − g| ≠ 0 on [a, b] . By continuity<br />

of h, there exists a compact subinterval [c, d] such that |f − g| ≠ 0. So, there<br />

exists m > 0 such that h = |f − g| ≥ m > 0 on [c, d] . Since<br />

∫ b<br />

a<br />

|f n − g| 2 dx → 0 as n → ∞,<br />

26


we have<br />

∫ d<br />

c<br />

|f n − g| 2 dx → 0 as n → ∞.<br />

then by Egorov’s <strong>The</strong>orem, given ε > 0, there exists a closed susbet F of<br />

[c, d] , where |[c, d] − F | < ε such that<br />

which implies that<br />

f n → f uniformly on F<br />

0 = lim |f n − g|<br />

n→∞<br />

∫F<br />

2 dx<br />

∫<br />

= lim |f n − g| 2 dx<br />

F<br />

n→∞<br />

∫<br />

= |f − g| 2 dx ≥ m 2 |F |<br />

F<br />

which implies that |F | = 0. If we choose ε < d−c, then we get a contradiction.<br />

<strong>The</strong>refore, f = g on [a, b] .<br />

Note: <strong>The</strong> reader can see the book, Measure and Integral (An Introduction<br />

to <strong>Real</strong> Analysis) written by Richard L. Wheeden and<br />

Antoni Zygmund, pp 57.<br />

9.28 Let f n (x) = cos n x if 0 ≤ x ≤ π.<br />

(a) Prove that l.i.m. n→∞ f n<br />

converge.<br />

= 0 on [0, π] but that {f n (π)} does not<br />

Proof: It is clear that {f n (π)} does not converge since f n (π) = (−1) n .<br />

It remains to show that l.i.m. n→∞ f n = 0 on [0, π] . Consider cos 2n x := g n (x)<br />

on [0, π] , then it is clear that {g n (x)} is boundedly convergent with limit<br />

function<br />

{ 0 if x ∈ (0, π)<br />

g =<br />

1 if x = 0 or π .<br />

Hence, by Arzela’s <strong>The</strong>orem,<br />

∫ π<br />

lim<br />

n→∞<br />

0<br />

So, l.i.m. n→∞ f n = 0 on [0, π] .<br />

cos 2n xdx =<br />

27<br />

∫ π<br />

0<br />

g (x) dx = 0.


(b) Prove that {f n } converges pointwise but not uniformly on [0, π/2] .<br />

Proof: Note that each f n (x) is continuous on [0, π/2] , and the limit<br />

function<br />

{ 0 if x ∈ (0, π/2]<br />

f =<br />

.<br />

1 if x = 0<br />

Hence, by <strong>The</strong>orem9.2, we know that {f n } converges pointwise but not<br />

uniformly on [0, π/2] .<br />

9.29 Let f n (x) = 0 if 0 ≤ x ≤ 1/n or 2/n ≤ x ≤ 1, and let f n (x) = n if<br />

1/n < x < 2/n. Prove that {f n } converges pointwise to 0 on [0, 1] but that<br />

l.i.m. n→∞ f n ≠ 0 on [0, 1] .<br />

Proof: It is clear that {f n } converges pointwise to 0 on [0, 1] . In order<br />

to show that l.i.m. n→∞ f n ≠ 0 on [0, 1] , it suffices to note that<br />

∫ 1<br />

Hence, l.i.m. n→∞ f n ≠ 0 on [0, 1] .<br />

Power series<br />

0<br />

f n (x) dx = 1 for all n.<br />

9.30 If r is the radius of convergence if ∑ a n (z − z 0 ) n , where each a n ≠ 0,<br />

show that<br />

∣ ∣ ∣ ∣∣∣<br />

lim inf a n ∣∣∣ ≤ r ≤ lim sup<br />

a n ∣∣∣<br />

n→∞ a n+1 n→∞ ∣ .<br />

a n+1<br />

Proof: By Exercise 8.4, we have<br />

1<br />

∣ lim n→∞ sup ∣ a n+1 ∣∣<br />

≤ r =<br />

a n<br />

1<br />

lim n→∞ sup |a n | 1 n<br />

≤<br />

1<br />

lim n→∞ inf<br />

∣ ∣ a n+1 ∣∣<br />

.<br />

a n<br />

Since<br />

and<br />

1<br />

∣<br />

lim n→∞ sup<br />

1<br />

lim n→∞ inf<br />

∣<br />

∣ a n+1<br />

a n<br />

∣ a n+1<br />

a n<br />

∣ ∣∣<br />

= lim<br />

n→∞<br />

inf<br />

∣ ∣∣<br />

= lim<br />

n→∞<br />

sup<br />

∣ a n ∣∣∣<br />

∣a n+1<br />

∣ a n ∣∣∣<br />

∣ ,<br />

a n+1<br />

28


we complete it.<br />

9.31 Given that two power series ∑ a n z n has radius of convergence 2.<br />

Find the radius convergence of each of the following series: In (a) and (b), k<br />

is a fixed positive integer.<br />

(a) ∑ ∞<br />

n=0 ak nz n<br />

Proof: Since<br />

2 =<br />

we know that the radius of ∑ ∞<br />

n=0 ak nz n is<br />

1<br />

, (*)<br />

1/n<br />

lim n→∞ sup |a n |<br />

1<br />

lim n→∞ sup |a k n| 1/n = 1<br />

(lim n→∞ sup |a n | 1/n) k = 2k .<br />

(b) ∑ ∞<br />

n=0 a nz kn<br />

Proof: Consider<br />

which implies that<br />

lim sup ∣ an z kn∣ ∣ 1/n = lim sup |a n | 1/n |z| k < 1<br />

n→∞ n→∞<br />

(<br />

) 1/k<br />

1<br />

|z| <<br />

= 2 1/k by (*).<br />

lim n→∞ sup |a n | 1/n<br />

So, the radius of ∑ ∞<br />

n=0 a nz kn is 2 1/k .<br />

(c) ∑ ∞<br />

n=0 a nz n2<br />

Proof: Consider<br />

∣<br />

∣ ∣∣<br />

1/n<br />

lim sup ∣a n z n2 = lim sup |a n | 1/n |z| n<br />

n→∞<br />

and claim that the radius of ∑ ∞<br />

n=0 a nz n2 is 1 as follows.<br />

If |z| < 1, it is clearly seen that the series converges. However, if |z| > 1,<br />

lim sup |a n| 1/n lim inf |z| n ≤ lim sup |a n | 1/n |z| n<br />

n→∞ n→∞ n→∞<br />

29


which impliest that<br />

lim sup |a n| 1/n |z| n = +∞.<br />

n→∞<br />

so, the series diverges. From above, we have proved the claim.<br />

9.32 Given a power series ∑ a n x n whose coefficents are related by an<br />

equation of the form<br />

a n + Aa n−1 + Ba n−2 = 0 (n = 2, 3, ...).<br />

Show that for any x for which the series converges, its sum is<br />

a 0 + (a 1 + Aa 0 ) x<br />

1 + Ax + Bx 2 .<br />

Proof: Consider<br />

which implies that<br />

n=2<br />

which implies that<br />

n=0<br />

∞∑<br />

(a n + Aa n−1 + Ba n−2 ) x n = 0<br />

n=2<br />

∞∑<br />

∞∑<br />

a n x n + Ax a n−1 x n−1 + Bx 2<br />

n=2<br />

∞∑<br />

∞∑<br />

a n x n + Ax a n x n + Bx 2<br />

which implies that<br />

n=0<br />

∞∑<br />

n=0<br />

∞<br />

∑<br />

n=0<br />

∞<br />

∑<br />

n=2<br />

a n x n = a 0 + (a 1 + Aa 0 ) x<br />

1 + Ax + Bx 2 .<br />

a n−2 x n−2 = 0<br />

a n x n = a 0 + a 1 x + Aa 0 x<br />

Remark: We prove that for any x for which the series converges, then<br />

1 + Ax + Bx 2 ≠ 0 as follows.<br />

Proof: Consider<br />

( ) ∑<br />

∞<br />

1 + Ax + Bx<br />

2<br />

a n x n = a 0 + (a 1 + Aa 0 ) x,<br />

n=0<br />

30


if x = λ (≠ 0) is a root of 1 + Ax + Bx 2 , and ∑ ∞<br />

n=0 a nλ n exists, we have<br />

1 + Aλ + Bλ 2 = 0 and a 0 + (a 1 + Aa 0 ) λ = 0.<br />

Note that a 1 + Aa 0 ≠ 0, otherwise, a 0 = 0 (⇒ a 1 = 0) , and therefore, a n =<br />

0 for all n. <strong>The</strong>n there is nothing to prove it. So, put λ = −a 0<br />

a 1 +Aa 0<br />

into<br />

1 + Aλ + Bλ 2 = 0, we then have<br />

a 2 1 = a 0 a 2 .<br />

Note that a 0 ≠ 0, otherwise, a 1 = 0 and a 2 = 0. Similarly, a 1 ≠ 0, otherwise,<br />

we will obtain a trivial thing. Hence, we may assume that all a n ≠ 0 for all<br />

n. So,<br />

a 2 2 = a 1 a 3 .<br />

<strong>And</strong> it is easy to check that a n = a 0<br />

1<br />

λ n for all n ≥ N. <strong>The</strong>refore, ∑ a n λ n =<br />

∑<br />

a0 diverges. So, for any x for whcih the series converges, we have 1+Ax+<br />

Bx 2 ≠ 0.<br />

9.33 Let f (x) = e −1/x2 if x ≠ 0, f (0) = 0.<br />

(a) Show that f (n) (0) exists for all n ≥ 1.<br />

Proof: By Exercise 5.4, we complete it.<br />

(b) Show that the Taylor’s series about 0 generated by f converges everywhere<br />

on R but that it represents f only at the origin.<br />

Proof: <strong>The</strong> Taylor’s series about 0 generated by f is<br />

∞∑<br />

n=0<br />

f (n) (0)<br />

x n =<br />

n!<br />

∞∑<br />

0x n = 0.<br />

So, it converges everywhere on R but that it represents f only at the origin.<br />

Remark: It is an important example to tell us that even for functions<br />

f ∈ C ∞ (R) , the Taylor’s series about c generated by f may NOT represent<br />

f on some open interval. Also see the textbook, pp 241.<br />

9.34 Show that the binomial series (1 + x) α = ∑ ∞<br />

n=0 (α n) x n exhibits the<br />

following behavior at the points x = ±1.<br />

(a) If x = −1, the series converges for α ≥ 0 and diverges for α < 0.<br />

n=0<br />

31


Proof: If x = −1, we consider three cases: (i) α < 0, (ii) α = 0, and (iii)<br />

α > 0.<br />

(i) As α < 0, then<br />

∞∑<br />

( α n) (−1) n =<br />

n=0<br />

∞∑<br />

n=0<br />

(−1) n α (α − 1) · · · (α − n + 1)<br />

,<br />

n!<br />

say a n = (−1) n α(α−1)···(α−n+1)<br />

n!<br />

, then a n ≥ 0 for all n, and<br />

a n<br />

1/n<br />

=<br />

−α (−α + 1) · · · (−α + n − 1)<br />

(n − 1)!<br />

≥ −α > 0 for all n.<br />

Hence, ∑ ∞<br />

n=0 (α n) (−1) n diverges.<br />

(ii) As α = 0, then the series is clearly convergent.<br />

(iii) As α > 0, define a n = n (−1) n ( α n) , then<br />

a n+1<br />

a n<br />

= n − α<br />

n<br />

≥ 1 if n ≥ [α] + 1. (*)<br />

It means that a n > 0 for all n ≥ [α] + 1 or a n < 0 for all n ≥ [α] + 1. Without<br />

loss of generality, we consider a n > 0 for all n ≥ [α] + 1 as follows.<br />

Note that (*) tells us that<br />

and<br />

So,<br />

m∑<br />

n=[α]+1<br />

a n > a n+1 > 0 ⇒ lim<br />

n→∞<br />

a n exists.<br />

a n − a n+1 = α (−1) n ( α n) .<br />

(−1) n ( α n) = 1 α<br />

m∑<br />

n=[α]+1<br />

(a n − a n+1 ) .<br />

By <strong>The</strong>orem 8.10, we have proved the convergence of the series ∑ ∞<br />

n=0 (α n) (−1) n .<br />

(b) If x = 1, the series diverges for α ≤ −1, converges conditionally for α<br />

in the interval −1 < α < 0, and converges absolutely for α ≥ 0.<br />

Proof: If x = 1, we consider four cases as follows: (i) α ≤ −1, (ii)<br />

−1 < −α < 0, (iii) α = 0, and (iv) α > 0 :<br />

(i) As α ≤ −1, say a n = α(α−1)···(α−n+1)<br />

n!<br />

. <strong>The</strong>n<br />

|a n | =<br />

−α (−α + 1) · · · (−α + n − 1)<br />

n!<br />

32<br />

≥ 1 for all n.


So, the series diverges.<br />

(ii) As −1 < α < 0, say a n = α(α−1)···(α−n+1)<br />

n!<br />

. <strong>The</strong>n a n = (−1) n b n , where<br />

with<br />

b n =<br />

b n+1<br />

b n<br />

−α (−α + 1) · · · (−α + n − 1)<br />

n!<br />

= n − α<br />

n<br />

> 0.<br />

< 1 since − 1 < −α < 0<br />

which implies that {b n } is decreasing with limit L. So, if we can show L = 0,<br />

then ∑ a n converges by <strong>The</strong>orem 8.16.<br />

Rewrite<br />

n∏<br />

(<br />

b n = 1 − α + 1 )<br />

k<br />

k=1<br />

and since ∑ α+1<br />

k<br />

diverges, then by <strong>The</strong>roem 8.55, we have proved L = 0.<br />

In order to show the convergence is conditionally, it suffices to show the<br />

divergence of ∑ b n . <strong>The</strong> fact follows from<br />

b n<br />

1/n<br />

=<br />

−α (−α + 1) · · · (−α + n − 1)<br />

(n − 1)!<br />

≥ −α > 0.<br />

(iii) As α = 0, it is clearly that the series converges absolutely.<br />

(iv) As α > 0, we consider ∑ |( α n)| as follows. Define a n = |( α n)| , then<br />

a n+1<br />

a n<br />

= n − α<br />

n + 1<br />

< 1 if n ≥ [α] + 1.<br />

It implies that na n − (n + 1) a n = αa n and (n + 1) a n+1 < na n . So, by<br />

<strong>The</strong>roem 8.10,<br />

∑<br />

an = 1 ∑<br />

nan − (n + 1) a n<br />

α<br />

converges since lim n→∞ na n exists. So, we have proved that the series converges<br />

absolutely.<br />

9.35 Show that ∑ a n x n converges uniformly on [0, 1] if ∑ a n converges.<br />

Use this fact to give another proof of Abel’s limit theorem.<br />

Proof: Define f n (x) = a n on [0, 1] , then it is clear that ∑ f n (x) converges<br />

uniformly on [0, 1] . In addition, let g n (x) = x n , then g n (x) is unifomrly<br />

bouned with g n+1 (x) ≤ g n (x) . So, by Abel’s test for uniform convergence,<br />

33


∑<br />

an x n converges uniformly on [0, 1] . Now, we give another proof of Abel’s<br />

Limit <strong>The</strong>orem as follows. Note that each term of ∑ a n x n is continuous<br />

on [0, 1] and the convergence is uniformly on [0, 1] , so by <strong>The</strong>orem 9.2, the<br />

power series is continuous on [0, 1] . That is, we have proved Abel’s Limit<br />

<strong>The</strong>orem:<br />

lim<br />

x→1 − ∑<br />

an x n = ∑ a n .<br />

9.36 If each a n > 0 and ∑ a n diverges, show that ∑ a n x n → +∞ as<br />

x → 1 − . (Assume ∑ a n x n converges for |x| < 1.)<br />

Proof: Given M > 0, if we can find a y near 1 from the left such that<br />

∑<br />

an y n ≥ M, then for y ≤ x < 1, we have<br />

M ≤ ∑ a n y n ≤ ∑ a n x n .<br />

That is, lim x→1 −<br />

∑<br />

an x n = +∞.<br />

Since ∑ a n diverges, there is a positive integer p such that<br />

p∑<br />

a k ≥ 2M > M. (*)<br />

k=1<br />

Define f n (x) = ∑ n<br />

k=1 a kx k , then by continuity of each f n , given 0 < ε (< M) ,<br />

there exists a δ n > 0 such that as x ∈ [δ n , 1), we have<br />

n∑<br />

a k − ε <<br />

k=1<br />

n∑<br />

a k x k <<br />

k=1<br />

n∑<br />

a k + ε (**)<br />

k=1<br />

By (*) and (**), we proved that as y = δ p<br />

M ≤<br />

p∑<br />

a k − ε <<br />

k=1<br />

p∑<br />

a k y k .<br />

k=1<br />

Hence, we have proved it.<br />

9.37 If each a n > 0 and if lim x→1 −<br />

∑<br />

an x n exists and equals A, prove<br />

that ∑ a n converges and has the sum A. (Compare with <strong>The</strong>orem 9.33.)<br />

Proof: By Exercise 9.36, we have proved the part, ∑ a n converges. In<br />

order to show ∑ a n = A, we apply Abel’s Limit <strong>The</strong>orem to complete it.<br />

34


9.38 For each real t, define f t (x) = xe xt / (e x − 1) if x ∈ R, x ≠ 0,<br />

f t (0) = 1.<br />

(a) Show that there is a disk B (0; δ) in which f t is represented by a power<br />

series in x.<br />

Proof: First, we note that ex −1<br />

= ∑ ∞ x n<br />

x n=0<br />

:= p (x) , then p (0) = 1 ≠<br />

(n+1)!<br />

0. So, by <strong>The</strong>orem 9. 26, there exists a disk B (0; δ) in which the reciprocal<br />

of p has a power series exapnsion of the form<br />

∞<br />

1<br />

p (x) = ∑<br />

q n x n .<br />

So, as x ∈ B (0; δ) by <strong>The</strong>orem 9.24.<br />

n=0<br />

f t (x) = xe xt / (e x − 1)<br />

( ∞<br />

) (<br />

∑ (xt) n ∞<br />

)<br />

∑ x n<br />

=<br />

n! (n + 1)!<br />

n=0<br />

n=0<br />

∞∑<br />

= r n (t) x n .<br />

n=0<br />

(b) Define P 0 (t) , P 1 (t) , P 2 (t) , ..., by the equation<br />

and use the identity<br />

f t (x) =<br />

n=0<br />

∞∑<br />

n=0<br />

P n (t) xn<br />

, if x ∈ B (0; δ) ,<br />

n!<br />

∞∑<br />

P n (t) xn<br />

n! = ∑ ∞ etx P n (0) xn<br />

n!<br />

to prove that P n (t) = ∑ n<br />

k=0 (n k ) P k (0) t n−k .<br />

Proof: Since<br />

f t (x) =<br />

∞∑<br />

n=0<br />

n=0<br />

P n (t) xn<br />

n! = etx x<br />

e x − 1 ,<br />

35


and<br />

f 0 (x) =<br />

n=0<br />

∞∑<br />

n=0<br />

P n (0) xn<br />

n! = x<br />

e x − 1 .<br />

So, we have the identity<br />

∞∑<br />

P n (t) xn<br />

n! = ∑ ∞ etx P n (0) xn<br />

n! .<br />

n=0<br />

Use the identity with e tx = ∑ ∞ t n<br />

n=0 n! xn , then we obtain<br />

n∑<br />

which implies that<br />

P n (t)<br />

n!<br />

=<br />

k=0<br />

= 1 n!<br />

P n (t) =<br />

t n−k P k (0)<br />

(n − k)! k!<br />

n∑<br />

( n k) P k (0) t n−k<br />

k=0<br />

n∑<br />

( n k) P k (0) t n−k .<br />

k=0<br />

This shows that each function P n is a polynomial. <strong>The</strong>re are the Bernoulli<br />

polynomials. <strong>The</strong> numbers B n = P n (0) (n = 0, 1, 2, ...) are called the<br />

Bernoulli numbers. Derive the following further properties:<br />

(c) B 0 = 1, B 1 = − 1 2 , ∑ n−1<br />

k=0 (n k ) B k = 0, if n = 2, 3, ...<br />

Proof: Since 1 = p(x) , where p (x) := ∑ ∞ x n<br />

p(x) n=0<br />

, and 1 := ∑ ∞<br />

(n+1)! p(x) n=0 P n (0) xn . n!<br />

So,<br />

where<br />

1<br />

1 = p (x)<br />

p (x)<br />

∞∑ x n<br />

=<br />

(n + 1)!<br />

n=0<br />

∞∑<br />

= C n x n<br />

C n =<br />

n=0<br />

1<br />

(n + 1)!<br />

n∑<br />

k=0<br />

36<br />

∞∑<br />

n=0<br />

P n (0) xn<br />

n!<br />

( n+1<br />

)<br />

k Pk (0) .


So,<br />

B 0 = P 0 (0) = C 0 = 1,<br />

B 1 = P 1 (0) = C 1 − P 0 (0)<br />

= − 1 2 2 , by C 1 = 0<br />

and note that C n = 0 for all n ≥ 1, we have<br />

0 = C n−1<br />

= 1 ∑n−1<br />

( n<br />

n!<br />

k) P k (0)<br />

k=0<br />

= 1 ∑n−1<br />

( n<br />

n!<br />

k) B k for all n ≥ 2.<br />

k=0<br />

(d) P ′ n (t) = nP n−1 (t) , if n = 1, 2, ...<br />

Proof: Since<br />

P ′ n (t) =<br />

n∑<br />

( n k) P k (0) (n − k) t n−k−1<br />

k=0<br />

n−1<br />

∑<br />

= ( n k) P k (0) (n − k) t n−k−1<br />

k=0<br />

n−1<br />

∑<br />

=<br />

k=0<br />

n−1<br />

n! (n − k)<br />

k! (n − k)! P k (0) t (n−1)−k<br />

∑ (n − 1)!<br />

= n<br />

k! (n − 1 − k)! P k (0) t (n−1)−k<br />

k=0<br />

n−1<br />

∑<br />

= n<br />

k=0<br />

( n−1<br />

)<br />

k Pk (0) t (n−1)−k<br />

= nP n−1 (t) if n = 1, 2, ...<br />

(e) P n (t + 1) − P n (t) = nt n−1 if n = 1, 2, ...<br />

37


Proof: Consider<br />

f t+1 (x) − f t (x) =<br />

so as n = 1, 2, ..., we have<br />

∞∑<br />

n=0<br />

[P n (t + 1) − P n (t)] xn<br />

n!<br />

= xe xt by f t (x) = xe xt / (e x − 1)<br />

∞∑<br />

= (n + 1) t n xn+1<br />

(n + 1)! ,<br />

n=0<br />

P n (t + 1) − P n (t) = nt n−1 .<br />

by (b)<br />

(f) P n (1 − t) = (−1) n P n (t)<br />

Proof: Note that<br />

so we have<br />

n=0<br />

Hence, P n (1 − t) = (−1) n P n (t) .<br />

(g) B 2n+1 = 0 if n = 1, 2, ...<br />

f t (−x) = f 1−t (x) ,<br />

∞∑<br />

∞<br />

(−1) n P n (t) xn<br />

n! = ∑<br />

P n (1 − t) xn<br />

n! .<br />

Proof: With help of (e) and (f), let t = 0 and n = 2k + 1, then it is clear<br />

that B 2k+1 = 0 if k = 1, 2, ...<br />

n=0<br />

(h) 1 n + 2 n + ... + (k − 1) n = P n+1(k)−P n+1 (0)<br />

n+1<br />

(n = 2, 3, ...)<br />

Proof: With help of (e), we know that<br />

P n+1 (t + 1) − P n+1 (t)<br />

n + 1<br />

= t n<br />

which implies that<br />

1 n + 2 n + ... + (k − 1) n = P n+1 (k) − P n+1 (0)<br />

n + 1<br />

(n = 2, 3, ...)<br />

38


Remark: (1) <strong>The</strong> reader can see the book, Infinite Series by Chao<br />

Wen-Min, pp 355-366. (Chinese Version)<br />

(2) <strong>The</strong>re are some special polynomials worth studying, such as Legengre<br />

Polynomials. <strong>The</strong> reader can see the book, Essentials of Ordinary<br />

Differential Equations by Ravi P. Agarwal and Ramesh C. Gupta.<br />

pp 305-312.<br />

(3) <strong>The</strong> part (h) tells us one formula to calculte the value of the finite<br />

series ∑ m<br />

k=1 kn . <strong>The</strong>re is an interesting story from the mail that Fermat,<br />

pierre de (1601-1665) sent to Blaise Pascal (1623-1662). Fermat<br />

used the Mathematical Induction to show that<br />

n∑<br />

k (k + 1) · · · (k + p) =<br />

k=1<br />

n (n + 1) · · · (n + p + 1)<br />

. (*)<br />

p + 2<br />

In terms of (*), we can obtain another formula on ∑ m<br />

k=1 kn .<br />

39


Limit sup and limit inf.<br />

Introduction<br />

In order to make us understand the information more on approaches of a given real<br />

sequence a n <br />

n1<br />

, we give two definitions, thier names are upper limit and lower limit. It<br />

is fundamental but important tools in analysis.<br />

Definition<br />

Definition of limit sup and limit inf<br />

and<br />

Example<br />

Example<br />

Example<br />

Given a real sequence a n <br />

n1<br />

,wedefine<br />

b n supa m : m n<br />

1 1 n <br />

n1<br />

1 n <br />

n n1<br />

<br />

n n1<br />

c n infa m : m n.<br />

0,2,0,2,..., sowehave<br />

b n 2andc n 0 for all n.<br />

1, 2, 3,4,..., sowehave<br />

b n and c n for all n.<br />

1, 2, 3, . . . , sowehave<br />

b n n and c n for all n.<br />

Proposition Given a real sequence a n <br />

n1<br />

, and thus define b n and c n as the same as<br />

before.<br />

1 b n ,andc n n N.<br />

2 If there is a positive integer p such that b p , then b n n N.<br />

If there is a positive integer q such that c q , then c n n N.<br />

3 b n is decreasing and c n is increasing.<br />

By property 3, we can give definitions on the upper limit and the lower limit of a given<br />

sequence as follows.<br />

Definition Given a real sequence a n and let b n and c n as the same as before.<br />

(1) If every b n R, then<br />

infb n : n N<br />

is called the upper limit of a n , denoted by<br />

lim n<br />

sup a n .<br />

That is,<br />

lim n<br />

sup a n inf b n.<br />

n<br />

If every b n , then we define<br />

lim n<br />

sup a n .<br />

(2) If every c n R, then<br />

supc n : n N<br />

is called the lower limit of a n , denoted by<br />

lim n<br />

inf a n .


That is,<br />

lim n<br />

inf a n sup c n .<br />

n<br />

If every c n , then we define<br />

lim n<br />

inf a n .<br />

Remark <strong>The</strong> concept of lower limit and upper limit first appear in the book (Analyse<br />

Alge’brique) written by Cauchy in 1821. But until 1882, Paul du Bois-Reymond<br />

gave explanations on them, it becomes well-known.<br />

Example 1 1 n <br />

n1<br />

0,2,0,2,..., sowehave<br />

b n 2andc n 0 for all n<br />

which implies that<br />

lim sup a n 2 and lim inf a n 0.<br />

Example 1 n <br />

n n1<br />

1, 2, 3,4,..., sowehave<br />

b n and c n for all n<br />

which implies that<br />

lim sup a n and lim inf a n .<br />

<br />

n n1<br />

Example 1, 2, 3, . . . , sowehave<br />

b n n and c n for all n<br />

which implies that<br />

lim sup a n and lim inf a n .<br />

Relations with convergence and divergence for upper (lower) limit<br />

<strong>The</strong>orem Let a n be a real sequence, then a n converges if, and only if, the upper<br />

limit and the lower limit are real with<br />

lim n<br />

sup a n lim n<br />

inf a n lim n<br />

a n .<br />

<strong>The</strong>orem Let a n be a real sequence, then we have<br />

(1) lim n sup a n a n has no upper bound.<br />

(2) lim n sup a n for any M 0, there is a positive integer n 0 such<br />

that as n n 0 ,wehave<br />

a n M.<br />

(3) lim n sup a n a if, and only if, (a) given any 0, there are infinite<br />

many numbers n such that<br />

a a n<br />

and (b) given any 0, there is a positive integer n 0 such that as n n 0 ,wehave<br />

a n a .<br />

Similarly, we also have<br />

<strong>The</strong>orem Let a n be a real sequence, then we have<br />

(1) lim n inf a n a n has no lower bound.<br />

(2) lim n inf a n for any M 0, there is a positive integer n 0 such


that as n n 0 ,wehave<br />

a n M.<br />

(3) lim n inf a n a if, and only if, (a) given any 0, there are infinite<br />

many numbers n such that<br />

a a n<br />

and (b) given any 0, there is a positive integer n 0 such that as n n 0 ,wehave<br />

a n a .<br />

From <strong>The</strong>orem 2 an <strong>The</strong>orem 3, the sequence is divergent, we give the following<br />

definitios.<br />

Definition Let a n be a real sequence, then we have<br />

(1) If lim n sup a n , then we call the sequence a n diverges to ,<br />

denoted by<br />

lim n<br />

a n .<br />

<strong>The</strong>orem<br />

<strong>The</strong>orem<br />

<strong>The</strong>orem<br />

(2) If lim n inf a n , then we call the sequence a n diverges to ,<br />

denoted by<br />

lim n<br />

a n .<br />

Let a n be a real sequence. If a is a limit point of a n , then we have<br />

lim n<br />

inf a n a lim n<br />

sup a n .<br />

Some useful results<br />

Let a n be a real sequence, then<br />

(1) lim n inf a n lim n sup a n .<br />

(2) lim n infa n lim n sup a n and lim n supa n lim n inf a n<br />

(3) If every a n 0, and 0 lim n inf a n lim n sup a n , then we<br />

have<br />

lim n<br />

sup a 1 n<br />

1<br />

lim n inf a n<br />

Let a n and b n be two real sequences.<br />

(1) If there is a positive integer n 0 such that a n b n , then we have<br />

lim n<br />

inf a n lim n<br />

inf b n and lim n<br />

sup a n lim n<br />

sup b n .<br />

(2) Suppose that lim n inf a n , lim n inf b n , lim n sup a n ,<br />

lim n sup b n , then<br />

lim n<br />

inf a n lim n<br />

inf b n<br />

and lim n<br />

inf 1 a n<br />

1<br />

lim n sup a n<br />

.<br />

lim n<br />

infa n b n <br />

lim n<br />

inf a n lim n<br />

sup b n (or lim n<br />

sup a n lim n<br />

inf b n )<br />

lim n<br />

supa n b n <br />

lim n<br />

sup a n lim n<br />

sup b n .<br />

In particular, if a n converges, we have<br />

lim n<br />

supa n b n lim n<br />

a n lim n<br />

sup b n


and<br />

lim n<br />

infa n b n lim n<br />

a n lim n<br />

inf b n .<br />

(3) Suppose that lim n inf a n , lim n inf b n , lim n sup a n ,<br />

lim n sup b n , anda n 0, b n 0 n, then<br />

lim n<br />

inf a n<br />

lim n<br />

inf b n<br />

lim n<br />

infa n b n <br />

lim n<br />

inf a n lim n<br />

sup b n (or lim n<br />

inf b n<br />

lim n<br />

supa n b n <br />

lim n<br />

sup a n lim n<br />

sup b n .<br />

In particular, if a n converges, we have<br />

and<br />

lim n<br />

supa n b n <br />

lim n<br />

infa n b n <br />

lim n<br />

a n<br />

lim n<br />

a n<br />

lim n<br />

sup b n<br />

lim n<br />

inf b n .<br />

lim n<br />

sup a n )<br />

<strong>The</strong>orem Let a n be a positive real sequence, then<br />

lim n<br />

inf a n1<br />

a n<br />

lim n<br />

infa n 1/n lim n<br />

supa n 1/n lim n<br />

sup a n1<br />

a n<br />

.<br />

Remark We can use the inequalities to show<br />

n! 1/n<br />

lim n n 1/e.<br />

<strong>The</strong>orem Let a n be a real sequence, then<br />

lim n<br />

inf a n lim n<br />

inf a 1 ...a n<br />

n<br />

lim n<br />

sup a 1 ...a n<br />

n<br />

lim n<br />

sup a n .<br />

Exercise Let f : a, d R be a continuous function, and a n is a real sequence. If f is<br />

increasing and for every n, lim n inf a n , lim n sup a n a, d, then<br />

lim n<br />

sup fa n f lim n<br />

sup a n<br />

and lim n<br />

inf fa n f lim n<br />

inf a n .<br />

Remark: (1) <strong>The</strong> condition that f is increasing cannot be removed. For<br />

example,<br />

fx |x|,<br />

and<br />

1/k if k is even<br />

a k <br />

1 1/k if k is odd.<br />

(2) <strong>The</strong> proof is easy if we list the definition of limit sup and limit inf. So, we<br />

omit it.<br />

Exercise Let a n be a real sequence satisfying a np a n a p for all n, p. Show that<br />

an<br />

n converges.<br />

Hint: Consider its limit inf.


Something around the number e<br />

1. Show that the sequence 1 1 n n converges, and denote the limit by e.<br />

Proof: Since<br />

1 1 n<br />

n<br />

n<br />

kn 1 n<br />

k<br />

k0<br />

1 n 1 n<br />

<br />

nn 1<br />

2!<br />

1<br />

n<br />

1 1 <br />

2!<br />

1 1 1 n ...<br />

n!<br />

1<br />

1 1 1 2 1 2 .. 1 ...<br />

2 2 n1<br />

3,<br />

2 nn 1 1<br />

.. 1<br />

n! n<br />

1 1 n 1 n n<br />

1<br />

and by (1), we know that the sequence is increasing. Hence, the sequence is convergent.<br />

We denote its limit e. Thatis,<br />

lim n<br />

1 1 n<br />

n e.<br />

n<br />

1<br />

Remark: 1. <strong>The</strong> sequence and e first appear in the mail that Euler wrote to Goldbach.<br />

It is a beautiful formula involving<br />

e i 1 0.<br />

1<br />

2. Use the exercise, we can show that k0<br />

e as follows.<br />

k!<br />

Proof: Let x n 1 1 n n , and let k n, wehave<br />

1 1 <br />

2!<br />

1 1 1 ..<br />

k n!<br />

1 1 1 1 <br />

k<br />

n k<br />

1 x k<br />

which implies that ( let k )<br />

On the other hand,<br />

So,by(2)and(3),wefinallyhave<br />

n<br />

y n : <br />

i0<br />

<br />

<br />

k0<br />

1<br />

i! e. 2<br />

x n y n 3<br />

1<br />

k! e. 4<br />

3. e is an irrational number.<br />

Proof: Assume that e is a rational number, say e p/q, where g.c.d. p, q 1. Note<br />

that q 1. Consider<br />

q!e q!<br />

q!<br />

<br />

<br />

k0<br />

q<br />

<br />

k0<br />

1<br />

k!<br />

1<br />

k!<br />

q!<br />

<br />

<br />

kq1<br />

and since q! k0<br />

q 1<br />

k!<br />

and q!e are integers, we have q! kq1<br />

1<br />

k!<br />

is also an<br />

integer. However,<br />

1<br />

k!<br />

,


q!<br />

<br />

<br />

kq1<br />

1<br />

k!<br />

<br />

<br />

kq1<br />

q!<br />

k!<br />

1<br />

q 1 1<br />

q 1q 2 ...<br />

1<br />

q 1 1<br />

q 1<br />

2<br />

...<br />

1 q<br />

1,<br />

a contradiction. So, we know that e is not a rational number.<br />

n 1<br />

4. Here is an estimate about e k0 k!<br />

, where 0 1. ( In fact, we know<br />

nn!<br />

that e 2. 71828 18284 59045 . . . . )<br />

<br />

Proof: Since e k0<br />

1<br />

k!<br />

<br />

0 e x n <br />

kn1<br />

1<br />

n 1!<br />

1<br />

n 1!<br />

1<br />

k! ,wherex n <br />

n<br />

k0<br />

1<br />

k!<br />

1 1<br />

n 2 1<br />

n 2n 3 ...<br />

1 1<br />

n 2 1<br />

n 2 2<br />

1<br />

n 1! n n 2<br />

1<br />

1<br />

nn! since n 2 1<br />

n 1 2 n .<br />

So, we finally have<br />

n<br />

e <br />

1<br />

k! , where 0 1.<br />

nn!<br />

k0<br />

Note: We can use the estimate dorectly to show e is an irrational number.<br />

2. For continuous variables, we have the samae result as follows. That is,<br />

x<br />

e.<br />

lim<br />

x<br />

1 1 x<br />

Proof: (1) Since 1 1 n n e as n , we know that for any sequence a n N,<br />

with a n , wehave<br />

lim n<br />

1 a 1 a n<br />

n<br />

e. 5<br />

(2) Given a sequence x n with x n , and define a n x n , then<br />

a n x n a n 1, then we have<br />

a n x<br />

1 1 1 1 n<br />

a n 1<br />

xn<br />

1 a 1 a n1<br />

n<br />

.<br />

Since<br />

a n a<br />

1 1 e and 1 1 n1<br />

a n 1<br />

an e as x by (5)<br />

we know that<br />

...


lim 1 1 x n<br />

n x n<br />

e.<br />

Since x n is arbitrary chosen so that it goes infinity, we finally obtain that<br />

lim 1 1 x<br />

x x e. 6<br />

(3) In order to show 1 1 x x e as x , weletx y, then<br />

1 1 x y<br />

x 1 1 y<br />

<br />

y<br />

y 1<br />

1 1 1 1<br />

y 1 y 1 .<br />

Note that x y , by (6), we have shown that<br />

e y<br />

lim 1 1<br />

y 1<br />

lim x<br />

1 1 x<br />

x<br />

.<br />

y<br />

y1<br />

y1<br />

1 1<br />

y 1<br />

3. Prove that as x 0, we have1 1 x x is strictly increasing, and 1 1 x x1 is<br />

dstrictly ecreasing.<br />

Proof: Since, by Mean Value <strong>The</strong>orem<br />

1<br />

x 1 log 1 1 x logx 1 logx 1 1 x for all x 0,<br />

we have<br />

x log 1 1 <br />

x log 1 1 x 1 0 for all x 0<br />

x 1<br />

and<br />

<br />

x 1 log 1 1 x log 1 1 x 1 x 0 for all x 0.<br />

Hence, we know that<br />

x log 1 1 x is strictly increasing on 0, <br />

and<br />

x 1 log 1 1 x is strictly decreasing on 0, .<br />

It implies that<br />

1 1 x<br />

x<br />

is strictly increasing 0, , and 1 1 x<br />

x1<br />

is strictly decreasing on 0, .<br />

Remark: By exercise 2, we know that<br />

lim<br />

x<br />

1 1 x<br />

x<br />

e lim<br />

x<br />

1 1 x<br />

4. Follow the Exercise 3 to find the smallest a such that 1 1 x xa e and strictly<br />

decreasing for all x 0, .<br />

Proof: Let fx 1 1 x xa , and consider<br />

log fx x a log 1 1 x : gx,<br />

Let us consider<br />

x1<br />

.


g x log 1 1 x x a<br />

x 2 x<br />

log1 y y 1 ay 2 1<br />

1 y , where 0 y 1<br />

1 x 1<br />

<br />

<br />

y<br />

k<br />

k y 1 ay2 y k<br />

k1<br />

k0<br />

1<br />

2<br />

a y 2 1<br />

3<br />

a y 3 ... 1 n a yn ...<br />

It is clear that for a 1/2, we have g x 0 for all x 0, . Note that for a 1/2,<br />

if there exists such a so that f is strictly decreasing for all x 0, . <strong>The</strong>n g x 0for<br />

all x 0, . However, it is impossible since<br />

g x 1 a y 2 1 a y 3 ... 1 2 3 n a yn ...<br />

1 2 a 0asy 1 .<br />

So, we have proved that the smallest value of a is 1/2.<br />

Remark: <strong>The</strong>re is another proof to show that 1 1 x x1/2 is strictly decreasing on<br />

0, .<br />

Proof: Consider ht 1/t, and two points 1, 1 and 1 1 1<br />

x , lying on the graph<br />

1 1 x<br />

From three areas, the idea is that<br />

<strong>The</strong> area of lower rectangle <strong>The</strong>areaofthecurve <strong>The</strong>areaoftrapezoid<br />

So, we have<br />

1<br />

1 x 1 x<br />

Consider<br />

1 1 x<br />

1<br />

1 1 x<br />

x1/2<br />

<br />

<br />

log 1 1 x 1 2x<br />

1 1 x<br />

x1/2<br />

1 1<br />

1 1 x<br />

log 1 1 x x 1 2<br />

x 1 2<br />

1<br />

xx 1<br />

1<br />

xx 1<br />

0by(7);<br />

hence, we know that 1 1 x x1/2 is strictly decreasing on 0, .<br />

Note: Use the method of remark, we know that 1 1 x x is strictly increasing on<br />

0, .<br />

. 7

Hooray! Your file is uploaded and ready to be published.

Saved successfully!

Ooh no, something went wrong!